Sie sind auf Seite 1von 294

CRIMINAL LAW 1

(Sunday, Section 52) 1

CRIMINAL LAW 1
CASE DIGESTS

MARCH 2019
CRIMINAL LAW 1
(Sunday, Section 52) 2

CANCERAN VS. PEOPLE OF THE PHIL.


G.R. NO. 206442 JULY 1, 2015
(Unilever – attempted theft – convicted)

FACTS:
On October 6, 2002, at more or less 12:00 noon, at Ororama Mega Center Grocery
Department, Lapasan, Cagayan de Oro City, Jovito Canceran, conspiring, confederating together and
mutually helping one another with his co-accused Frederick Vequizo, URC Merchandiser,
and Marcial Diaz, Jr., a Unilever Philippines merchandiser both of Ororama Mega Center.
With intent to gain and without the knowledge
and consent of the owner, did they willfully, unlawfully and feloniously take, steal and carry away
14 cartons of Ponds White Beauty Cream valued at ₱28,627,20, belonging to Ororama Mega
Center, represented by William Michael N. Arcenio

Thus, performing all the acts of execution which would produce the crime of theft as a consequence
but, nevertheless, did not produce it by reason of some cause independent of accused’s will.

They were discovered by the employees of Ororama Mega Center who prevented them from further
carrying away said 14 cartons of Ponds White Beauty Cream, to the damage and prejudice of the
Ororama Mega Center.

Version of the Prosecution


The prosecution presented Damalito Ompoc (Ompoc), a security guard; and William Michael N.
Arcenio (Arcenio), the Customer Relation Officer of Ororama Mega Center (Ororama), as its
witnesses. Through their testimonies, the prosecution established that on or about October 6,
2002, Ompoc saw Canceran approach one of the counters in Ororama Canceran was pushing
a cart which contained 2 boxes of Magic Flakes which he paid ₱1,423.00

Ompoc went to the packer and asked if the boxes had been checked; upon inspection by Ompoc and
the packer, they found out that the contents of the 2 boxes were not Magic Flakes biscuits, but 14
smaller boxes of Ponds White Beauty Cream worth ₱28,627.20

Canceran hurriedly left and a chase ensued; that upon reaching the Don Mariano gate,
Canceran stumbled as he attempted to ride a jeepney

After being questioned, he tried to settle with the guards and even offered his personal effects to pay
for the items he tried to take; But, Arcenio refused to settle and his personal belongings were
deposited in the office of Arcenio.

Version of the Defense


Canceran vehemently denied the charges against him and claimed that he was a
promo merchandiser of La Tondeña, Inc. On October 6, 2002, he was in Ororama to buy medicine for
his wife.

On his way out, after buying medicine and mineral water, an unknown male person around 20 yrs
old requested him to pay for the items in his cart at the cashier at the amount of ₱1,440.00.
CRIMINAL LAW 1
(Sunday, Section 52) 3

After paying at the cashier, he went out of Ororama towards Limketkai to take a jeepney; that three
persons ran after him, and he was caught; that he was brought to the 4th floor of Ororama, where he
was mauled and kicked by one of those who chased him

They took his Nokia 5110 cellular phone and cash amounting to ₱2,500.00; and Ompoc took
his Seiko watch and ring, while a certain Amion took his necklace.
Canceran further claimed that an earlier Information for theft was already filed on October 9,2002
which was eventually dismissed. In January 2003, a second Information was filed for the same
offense over the same incident and became the subject of the present case.

ISSUES:
1. WON Canceran should be acquitted in the crime of theft as it was not charged in the information
2. WON there was double jeopardy.

RULING:
The Court finds the petition partially meritorious.

Constitutional Right of the Accused to be Informed of the Nature and Cause of Accusation against
Him.

No less than the Constitution guarantees the right of every person accused in a criminal prosecution
to be informed of the nature and cause of accusation against him. It is fundamental that every element
of which the offense is composed must be alleged in the complaint or information. The main purpose
of requiring the various elements of a crime to be set out in the information is to enable the accused
to suitably prepare his defense. He is presumed to have no independent knowledge of the facts that
constitute the offense.

Under Article 308 of the RPC, the essential elements of theft are (1) the taking of personal property;
(2) the property belongs to another; (3) the taking away was done with intent of gain; (4) the taking
away was done without the consent of the owner; and (5) the taking away is accomplished without
violence or intimidation against person or force upon things. "Unlawful taking, which is the
deprivation of one's personal property, is the element which produces the felony in its consummated
stage. At the same time, without unlawful taking as an act of execution, the offense could only be
attempted theft, if at all."

As stated earlier, there is no crime of Frustrated Theft. The Information can never be read to charge
Canceran of consummated Theft because the indictment itself stated that the crime was never
produced. Instead, the Information should be construed to mean that Canceran was being charged
with theft in its attempted stage only. Necessarily, Canceran may only be convicted of the lesser crime
of Attempted Theft.

"[A]n accused cannot be convicted of a higher offense than that with which he was charged in the
complaint or information and on which he was tried. It matters not how conclusive and convincing
the evidence of guilt may be, an accused cannot be convicted in the courts of any offense, unless it is
charged in the complaint or information on which he is tried, or necessarily included therein. He has
a right to be informed as to the nature of the offense with which he is charged before he is put on
trial, and to convict him of an offense higher than that charged in the complaint or information on
which he is tried would be an unauthorized denial of that right."
CRIMINAL LAW 1
(Sunday, Section 52) 4

Indeed, an accused cannot be convicted of a crime, even if duly proven, unless it is alleged or
necessarily included in the information filed against him. An offense charged necessarily includes the
offense proved when some of the essential elements or ingredients of the former, as alleged in the
complaint or information, constitute the latter.

The crime of theft in its consummated stage undoubtedly includes the crime in its attempted stage.
In this case, although the evidence presented during the trial prove the crime of consummated Theft,
he could be convicted of Attempted Theft only. Regardless of the overwhelming evidence to convict
him for consummated Theft, because the Information did not charge him with consummated Theft,
the Court cannot do so as the same would violate his right to be informed of the nature and cause of
the allegations against him, as he so protests.

No double jeopardy when the first jeopardy never attached. Anent the issue of double jeopardy, the
Court finds no reason to deviate from the ruling of the CA.

No person shall be twice put in jeopardy for punishment for the same offense. The rule of double
jeopardy has a settled meaning in this jurisdiction. It means that when a person is charged with an
offense and the case is terminated either by acquittal or conviction or in any other manner without
the consent of the accused, the latter cannot again be charged with the same or identical offense. This
principle is founded upon the law of reason, justice and conscience.

Canceran argues that double jeopardy exists as the first case was scheduled for arraignment and he,
already bonded, was ready to enter a plea. It was the RTC who decided that there was insufficient
evidence to constitute the crime of theft.

To raise the defense of double jeopardy, three requisites must be present: (1) a first jeopardy must
have attached prior to the second; (2) the first jeopardy must have been validly terminated; and (3)
the second jeopardy must be for the same offense as that in the first. Legal jeopardy attaches only (a)
upon a valid indictment, (b) before a competent court, (c) after arraignment, (d) a valid plea having
been entered; and (e) the case was dismissed or otherwise terminated without the express consent
of the accused.

Penalty of Attempted Theft

The penalty for consummated theft is prision mayor in its minimum and medium periods. The
penalty lower by two degrees than that prescribed by law for the consummated felony shall be
imposed upon principals in an attempt to commit a felony.[30] The basis for reduction of penalty by
two degrees is the penalty prescribed by law for the consummated crime. Also, when the offenses
defined in the RPC are punished with a penalty composed of two periods, like in the crime of theft,
the penalty lower by one degree is formed by two periods to be taken from the same penalty
prescribed.

Here, the products stolen were worth P28,627.20. Following Article 309 par. 1 of the RPC, the penalty
shall be the maximum period of the penalty prescribed in the same paragraph, because the value of
the things stolen exceeded P22,000.00. In other words, a special aggravating circumstance shall affect
the imposable penalty.

Applying the Indeterminate Sentence Law, the minimum penalty should be within the range of
Arresto Mayor Minimum to Arresto Mayor Medium. In view of the special aggravating circumstance
CRIMINAL LAW 1
(Sunday, Section 52) 5

under Article 309 (1), the maximum penalty should be Arresto Mayor Maximum to Prision
Correccional Minimum in its maximum period.

BAYAN VS. ZAMORA


G.R. NO. 138570. OCTOBER 10, 2000
[VFA-constitutional]

FACTS:
On March 14, 1947, the Philippines and the United States of America forged a Military Bases
Agreement which formalized, among others, the use of installations in the Philippine territory by
United States military personnel. In view of the impending expiration of the RP-US Military Bases
Agreement in 1991, the Philippines and the United States negotiated for a possible extension of the
military bases agreement. On September 16, 1991, the Philippine Senate rejected the proposed RP-
US Treaty of Friendship, Cooperation and Security which, in effect, would have extended the presence
of US military bases in the Philippines. On July 18, 1997, the United States panel, headed by US
Defense Deputy Assistant Secretary for Asia Pacific Kurt Campbell, met with the Philippine panel,
headed by Foreign Affairs Undersecretary Rodolfo Severino Jr., to exchange notes on “the
complementing strategic interests of the United States and the Philippines in the Asia-Pacific region.”
Both sides discussed, among other things, the possible elements of the Visiting Forces Agreement
(VFA for brevity). Thereafter, then President Fidel V. Ramos approved the VFA, which was
respectively signed by public respondent Secretary Siazon and Unites States Ambassador Thomas
Hubbard. On October 5, 1998, President Joseph E. Estrada, through respondent Secretary of Foreign
Affairs, ratified the VFA. On October 6, 1998, the President, acting through respondent Executive
Secretary Ronaldo Zamora, officially transmitted to the Senate of the Philippines, the Instrument of
Ratification, the letter of the President and the VFA, for concurrence pursuant to Section 21, Article
VII of the 1987 Constitution

ISSUES:
(1) Whether or not petitioners have legal standing as concerned citizens, taxpayers, or legislators to
question the constitutionality of the VFA;
(2) whether the VFA is governed by the provisions of Section 21, Article VII or of Section 25, Article
XVIII of the Constitution; (3) and whether or not the Supreme Court has jurisdiction.

RULING:
(1) No. Petitioners failed to show that they have sustained, or are in danger of sustaining any direct
injury as a result of the enforcement of the VFA. As taxpayers, petitioners have not established that
the VFA involves the exercise by Congress of its taxing or spending powers. On this point, it bears
stressing that a taxpayer’s suit refers to a case where the act complained of directly involves the
illegal disbursement of public funds derived from taxation.

(2) Yes.The fact that the President referred the VFA to the Senate under Section 21, Article VII, and
that the Senate extended its concurrence under the same provision, is immaterial. For in either case,
whether under Section 21, Article VII or Section 25, Article XVIII, the fundamental law is crystalline
that the concurrence of the Senate is mandatory to comply with the strict constitutional
requirements.

(3) No. In fine, absent any clear showing of grave abuse of discretion on the part of respondents, the
Court as the final arbiter of legal controversies and staunch sentinel of the rights of the people is then
without power to conduct an incursion and meddle with such affairs purely executive and legislative
in character and nature. For the Constitution no less, maps out the distinct boundaries and limits the
CRIMINAL LAW 1
(Sunday, Section 52) 6

metes and bounds within which each of the three political branches of government may exercise the
powers exclusively and essentially conferred to it by law.

SOLIVEN VS. MAKASIAR


G.R. NO. L-82585, 14 NOVEMBER 1988

● While the President is immune from suit, she may not be prevented from instituting suit. The
privilege of immunity from suit, pertains to the President by virtue of the office and may be invoked
only by the holder of the office; not by any other person in the President's behalf.

● Due process of law does not require that the respondent in a criminal case actually file his counter-
affidavits before the preliminary investigation is deemed completed. All that is required is that the
respondent be given the opportunity to submit counter-affidavits if he is so minded.

● What the Constitution underscores is the exclusive and personal responsibility of the issuing judge
to satisfy himself of the existence of probable cause. In satisfying himself of the existence of probable
cause for the issuance of a warrant of arrest, the judge is not required to personally examine the
complainant and his witnesses.

FACTS:
Pres. Cory Aquino filed a criminal complaint for libel against Beltran. Beltranargues that "the reasons
which necessitate presidential immunity from suit impose a correlative disability to file suit".
He contends that if criminal proceedings ensue by virtue of the President's filing of her complaint-
affidavit, she may subsequently have to be a witness for the prosecution, bringing her under the trial
court's jurisdiction. This would in an indirect way defeat her privilege of immunity from suit, as
by testifying on the witness stand, she would be exposing herself to possible contempt of court or
perjury. Beltran also contends that he could not be held liable for libel because of the privileged
character of the publication. He also says that to allow the libel case to proceed would produce a
“chilling effect” on press freedom.

ISSUES:
(1) whether or not petitioners were denied due process when informations for libel were filed
against them although the finding of the existence of a prima faciecase was still under review by the
Secretary of Justice and, subsequently, by the President;

(2) whether or not the constitutional rights of Beltran were violated when respondent RTC judge
issued a warrant for his arrest without personally examining the complainant and the witnesses, if
any, to determine probable cause; and

(3) whether or not the President of the Philippines, under the Constitution, may initiate criminal
proceedings against the petitioners through the filing of a complaint-affidavit.

RULING:
(1) The allegation of denial of due process of law in the preliminary investigation is negated by the
fact that instead of submitting his counter- affidavits, he filed a "Motion to Declare Proceedings
Closed," in effect waiving his right to refute thecomplaint by filing counter-affidavits. Due process of
law does not require that the respondent in a criminal case actually file his counter-affidavits before
the preliminary investigation is deemed completed. All that is required is that the respondent be
given the opportunity to submit counter-affidavits if he is so minded.
CRIMINAL LAW 1
(Sunday, Section 52) 7

(2) What the Constitution underscores is the exclusive and personal responsibility of the issuing
judge to satisfy himself of the existence of probable cause. In satisfying himself of the existence of
probable cause for the issuance of a warrant of arrest, the judge is not required to personally examine
the complainant and his witnesses. Following established doctrine and procedure, he shall: (1)
personally evaluate the report and the supporting documents submitted by the fiscal regarding the
existence of probable cause and, on the basis thereof, issue a warrant of arrest; or (2) if on the basis
thereof he finds no probable cause, he may disregard the fiscal's report and require the submission
of supporting affidavits of witnesses to aid him in arriving at a conclusion as to the existence of
probable cause.

Sound policy dictates this procedure, otherwise judges would be unduly laden with the preliminary
examination and investigation of criminal complaints instead of concentrating on hearing and
deciding cases filed before their courts.

(3) The rationale for the grant to the President of the privilege of immunity from suit is to assure the
exercise of Presidential duties and functions free from any hindrance or distraction, considering that
being the Chief Executive of the Government is a job that, aside from requiring all of the office holder's
time, also demands undivided attention.

But this privilege of immunity from suit, pertains to the President by virtue of the office and may be
invoked only by the holder of the office; not by any other person in the President's behalf. Thus, an
accused in a criminal case in which the President is complainant cannot raise the presidential
privilege as a defense to prevent the case from proceeding against such accused.

Moreover, there is nothing in our laws that would prevent the President from waiving the privilege.
Thus, if so minded the President may shed the protection afforded by the privilege and submit to the
court's jurisdiction. The choice of whether to exercise the privilege or to waive it is solely the
President's prerogative. It is a decision that cannot be assumed and imposed by any other person.

(4) Court reiterates that it is not a trier of facts. Court finds no basis at this stage to rule on the “chilling
effect” point. (Beltran vs. Makasiar, G.R. No. 82585 November 14, 1988)

LOURDES RUBRICO ET. AL. VS. GLORIA MACAPAGAL-ARROYO, ET. AL.


G.R. NO. 183871, 2010

FACTS:
Rubrico, in her petition, said she was abducted on April 3, 2007 by armed men belonging to the 301st
Air Intelligence and Security Squadron, based at the Philippine Air Force Field Station at Fernando
Air Base in Lipa City, Batangas. During her detention, the petitioner added, her daughters Mary Joy
Rubrico Carbonel and Jean Rubrico Apruebo were harassed by Senior Insp. Arsenio Gomez and that
there were also armed men following them. The petitioners prayed that a writ of amparo be issued,
ordering the individual respondents to desist from performing any threatening act against the
security of the petitioners and for the Office of the Ombudsman (OMB) to immediately file an
information for kidnapping qualified with the aggravating circumstance of gender of the offended
party. It also prayed for damages and for respondents to produce documents submitted to any of
them on the case of Lourdes.

The respondents then filed a joint return on the writ specifically denying the material inculpatory
averments against them. Respondents interposed the defense that the President may not be sued
during her incumbency.
CRIMINAL LAW 1
(Sunday, Section 52) 8

Petitioners pleaded back to be allowed to present evidence ex parte against the President, et al.

By a separate resolution, the CA dropped the President as respondent in the case .

ISSUE:
WHETHER OR NOT the [CA] committed reversible error in dismissing [their] Petition and dropping
President Gloria Macapagal Arroyo as party respondent.

RULING:
The presidential immunity from suit remains preserved under our system of government, albeit not
expressly reserved in the present constitution. Addressing a concern of his co-members in the 1986
Constitutional Commission on the absence of an express provision on the matter, Fr. Joaquin Bernas,
S.J. observed that it was already understood in jurisprudence that the President may not be sued
during his or her tenure.

Settled is the doctrine that the President, during his tenure of office or actual incumbency, may not
be sued in any civil or criminal case, and there is no need to provide for it in the Constitution or law.
It will degrade the dignity of the high office of the President, the Head of State, if he can be dragged
into court litigations while serving as such.

The Court also affirmed the dismissal of the amparo case against other respondents for failure of the
petition to allege ultimate facts as to make out a case against that body for the enforced
disappearance of Lourdes and the threats and harassment that followed.

DAVID ET. AL. VS. MACAPAGAL-ARROYO ET. AL.


G.R. NO. 171396, MAY 3, 2006

FACTS:
On February 24, 2006, as the Filipino nation celebrated the 20th Anniversary of the EDSA People
Power I, President Arroyo issued PP 1017, implemented by G.O. No. 5, declaring a state of national
emergency, thus:

NOW, THEREFORE, I, Gloria Macapagal-Arroyo, President of the Republic of the Philippines and
Commander-in-Chief of the Armed Forces of the Philippines, by virtue of the powers vested upon me
by Section 18, Article 7 of the Philippine Constitution which states that: “The President. . . whenever
it becomes necessary, . . . may call out (the) armed forces to prevent or suppress. . .rebellion. . .,” and
in my capacity as their Commander-in-Chief, do hereby command the Armed Forces of the
Philippines, to maintain law and order throughout the Philippines, prevent or suppress all forms of
lawless violence as well as any act of insurrection or rebellion and to enforce obedience to all the
laws and to all decrees, orders and regulations promulgated by me personally or upon my direction;
and as provided in Section 17, Article 12 of the Constitution do hereby declare a State of National
Emergency.

In their presentation of the factual bases of PP 1017 and G.O. No. 5, respondents stated that the
proximate cause behind the executive issuances was the conspiracy among some military officers,
leftist insurgents of the New People’s Army, and some members of the political opposition in a plot
to unseat or assassinate President Arroyo. They considered the aim to oust or assassinate the
President and take-over the reins of government as a clear and present danger.
CRIMINAL LAW 1
(Sunday, Section 52) 9

Petitioners David and Llamas were arrested without warrants on February 24, 2006 on their way to
EDSA. Meanwhile, the offices of the newspaper Daily Tribune, which was perceived to be anti-Arroyo,
was searched without warrant at about 1:00 A.M. on February 25, 2006. Seized from the premises –
in the absence of any official of the Daily Tribune except the security guard of the building – were
several materials for publication. The law enforcers, a composite team of PNP and AFP officers, cited
as basis of the warrantless arrests and the warrantless search and seizure was Presidential
Proclamation 1017 issued by then President Gloria Macapagal-Arroyo in the exercise of her
constitutional power to call out the Armed Forces of the Philippines to prevent or suppress lawless
violence.

ISSUE:
1. Were the warrantless arrests of petitioners David, et al., made pursuant to PP 1017, valid?
2. Was the warrantless search and seizure on the Daily Tribune’s offices conducted pursuant to PP
1017 valid?

RULING:
[The Court partially GRANTED the petitions.]

1. NO, the warrantless arrests of petitioners David, et al., made pursuant to PP 1017, were NOT
valid.

[S]earches, seizures and arrests are normally unreasonable unless authorized by a validly issued
search warrant or warrant of arrest. Section 5, Rule 113 of the Revised Rules on Criminal Procedure
provides [for the following circumstances of valid warrantless arrests]:

Sec. 5. Arrest without warrant; when lawful. - A peace officer or a private person may, without a
warrant, arrest a person:
(a) When, in his presence, the person to be arrested has committed, is actually committing, or is
attempting to commit an offense.
(b) When an offense has just been committed and he has probable cause to believe based on personal
knowledge of facts or circumstances that the person to be arrested has committed it; and
x x x.

Neither of the [provisions on in flagrante nor hot pursuit warrantless arrests] justifies petitioner
David’s warrantless arrest. During the inquest for the charges of inciting to sedition and violation of
BP 880, all that the arresting officers could invoke was their observation that some rallyists were
wearing t-shirts with the invective “Oust Gloria Now” and their erroneous assumption that petitioner
David was the leader of the rally. Consequently, the Inquest Prosecutor ordered his immediate
release on the ground of insufficiency of evidence. He noted that petitioner David was not wearing
the subject t-shirt and even if he was wearing it, such fact is insufficient to charge him with inciting
to sedition.

2. NO, the warrantless search and seizure on the Daily Tribune’s offices conducted pursuant to PP
1017 was NOT valid.

[T]he search [and seizure in the Daily Tribune premises] is illegal. Rule 126 of The Revised Rules on
Criminal Procedure lays down the steps in the conduct of search and seizure. Section 4 requires that
a search warrant be issued upon probable cause in connection with one specific offence to be
determined personally by the judge after examination under oath or affirmation of the complainant
and the witnesses he may produce. Section 8 mandates that the search of a house, room, or any other
CRIMINAL LAW 1
(Sunday, Section 52) 10

premise be made in the presence of the lawful occupant thereof or any member of his family or in the
absence of the latter, in the presence of two (2) witnesses of sufficient age and discretion residing in
the same locality. And Section 9 states that the warrant must direct that it be served in the daytime,
unless the property is on the person or in the place ordered to be searched, in which case a direction
may be inserted that it be served at any time of the day or night. All these rules were violated by the
CIDG operatives.

MINUCHER V. SCALZO
G.R. NO. 142396, 11 FEBRUARY 2003

FACTS:
Violation of the “Dangerous Drugs Act of 1972,” was filed against Minucher following a “buy-bust
operation” conducted by Philippine police narcotic agents accompanied by Scalzo in the house of
Minucher, an Iranian national, where heroin was said to have been seized. Minucher was later
acquitted by the court.

Minucher later on filed for damages due to trumped-up charges of drug trafficking made by Arthur
Scalzo.

Scalzo on his counterclaims that he had acted in the discharge of his official duties as being merely
an agent of the Drug Enforcement Administration of the United States Department of Justice.

Scalzo subsequently filed a motion to dismiss the complaint on the ground that, being a special agent
of the United States Drug Enforcement Administration, he was entitled to diplomatic immunity. He
attached to his motion Diplomatic Note of the United States Embassy addressed to DOJ of the
Philippines and a Certification of Vice Consul Donna Woodward, certifying that the note is a true and
faithful copy of its original. Trial court denied the motion to dismiss.

ISSUE:
Whether or not Arthur Scalzo is indeed entitled to diplomatic immunity.

RULING:
YES. A foreign agent, operating within a territory, can be cloaked with immunity from suit as long as
it can be established that he is acting within the directives of the sending state.

The consent or imprimatur of the Philippine government to the activities of the United States Drug
Enforcement Agency, however, can be gleaned from the undisputed facts in the case.

The official exchanges of communication between agencies of the government of the two countries
Certifications from officials of both the Philippine Department of Foreign Affairs and the United
States Embassy
Participation of members of the Philippine Narcotics Command in the “buy-bust operation”
conducted at the residence of Minucher at the behest of Scalzo
These may be inadequate to support the “diplomatic status” of the latter but they give enough
indication that the Philippine government has given its imprimatur, if not consent, to the activities
within Philippine territory of agent Scalzo of the United States Drug Enforcement Agency.

The job description of Scalzo has tasked him to conduct surveillance on suspected drug suppliers and,
after having ascertained the target, to inform local law enforcers who would then be expected to
make the arrest.
CRIMINAL LAW 1
(Sunday, Section 52) 11

In conducting surveillance activities on Minucher, later acting as the poseur-buyer during the buy-
bust operation, and then becoming a principal witness in the criminal case against Minucher,

Scalzo hardly can be said to have acted beyond the scope of his official function or duties.

LIANG V. PEOPLE
G.R. NO. 125865, 28 JANUARY 2000

FACTS:
Petitioner is an economist working with the Asian Development Bank (ADB). Sometime in 1994, for
allegedly uttering defamatory words against fellow ADB worker Joyce Cabal, he was charged before
the MeTC of Mandaluyong City with two counts of oral defamation. Petitioner was arrested by virtue
of a warrant issued by the MeTC. After fixing petitioner’s bail, the MeTC released him to the custody
of the Security Officer of ADB. The next day, the MeTC judge received an “office of protocol” from the
DFA stating that petitioner is covered by immunity from legal process under section 45 of the
Agreement between the ADB and the Philippine Government regarding the Headquarters of the ADB
in the country. Based on the said protocol communication that petitioner is immune from suit, the
MeTC judge without notice to the prosecution dismissed the criminal cases. The latter filed a motion
for reconsideration which was opposed by the DFA. When its motion was denied, the prosecution
filed a petition for certiorari and mandamus with the RTC of Pasig City which set aside the MeTC
rulings and ordered the latter court to enforce the warrant of arrest it earlier issued. After the motion
for reconsideration was denied, the petitioner elevated the case to the SC via a petition for review
arguing that he is covered by immunity under the Agreement and that no preliminary investigation
was held before the criminal case.

ISSUES:
(1) Whether or not the petitioner’s case is covered with immunity from legal process with regard to
Section 45 of the Agreement between the ADB and the Philippine Gov’t.
(2) Whether or not the conduct of preliminary investigation was imperative.

RULING:
(1) NO. The petitioner’s case is not covered by the immunity. Courts cannot blindly adhere to the
communication from the DFA that the petitioner is covered by any immunity. It has no binding effect
in courts. The court needs to protect the right to due process not only of the accused but also of the
prosecution. Secondly, the immunity under Section 45 of the Agreement is not absolute, but subject
to the exception that the acts must be done in “official capacity”. Hence, slandering a person could
not possibly be covered by the immunity agreement because our laws do not allow the commission
of a crime, such as defamation, in the name of official duty.
(2) NO. Preliminary Investigation is not a matter of right in cases cognizable by the MeTC such as
this case. Being purely a statutory right, preliminary investigation may be invoked only when
specifically granted by law. The rule on criminal procedure is clear that no preliminary investigation
is required in cases falling within the jurisdiction of the MeTC.

Hence, SC denied the petition.


CRIMINAL LAW 1
(Sunday, Section 52) 12

WHO VS. AQUINO


G.R. NO. L-35131, 29 NOVEMBER 1972

FACTS:
Dr. Leonce Verstuyft was assigned by WHO to its regional office in Manila as Acting Assistant Director
of Health Services. His personal effects, contained in twelve (12) crates, were allowed free entry from
duties and taxes. Constabulary Offshore Action Center (COSAC) suspected that the crates
“contain large quantities of highly dutiable goods” beyond the official needs of Verstuyft. Upon
application of the COSAC officers, Judge Aquino issued a search warrant for the search and seizure of
the personal effects of Verstuyft.

Secretary of Foreign Affairs Carlos P. Romulo advised Judge Aquino that Dr. Verstuyft is entitled to
immunity from search in respect for his personal baggage as accorded to members of diplomatic
missions pursuant to the Host Agreement and requested that the search warrant be suspended. The
Solicitor General accordingly joined Verstuyft for the quashal of the search warrant but
respondent judge nevertheless summarily denied the quashal. Verstuyft, thus, filed a petition for
certiorari and prohibition with the SC. WHO joined Verstuyft in asserting diplomatic immunity.

ISSUE:
Whether or not personal effect of Verstuyft can be exempted from search and seizure under the
diplomatic immunity.

RULING:
Yes. The executive branch of the Phils has expressly recognized that Verstuyft is entitled to
diplomatic immunity, pursuant to the provisions of the Host Agreement. The DFA formally advised
respondent judge of the Philippine Government's official position. The Solicitor General, as principal
law officer of the gorvernment, likewise expressly affirmed said petitioner's right to diplomatic
immunity and asked for the quashal of the search warrant.

It is a recognized principle of international law and under our system of separation of powers
that diplomatic immunity is essentially a political question and courts should refuse to look beyond
a determination by the executive branch of the government, and where the plea of diplomatic
immunity is recognized and affirmed by the executive branch of the government as in the case at bar,
it is then the duty of the courts to accept the claim of immunity upon appropriate suggestion by the
principal law officer of the government, the Solicitor General in this case, or other officer acting
under his discretion. Courts may not so exercise their jurisdiction by seizure and detention of
property, as to embarass the executive arm of the government in conducting foreign relations.

The Court, therefore, holds the respondent judge acted without jurisdiction and with grave abuse of
discretion in not ordering the quashal of the search warrant issued by him in disregard of the
diplomatic immunity of petitioner Verstuyft. (World Health Organization vs. Aquino, G.R. No. L-
35131, November 29, 1972, 48 SCRA 243)
CRIMINAL LAW 1
(Sunday, Section 52) 13

PEOPLE V JALOSJOS
GR Nos. 132875-76, February 3, 2000

Facts:
The accused-appellant, Romeo Jalosjos, is a full-fledged member of Congress who is confined at the
national penitentiary while his conviction for statutory rape and acts of lasciviousness is pending
appeal. The accused-appellant filed a motion asking that he be allowed to fully discharge the duties
of a Congressman, including attendance at legislative sessions and committee meetings despite his
having been convicted in the first instance of a non-bailable offense on the basis of popular
sovereignty and the need for his constituents to be represented.

Issue:
Whether or not accused-appellant should be allowed to discharge mandate as member of House of
Representatives

Held:
Election is the expression of the sovereign power of the people. However, inspite of its importance,
the privileges and rights arising from having been elected may be enlarged or restricted by law.
The immunity from arrest or detention of Senators and members of the House of Representatives
arises from a provision of the Constitution. The privilege has always been granted in a restrictive
sense. The provision granting an exemption as a special privilege cannot be extended beyond the
ordinary meaning of its terms. It may not be extended by intendment, implication or equitable
considerations.

The accused-appellant has not given any reason why he should be exempted from the operation of
Sec. 11, Art. VI of the Constitution. The members of Congress cannot compel absent members to
attend sessions if the reason for the absence is a legitimate one. The confinement of a Congressman
charged with a crime punishable by imprisonment of more than six years is not merely authorized
by law, it has constitutional foundations. To allow accused-appellant to attend congressional sessions
and committee meetings for 5 days or more in a week will virtually make him a free man with all the
privileges appurtenant to his position. Such an aberrant situation not only elevates accused-
appellant’s status to that of a special class, it also would be a mockery of the purposes of the
correction system.

TRILLANES VS HON. PIMENTEL


GR. No. 179817, June 27, 2008

FACTS:
On July 27, 2003, more than 300 heavily armed soldiers led by junior officers of the Armed Forces of
the Philippines (AFP) stormed into the Oakwood Premier Apartments in Makati City and publicly
demanded the resignation of the President and key national officials. After a series of negotiations,
military soldiers surrendered that evening.

In the aftermath of such event dubbed as the Oakwood Incident, petitioner Antonio F. Trillanes IV
was charged with coup d’état before the Regional Trial Court of Makati. Four years later, Trillanes
remained in detention and won a seat in the Senate. Before starting his term, Trillanes filed with RTC
an Omnibus Motion for Leave of Court to be Allowed to Attend Senate Sessions and Related Requests.
CRIMINAL LAW 1
(Sunday, Section 52) 14

Trillanes requested to be allowed to attend senate sessions and fulfill his functions as senator. The
RTC however denied his motion. Thus, he filed Petition for Certiorari with the Supreme Court to set
aside orders of the RTC.

ISSUES:
Whether or not Trillanes‘ case is different from that of the Jalosjos case
Whether or not Trillanes‘ election as senator provides legal justification to allow him to work and
serve his mandate as senator
Whether or not there are enough precedents that allows for a liberal treatment of detention prisoners
who are held without bail

HELD:
No distinction between Trillanes’ case and that of Jalosjos case
The distinctions cited by petitioner were not elemental in the pronouncement in Jalosjos that election
to Congress is not a reasonable classification in criminal law enforcement as the functions and duties
of the office are not substantial distinctions which lift one from the class of prisoners interrupted in
their freedom and restricted in liberty of movement.

The Constitution provides: All persons, except those charged with offenses punishable by reclusion
perpetua when evidence of guilt is strong, shall, before conviction, be bailable by sufficient sureties,
or be released on recognizance as may be provided by law. The Rules also state that no person
charged with a capital offense, or an offense punishable by reclusion perpetua or life imprisonment,
shall be admitted to bail when evidence of guilt is strong, regardless of the stage of the criminal action.
That the cited provisions apply equally to rape and coup d’état cases, both being punishable by
reclusion perpetua, is beyond cavil. Within the class of offenses covered by the stated range of
imposable penalties, there is clearly no distinction as to the political complexion of or moral turpitude
involved in the crime charged.

In the present case, it is uncontroverted that petitioner's application for bail and for release on
recognizance was denied. The determination that the evidence of guilt is strong, whether ascertained
in a hearing of an application for bail or imported from a trial court's judgment of conviction, justifies
the detention of an accused as a valid curtailment of his right to provisional liberty. This accentuates
the proviso that the denial of the right to bail in such cases is "regardless of the stage of the criminal
action."

Such justification for confinement with its underlying rationale of public self-defense applies equally
to detention prisoners like Trillanes or convicted prisoners-appellants like Jalosjos. The Court in
People v. Hon. Maceda said that all prisoners whether under preventive detention or serving final
sentence can not practice their profession nor engage in any business or occupation, or hold office,
elective or appointive, while in detention. This is a necessary consequence of arrest and detention.

Trillanes’ election as Senator not a legislative justification to allow him to serve his mandate

The case against Trillanes is not administrative in nature. And there is no "prior term" to speak of. In
a plethora of cases, the Court categorically held that the doctrine of condonation does not apply to
criminal cases. Election, or more precisely, re-election to office, does not obliterate a criminal charge.
Petitioner's electoral victory only signifies pertinently that when the voters elected him to the Senate,
"they did so with full awareness of the limitations on his freedom of action [and] x x x with the
knowledge that he could achieve only such legislative results which he could accomplish within the
confines of prison.
CRIMINAL LAW 1
(Sunday, Section 52) 15

It is opportune to wipe out the lingering misimpression that the call of duty conferred by the voice of
the people is louder than the litany of lawful restraints articulated in the Constitution and echoed by
jurisprudence. The apparent discord may be harmonized by the overarching tenet that the mandate
of the people yields to the Constitution which the people themselves ordained to govern all under the
rule of law. The performance of legitimate and even essential duties by public officers has never been
an excuse to free a person validly in prison. The duties imposed by the "mandate of the people" are
multifarious. The accused-appellant asserts that the duty to legislate ranks highest in the hierarchy
of government. The accused-appellant is only one of 250 members of the House of Representatives,
not to mention the 24 membersof the Senate, charged with the duties of legislation. Congress
continues to function well in the physical absence of one or a few of its members. x x x Never has the
call of a particular duty lifted a prisoner into a different classification from those others who are
validly restrained by law.

Trillanes’ case fails to compare with the species of allowable leaves

Emergency or compelling temporary leaves from imprisonment are allowed to all prisoners, at the
discretion of the authorities or upon court orders. That this discretion was gravely abused, petitioner
failed to establish. In fact, the trial court previously allowed petitioner to register as a voter in
December 2006, file his certificate of candidacy in February 2007, cast his vote on May 14, 2007, be
CRIMINAL LAW 1
(Sunday, Section 52) 16

proclaimed as senator-elect, and take his oath of office on June 29, 2007. In a seeming attempt to bind
or twist the hands of the trial court lest it be accused of taking a complete turn-around, petitioner
largely banks on these prior grants to him and insists on unending concessions and blanket
authorizations.

JIMENEZ VS CABANGBANG
GR. No. L-15905, August 3, 1966

Facts:
Cabangbang was a member of the House of Representatives and Chairman of its Committee on
National Defense. On 14 Nov 1958, Cabangbang caused the publication of an open letter addressed
to the Philippines. Said letter alleged that there have been allegedly three operational plans under
serious study by some ambitious AFP officers, with the aid of some civilian political strategists. That
such strategists have had collusions with communists and that the Secretary of Defense, Jesus Vargas,
was planning a coup d’état to place him as the president. The “planners” allegedly have Nicanor
Jimenez, among others, under their guise and that Jimenez et al may or may not be aware that they
are being used as a tool to meet such an end. The letter was said to have been published in
newspapers of general circulation. Jimenez then filed a case against Cabangbang to collect a sum of
damages against Cabangbang alleging that Cabangbang’s statement is libelous. Cabangbang
petitioned for the case to be dismissed because he said that as a member of the HOR he is immune
from suit and that he is covered by the privileged communication rule and that the said letter is not
even libelous.

ISSUE:
Whether or not the open letter is covered by privilege communication endowed to members of
Congress. Whether or not the said letter is libelous.

HELD:
Article VI, Section 15 of the Constitution provides “The Senators and Members of the House of
Representatives shall in all cases except treason, felony, and breach of the peace. Be privileged from
arrest during their attendance at the sessions of the Congress, and in going to and returning from the
same; and for any speech or debate therein, they shall not be questioned in any other place.” The
publication of the said letter is not covered by said expression which refers to utterances made by
Congressmen in the performance of their official functions, such as speeches delivered, statements
made, or votes cast in the halls of Congress, while the same is in session as well as bills introduced in
Congress, whether the same is in session or not, and other acts performed by Congressmen, either in
Congress or outside the premises housing its offices, in the official discharge of their duties as
members of Congress and of Congressional Committees duly authorized to perform its functions as
such at the time of the performance of the acts in question. Congress was not in session when the
letter was published and at the same time he, himself, caused the publication of the said letter. It is
obvious that, in thus causing the communication to be so published, he was not performing his official
duty, either as a member of Congress or as officer of any Committee thereof. Hence, contrary to the
finding made by the lower court the said communication is not absolutely privileged.
The SC is satisfied that the letter in question is not sufficient to support Jimenez’ action for damages.
Although the letter says that plaintiffs are under the control of the persons unnamed therein alluded
to as “planners”, and that, having been handpicked by Vargas, it should be noted that defendant,
likewise, added that “it is of course possible” that plaintiffs “are unwitting tools of the plan of which
they may have absolutely no knowledge”. In other words, the very document upon which plaintiffs’
action is based explicitly indicates that they might be absolutely unaware of the alleged operational
plans, and that they may be merely unwitting tools of the planners. The SC does not think that this
CRIMINAL LAW 1
(Sunday, Section 52) 17

statement is derogatory to Jimenez to the point of entitling them to recover damages, considering
that they are officers of our Armed Forces, that as such they are by law, under the control of the
Secretary of National Defense and the Chief of Staff, and that the letter in question seems to suggest
that the group therein described as “planners” include these two (2) high ranking officers.Petition is
dismissed.

BAYAN VS ZAMORA
G.R. No. 138570 October 10, 2000

FACTS
The Republic of the Philippines and the United States of America entered into an agreement called
the Visiting Forces Agreement (VFA). The agreement was treated as a treaty by the Philippine
government and was ratified by then-President Joseph Estrada with the concurrence of 2/3 of the
total membership of the Philippine Senate.

The VFA defines the treatment of U.S. troops and personnel visiting the Philippines. It provides for
the guidelines to govern such visits, and further defines the rights of the U.S. and the Philippine
governments in the matter of criminal jurisdiction, movement of vessel and aircraft, importation and
exportation of equipment, materials and supplies.

Petitioners argued, inter alia, that the VFA violates §25, Article XVIII of the 1987 Constitution, which
provides that “foreign military bases, troops, or facilities shall not be allowed in the Philippines except
under a treaty duly concurred in by the Senate . . . and recognized as a treaty by the other contracting
State.”

ISSUE
Was the VFA unconstitutional?

RULING
[The Court DISMISSED the consolidated petitions, held that the petitioners did not commit grave abuse
of discretion, and sustained the constitutionality of the VFA.]

NO, the VFA is not unconstitutional.

Section 25, Article XVIII disallows foreign military bases, troops, or facilities in the country, unless
the following conditions are sufficiently met, viz: (a) it must be under a treaty; (b) the treaty must
be duly concurred in by the Senate and, when so required by congress, ratified by a majority of the
votes cast by the people in a national referendum; and (c) recognized as a treaty by the other
contracting state.

There is no dispute as to the presence of the first two requisites in the case of the VFA. The
concurrence handed by the Senate through Resolution No. 18 is in accordance with the provisions of
the Constitution . . . the provision in [in §25, Article XVIII] requiring ratification by a majority of the
votes cast in a national referendum being unnecessary since Congress has not required it.

This Court is of the firm view that the phrase “recognized as a treaty” means that the other
contracting party accepts or acknowledges the agreement as a treaty. To require the other
contracting state, the United States of America in this case, to submit the VFA to the United States
Senate for concurrence pursuant to its Constitution, is to accord strict meaning to the phrase.
CRIMINAL LAW 1
(Sunday, Section 52) 18

Well-entrenched is the principle that the words used in the Constitution are to be given their
ordinary meaning except where technical terms are employed, in which case the significance thus
attached to them prevails. Its language should be understood in the sense they have in common use.

Moreover, it is inconsequential whether the United States treats the VFA only as an executive
agreement because, under international law, an executive agreement is as binding as a treaty. To be
sure, as long as the VFA possesses the elements of an agreement under international law, the said
agreement is to be taken equally as a treaty.

The records reveal that the United States Government, through Ambassador Thomas C. Hubbard, has
stated that the United States government has fully committed to living up to the terms of the VFA. For
as long as the United States of America accepts or acknowledges the VFA as a treaty, and binds itself
further to comply with its obligations under the treaty, there is indeed marked compliance with the
mandate of the Constitution.

PEOPLE VS TULIN
GR. No. 111709, August 30, 200
CRIMINAL LAW 1
(Sunday, Section 52) 19

PEOPLE VS WONG CHENG


GR. No. 18924, October 19, 1922

Facts:
The appellant, in representation of the Attorney General, filed an appeal that urges the revocation of
a demurrer sustained by the Court of First Instance of Manila presented by the defendant. The
defendant, accused of having illegally smoked opium aboard the merchant vessel Changsa of English
nationality while the said vessel was anchored in Manila Bay, two and a half miles from the shores of
the city. In the said demurrer, the defendant contended the lack of jurisdiction of the lower court of
the said crime, which resulted to the dismissal of the case.

Issue:
Whether or not the Philippine courts have jurisdiction over the crime committed aboard merchant
vessels anchored in our jurisdictional waters.

Held:
Yes. The crime in the case at bar was committed in our internal waters thus the Philippine courts
have a right of jurisdiction over the said offense. The Court said that having the opium smoked within
our territorial waters even though aboard a foreign merchant ship is a breach of the public order
because it causes such drugs to produce pernicious effects within our territory. Therefore, the
demurrer is revoked and the Court ordered further proceedings.

PEOPLE VS ELKANISH

PEOPLE v. LOL-LO
GR No. 17958, 1922-02-27
Facts:
Two boats of Dutch possession left matuta. In one of the boats was one individual, a Dutch subject,
and in the other boat 11 men, women, and children, subjects of Holland. The 2nd boat arrived
between the Islands of Buang and Bukid in the Dutch East Indies. There the boat was surrounded by
6 vintas manned by 24 Moros all armed. The Moros first asked for food, but once on the Dutch boat,
too for themselves all of the cargo, attacked some of the men, and brutally violated 2 of the women.
All of the persons on the Dutch boat, except the 2 young women, were again placed on it and holes
were made in it, the idea that it would submerge. The Moros finally arrived at Maruro, a Dutch
possession. Two of the Moro marauder were Lol-lo, who also raped one of the women, and Saraw. At
Maruro the 2 women were able to escape.Lol-lo and Saraw later returned to their home in South
Ubian, Tawi-Tawi, Sulu, Philippine Islands. There they were arrested and were charged in the Court
of First Instance of Sulu with the crime of piracy.

Issues:
Whether Lo-lo and saraw are guilty of piracy and if the crime of piracy under the RPC is still in force?

Ruling: Yes. The Penal code dealing with the crime of Piracy is still inforce.
Yes. The crime of piracy was accompanied by (1) an offense against chastity and (2) the abandonment
of persons without apparent means of saving themselves. It is, therefore, only necessary for us to
determine as to whether the penalty of cadena perpetua or death should be imposed. At least 3
aggravating circumstances, that the wrong done in the commission of the crime was deliberately
augmented by causing other wrongs not necessary for its commission, that advantage was taken of
CRIMINAL LAW 1
(Sunday, Section 52) 20

superior strength, and that means were employed which added ignominy to the natural effects of the
act, must also be taken into consideration in fixing the penalty.

BAYAN VS ZAMORA
G. R. No. 138570October 10, 2000

Facts:
The United States panel met with the Philippine panel to discussed, among others, the
possibleelements of the Visiting Forces Agreement (VFA). This resulted to a series of conferences
andnegotiations which culminated on January 12 and 13, 1998. Thereafter, President Fidel
Ramosapproved the VFA, which was respectively signed by Secretary Siazon and United
StatesAmbassador Thomas Hubbard.Pres. Joseph Estrada ratified the VFA on October 5, 1998 and on
May 27, 1999, the senate approvedit by (2/3) votes.

Issue:
Is the VFA governed by the provisions of Section 21, Art VII or of Section 25, Article XVIII of
theConstitution?

Ruling:
Yes.The fact that the President referred the VFA to the Senate under Section 21, Article VII, and that
the Senate extended its concurrence under the same provision, is immaterial. For in either case,
whether under Section 21, Article VII or Section 25, Article XVIII, the fundamental law is clear that
the concurrence of the Senate is mandatory to comply with the strict constitutional requirements.

US VS BULL 15 PHIL. 7

Facts
On December 2, 1908, a steamship vessel engaged in the transport of animals named Stanford
commanded by H.N. Bull docked in the port of Manila, Philippines. It was found that said vessel from
Ampieng, Formosa carried 674 heads of cattle without providing appropriate shelter and proper
suitable means for securing the animals which resulted for most of the animals to get hurt and others
to have died while in transit.

This cruelty to animals is said to be contrary to Acts No. 55 and No. 275 of the Philippine Constitution.
It is however contended that cases cannot be filed because neither was it said that the court sitting
where the animals were disembarked would take jurisdiction, nor did it say about ships not licensed
under Philippine laws, like the ships involved.

Issue:
Whether or not the court had jurisdiction over an offense committed on board a foreign ship while
inside the territorial waters of the Philippines.

Ruling:
Yes. When the vessel comes within 3 miles from the headlines which embrace the entrance of Manila
Bay, the vessel is within territorial waters and thus, the laws of the Philippines shall apply. A crime
committed on board a Norwegian merchant vessel sailing to the Philippines is within the jurisdiction
of the courts of the Philippines if the illegal conditions existed during the time the ship was within
the territorial waters - regardless of the fact that the same conditions existed when the ship settled
from the foreign port and while it was on the high seas.
CRIMINAL LAW 1
(Sunday, Section 52) 21

In light of the above restriction, the defendant was found guilty and sentenced to pay a fine of two
hundred and fifty pesos with subsidiary imprisonment in case of insolvency, and to pay the costs.

US VS SOLIMAN
36 Phil.5 1917

Facts:
Gabino Soliman, the defendant/appellant, was found guilty for false testimony(perjury) in another
criminal case, for falsely imputing to some other personsthe commission of the crime of estafa. The
trial judge on the ground thatthere was room for reasonable doubt acquitted him. However, appellant
wassentenced to 6 months imprisonment and P300 fine was imposed by the trial judge for there can
be no doubt that the accused was guilty of the crime of perjury as defined and penalized in Act No.
1697 Sec .3. The enactment of the Administrative Code (Act No. 2657) was alleged tohave expressly
repealed Act No. 1697 Sec.3 where the former becameeffective on July 1, 1916. On the other hand,
the judgment was entered onNovember 23, 1915. It was suggested that the repealed Act No. 1697
Sec.3should be held to have the effect of remitting and extinguishing the criminalresponsibility of the
accused incurred under the provisions of the repealedlaw prior to the enactment of the
Administrative Code.

Issue:
Whether or not the enactment of Administrative Code repealing Act No. 1697 Sec.3relieved Soliman
of his penalties.

Ruling:
The repealed Act No. 1697 does not have the effect of relieving an offender inwhole or in part of
penalties already incurred under the old law, unless thenew law favors the defendant by diminishing
the penalty or doing away withit altogether, and then only to the extent to which the new law is
favorable to the offender. It will not be presumed that in the absence of an express language.

REPUBLIC OF THE PHILIPPINESvs.SANDIGANBAYAN (THIRD DIVISION) and MACARIO


ASISTIO, JR.G.R.
No. 90529 August 16, 1991

Facts:
In a Joint Letter-Complaint to the Ombudsman dated January 8,1989, Messrs. Arnel Blancaflorand
Rodolfo Santos, residents of Kalookan City, Macario Asistio, Jr., who is theincumbent Mayor of
Kalookan City, with having violated the Anti-Graft and Corrupt Practices Act (R.A.3019), specifically
Section 8 thereof. In said Joint Sworn Letter-Complaint, they alleged that during his incumbency as
Kalookan CityMayor in 1981, 1982 and 1983, Asistio acquired a total of P17,264,722.90, which he
deposited inhis personal account in the Republic Planters Bank, Sangandaan Branch, KalookanCity.
In support of their allegations, they attached the original copies of the bank deposits andreceipts
which indicated the various sums deposited within the three-year period and which had
beenmachine validated from January 5.

However, in his Sworn Statementsof Assets and Liabilities for the period ending December 31,1982
and December 31, 1984, saidrespondent had a total income of only P234,128.68 and P255,324.02,
respectively; and as against itstotal assets of P2,859,327.94.
CRIMINAL LAW 1
(Sunday, Section 52) 22

Issue: Whether or not the sequestration automatically deprive a stockholder of his right of inspection.

Ruling:
No. The right of a stockholder to inspect and/or examine the records of a corporation is
explicitlyprovided in Section 74 of the Corporation Code. The records of all business transactions of
thecorporation and the minutes of any meeting shall be open to the inspection of any director,
trustee,stockholder or member of the corporation at reasonable hours on business days and he
maydemand, in writing, for a copy of excerpts from said records or minutes, at his expense.One thing
is certain, and should be stated at the outset: the PCGG cannot exercise acts ofdominion over property
sequestered, frozen or provisionally taken over. As already earlier stressedwith no little insistence,
the act of sequestration; freezing or provisional takeover of property doesnot import or bring about
a divestment of title over said property; does not make the PCGG the ownerthereof. In relation to the
property sequestered, frozen or provisionally taken over, the PCGG is a conservator, not an owner.

US v. CUNA
Facts:
Cuna, on June 30, 1907, sold opium to Apolinanaria Gumpal, Filipina, who was not a doctor or a
registered user of the drug. He was charged in violation of Sec. 5 of Act No. 1461 of the Philippine
Commission.Cuna demurred on the ground that the said act was repealed by Act no. 1761 on October
10, 1907 and because having repealed during the pendency of the case with no exception regarding
the pendency, there was no law in force thus no jurisdiction of any courts. The trial court dismissed
the case. The Government appealed and argued that both Acts penalizes the same crime and should
not be construed as having the courts deprive Cuna of trial.
Issue:
Whether Cuna, who committed the crime before the repeal, should be convicted.

Ruling:
The trial court cited the American and English common-law doctrine in their decision. The Supreme
Court (SC), although they consider it right, said that those laws were not in force in the country. The
SC said that no retroactive effect of the law shall take effect except when the punishment will be more
favorable to the accused. Being that the new law penalized the same act in the repealed law, there is
no retroactivity. The old law should still be prescribed. The SC decided that the Courts have
jurisdiction over the case. The decision of the trial court was reversed.

PEOPLE VS TAMAYO
61 Phil 225

Facts:
Crisanto Tamayo was convicted in the justice of the peace court of Magsingal, Province of Ilocos Sur,
of a violation of section 2, municipal ordinance no. 5 series of 1932, of said municipality. Upon appeal
to the Court of First Instance of Ilocos Sur conviction resulted and a fine was imposed. While this
appeal was pending the municipal council repealed section 2 in question, which repeal was duly
approved by the provincial board, and the act complained of, instead of being a violation of the
municipal ordinances, is now legal in that municipality. Crisanto moved for a dismissal of the action
against him on account of that repeal.

Issue:
Is Tamayo still guilty of the crime wherein the law governing it has been repealed?
CRIMINAL LAW 1
(Sunday, Section 52) 23

Ruling:
The repeal here was absolute, and not a reenactment and repeal by implication. Nor was there any
saving clause. The legislative intent as shown by the action of the municipal council is that such
conduct, formerly denounced, is no longer deemed criminal, and it would be illogical for this court to
attempt to sentence appellant for an offense that no longer exists.

PEOPLE VS LACSON
April 1, 2003

Facts:
Before the court is the petitioner’s motion of reconsideration of the resolution dated May 23, 2002,
for the determination of several factual issues relative to the application of Sec. 8 Rule 117 of RRCP
on the dismissal of the cases Q-99- 81679 and Q-99-81689 against the respondent. The respondent
was charged with the shooting and killing of eleven male persons. The court confirmed the express
consent of the respondent in the provisional dismissal of the aforementioned cases when he filed for
judicial determination. The court also ruled the need to determine whether the other facts for its
application are attendant.

Issues:
1. Whether or not the requisites for the applicability of Sec. 8, Rule 117 of 2000 Rules on Criminal
Procedure were complied with in the Kuratong Baleleng cases
a. Was express consent given by the respondent?
b. Was notice for the motion, the hearing and the subsequent dismissal given to the heirs of the
victims?

Section 8, Rule 117 is not applicable to the case since the conditions for its applicability, namely:
1) prosecution with the express consent of the accused or both of them move for provisional
dismissal,
2) offended party notified,
3) court grants motion and dismisses cases provisionally,
4) public prosecutor served with copy of orders of provisional dismissal, which is the defendants
burden to prove, which in this case has not been done
a. The defendant never filed and denied unequivocally in his statements, through counsel at the Court
of Appeals, that he filed for dismissal nor did he agree to a provisional dismissal thereof.
b. No notice of motion for provisional dismissal, hearing and subsequent dismissal was given to the
heirs of the victims.

2. WON time-bar in Sec 8 Rule 117 should be applied prospectively or retroactively.

Ruling:
Time-bar should not be applied retroactively. Though procedural rules may be applied retroactively,
it should not be if to do so would work injustice or would involve intricate problems of due process.
Statutes should be construed in light of the purposes to be achieved and the evils to be remedied.
This is because to do so would be prejudicial to the State since, given that the Judge dismissed the
case on March 29,1999, and the New rule took effect on Dec 1,2000, it would only in effect give them
1 year and three months to work instead of 2 years. At that time, they had no knowledge of the said
rule and therefore they should not be penalized for that. “Indeed for justice to prevail, the scales must
balance; justice is not to be dispensed for the accused alone.” The two-year period fixed in the new
rule is for the benefit of both the State and the accused. It should not be emasculated and reduced by
an inordinate retroactive application of the time-bar therein provided merely to benefit the accused.
CRIMINAL LAW 1
(Sunday, Section 52) 24

To do so would cause an injustice of hardship to the state and adversely affect the administration of
justice.

GUMABON VS DIRECTOR OF PRISONS


37 SCRA 420 (1971)

Facts:
Gumabon, after pleading guilty, was sentenced on May 5, 1953 to reclusion perpetua for the complex
crime of rebellion with multiple murder, robbery, arson and kidnapping (along with
Agapito, Palmares and Padua). The decision for the first two petitioners was rendered on March 8,
1954 and the third on Dec. 5, 1955. The last petitioner Bagolbagol was penalized with reclusion
perpetua on Jan. 12, 1954. Each of the petitioners have been imprisoned for more than 13 years by
virtue of their convictions.
They now invoke the doctrine laid down in People v. Hernandez which negated such complex crime,
a ruling which was not handed down until after their convictions have become final. In
People v. Hernandez, the SC ruled that the information against the accused for rebellion complexed
with murder, arson and robbery was not warranted under Art. 134 of the RPC, there
being no such complex offense. This ruling was not handed down until after their convictions have
become final. Since Hernandez served more than the maximum penalty that could
have been served against him, he is entitled to freedom, and thus, his continued detention is illegal.
Issue: Whether or not Art. 22 of the RPC which gives a penal judgment a retroactive effect is
applicable in this case.

Issue:
WON judicial decisions favourable to the accused/convicted for the same
crime can be applied retroactively)

Ruling:
Yes. Judicial decisions favourable to the accused must be applied retroactively. Petitioners relied on
Art. 22 of the RPC, which states the penal laws shall have a retroactive effect insofar as they favour
the accused who is not a habitual criminal. The Civil Code also provides that judicial decisions
applying or interpreting the Constitution forms part of our legal
system. Petitioners even raised their constitutional right to equal protection, given that Hernandez
et al., has been convicted for the same offense as they have, though their sentences were lighter.
Habeas corpus is the only means of benefiting the accused by the retroactive character of a favorable
decision.

PEOPLE V. PIMENTEL
GR. No. 100210, April 1, 1998

Facts:
As early as 1983, Tujan was charged with Subversion under RA 1700 ( Anti-Subversion Law) as
amended before the RTC Manila. A warrant for his arrest was issued on July 1983 but was unserved
as he could not be found.
Seven years after, Tujan was arrested on the basis of warrant of arrest in the subversion case. When
arrested, an unlicensed revolver and six rounds of live ammunition was found in his possession. On
June 1990, Tujan was charged with Illegal Possession of Firearms and Ammunition in furtherance of
Subversion under PD No. 1866 before RTC Makati. Tujan filed a motion to quash the information
CRIMINAL LAW 1
(Sunday, Section 52) 25

invoking protection versus double jeopardy since he claims that alleged possession of firearms was
absorbed in subversion. It was granted by RTC and CA.

Issue:
Whether or not RA 7363 (An Act Repealing RA 1700) should be applied retroactively to Tujan.

Held:
Yes, RA 7363 should be applied retroactively. The repeal by said law of RA 1700, as amended was
absolute. There was no saving clause in the repeal.
Where, as here, the repeal of a penal law is total and absolute and the act which was penalized by a
prior law ceases to be criminal under the new law, the previous offense is obliterated. It is a
recognized rule in this jurisdiction that a total repeal deprives the courts of jurisdiction to try, convict
and sentence persons charged with violation of the old law prior to the repeal.
With the enactment of R.A. No. 7636, the charge of subversion against the accused-private
respondent has no more legal basis and should be dismissed.

BERNARDO V. BALAGOT
G.R. No. 86561, November 10, 1992

Facts:
Pablo Bernardo, the herein petitioner, was convicted of estafa in the Municipal Trial Court of San
Antonio, Nueva Ecija, and sentenced on September 5, 1984 to 1 year, 8 months and 21 days of prision
correccional as minimum to 2 years, 11 months and 10 days of prision correccional medium as
maximum, with the accessory and other penalties.

He appealed to the Regional Trial Court of Nueva Ecija, which affirmed the decision with
modifications. On April 25, 1985, he filed a petition for review with the Court of Appeals, which on
December 24, 1985, sustained the appealed decision with modifications. On January 17, 1986, the
petitioner filed a motion for new trial and/or reconsideration, but this was not granted. Bernardo
then filed a petition for review with this Court on November 16, 1986, which we also denied.
While his motion for new trial and/or reconsideration was pending in the Court of Appeals, Bernardo
filed an application for probation. The application was dated February 3, 1986. It was referred to the
Probation Officer of Nueva Ecija, who subsequently recommended its approval. On October 11, 1987,
however, Municipal Judge Francisco R. Andres denied it. 1 The denial was based on Section 4 of P.D.
968 as amended by P.D. 1990, prohibiting the grant of probation to an applicant who has appealed
his conviction, and also on Bernardo's unsatisfactory conduct.

Issue:
Whether or not Pablo Bernardo is entitled to probation although he had previously appealed his
conviction

Ruling:
No. He is no longer entitled for Probation. Section 4 of P.D. 968 as amended by P.D. 1990 provides
that: Sec. 4. Grant of Probation. — Subject to the provisions of this Decree, the trial court may, after it
shall have convicted and sentenced a defendant, and upon his application by said defendant within
the period for perfecting an appeal, suspend the execution of the sentence and place the defendant
on probation for such period and upon such terms and conditions as it may deem best; Provided, that
no application for probation shall be entertained or granted if the defendant has perfected the appeal
from the judgment of conviction.
CRIMINAL LAW 1
(Sunday, Section 52) 26

P.D. 1990 was issued when it was observed that even if a person's conviction was finally affirmed
after he had exhausted the appeal process (usually up to this Court), he nevertheless could still apply
for probation and thus in effect undo such affirmance. To prevent loss of time, money, and effort on
the part of the State in this wasteful exercise, the law was amended to make appeal and probation
mutually exclusive remedies.

The present case falls squarely within the objectives of P.D. 1990. Bernardo appealed his case all the
way to this Court, which sustained the courts below and denied his petition. He filed his application
for probation after his conviction was affirmed by both the Regional Trial Court and the Court of
Appeals. It was only while his motion for new trial and/or reconsideration was pending with the
latter court that it occurred to him to apply for probation. It was already too late then because P.D.
1990 was already in effect.

PEOPLE V. VENUS
63 Phil 435 (1936)

Facts:
On March 16, 1936, the prosecuting attorney of the City of Manila filed with the Court of First Instance
of that city an information charging the defendant, Bienvenido Venus, with the crime of robbery in
an inhabited house. The information alleges that on or about the 9th day of March, 1936, the
defendant entered the house then occupied by Zoila de Talaban at 1328 M. Natividad Street, Manila,
Philippines, by breaking the hasp of the door of said house which was secured by a padlock and once
inside took and carried away, with the intent of gain and without the consent of the owner thereof,
various personal properties belonging to Zoila de Talaban of the total value of one hundred and
eighty-eight and fifty centavos. The information further alleges "that the said accused is a habitual
delinquent, having previously been convicted by final judgment rendered by a competent court, once
for the crime of attempted robbery in an inhabited house and once for theft, the date of his last
conviction being November 14, 1934."

Issue:
Whether or not Bienvenido Venus is a Habitual Delinquent.

Ruling:
No. It is provided in article 62 of the Revised Penal Code that a person shall be deemed to be a habitual
delinquent, if within a period of ten years from the date of his release or last conviction of the crimes
of robo, hurto, estafa, or falsification, he is found guilty of said crimes a third time or oftener. In the
case before us there is no allegation as to the date of the last conviction or release of the defendant, or
as to the date of his other convictions or when the crimes for which he was convicted were committed.
It is important whenever the evidence warrants it that prosecuting attorneys should take pains to
allege and prove the facts necessary to constitute habitual delinquency.
CRIMINAL LAW 1
(Sunday, Section 52) 27

WHITE LIGHT CORP. V. CITY OF MANILA


GR. No. 122846, January 20, 2009

Facts:
On December 3, 1992, City Mayor Alfredo S. Lim signed into a law Manila City Ordinance No. 7774
entitled “An Ordinance Prohibiting Short-Time Admission, Short-Time Admission Rates, and Wash-
Up Rate Schemes in Hotels, Motels, Inns, Lodging Houses, Pension Houses, and Similar
Establishments in the City of Manila.” On December 15, 1992, the Malate Tourist and Development
Corporation (MTDC) filed a complaint for declaratory relief with prayer for a writ of preliminary
injunction and/or temporary restraining order (TRO) impleading as defendant, herein respondent
City of Manila represented by Mayor Lim with the prayer that the Ordinance be declared invalid and
unconstitutional.

On December 21, 1992, petitioners White Light Corporation (WLC), Titanium Corporation (TC) and
Sta. Mesa Tourist and Development Corporation (STDC) filed a motion to intervene and to admit
attached complaint-in-intervention on the ground that the Ordinance directly affects their business
interests as operators of drive-in-hotels and motels in Manila. The RTC issued a TRO directing the
City to cease and desist from enforcing the Ordinance. The City alleges that the Ordinance is a
legitimate exercise of police power. On October 20, 1993, the RTC rendered a decision declaring the
Ordinance null and void. On a petition for review on certiorari, the Court of Appeals reversed the
decision of the RTC and affirmed the constitutionality of the Ordinance.

Issue:
Whether Manila City Ordinance No. 7774 is a valid exercise of police power

Ruling:
Police power, while incapable of an exact definition, has been purposely veiled in general terms to
underscore its comprehensiveness to meet all exigencies and provide enough room for an efficient
and flexible response as the conditions warrant. Police power is based upon the concept of necessity
of the State and its corresponding right to protect itself and its people. Police power has been used as
justification for numerous and varied actions by the State. The apparent goal of the Ordinance is to
minimize if not eliminate the use of the covered establishments for illicit sex, prostitution, drug use
and alike. These goals, by themselves, are unimpeachable and certainly fall within the ambit of the
police power of the State. Yet the desirability of these ends do not sanctify any and all means for their
achievement. Those means must align with the Constitution, and our emerging sophisticated analysis
of its guarantees to the people.
That the Ordinance prevents the lawful uses of a wash rate depriving patrons of a product and the
petitioners of lucrative business ties in with another constitutional requisite for the legitimacy of the
Ordinance as a police power measure. It must appear that the interests of the public generally, as
distinguished from those of a particular class, require an interference with private rights and the
means must be reasonably necessary for the accomplishment of the purpose and not unduly
oppressive of private rights. It must also be evident that no other alternative for the accomplishment
of the purpose less intrusive of private rights can work. More importantly, a reasonable relation must
exist between the purposes of the measure and the means employed for its accomplishment, for even
under the guise of protecting the public interest, personal rights and those pertaining to private
property will not be permitted to be arbitrarily invaded. Lacking a concurrence of these requisites,
the police measure shall be struck down as an arbitrary intrusion into private rights. As held in Morfe
v. Mutuc, the exercise of police power is subject to judicial review when life, liberty or property is
affected. However, this is not in any way meant to take it away from the vastness of State police power
CRIMINAL LAW 1
(Sunday, Section 52) 28

whose exercise enjoys the presumption of validity. Ordinance No. 7774 is hereby declared
UNCONSTITUTIONAL.

GARCIA V. DRILON
G.R. No. 179267, June 25, 2013

Facts: Rosalie Jaype-Garcia filed, for herself and in behalf of her minor children for a Temporary
Protection Order against her husband, Jesus C. Garcia pursuant to R.A. 9262. She claimed to be a
victim of physical abuse; emotional, psychological, and economic violence as a result of marital
infidelity on the part of petitioner, with threats of deprivation of custody of her children and of
financial support. The husband now, assails the constitutionality of RA 9262 as being violative of the
equal protection clause.

Issue:
Whether there is a violation of equal protection clause.

Held:
R.A. 9262 does not violate the guaranty of equal protection of the laws.
Equal protection simply requires that all persons or things similarly situated should be treated alike,
both as to rights conferred and responsibilities imposed. R.A. 9262 is based on a valid classification
as shall hereinafter be discussed and, as such, did not violate the equal protection clause by favoring
women over men as victims of violence and abuse to whom the State extends its protection.

There is likewise no merit to the contention that R.A. 9262 singles out the husband or father as the
culprit. As defined above, VAWC may likewise be committed “against a woman with whom the person
has or had a sexual or dating relationship.” Clearly, the use of the gender-neutral word “person” who
has or had a sexual or dating relationship with the woman encompasses even lesbian relationships.

R.A. 9262 is based on a valid classification as such, did not violate the equal protection clause by
favoring women over men as victims of violence and abuse to whom the State extends its protection.
The unequal power relationship between women and men; the fact that women are more likely than
men to be victims of violence; and the widespread gender bias and prejudice against women all make
for real differences justifying the classification under the law. As Justice McIntyre succinctly states,
“the accommodation of differences … is the essence of true equality.”

TAÑADA V. TUVERA
136 SCRA 27 (1985)

Facts:
The petitioners sought a writ of mandamus from the Court in order to compel the respondent public
officials to publish in the Official Gazette various presidential decrees, letters of instructions, general
orders, proclamations, executive implementations, and administrative orders. They did so because
of the right of the people to be informed on matters of public concern, a right recognized in Section
6, Article IV of the 1973 Constitution. In addition, petitioners stress that Article 2 of the Civil Code
requires the publication of laws as a requirement for their effectivity.

Issue:
Can laws of general application take effect even without being published as long as it provides the
date of effectivity?
CRIMINAL LAW 1
(Sunday, Section 52) 29

Ruling: No. “Article 2 does not preclude the requirement of publication in the Official Gazette, even if
the law itself provides for the date of its effectivity.” This is because if laws are allowed to take effect
without publication, the public would not be informed of the existence of the law that essentially
governs them. Without such publication, Article 3 of the Civil Code, which provides that “ignorance
of the law excuses no one from compliance therewith” would have no basis. Thus, the Court ruled
that all unpublished laws which are of general application have no binding force and effect.

GUINGGUING V. PEOPLE
G.R. NO. 128959, 30 SEPTEMBER 2005

FACTS:
CA affirmed with modification the decision rendered by the RTC finding Guingguing and Lim guilty
ofthe crime of libel. This is a petition filed by Guingguing alone. This case originated from
the case filed byTorralba. . Complainant was a broadcast journalist who handled two
programs for radio stations DYLA andDYFX . Lim c aus ed the publication of records of
c rimi nal cas es fi led agai ns t com plai nant as well as photographs of the latter being
arrested. These were published by means of a one-page advertisement paid forby Lim in the
Sunday Post, a weekly publication edited and published by petitioner.The lower court concluded
that the publication complained of was indeed libelous. The CA modifiedthe penalty imposed but
it affirmed the RTCs finding of guilt.

ISSUE:
Whether or not the publication subject matter of the instant case is indeed libelous.

RULING:
Criminal libel is defined as a public and malicious imputation of a crime, or of a vice or
defect,real or imaginary, or any act, omission, condition, status, or circumstance tending
to cause the dishonor,discredit, or contempt of a natural or juridical person, or to blacken the
memory of one who is dead.Two major propositions in the prosecution of defamatory remarks
were established: first, that libelagainst a public person is a greater offense than one directed
against an ordinary man, and second, that it isimmaterial that the libel be true. This Court has
accepted the proposition that the actual malice standard governsthe prosecution of criminal libel
cases concerning public figures. As it has been established that complainant was a public figure, it
was incumbent upon the prosecutionto prove actual malice on the part of Lim and petitioner
when the latter published the article subject matter ofthe complaint. It should thus proceed that
if the statements made against the public figure are essentially true,then no conviction for libel
can be had.From the foregoing, it is clear that there was nothing untruthful about what was
published in theSunday Post. The criminal cases listed in the advertisement as pending against
the complainant had indeed beenfiled.To this end, the publication of the subject advertisement
by petitioner and Lim cannot be deemed bythis Court to have been done with actual malice.
Aside from the fact that the information contained in saidpublication was true, the intention to
let the public know the character of their radio commentator can at best besubsumed under the
mantle of having been done with good motives and for justifiable ends.Wherefore, petitionis
GRANTED. Decision of the RTC and CA is REVERSED and SET ASIDE.Petitioner is ACQUITTED.
CRIMINAL LAW 1
(Sunday, Section 52) 30

ESTRADA V. ESCRITORAM
NO. P-02-1651, 22 JUNE 2006, 492 SCRA 1
FACTS:
Escritor is a court interpreter since 1999 in the RTC of Las Pinas City. She has been living
with Quilapio, a man who is not her husband, for more than twenty five years and had a son with him
as well. Respondent’s husband died a year before she entered into the judiciary while Quilapio is still
legally married to another woman.
Complainant Estrada requested the Judge of said RTC to investigate respondent. According
to complainant, respondent should not be allowed to remain employed therein for it will appear as if
the court allows such act.
Respondent claims that their conjugal arrangement is permitted by her religion—the Jehovah’s
Witnesses and the Watch Tower and the Bible Trace Society. They allegedly have a ‘Declaration of
Pledging Faithfulness’ under the approval of their congregation. Such a declaration is effective when
legal impediments render it impossible for a couple to legalize their union.

ISSUE:
Whether or Not the State could penalize respondent for such conjugal arrangement.

RULING:
No. The State could not penalize respondent for she is exercising her right to freedom of religion. The
free exercise of religion is specifically articulated as one of the fundamental rights in
our Constitution. As Jefferson put it, it is the most inalienable and sacred of human rights. The State’s
interest in enforcing its prohibition cannot be merely abstract or symbolic in order to be sufficiently
compelling to outweigh a free exercise claim. In the case at bar, the State has not evinced any concrete
interest in enforcing the concubinage or bigamy charges against respondent or her partner. Thus the
State’s interest only amounts to the symbolic preservation of an unenforced
prohibition. Furthermore, a distinction between public and secular morality and religious morality
should be kept in mind. The jurisdiction of the Court extends only to public and secular morality.
The Court further states that our Constitution adheres the benevolent neutrality approach that gives
room for accommodation of religious exercises as required by the Free Exercise Clause.
This benevolent neutrality could allow for accommodation of morality based on religion, provided it
does not offend compelling state interests. Assuming arguendo that the OSG has proved a compelling
state interest, it has to further demonstrate that the state has used the least intrusive means possible
so that the free exercise is not infringed any more than necessary to achieve the legitimate goal of the
state. Thus the conjugal arrangement cannot be penalized for it constitutes an exemption to the law
based on her right to freedom of religion.

PEOPLE V. ECHAGARAY
G.R. NO. 117472, 7 FEB. 1997, 267 SCRA 682

FACTS:
The SC rendered a decision in the instant case affirming the conviction of the accused-appellant for
the crime of raping his ten-year old daughter. The crime having been committed sometime in April,
1994, during which time Republic Act (R.A.) No. 7659, commonly known as the Death Penalty Law,
was already in effect, accused-appellant was inevitably meted out the supreme penalty of death. The
accused-appellant timely filed a Motion for Reconsideration which focused on the sinister motive of
the victim's grandmother that precipitated the filing of the alleged false accusation of rape against
the accused. The motion was dismissed as the SC found no substantial arguments on the said motion
that can disturb the verdict.
CRIMINAL LAW 1
(Sunday, Section 52) 31

On August 6, 1996, accused-appellant discharged the defense counsel, Atty. Julian R. Vitug, and
retained the services of the Anti-Death Penalty Task Force of the Free Legal Assistance Group of the
Philippines. (FLAG)

A supplemental Motion for Reconsideration prepared by the FLAG on behalf of accused-appellant


aiming for the reversal of the death sentence.
In sum, the Supplemental Motion for Reconsideration raises three (3) main issues: (1) mixed factual
and legal matters relating to the trial proceedings and findings; (2) alleged incompetence of accused-
appellant's former counsel; and (3) purely legal question of the constitutionality of R.A. No. 7659.

ISSUE: WON the death penalty law (RA no. 7659) is unconstitutional

RULING: No. Wherefore, the motion for reconsideration & supplemental motion for reconsideration
are denied for lack of merit.

Accused-appellant first claims that the death penalty is per se a cruel, degrading or inhuman
punishment as ruled by the United States (U.S.) Supreme Court in Furman v. Georgia. To state,
however, that the U.S. Supreme Court, in Furman, categorically ruled that the death penalty is a cruel,
degrading or inhuman punishment, is misleading and inaccurate.

The issue in Furman was not so much death penalty itself but the arbitrariness pervading the
procedures by which the death penalty was imposed on the accused by the sentencing jury. Thus,
the defense theory in Furman centered not so much on the nature of the death penalty as a criminal
sanction but on the discrimination against the black accused who is meted out the death penalty by
a white jury that is given the unconditional discretion to determine whether or not to impose the
death penalty.

Furman, thus, did not outlaw the death penalty because it was cruel and unusual per se. While the
U.S. Supreme Court nullified all discretionary death penalty statutes in Furman, it did so because the
discretion which these statutes vested in the trial judges and sentencing juries was uncontrolled and
without any parameters, guidelines, or standards intended to lessen, if not altogether eliminate, the
intervention of personal biases, prejudices and discriminatory acts on the part of the trial judges and
sentencing juries. accused-appellant asseverates that the death penalty is a cruel, inhuman or
degrading punishment for the crime of rape mainly because the latter, unlike murder, does not
involve the taking of life.

In support of his contention, accused-appellant largely relies on the ruling of the U.S. Supreme Court
in Coker v. Georgia:: "Rape is without doubt deserving of serious punishment; but in terms of moral
depravity and of the injury to the person and to the public, it does not compare with murder, which
does involve the unjustified taking of human life. Although it may be accompanied by another crime,
rape by definition does not include the death of or even the serious injury to another person. The
murderer kills; the rapist, if no more than that, does not. Life is over for the victim of the murderer;
for the rape victim, life may not be nearly so happy as it was, but it is not over and normally is not
beyond repair. We have the abiding conviction that the death penalty, which 'is unique in its severity
and irrevocability' x x x is an excessive penalty for the rapist who, as such, does not take human life"

The U.S. Supreme Court based its foregoing ruling on two grounds:
first, that the public has manifested its rejection of the death penalty as a proper punishment for the
crime of rape through the willful omission by the state legislatures to include rape in their new death
penalty statutes in the aftermath of Furman;
CRIMINAL LAW 1
(Sunday, Section 52) 32

Phil. SC: Anent the first ground, we fail to see how this could have any bearing on the Philippine
experience and in the context of our own culture.

second, that rape, while concededly a dastardly contemptuous violation of a woman's spiritual
integrity, physical privacy, and psychological balance, does not involve the taking of life.

Phil. SC: we disagree with the court's predicate that the gauge of whether or not a crime warrants the
death penalty or not, is the attendance of the circumstance of death on the part of the victim. Such a
premise is in fact an ennobling of the biblical notion of retributive justice of "an eye for an eye, a tooth
for a tooth".

The Revised Penal Code, as it was originally promulgated, provided for the death penalty in specified
crimes under specific circumstances. As early as 1886, though, capital punishment had entered our
legal system through the old Penal Code, which was a modified version of the Spanish Penal Code of
1870.
Under the Revised Penal Code, death is the penalty for the crimes of treason, correspondence with
the enemy during times of war, qualified piracy, parricide, murder, infanticide, kidnapping, rape with
homicide or with the use of deadly weapon or by two or more persons resulting in insanity, robbery
with homicide, and arson resulting in death.

The opposition to the death penalty uniformly took the form of a constitutional question of whether
or not the death penalty is a cruel, unjust, excessive or unusual punishment in violation of the
constitutional proscription against cruel and unusual punishment
Harden v. Director of Prison- "The penalty complained of is neither cruel, unjust nor excessive. In
Ex-parte Kemmler, 136 U.S., 436, the United States Supreme Court said that 'punishments are cruel
when they involve torture or a lingering death, but the punishment of death is not cruel, within the
meaning of that word as used in the constitution. It implies there something inhuman and barbarous,
something more than the mere extinguishment of life.

People v. Limaco- "x x x there are quite a number of people who honestly believe that the supreme
penalty is either morally wrong or unwise or ineffective. However, as long as that penalty remains
in the statute books, and as long as our criminal law provides for its imposition in certain cases, it is
the duty of judicial officers to respect and apply the law regardless of their private opinions,"

Article III, Section 19 (1) of the 1987 Constitution simply states that congress, for compelling reasons
involving heinous crimes, may re-impose the death penalty. Nothing in the said provision imposes a
requirement that for a death penalty bill to be valid, a positive manifestation in the form of a higher
incidence of crime should first be perceived and statistically proven following the suspension of the
death penalty. Neither does the said provision require that the death penalty be resorted to as a last
recourse when all other criminal reforms have failed to abate criminality in society what R.A. No.
7659 states is that "the Congress, in the interest of justice, public order and rule of law, and the need
to rationalize and harmonize the penal sanctions for heinous crimes, finds compelling reasons to
impose the death penalty for said crimes.

Heinous crime is an act or series of acts which, by the flagrantly violent manner in which the same
was committed or by the reason of its inherent viciousness, shows a patent disregard and mockery
of the law, public peace and order, or public morals. It is an offense whose essential and inherent
viciousness and atrocity are repugnant and outrageous to a civilized society and hence, shock the
moral self of a people.
CRIMINAL LAW 1
(Sunday, Section 52) 33

The right of a person is not only to live but to live a quality life, and this means that the rest of society
is obligated to respect his or her individual personality, the integrity and the sanctity of his or her
own physical body, and the value he or she puts in his or her own spiritual, psychological, material
and social preferences and needs.

Seen in this light, the capital crimes of kidnapping and serious illegal detention for ransom resulting
in the death of the victim or the victim is raped, tortured, or subjected to dehumanizing acts;
destructive arson resulting in death, and drug offenses involving minors or resulting in the death of
the victim in the case of other crimes; as well as murder, rape, parricide, infanticide, kidnapping and
serious illegal detention where the victim is detained for more than three days or serious physical
injuries were inflicted on the victim or threats to kill him were made or the victim is a minor, robbery
with homicide, rape or intentional mutilation, destructive arson, and carnapping where the owner,
driver or occupant of the carnapped vehicle is killed or raped, which are penalized by reclusion
perpetua to death, are clearly heinous by their very nature.

SC: the death penalty is imposed in heinous crimes because:


the perpetrators thereof have committed unforgivably execrable acts that have so deeply
dehumanized a person or criminal acts with severely destructive effects on the national efforts to lift
the masses from abject poverty through organized governmental strategies based on a disciplined
and honest citizenry
they have so caused irreparable and substantial injury to both their victim and the society and a
repetition of their acts would pose actual threat to the safety of individuals and the survival of
government, they must be permanently prevented from doing so

People v. Cristobal: "Rape is the forcible violation of the sexual intimacy of another person. It does
injury to justice and charity. Rape deeply wounds the respect, freedom, and physical and moral
integrity to which every person has a right. It causes grave damage that can mark the victim for life.
It is always an intrinsically evil act xxx an outrage upon decency and dignity that hurts not only the
victim but the society itself.

CORPUZ V. PEOPLE
G.R. NO. 180016, 29 APRIL 2014

FACTS:
Accused Corpuz received from complainant Tangcoy pieces of jewelry with an obligation to sell the
same and remit the proceeds of the sale or to return the same if not sold, after the expiration of 30
days.
The period expired without Corpuz remitting anything to Tangcoy.
When Corpuz and Tangcoy met, Corpuz promised that he will pay, but to no avail.
Tangcoy filed a case for estafa with abuse of confidence against Corpuz.
Corpuz argued as follows:
a. The proof submitted by Tangcoy (receipt) is inadmissible for being a mere photocopy.
b. The information was defective because the date when the jewelry should be returned and the date
when crime occurred is different from the one testified to by Tangcoy.
c. Fourth element of estafa or demand is not proved.
d. Sole testimony of Tangcoy is not sufficient for conviction
CRIMINAL LAW 1
(Sunday, Section 52) 34

ISSUES:
Can the court admit as evidence a photocopy of document without violating the best evidence rule
(only original documents, as a general rule, is admissible as evidence)?
Is the date of occurrence of time material in estafa cases with abuse of confidence?
3 What is the form of demand required in estafa with abuse of confidence?
4 May a sole witness be considered credible?

RULING
Yes. The established doctrine is that when a party failed to interpose a timely objection to evidence
at the time they were offered in evidence, such objection shall be considered as waived.
Here, Corpuz never objected to the admissibility of the said evidence at the time it was identified,
marked and testified upon in court by Tangcoy. Corpuz also failed to raise an objection in his
Comment to the prosecution’s formal offer of evidence and even admitted having signed the said
receipt.

No. It is true that the gravamen of the crime of estafa with abuse of confidence under Article 315,
paragraph 1, subparagraph (b) of the RPC is the appropriation or conversion of money or property
received to the prejudice of the owner and that the time of occurrence is not a material ingredient of
the crime. Hence, the exclusion of the period and the wrong date of the occurrence of the crime, as
reflected in the Information, do not make the latter fatally defective.
Further, the following satisfies the sufficiency of information:
1. The designation of the offense by the statute;
2. The acts or omissions complained of as constituting the offense;
3. The name of the offended party; and
4. The approximate time of the commission of the offense, and the place wherein the offense was
committed.
The 4th element is satisfied. Even though the information indicates that the time of offense was
committed “on or about the 5th of July 1991,” such is not fatal to the prosecution’s cause considering
that Section 11 of the same Rule requires a statement of the precise time only when the same is a
material ingredient of the offense.

3 Note first that the elements of estafa with abuse of confidence are as follows:
(a) that money, goods or other personal property is received by the offender in trust, or on
commission, or for administration, or under any other obligation involving the duty to make delivery
of, or to return the same;
(b) that there be misappropriation or conversion of such money or property by the offender or denial
on his part of such receipt;
(c) that such misappropriation or conversion or denial is to the prejudice of another; and
(d) that there is a demand made by the offended party on the offender.
No specific type of proof is required to show that there was demand. Demand need not even be
formal; it may be verbal. The specific word “demand” need not even be used to show that it has indeed
been made upon the person charged, since even a mere query as to the whereabouts of the money
[in this case, property], would be tantamount to a demand.
In Tubb v. People, where the complainant merely verbally inquired about the money entrusted to the
accused, the query was tantamount to a demand.

4 Yes. Note first that settled is the rule that in assessing the credibility of witnesses, SC gives great
respect to the evaluation of the trial court for it had the unique opportunity to observe the demeanor
of witnesses and their deportment on the witness stand, an opportunity denied the appellate courts,
which merely rely on the records of the case.
CRIMINAL LAW 1
(Sunday, Section 52) 35

The assessment by the trial court is even conclusive and binding if not tainted with arbitrariness or
oversight of some fact or circumstance of weight and influence, especially when such finding is
affirmed by the CA. Truth is established not by the number of witnesses, but by the quality of their
testimonies, for in determining the value and credibility of evidence, the witnesses are to be weighed
not numbered.

PEOPLE V. FERRER
L-32613-14, 27 DECEMBER 1972, 48 SCRA 382

FACTS:
Hon. Judge Simeon Ferrer is the Tarlac trial court judge that declared RA1700 or the Anti-Subversive
Act of 1957 as a bill of attainder. Thus, dismissing the information of subversion against the
following: 1.) Feliciano Co for being an officer/leader of the CommunistParty of the Philippines (CPP)
aggravated by circumstances of contempt and insult to public officers, subversion by a band and aid
of armed men to afford impunity. 2.) Nilo Tayag and 5 others, for being members/leaders of the NPA,
inciting, instigating people to unite and overthrow the Philippine Government. Attended by
Aggravating Circumstances of Aid or Armed Men, Craft, and Fraud. The trial court is of opinion that
1.) The Congress usurped the powers of the judge 2.) Assumed judicial magistracy by pronouncing
the guilt of the CPP without any forms of safeguard of a judicial trial. 3.) It created a presumption of
organizational guilt by being members of the CPP regardless of voluntariness.

The Anti Subversive Act of 1957 was approved 20June1957. It is an act to outlaw the CPP and similar
associations penalizing membershiptherein, and for other purposes. It defined the Communist Party
being although a political party is in fact an organized conspiracy to overthrow the Government, not
only by force and violence but also by deceit, subversion and other illegal means. It declares that the
CPP is a clear and present danger to the security of the Philippines. Section 4 provided
that affiliation with full knowledge of the illegal acts of the CPP is punishable. Section 5 states that
due investigation by a designatedprosecutor by the Secretary of Justice be made prior to filing
of information in court. Section 6 provides for penalty for furnishing false evidence. Section 7
provides for 2 witnesses in open court for acts penalized by prision mayor to death. Section 8 allows
the renunciation of membership to the CCP through writing under oath. Section 9 declares the
constitutionality of the statute and its valid exercise under freedom if thought, assembly and
association.

ISSUES:
(1)Whether or not RA1700 is a bill of attainder/ ex post facto law.
(2)Whether or Not RA1700 violates freedom of expression.

RULING:
The court holds the VALIDITY Of the Anti-Subversion Act of 1957.
A bill of attainder is solely a legislative act. It punishes without the benefit of the trial. It is the
substitution of judicial determination to a legislative determination of guilt. In order for a statute be
measured as a bill of attainder, the following requisites must be present: 1.) The statute specifies
persons, groups. 2.) the statute is applied retroactively and reach past conduct. (A bill of attainder
relatively is also an ex post facto law.)

In the case at bar, the statute simply declares the CPP as an organized conspiracy for the overthrow
of the Government for purposes of example of SECTION 4 of the Act. The Act applies not only to the
CRIMINAL LAW 1
(Sunday, Section 52) 36

CPP but also to other organizations having the same purpose and their successors. The Act’s focus is
on the conduct not person.

Membership to this organizations, to be UNLAWFUL, it must be shown that membership was


acquired with the intent to further the goals of the organization by overt acts. This is the element
of MEMBERSHIP with KNOWLEDGE that is punishable. This is the required proof of a member’s
direct participation. Why is membership punished. Membership renders aid and encouragement to
the organization. Membership makes himself party to its unlawful acts.

Furthermore, the statute is PROSPECTIVE in nature. Section 4 prohibits acts committed after
approval of the act. The members of the subversive organizations before the passing of this Act is
given an opportunity to escape liability by renouncing membership in accordance with Section 8. The
statute applies the principle of mutatis mutandis or that the necessary changes having been made.

The declaration of that the CPP is an organized conspiracy to overthrow the Philippine Government
should not be the basis of guilt. This declaration is only a basis of Section 4 of the Act. The EXISTENCE
OF SUBSTANTIVE EVIL justifies the limitation to the exercise of “Freedom of Expression and
Association” in this matter. Before the enactment of the statute and statements in the preamble,
careful investigations by the Congress were done. The court further stresses that whatever interest
in freedom of speech and association is excluded in the prohibition of membership in the CPP are
weak considering NATIONAL SECURITY and PRESERVATION of DEMOCRACY.

The court set basic guidelines to be observed in the prosecution under RA1700. In addition to proving
circumstances/ evidences of subversion, the following elements must also be established:

1. Subversive Organizations besides the CPP, it must be proven that the organization purpose is to
overthrow the present Government of the Philippines and establish a domination of a FOREIGN
POWER. Membership is willfully and knowingly done by overt acts.
2. In case of CPP, the continued pursuance of its subversive purpose. Membership is willfully and
knowingly done by overt acts.

The court did not make any judgment on the crimes of the accused under the Act. The Supreme Court
set aside the resolution of the TRIAL COURT.

US V. DIAZ CONDE
L-18208, 14 FEBRUARY 1922

FACTS:On December 30, 1915, Bartolome Oliveros and Engracio Liaco borrowed from Vicente Diaz-
Conde and Apolinaria R. De Conde the sum of P300. They obligated themselves to pay the defendants
5% per month, payable within the first ten days beginning on January 1916.
On May 1, 1916, Act No. 2655 (Usury Law) took effect.

ISSUE:
Whether or not the defendants violated Act No. 2655.

RULING:
No. If a contract is legal at its inception, it cannot be rendered illegal by any subsequent legislation.
The obligation of the contract is the law which binds the parties to perform their agreement if it is
not contrary to the law of the land, morals or public order. That law must govern and control the
contract in every aspect in which it is intended to bear upon it, whether it affect its validity,
CRIMINAL LAW 1
(Sunday, Section 52) 37

construction, or discharge. In the present case, making Act No. 2655 applicable to the act complained
of which had been done before the law was adopted, a criminal act, would give it an ex post facto
operation.

An ex post facto law, is a law that makes an action, done before the passage of the law, and which was
innocent when done, criminal. Ex post facto laws are absolutely prohibited unless its retroactive
effect is favorable to the defendant.

The decision of the lower court is revoked and the complaint dismissed.

REPUBLIC VS. ROSEMOORE MINING


G.R. NO. 149927, 30 MAR. 2004

FACTS:
Petitioner Rosemoor Mining and Development Corporation after having been granted permission to
prospect for marble deposits in the mountains of Biak-na-Bato, San Miguel, Bulacan, succeeded in
discovering marble deposits of high quality and in commercial quantities in Mount Mabio which
forms part of the Biak-na-Bato mountain range.
The petitioner then applied with the Bureau of Mines, now Mines and Geosciences Bureau, for the
issuance of the corresponding license to exploit said marble deposits.
License No. 33 was issued by the Bureau of Mines in favor of the herein petitioners. Shortly thereafter,
Respondent Ernesto Maceda cancelled the petitioner’s license stating that their license had illegally
been issued, because it violated Section 69 of PD 463; and that there was no more public interest
served by the continued existence or renewal of the license. The latter reason was confirmed by the
language of Proclamation No. 84. According to this law, public interest would be served by reverting
the parcel of land that was excluded by Proclamation No. 2204 to the former status of that land as
part of the Biak-na-Bato national park.

ISSUE:
Whether or not Presidential Proclamation No. 84 is valid.

RULING:
Yes. We cannot sustain the argument that Proclamation No. 84 is a bill of attainder; that is, a
legislative act which inflicts punishment without judicial trial.” Its declaration that QLP No. 33 is a
patent nullity is certainly not a declaration of guilt. Neither is the cancellation of the license a
punishment within the purview of the constitutional proscription against bills of attainder.
Too, there is no merit in the argument that the proclamation is an ex post facto law. It is settled that
an ex post facto law is limited in its scope only to matters criminal in nature. Proclamation 84, which
merely restored the area excluded from the Biak-na-Bato national park by canceling respondents’
license, is clearly not penal in character.
Also at the time President Aquino issued Proclamation No. 84 on March 9, 1987, she was still validly
exercising legislative powers under the Provisional Constitution of 1986. Section 1 of Article II of
Proclamation No. 3, which promulgated the Provisional Constitution, granted her legislative power
until a legislature is elected and convened under a new Constitution. The grant of such power is also
explicitly recognized and provided for in Section 6 of Article XVII of the 1987 Constitution.
CRIMINAL LAW 1
(Sunday, Section 52) 38

LACSON VS. EXECUTIVE SECRETARY


G.R. NO. 128096, 20 JAN. 1999

FACTS:
Eleven persons believed to be members of the Kuratong Baleleng gang, an organized crime syndicate
involved in bank robberies, were slain by elements of the Anti-Bank Robbery andIntelligence Task
Group (ABRITG). Among those included in the ABRITG were petitioners and petitioner-intervenors.

Acting on a media expose of SPO2 Eduardo delos Reyes, a member of the Criminal Investigation
Command, that what actually transpired was a summary execution and not a shoot-out between the
Kuratong Baleleng gang members and the ABRITG, Ombudsman Aniano Desiertoformed a panel of
investigators to investigate the said incident. Said panel found the incident as a legitimate police
operation. However, a review board modified the panel’s finding and recommended the indictment
for multiple murder against twenty-six respondents including herein petitioner, charged as principal,
and herein petitioner-intervenors, charged as accessories. After a reinvestigation, the Ombudsman
filed amended informations before the Sandiganbayan, where petitioner was charged only as an
accessory.

The accused filed separate motions questioning the jurisdiction of the Sandiganbayan, asserting that
under the amended informations, the cases fall within the jurisdiction of the Regional Trial Court
pursuant to Section 2 of R.A. 7975. They contend that the said law limited the jurisdiction of the
Sandiganbayan to cases where one or ore of the “principal accused” are government officals with
Salary Grade 27 or higher, or PNP officials with rank of Chief Superintendent or higher. Thus, they
did not qualify under said requisites. However, pending resolution of their motions, R.A. 8249 was
approved amending the jurisdiction of the Sandiganbayan by deleting the word “principal” from the
phrase “principal accused” in Section 2 of R.A. 7975.

Petitioner questions the constitutionality of Section 4 of R.A. 8249, including Section 7 which
provides that the said law shall apply to all cases pending in any court over which trial has not begun
as of the approval hereof.

ISSUES:
(1) Whether or not Sections 4 and 7 of R.A. 8249 violate the petitioners’ right to due process and the
equal protection clause of the Constitution as the provisions seemed to have been introduced for the
Sandiganbayan to continue to acquire jurisdiction over the Kuratong Baleleng case.

(2) Whether or not said statute may be considered as an ex-post facto statute.

(3) Whether or not the multiple murder of the alleged members of the Kuratong Baleleng was
committed in relation to the office of the accused PNP officers which is essential to the determination
whether the case falls within the Sandiganbayan’s or Regional Trial Court’s jurisdiction.

RULING:
Petitioner and intervenors’ posture that Sections 4 and 7 of R.A. 8249 violate their right to equal
protection of the law is too shallow to deserve merit. No concrete evidence and convincing argument
were presented to warrant such a declaration. Every classification made by the law is presumed
reasonable and the party who challenges the law must present proof of arbitrariness. The
classification is reasonable and not arbitrary when the following concur: (1) it must rest on
substantial distinction; (2) it must be germane to the purpose of the law; (3) must not be limited to
CRIMINAL LAW 1
(Sunday, Section 52) 39

existing conditions only, and (4) must apply equally to all members of the same class; all of which
are present in this case.

Paragraph a of Section 4 provides that it shall apply “to all cases involving” certain public officials and
under the transitory provision in Section 7, to “all cases pending in any court.” Contrary to petitioner
and intervenors’ argument, the law is not particularly directed only to the Kuratong Baleleng cases.
The transitory provision does not only cover cases which are in the Sandiganbayan but also in “any
court.”

There is nothing ex post facto in R.A. 8249. Ex post facto law, generally, provides retroactive effect of
penal laws. R.A. 8249 is not apenal law. It is a substantive law on jurisdiction which is not penal in
character. Penal laws are those acts of the Legislature which prohibit certain acts and establish
penalties for their violations or those that define crimes and provide for their punishment. R.A. 7975,
as regards the Sandiganbayan’s jurisdiction, its mode of appeal and other procedural matters, has
been declared by the Court as not a penal law, but clearly a procedural statute, one which prescribes
rules of procedure by which courts applying laws of all kinds can properly administer justice. Not
being a penal law, the retroactive application of R.A. 8249 cannot be challenged as unconstitutional.

In People vs. Montejo, it was held that an offense is said to have been committed in relation to the
office if it is intimately connected with the office of the offender and perpetrated while he was in the
performance of his official functions. Such intimate relation must be alleged in the information which
is essential in determining the jurisdiction of the Sandiganbayan. However, upon examination of the
amended information, there was no specific allegation of facts that the shooting of the victim by the
said principal accused was intimately related to the discharge of their official duties as police officers.
Likewise, the amended information does not indicate that the said accused arrested and investigated
the victim and then killed the latter while in their custody. The stringent requirement that the charge
set forth with such particularity as will reasonably indicate the exact offense which the accused is
alleged to have committed in relation to his office was not established.

Consequently, for failure to show in the amended informations that the charge of murder was
intimately connected with the discharge of official functions of the accused PNP officers, the offense
charged in the subject criminal cases is plain murder and, therefore, within the exclusive original
jurisdiction of the Regional Trial Court and not the Sandiganbayan.

CALDER VS. BULL


3 DALL [US] 386

FACTS:
Mr. and Mrs. Caleb Bull, the stated beneficiaries of the will of Norman Morrison, were denied an
inheritance by a Connecticut probate court. When the Bulls attempted to appeal the decision more
than a year and a half later, they found that a state law prohibited appeals not made within 18 months
of the original ruling. The Bulls persuaded the Connecticut legislature to change the restriction, which
enabled them to successfully appeal the case. Calder, the initial inheritor of Morrison's estate, took
the case to the Supreme Court.

ISSUE:
Was the Connecticut legislation a violation of Article 1, Section 10, of the Constitution, which
prohibits ex post facto laws?
CRIMINAL LAW 1
(Sunday, Section 52) 40

RULING:
In a unanimous decision, the Court held that the legislation was not an ex post facto law. The Court
drew a distinction between criminal rights and "private rights," arguing that restrictions against ex
post facto laws were not designed to protect citizens' contract rights. Justice Chase noted that while
all ex post facto laws are retrospective, all retrospective laws are not necessarily ex post facto. Even
"vested" property rights are subject to retroactive laws.

BAYOT VS. SANDIGANBAYAN


G.R. NO. L-61776 TO NO. L-61861, 23 MAR. 1984

FACTS:
Petitioner Reynaldo R. Bayot is one of the several persons accused in more than one hundred(100)
counts of Estafa thru Falsification of Public Documents before the Sandiganbayan. The said charges
stemmed from his alleged involvement, as a government auditor of the Commission on Audit
assigned to the Ministry of Education and Culture, together with some officers/employees of the said
Ministry, the Bureau of Treasury and the Teacher's Camp in Baguio City, in the preparation and
encashment of fictitious TCAA checks for non-existent obligations of the Teacher's Camp resulting in
damage to the government of several million pesos. The first thirty-two (32) cases were filed on July
25, 1978. In the meantime, petitioner ran for the post of municipal mayor of Amadeo, Cavite in the
local elections held in January 1980. He was elected. On May 30, 1980, the Sandiganbayan
promulgated a decision convicting herein petitioner and some of his co-accused in all but one of the
thirty-two (32) cases filed against them. Whereupon, appeals were taken to this Court and the cases
are now pending review in G.R. Nos. L-54645-76. However, on March 16, 1982, Batas Pambansa Blg.
195 was passed amending, among others, Section 13 of Republic Act No. 3019. Petitioner filed a
motion for reconsideration alleging that "to apply the provision of Batas Pambansa Blg. 195 to the
herein accused would be violative of the constitutional guarantee of protection against an ex post
facto law.

ISSUE:
Whether or not the retroactive application of Batas Pambansa Blg. 195 would violate the
Constitutional provision against enactment of ex post facto law.

RULING:
NO. The Court finds no merit in petitioner's contention that Section 13 of Republic Act 3019, as
amended by Batas Pambansa Blg. 195, which includes the crime of Estafa thru Falsification of Public
Document as among the crimes subjecting the public officer charged therewith with suspension from
office pending action in court, is a penal provision which violates the constitutional prohibition
against the enactment of ex post facto law. Paragraph 3 of Article 24 of the Revised Penal Code clearly
states that suspension from the employment or public office during the trial or in order to institute
proceedings shall not be considered as penalty. It is not a penalty because it is not imposed as a result
of judicial proceedings. In fact, if acquitted, the official concerned shall be entitled to reinstatement
and to the salaries and benefits which he failed to receive during suspension. Those mentioned in
paragraph Nos. 1, 3 and 4 of said Article 24 are merely preventive measures before final judgment.
Not being a penal provision, therefore, the suspension from office, pending trial, of the public officer
charged with crimes mentioned in the amendatory provision committed before its effectivity does
not violate the constitutional provision on ex post facto law.
CRIMINAL LAW 1
(Sunday, Section 52) 41

US VS. WILTBERGER
5 WHEAT (US) 76, 95, 5 L. ED. 37, 42

FACTS:
On the afternoon of the 24th of November, 1906, several laborers working for the street-railway
company, for the purpose of receiving their wages, were directed to remain in single file and then
successively approach a small house, where the paymaster had his office, in order to be regularly
paid. The accused, as foreman in the service of the company, was in charge of the preservation of
order and for such purpose he provided himself with a pick or spade handle, one end of which he
daubed with mud and threatened to push against the clothing of the laborers if they left the line or
created trouble. Benedicto Dio Pito left his place and forced his way into the file; the accused ordered
him out, but still persisted, and the accused then gave him a blow with the stick on the right side of
the head, above the ear, in consequence of which the deceased, Benedicto Dio Pito, fell to the ground
holding his head with his hands; thereupon the accused and another laborer rendered him prompt
assistance and took him to the interior of a warehouse close by; the injured man shortly afterwards
asked for water, which, however, he was unable to drink, and became unconscious, and upon being
taken to St. Paul's Hospital died there a few hours later.

The accused pleaded not guilty, although he confessed to having struck the deceased on the right side
of his head with a pick handle, one of the working tools; it is therefore unquestionable that he is the
author of the violent death of Benedicto Dio Pito.

The accused, however, alleges that when he struck the blow he merely acted in self-defense in view
of the attitude of the deceased, who, after insulting him, thrust his hand into his pocket as if for the
purpose of drawing a dagger or a pocketknife and that in order to defend himself, because he believed
that the deceased, owing to his attitude, was about to attack him, he struck him the blow with the
stick he had provided himself with, in consequence of which the deceased fell to the ground, then
arose at once, vomited, and then sat down; that he and two other men then assisted the deceased and
conducted him to the interior of a warehouse, where he became unconscious, and later on he died in
St. Paul's Hospital, whereto he was removed.

ISSUE: W/N respondent can be held liable

HELD: YES. Although, In the commission of the homicide in question the circumstance of the accused
being under 18 years of age must be considered, according to No. 2 of article 9 of the Penal Code,
which requires, in connection with paragraph 2 of article 85 of said code, that the penalty next lower
to the one imposed by article 404 should be applied. The penalty of prision mayor should ordinarily
be imposed on the accused, but taking into account the mitigating circumstances 3 and 4 of said
article 9, viz, that he never intended to cause so serious a harm as the death of Benedicto Dio Pito,
and owing to the fact that provocation on the part of the latter immediately preceded the attack by
the accused, and also considering that no aggravating circumstance is present, and, under rule 5 of
article 81 of the code, the appropriate penalty to be inflicted on the accused should be the next lower
to the above-mentioned penalty of prision mayor, that is, prision correccional in its medium degree.
CRIMINAL LAW 1
(Sunday, Section 52) 42

CENTENO VS. VILLALON-PORNILLOS


G.R. NO. 113092, 1 SEPT. 1994

Solicitation for religious purposes may be subject to proper regulation by the State in the exercise of
police power. The State has authority under the exercise of its police power to determine whether or
not there shall be restrictions on soliciting by unscrupulous persons or for unworthy causes or for
fraudulent purposes. Certainly the solicitation of contributions in good faith for worthy purposes
should not be denied, but somewhere should be lodged the power to determine within reasonable
limits the worthy from the unworthy.

FACTS:
This petition is an appeal on the decision of the Trial Court convicting Centeno and Yco for violating
P.D. 1564 known as the Solicitation Permit Law when they both solicited money for the renovation
of their chapel without a permit from the DSWD.

In 1985, the petitioners, officers of Samahang Katandaan ng Nayon ng Tikay, launched a fund drive
for the renovation of their chapel in Bulacan.

The petitioners approached and solicited from Judge Adoracion G. Angeles, a resident of Tikay, a
contribution of P1,500.00. The solicitation was made without a permit from the Department of Social
Welfare and Development (DSWD). Hon. Angeles filed a complaint against the petitioners for
violation of P.D. 1564 known as the Soliciation Permit Law.

P.D. 1564 provides as follows:


Sec. 2. Any person, corporation, organization, or association desiring to solicit or receive
contributions for charitable or public welfare purposes shall first secure a permit from the Regional
Offices of the Department of Social Services and Development as provided in the Integrated
Reorganization Plan.

In 1992, the trial court found the petitioners guilty of violating the Solicitation Permit Law.
In this instant case, the petitioners assert among others that the term “religious purpose” is not
expressly included in the provisions of the statute, hence what the law does not include, it excludes.
ISSUE: Whether or not the phrase “charitable purposes” should be construed in the broadest sense
so as to include a religious purpose.

RULING:
The 1987 Constitution and other statutes treat the words “charitable” and “religious” separately and
independently of each other.

In P.D. 1564, it merely stated “charitable or public welfare purposes” which means that it was not the
intention of the framers of the law to include solicitations for religious purposes. The world “religious
purpose” is not interchangeable with the expression “charitable purpose”.

The acts of the petitioners cannot be punished under the said law because the law does not
contemplate solicitation for religious purposes.

The solicitation for religious purposes may be subject to proper regulation by the State in the exercise
of police power. However, in the case at bar, considering that solicitations intended for a religious
purpose are not within the coverage of Presidential Decree No. 1564, as earlier demonstrated,
petitioner cannot be held criminally liable therefor.
CRIMINAL LAW 1
(Sunday, Section 52) 43

The decision appealed from is reversed and set aside, and petitioner Martin Centeno is acquitted of
the offense charged.

RIMANDO VS. COMELEC

FACTS:
Private respondent NORMA MAGNO filed a complaint against two security guards, JACINTO CARAG
& JONRY ENAYA, for:
A. Possession of firearms pursuant to COMELEC Resolution No. 3328 Sec. 2(a) and
N. for killing her son JONATHAN MAGNO, a 19-year old nautical student in his school uniform in Sta.
Rosa Homes, Sta. Rosa Laguna, with a shotgun, on 27 FEB 2001, during the 2001 election period.
2. Petitioner, thru their counter-affidavit, denied the aforementioned violations and argued that:
a. The security guards acted on the premise of work
b. Firearms were licensed and were never carried out of the aforementioned vicinity
3.8 OCT 2001, Provincial Election Supervisor of Sta. Cruz, Laguna, dismissed respondent’s complaint
finding that the licensed firearms were carried and used by the security guards within their place of
work, for which no exemption and/or permit was needed in pursuant to Sec. 2(e) of the COMELEC
Resolution 3328
4. Respondent appealed to COMELEC, Intramuros, citing Sec. 3(d) of the COMELEC Resolution 3328,
and argued that an approval to possess firearms were necessary
5. 6 MAY 2002, COMELECs resolution affirmed dismissal of complaint but redirected said complaint
to owner JUANITO RIMANDO (Ret. Brig. Gen.) for violation of Art XXII, Sec 261 of the Omnibus
Election Code, requiring a permit from the commission before security guards of a security agency
can bear firearms in their place of assignment during the election gun ban
The Commission held RIMANDO liable for such violation as he applied for an exemption from the
Committee but was denied for lack of endorsement of the CSG Director
6. Petitioner filed a motion for reconsideration contending that:
a. The said resolution went beyond the scope of law when it held petitioner, as President of the
security agency, criminally liable for an act that was not prohibited pursuant to Sec. 261(s) of OEC
b. There was no conflict between Sec. (2) & (3) of COMELEC Resolution No. 3382 and if there was, it
should be resolved in his favor since penal laws were construed strictly against the State and in favor
of the accused.
c. The application for exemption filed by petitioner’s security agency with the COMELEC through
PNP-SAGD was to transport firearms not to bear arms inside/within the place of work
d. SINCE NO ELECTION OFFENSE WAS COMMITTED THE FILING OF A CRIMINAL CASE W AS
UNWARRANTED AND CONTRARY TO LAW.
7. 30 JAN 2004, COMELEC, granted the MR and reversed and set aside 6 MAY 2002 Resolution

ISSUE:
W/N respondent RIMANDO can be held liable, NO

RULING:
1. US v. ABAD SANTOS
: Courts will not hold one person criminally responsible for the acts of another, committed without
his knowledge or consent, unless there is a statute requiring it so plain in its terms that there is no
doubt of the intention of the Legislature
2.PEOPLE v. DELEVERIO
: It is a basic rule of statutory construction that penal statutes are to be liberally construed in favor
of the accused. Hence, any reasonable doubt must be resolved in favor of the accused.
CRIMINAL LAW 1
(Sunday, Section 52) 44

3. Even assuming for the sake of argument that Sec. 261(s) required petitioners security agency to
secure prior written approval from the COMELEC for its security guards to bear arms in their place
of work, the failure of the owner of the agency to secure such is not itself defined an election offense.

PEOPLE VS. DICK ONG


204 SCRA 942

FACTS:
Accused Dick Ong, one of the depositors of the Home Savings Bank and Trust Company (HSBTC)
opened a savings account with HSBTC with an initial deposit of P22.14 in cash and P10,000.00 in
check. Ong was allowed to withdraw from his savings account with the Bank the sum of P5,000.00,
without his check under going the usual and reglementary clearance. The withdrawal slip was signed
and approved by Lino Morfe, then the BranchManager, and accused Lucila Talabis, the Branch
Cashier. Subsequently, Ong deposited eleven checks in his savings account with the Bank and against
which he made withdrawals against its amount. Again, the withdrawal of the amount by Ong was
made before said checks were cleared and the Bank had collected their amounts and with the
approval of Talabis. However, when the Bank presented the eleven checks issued, deposited and
against which Ong made withdrawals against its amounts, to their respective drawee banks for
payment, they were all dishonored for lack or insufficiency of funds. Because of this, the Bank filed a
criminal action for Estafa against Ong, and the Bank’s officer in charge Villaran and Talabis. Talabis
testified that the approval of the withdrawals of Ong against his uncleared checks was in accordance
with the instruction of their then bank manager and that it is a kind of accommodation given to Ong
and also a common practice of the Bank. RTC ruled Ong as guilty for the crime of estafa but acquitted
Villarin and Talabis as their guilt were not proven beyond reasonable doubt. CA affirmed RTCs
decisions.

ISSUE:
1. What is the nature of bank deposits?
2. WON Ong is guilty of Estafa?

RULING:
1. The Supreme Court held that bank deposits are in the nature of irregular deposits. Bank deposits
are really loans because they earn interest. Whether fixed, savings, or current, all bank A deposits are
to be treated as loans and are to be covered by the law on loans.
2. The elements of this kind of estafa are the following: (1) postdating or issuance of a check in
payment of an obligation contracted at the time the check was issued; (2) lack or insufficiency of
funds to cover the check; and (3) damage to the payee thereof. In this case, the fact was established
that Ong either issued or indorsed the subject checks. However, it must be remembered that the
reason for the conviction of an accused of the crime of estafa is his guilty knowledge of the fact that
he had no funds in the bank when he negotiated the spurious check. In the present case, however, the
prosecution failed to prove that Ong had knowledge with respect to the check she indorsed.
Moreover, it has also been proven that it was the Bank which granted him a drawn against
uncollected deposit (DAUD) privilege without need of any pretensions on his part. The privilege this
privilege was not only for the subject checks, but for other past transactions. If ever, he, indeed acted
fraudulently, he could not have done so without the active cooperation of the Banks employees. Since
Talabis and Villaran were declared innocent of the crimes charged against them, the same should be
said for the Ong.
CRIMINAL LAW 1
(Sunday, Section 52) 45

MALILIN VS. PEOPLE


G.R. NO. 172953, 30 APRIL 2008

FACTS:
On the strength of a warrant of search and seizure, a team of five police officers raided the residence
of petitioner. The search allegedly yielded two (2) plastic sachets of shabu and five (5) empty plastic
sachets containing residual morsels of the said substance. Accordingly, petitioner was charged with
violation illegal possession of dangerous drugs under RA 9165.

Accused was convicted. He appealed from the said decision and averred that there was irregularity
on the conduct of the search.

During the trial, the following persons were presented as witnesses: (1) two members of the raiding
team, i.e., Esternon and Bulanon; and (2) the forensic chemist who examined the seized items. The
member of the raiding team who conducted the recording and marking of the seized items were not
presented. Furthermore the item seized was immediately sent to the laboratory without presenting
the same to the judge who issued the warrant. Furthermore, the forensic chemist admitted that all
seven sachets were delivered to the laboratory by Esternon in the afternoon of the same day that the
warrant was executed. The items seized were not presented to the judge who issued the warrant.

ISSUE:
Whether or not testimony of the raiding team and the forensic chemist is sufficient to establish the
identity of the seized items by virtue of a search warrant.

RULING:
No. Prosecutions for illegal possession of prohibited drugs necessitates that the elemental act of
possession of a prohibited substance be established with moral certainty, together with the fact that
the same is not authorized by law. The dangerous drug itself constitutes the very corpus delicti of the
offense and the fact of its existence is vital to a judgment of conviction. Essential therefore in these
cases is that the identity of the prohibited drug be established beyond doubt. Section 21 of the
Implementing Rules and Regulations of R.A. No. 9165 mandates that the officer acquiring initial
custody of drugs under a search warrant must conduct the photographing and the physical inventory
of the item at the place where the warrant has been served. The chain of custody rule requires that
there be testimony about every link in the chain, from the moment the object seized was picked up
to the time it is offered in evidence, in such a way that every person who touched it would describe
how and from whom it was received, where it was and what happened to it while in the witness’
possession, the condition in which it was received and the condition in which it was delivered to the
next link in the chain.

Esternon deviated he brought the seized items immediately to the police station for the alleged
purpose of making a "true inventory" thereof, but there appears to be no reason why a true inventory
could not be made in petitioner's house.

Likewise, Esternon's failure to deliver the seized items to the court demonstrates a departure from
the directive in the search warrant that the items seized be immediately delivered to the trial court
with a true and verified inventory of the same, as required by Rule 126, Section 1246 of the Rules of
Court. People v. Go characterized this requirement as mandatory in order to preclude the substitution
of or tampering with said items by interested parties. Thus, as a reasonable safeguard, People vs. Del
Castillo declared that the approval by the court which issued the search warrant is necessary before
CRIMINAL LAW 1
(Sunday, Section 52) 46

police officers can retain the property seized and without it, they would have no authority to retain
possession thereof and more so to deliver the same to another agency. Mere tolerance by the trial
court of a contrary practice does not make the practice right because it is violative of the mandatory
requirements of the law and it thereby defeats the very purpose for the enactment.

PEOPLE V. ABILONG
L-1960, 26 NOVEMBER 1948

FACTS:
That on or about the 17th day of September 1947, the accused, being a convict sentenced of destierro
during which he should not enter any place within the radius of 100 kilometers from the City of
Manila for robbery, evaded the service of said sentence by going beyond the limits made against him
and commit vagrancy.

Counsel of appellant contends that a person like the accused evading a sentence of destierro is not
criminally liable under the provisions of the RPC, particularly section 157 of the said Code for the
reason that said article 157 refers only to persons who are imprisoned in a penal institution and
completely deprived of their liberty. He bases the contention on the word “imprisonment” used in
the English text of said article which in part reads as follows:

“Evasion of service of sentence” – The penalty of prison correccional in its medium and maximum
periods shall be imposed upon any convict who shall evade service of his sentence by escaping during
the term of his imprisonment by reason of final judgment.”

The Solicitor General in his brief says that had the original text of the RPC been in the English
language, then the theory of the appellant could be upheld. However, it is the Spanish text that is
controlling in case of doubt.

The Supreme Court (SC) agreed that it is the Spanish texts that should govern, in as much as the RPC
was originally approved and enacted in Spanish.

The SC cleared that the word “imprisonment” used in the English text is a wrong or erroneous
translation of the phrase “sufriendo privacion de libertad” used in the Spanish text.

The SC also held that, it is clear that a person under sentence of destierro is suffering deprivation of
his liberty and escapes from the restrictions of the penalty when he enters the prohibited area.

ISSUE:
Whether the lower court erred in imposing a penalty on the accused under Article 157 of the Revised
Penal Code (RPC), which does not cover evasion of service of “destierro.”

RULING:
Appellant is guilty of evasion of service of sentence under article 157 of the RPC (Spanish text), in
that during the period of his sentence of destierro by virtue of final judgment wherein he was
prohibited from entering the City of Manila, he entered said City.
CRIMINAL LAW 1
(Sunday, Section 52) 47

PEOPLE V. FORMIGONES
L-3246 29 NOVEMBER 1950

FACTS:
From November to December 1946, defendant Abelardo Formigones together with his wife Julia
Agricola, and his five children lived in the house of his half-brother, Zacarias Formigones to find
employment as harvesters. One afternoon, the accused, without any previous quarrel or provocation
whatsoever, took his bolo from the wall of the house and stabbed his wife at the back, the blade
penetrating the right lung which latter caused her death. When she fall ont he ground the defendant
carried her up the house, laid her on the floor of the living room and then lay down beside her. He
was convicted of parricide and was sentenced to prison. The defendant entered a plea of not guilty.
His counsel presented testimonies of two guards of the provincial jail where defendant was confined.
They said that he behaved like an insane person, that sometimes he would remove his clothes in front
of others, would not take a bath, and remained silent and indifferent to his surroundings. His counsel
claimed that e is an imbecile therefore exempt from criminal liability. Dr. Francisco Gomez told that
Abelardo was suffering only from feeblemindedness and not imbecility and that he could distinguish
right from wrong. An imbecile so as to be exempt from criminal liability, he must be deprived
completely of reason or discernment and freedom of the will at the time of committing the crime.

ISSUE:
WON the defendant who is suffering from feeblemindedness is exempt from criminal liability.

RULING:
No. In order that an exempting circumstance may be taken into account, it is necessary that there be
a complete deprivation of intelligence in committing the act, that is, that the accused be deprived of
reason; that there be no responsibility for his own acts; that he acts without the least discernment;
that there be a complete absence of the power to discern, or that there be a total deprivation of
freedom of the will. As to the strange behaviour of the accused during his confinement, assuming that
it was not feigned to stimulate insanity, it may be attributed either to his being feebleminded or
eccentric, or to a morbid mental condition produced by remorse at having killed his wife. He could
distinguish right from wrong.

LADONGA V. PEOPLE
G.R. NO. 141066, 17 FEBRUARY 2005

FACTS:
Evangelina and Adronico Ladonga and spouse, conspiring and knowing fully well that they did not
have sufficient funds deposited with the United Coconut Planters Bank (UCPB), drew and issue UCPB
Check No. 284743 postdated July 7, 1990 in the amount of P9,075.55), payable to Alfredo Oculam,
and thereafter, without informing the latter that they did not have sufficient funds deposited with the
bank to cover up the amount of the check, did then and there willfully, unlawfully and feloniously
pass on, indorse, give and deliver the said check to Alfredo by way of rediscounting of the
aforementioned checks; however, upon presentation of the check to the drawee bank for
encashment, the same was dishonored for the reason that the account of the accused had already
been closed, to the damage and prejudice of Alfredo.

The RTC rendered a joint decision finding the Ladonga spouses guilty beyond reasonable doubt of
violating B.P. Blg. 22. Adronico applied for probation which was granted. On the other hand,
petitioner brought the case to the Court of Appeals, arguing that the RTC erred in finding her
CRIMINAL LAW 1
(Sunday, Section 52) 48

criminally liable for conspiring with her husband as the principle of conspiracy is inapplicable to B.P.
Blg. 22 which is a special law; moreover, she is not a signatory of the checks and had no participation
in the issuance thereof.

ISSUE:
a.) Whether conspiracy is applicable in violations of Batas Pambansa Bilang 22, by invoking art. 10 of
RPC?

b.) Whether or not the cases cited by the CA in affirming in toto the conviction of petitioner as
conspirator applying the suppletory character of the RPC to special laws like BP 22 is applicable?

RULING:
A.) YES. Some provisions of the Revised Penal Code, especially with the addition of the second
sentence in Article 10, are applicable to special laws. It submits that B.P. Blg. 22 does not provide any
prohibition regarding the applicability in a suppletory character of the provisions of the Revised
Penal Code to it.

Article 10 of the RPC reads as follows: ART. 10. Offenses not subject to the provisions of this Code. –
Offenses which are or in the future may be punishable under special laws are not subject to the
provisions of this Code. This Code shall be supplementary to such laws, unless the latter should
specially provide the contrary.

The article is composed of two clauses. The first provides that offenses which in the future are made
punishable under special laws are not subject to the provisions of the RPC, while the second makes
the RPC supplementary to such laws

B.) B.P. Blg. 22 does not expressly proscribe the suppletory application of the provisions of the RPC.
Thus, in the absence of contrary provision in B.P. Blg. 22, the general provisions of the RPC which, by
their nature, are necessarily applicable, may be applied suppletorily. Indeed, in the recent case of Yu
vs. People the Court applied suppletorily the provisions on subsidiary imprisonment under Article
39 of the RPC to B.P. Blg. 22.

The suppletory application of the principle of conspiracy in this case is analogous to the application
of the provision on principals under Article 17 in U.S. vs. Ponte. For once conspiracy or action in
concert to achieve a criminal design is shown, the act of one is the act of all the conspirators, and the
precise extent or modality of participation of each of them becomes secondary, since all the
conspirators are principals. BUT In the present case, the prosecution failed to prove that petitioner
performed any overt act in furtherance of the alleged conspiracy. Conspiracy must be established,
not by conjectures, but by positive and conclusive evidence. Thus, Petitioner Evangeline Ladonga is
ACQUITTED of the charges against her under B.P. Blg. 22 for failure of the prosecution to prove her
guilt beyond reasonable doubt. No pronouncement as to costs.
CRIMINAL LAW 1
(Sunday, Section 52) 49

PEOPLE V. SIMON
G.R. NO. 93028, 29 JULY 1994

FACTS:
Accused Martin Simon was charged with a violation of Section 4, Article II of Republic Act No. 6425
or the Dangerous Drugs Act of 1972. He sold tea bags of marijuana to a Narcotics Command
(NARCOM) poseur-buyer. The confiscated 4 tea bags, weighing a total of 3.8 grams, when subjected
to laboratory examination, were found positive for marijuana.

Simon denied the accusation against him, claiming that on the day of question, he was picked up by
the police at their house while watching TV. He was told that he was a pusher so he attempted to
alight from the jeep but he was handcuffed instead. When they finally reached the camp, he was
ordered to sign some papers and, when he refused, he was boxed in the stomach eight or nine times
by Sgt. Pejoro. He was then compelled to affix his signature and fingerprints on the documents
presented to him. He denied knowledge of the marked money or the 4 teabags of dried marijuana
leaves, and insisted that the marked money came from the pocket of Pejoro. Moreover, the reason
why he vomited blood was because of the blows he suffered at the hands of Pejoro.

Dr. Evelyn Gomez-Aguas, a resident physician of Romana Pangan District Hospital, declared that she
treated appellant for three days due to abdominal pain, but her examination revealed that the cause
for this ailment was appellant’s peptic ulcer. She did not see any sign of slight or serious external
injury, abrasion or contusion on his body.

Simon was sentenced to suffer the penalty of life imprisonment, to pay a fine of twenty thousand
pesos and to pay the costs. Simon then seek the reversal of the judgement

ISSUE:
Was the conviction of Simon correct?

RULING:
To sustain a conviction for selling prohibited drugs, the sale must be clearly and unmistakably
established. To sell means to give, whether for money or any other material consideration. It must,
therefore, be established beyond doubt that appellant actually sold and delivered two tea bags of
marijuana dried leaves to Sgt. Lopez, who acted as the poseur-buyer, in exchange for two twenty-
peso bills.

After careful review, the Court held that there were 2 tea bags of marijuana that was sold and there
were 2 other tea bags of marijuana confiscated. Thus, Simon should be charged of selling for the 2 tea
bags of marijuana only.
However, there is an overlapping error in the provisions on the penalty of reclusion perpetua by
reason of its dual imposition, that is, as the maximum of the penalty where the marijuana is less than
750 grams, and also as the minimum of the penalty where the marijuana involved is 750 grams or
more. The same error has been committed with respect to the other prohibited and regulated drugs
provided in said Section 20. To harmonize such conflicting provisions in order to give effect to the
whole law, the court hereby hold that the penalty to be imposed where the quantity of the drugs
involved is less than the quantities stated in the first paragraph shall range from prision
correccional to reclusion temporal, and not reclusion perpetua. This is also concordant with the
CRIMINAL LAW 1
(Sunday, Section 52) 50

fundamental rule in criminal law that all doubts should be construed in a manner favorable to the
accused.

The court held that Republic Act No. 6425, as now amended by Republic Act No. 7659, has
unqualifiedly adopted the penalties under the Revised Penal Code in their technical terms, hence with
their technical signification and effects. In fact, for purposes of determining the maximum of said
sentence, the court have applied the provisions of the amended Section 20 of said law to arrive
at prision correccional and Article 64 of the Code to impose the same in the medium period. Such
offense, although provided for in a special law, is now in effect punished by and under the Revised
Penal Code. Correlatively, to determine the minimum, the court applied first part of the aforesaid
Section 1 which directs that “in imposing a prison sentence for an offense punished by the Revised
Penal Code, or its amendments, the court shall sentence the accused to an indeterminate sentence
the maximum term of which shall be that which, in view of the attending circumstances, could be
properly imposed under the rules of said Code, and the minimum which shall be within the range of
the penalty next lower to that prescribed by the Code for the offense.”

Thus, in the case at bar, appellant should be begrudged the benefit of a minimum sentence within the
range of arresto mayor, the penalty next lower to prision correccional which is the maximum range
have fixed through the application of Articles 61 and 71 of the Revised Penal Code. For, with fealty to
the law, the court may set the minimum sentence at 6 months of arresto mayor, instead of 6 months
and 1 day of prision correccional.

PEOPLE VS. MANTALBA


G.R. NO. 186227, 20 JULY 2011

FACTS:
Task Forcer Regional Anti-Crime Emergency Response (RACER) in Butuan City received a report that
Mantalaba who was 17 yrs old was selling shabu. After a buy-bust operation, two informations was
filed against Mantalaba which was later on consolidated. Mantalaba pleaded not guilty.

RTC found Mantalaba guilty beyond reasonable doubt and was penalized of reclusion perpetua to
death and fine of 500k for selling shabu and (2) for illegally possessing shabu, Mantalaba was
penalized, in application of the ISL, 6 yrs and 1 day as minimum and 8 yrs as maximum of prision
mayor and fine of 300k. CA affirmed in toto the decision of the RTC. Thus, the present appeal.

Mantalaba: the lower court gravely erred in convicting him and that there was no evidence of actual
sale between him and the poser-buyer during the buy-bust operation. He also claims that the chain
of custody of the seized shabu was not established.

ISSUE:
Whether Mantalaba is guilty of drug trafficking and possession.

RULING:
The petition is without merit.

The buy-bust operation was valid, establishing the following: (1) the identity of the buyer and the
seller, the object, and the consideration; and (2) the delivery of the thing sold and the payment
therefore. From the above testimony of the prosecution witness, it was well established that the
elements have been satisfactorily met. The seller and the poseur-buyer were properly identified. The
CRIMINAL LAW 1
(Sunday, Section 52) 51

subject dangerous drug, as well as the marked money used, were also satisfactorily presented. The
testimony was also clear as to the manner in which the buy-bust operation was conducted.

Non-compliance by the apprehending/buy-bust team with Section 21 is not fatal as long as there is
justifiable ground therefor, and as long as the integrity and the evidentiary value of the
confiscated/seized items are properly preserved by the apprehending officer/team. Its non-
compliance will not render an accused arrest illegal or the items seized/confiscated from him
inadmissible.

As to his minority, Mantalaba was minor during the buy-bust operation but was of legal age during
the promulgation of the decision. It must be noted that RA 9344 took effect after the promulgation of
the RTC's decision against Mantalaba. The RTC did not suspend the sentence in accordance with PD
603 (Child and Youth Welfare Code) and Rule on Juveniles in Conflict with the Law that were
applicable at the time of the promulgation of the judgment. However, as ruled in People vs Sarcia,
suspension of sentence can still be applied but NOT when the offender upon the promulgation of
judgment is 21 yrs old. or older. Mantalaba is now 21 yrs old, therefore his suspension of sentence is
already moot and academic.

But as to the penalty, CA must have appreciated Mantalaba's minority as privileged mitigating
circumstance in fixing the penalty. Thus, applying the rules stated above, the proper penalty should
be one degree lower than reclusion perpetua, which is reclusion temporal, the privileged mitigating
circumstance of minority having been appreciated. Necessarily, also applying the Indeterminate
Sentence Law (ISLAW), the minimum penalty should be taken from the penalty next lower in degree
which is prision mayor and the maximum penalty shall be taken from the medium period of reclusion
temporal, there being no other mitigating circumstance nor aggravating circumstance.

ROMUALDEZ VS. MARCELO


G.R. NOS. 165510-33, 28 JULY 2006

FACTS:
Petitioner is being charged with violations of Section 7 of RA No. 3019 for failure to file his Statements
of Assets and Liabilities for the period 1967-1985 during his tenure as Ambassador Extraordinary
and Plenipotentiary and for the period 1963-1966 during his tenure as Technical Assistant in the
Department of Foreign Affairs.

Petitioner claims that the Office of the Ombudsman gravely abused its discretion in recommending
the filing of 24 informations against him for violation of Section 7 of Republic Act (RA) No. 3019 or
the Anti-Graft and Corrupt Practices Act; that the Ombudsman cannot revive the aforementioned
cases which were previously dismissed by the Sandiganbayan in its Resolution of February 10, 2004;
that the defense of prescription may be raised even for the first time on appeal and thus there is no
necessity for the presentation of evidence thereon before the court a quo. Thus, this Court may
accordingly dismiss Criminal Case Nos. 28031-28049 pending before the Sandiganbayan and
Criminal Case Nos. 04-23185704-231860 pending before the Regional Trial Court of Manila, all on
the ground of prescription.

In its Comment, the Ombudsman argues that the dismissal of the informations in Criminal Case Nos.
13406-13429 does not mean that petitioner was thereafter exempt from criminal prosecution; that
new informations may be filed by the Ombudsman should it find probable cause in the conduct of its
CRIMINAL LAW 1
(Sunday, Section 52) 52

preliminary investigation; that the filing of the complaint with the Presidential Commission on Good
Government (PCGG) in 1987 and the filing of the information with the Sandiganbayan in 1989
interrupted the prescriptive period; that the absence of the petitioner from the Philippines from 1986
until 2000 also interrupted the aforesaid period based on Article 91 of the Revised Penal Code.

For its part, the PCGG avers in its Comment[4] that, in accordance with the 1987 Constitution and RA
No. 6770 or the Ombudsman Act of 1989, the Omdudsman need not wait for a new complaint with a
new docket number for it to conduct a preliminary investigation on the alleged offenses of the
petitioner; that considering that both RA No. 3019 and Act No. 3326 or the Act To Establish Periods
of Prescription For Violations Penalized By Special Acts and Municipal Ordinances and to Provide
When Prescription Shall Begin To Run, are silent as to whether prescription should begin to run when
the offender is absent from the Philippines, the Revised Penal Code, which answers the same in the
negative, should be applied.

ISSUE:
Whether or not the offenses for which petitioner are being charged have already prescribed.

RULING:
YES. Rule 117 of the Rules of Court provides that the accused may, at any time before he enters his
plea, move to quash the complaint and information on the ground that the criminal action or liability
has been extinguished, which ground includes the defense of prescription considering that Article 89
of the Revised Penal Code enumerates prescription as one of those grounds which totally
extinguishes criminal liability. Indeed, even if there is yet to be a trial on the merits of a criminal case,
the accused can very well invoke the defense of prescription.
Thus, the question is whether or not the offenses charged in the subject criminal cases have
prescribed? We held in the case of Domingo v. Sandiganbayan that:

In resolving the issue of prescription of the offense charged, the following should be considered: (1)
the period of prescription for the offense charged; (2) the time the period of prescription starts to
run; and (3) the time the prescriptive period was interrupted.[

Section 11 of RA No. 3019 provides that all offenses punishable therein shall prescribe in 15 years.
Significantly, this Court already declared in the case of People v. Pacificador that:

It appears however, that prior to the amendment of Section 11 of R.A. No. 3019 by B.P. Blg. 195 which
was approved on March 16, 1982, the prescriptive period for offenses punishable under the said
statute was only ten (10) years. The longer prescriptive period of fifteen (15) years, as provided in
Section 11 of R.A. No. 3019 as amended by B.P. Blg. 195, does not apply in this case for the reason
that the amendment, not being favorable to the accused (herein private respondent), cannot be given
retroactive effect. Hence, the crime prescribed on January 6, 1986 or ten (10) years from January 6,
1976.

Thus, for offenses allegedly committed by the petitioner from 1962 up to March 15, 1982, the same
shall prescribe in 10 years. On the other hand, for offenses allegedly committed by the petitioner
during the period from March 16, 1982 until 1985, the same shall prescribe in 15 years.

As to when these two periods begin to run, reference is made to Act No. 3326 which governs the
computation of prescription of offenses defined by and penalized under special laws. Section 2 of Act
No. 3326 provides:
CRIMINAL LAW 1
(Sunday, Section 52) 53

SEC. 2. Prescription shall begin to run from the day of the commission of the violation of the law, and
if the same be not known at the time, from the discovery thereof and the institution of judicial
proceedings for its investigation and punishment.

The prescription shall be interrupted when proceedings are instituted against the guilty person, and
shall begin to run again if the proceedings are dismissed for reasons not constituting jeopardy.

Supreme Court ruled that the prescriptive period of the offenses herein began to run from the
discovery thereof or on May 8, 1987, which is the date of the complaint filed by the former Solicitor
General Francisco I. Chavez against the petitioner with the PCGG.

Section 2 of Act. No. 3326 is conspicuously silent as to whether the absence of the offender from the
Philippines bars the running of the prescriptive period. The silence of the law can only be interpreted
to mean that Section 2 of Act No. 3326 did not intend such an interruption of the prescription unlike
the explicit mandate of Article 91.

Article 10 of the Revised Penal Code provides:


ART. 10. Offenses not subject to the provisions of this Code. Offenses which are or in the future may
be punishable under special laws are not subject to the provisions of this Code. This Code shall be
supplementary to such laws, unless the latter should specially provide the contrary.

However, it must be pointed out that the suppletory application of the Revised Penal Code to special
laws, by virtue of Article 10 thereof, finds relevance only when the provisions of the special law are
silent on a particular matter as evident from the cases cited and relied upon in the Dissenting Opinion:

The term of prescription shall not run when the offender is absent from the Philippines.

In view of the foregoing, the applicable 10-and-15-year prescriptive periods in the instant case, were
not interrupted by any event from the time they began to run on May 8, 1987. As a consequence, the
alleged offenses committed by the petitioner for the years 1963-1982 prescribed 10 years from May
8, 1987 or on May 8, 1997. On the other hand, the alleged offenses committed by the petitioner for
the years 1983-1985 prescribed 15 years from May 8, 1987 or on May 8, 2002.

Therefore, when the Office of the Special Prosecutor initiated the preliminary investigation of
Criminal Case Nos. 13406-13429 on March 3, 2004 by requiring the petitioner to submit his counter-
affidavit, the alleged offenses subject therein have already prescribed. Indeed, the State has lost its
right to prosecute petitioner for the offenses subject of Criminal Case Nos. 28031-28049 pending
before the Sandiganbayan and Criminal Case Nos. 04-23185704-231860 pending before the Regional
Trial Court of Manila.

WHEREFORE, premises considered, petitioners Motion for Reconsideration is GRANTED. Criminal


Case Nos. 28031-28049 pending before the Sandiganbayan and Criminal Case Nos. 04-23185704-
231860 pending before the Regional Trial Court of Manila are all hereby ordered DISMISSED
CRIMINAL LAW 1
(Sunday, Section 52) 54

PEOPLE V. SYLVESTRE AND ATIENZA


G.R. NO. 35748, 14 DEC. 1931, 56 PHIL. 353

FACTS: Romana Silvestre is the wife of Domingo Joaquin by his second marriage. Romana cohabited
with codefendant Martin Atienza. Domingo filed a sworn complaint for adultery. After being arrested
and released on bail, the two defendants begged the municipal president of Paombong to speak to
the complainant and urge him to withdraw the complaint
The two accused bound themselves to discontinue cohabitation and promised not to live again in
Masocol (Atienza signed the promise). On May 20, 1930, Domingo Joaquin filed a motion for the
dismissal of his complaint and the justice of the peace dismissed the adultery case. The accused left
Masocol and wen to live in Santo Niño, in Paombong. About November 20, 1930: Romana met her
son by her former marriage, Nicolas de la Cruz, in Santo Niño and followed him home to Masocol
(under the pretext of asking him for some nipa leaves)
Martin Atienza, who continued to cohabit with Romana, followed her and lived in the home of Nicolas
On the night of November 25, 1930, while Nicolas, his wife Antonia, and the appellants were gathered
after supper, Martin told Nicolas and Antonia to take their furniture out of the house because he was
going to set fire to it. He said that that was the only way he could be revenged upon the people of
Masocol who, he said, had instigated the charge of adultery against him and Romana. Martin was
armed with a pistol so no one dared say anything to him. Nicolas and Antonia went to ask for help
but were too late. The fire destroyed about 48 houses and witnesses saw Martin and Romana leaving
the house on fire. The Court of First Instance of Bulacan convicted Martin and Romana of arson.
Martin was convicted as principal by direct participation (14 years, 8 months, and 1 day of cadena
temporal). Romana was convicted as accomplice (6 years and 1 day of presidio mayor).
The court-appointed counsel for the accused-appellant prays for the affirmance of the CFI decision
with regard to Martin, but assigns errors with reference to Romana:
The lower court erred in convicting Romana as acoomplice
The court erred in not acquitting Romana upon ground of insufficient evidence, or at least, of
reasonable doubt

ISSUE:
Whether or not Romana can be convicted as accomplice

RULING:
No.
Art. 14 of the Penal Code, in connection with Art. 13 defines an accomplice to be one who does not
take a direct part in the commission of the act, who does not force or induce other to commit it, nor
cooperates in the commission of the act by another act without which it would not have been
accomplished, yet cooperates in the execution of the act by previous or simultaneous actions.
In the case of Romana: there is no evidence of moral or material cooperation and none of an
agreement to commit the crime in question. Her mere presence and silence while they are
simultaneous acts, do not constitute cooperation, for it does not appear that they encouraged or
nerved Martin Atienza to commit the crime of arson; and as for her failure to give the alarm, that
being a subsequent act it does not make her liable as an accomplice.
Mere passive presence at the scene of another's crime, mere silence and failure to give the alarm,
without evidence of agreement or conspiracy, do not constitute the cooperation required by Art. 14
of the Penal Code for complicity in the commission of the crime witnessed passively, or with regard
to which one has kept silent
Decision is affirmed with reference to Martin Atienza, reversed with reference to Romana Silvestre,
who is acquitted.
CRIMINAL LAW 1
(Sunday, Section 52) 55

PEOPLE V. TALINGDAN
L-32126, 6 JULY 1978, 84 SCRA 19

FACTS:
Teresa Domogma was the supposed wife of the deceased Bernardo Bagabag
No certificate or any other proof of their marriage could be presented by the prosecution
They lived with their children in Sobosob, Salapadan, Abra
Their relationship had been strained and beset with troubles for Teresa had deserted her family
home a couple of times and each time Bernardo took time out to look for her
On 2 different occasions, appellant Nemesis Talingdan has visited Teresa in their house while
Bernardo was out at work, and during those visits Teresa had made Corazon, their then 12-year old
daughter to go down the house and leave them
Bernardo had gotten wind that an illicit relationship was going on between Talingdan and Teresa
About a month before Bernardo was killed, Teresa had again left their house and did not come back
for a period of more than 3 weeks, and Bernardo came to know later that she and Talingdan were
seen together in the town of Tayum Abra during that time
Just two days before Bernardo was killed (Thursday), Bernardo and Theresa had a violent quarrel;
Bernardo slapped Theresa several times, resulting in Theresa seeking the help of the police
Accused Talingdan, a policeman, came armed to the vicinity of Bernardo's house and called him to
come down; Bernardo ignored him; Talingdan instead left and warned Bernardo that someday he
would kill him
On Saturday, June 24, 1967, Bernardo was gunned down in his house
The defendants' and Corazon's accounts of what happened had variations
Corazon's version:
Friday morning: Corazon was in a creek to wash clothes. She saw her mother Teresa meeting with
Talingdan and their co-appellants Magellan Tobias, Augusto Berras, and Pedro Bides in a small hut
owned by Bernardo
She heard one of them say "Could he elude a bullet"
When Teresa noticed Corazon, she shoved her away saying "You tell your father that we will kill him"
Saturday, after sunset: Corazon was cooking food for supper when she saw her mother go down the
house to go to the yard where she again met with the other appellants.
She noted the long guns the appellants were carrying.
Teresa came back to the house and proceeded to her room.
Corazon informed Bernardo, who was then working on a plow, about the presence of persons
downstairs, but Bernardo paid no attention
Bernardo proceeded to the kitchen and sat himself on the floor near the door
He was suddenly fired upon form below the stairs of the batalan
The four accused climbed the stairs of the batalan and upon seeing that Bernardo was still alive,
Talingdan and Tobias fired at him again
Bides and Berras did not fire at that precise time but when Corazon tried to call for helo, Bides warned
her that he will kill her if she calls for help
Teresa came out of her room and when Corazon informed her that she recognized the killers, the former
threatened to kill the latter if she reveals the matter to anyone
The defendants'' version:
Teresa loved Bernardo dearly, they never quarreled, and her husband never maltreated her.
Teresa came to know Talingdan only when the latter became a policeman in Sallapadan; an illicit
relationship never existed between them
Talingdan was not in Sallapadan at the time of the killing on June 24; he escorted the Mayor in Bangued
from June 22 to June 26
CRIMINAL LAW 1
(Sunday, Section 52) 56

Tobias, Bides, and Berras claimed to be in the house of one Mrs. Bayongan in Sallapadan, 250-300
meters from the place of the killing

ISSUE:
Whether or not Teresa Domogma is an accessory to Bernardo's murder

RULING:
It is contended that there is no evidence proving that she actually joined in the conspiracy to kill her
husband because there is no showing of actual cooperation on her part with co-appellants in their
culpable acts that led to his death
It is claimed that what is apparent is "mere cognizance, acquiescence or approval thereof on her part,
which it is argued is less than what is required for her conviction as a conspirator
Holding:
Yes. She is an accessory to Bernardo's murder.
Ratio:
Note: The court believed Corazon's testimony.
It is true that proof of her direct participation in the conspiracy is not beyond reasonable doubt; she
cannot have the same liability as her co-appellants. She had no hand in the actual shooting. It is also
not clear if she helped directly in the planning and preparation thereof. But the court is convinced
that she knew it was going to be done and did not object.
There is in the record morally convincing proof that she is at the very least an accessory to the offense
committed.
She did not only order her daughter not to reveal what she knew to anyone, she also claimed to have
no suspects in mind when the peace officers came into their house later to investigate
Whereas before the actual shooting she was more or less passive in her attitude regarding the
conspiracy, after Bernardo was killed, she became active in her cooperation with her co-appellants
These acts constitute "concealing or assisting in the escape of the principal in the crime"

Male appellants sentenced to death. Guilty beyond reasonable doubt is Teresa Domogma, sentenced
to suffer the indeterminate penalty of 5 years of prision correccional as minimum to 8 years of prision
mayor as maximum.

MANUEL V. PEOPLE
G.R. NO. 165842, 29 NOVEMBER 2005

FACTS:
Eduardo P. Manuel, herein petitioner, was first married to Rubylus Gaña on July 18, 1975, who,
according to the former, was charged with estafa in 1975 and thereafter imprisoned and was never
seen again by him after his last visit. Manuel met Tina B. Gandalera in January 1996 when the latter
was only 21 years old. Three months after their meeting, the two got married through a civil wedding
in Baguio City without Gandalera’s knowledge of Manuel’s first marriage. In the course of their
marriage, things got rocky and Gandalera learned that Eduardo was in fact already married when he
married him. She then filed a criminal case of bigamy against Eduardo Manuel. The latter’s defense
being that his declaration of “single” in his marriage contract with Gandalera was done because he
believed in good faith that his first marriage was invalid and that he did not know that he had to go
to court to seek for the nullification of his first marriage before marrying Tina. The Regional Trial
Court ruled against him sentencing him of imprisonment of from 6 years and 10 months to ten years,
and an amount 0f P200,000.00 for moral damages.
Eduardo appealed the decision to the CA where he alleged that he was not criminally liable for bigamy
because when he married the private complainant, he did so in good faith and without any malicious
CRIMINAL LAW 1
(Sunday, Section 52) 57

intent. The CA ruled against the petitioner but with modification on the RTC’s decision. Imprisonment
was from 2 years, months and 1 day to ten years. Pecuniary reward for moral damages was affirmed.
Hence, this petition.

ISSUES:
1. Whether or not the Court of Appeals committed reversible error of law when it ruled that
petitioner’s wife cannot be legally presumed dead under Article 390 of the Civil Code as there was no
judicial declaration of presumptive death as provided for under Article 41 of the Family Code.
2. Whether or not the Court of Appeals committed reversible error of law when it affirmed the award
of Php200,000.00 as moral damages as it has no basis in fact and in law.

RULING:
1. The petition is denied for lack of merit. The petitioner is presumed to have acted with malice or
evil intent when he married the private complainant. As a general rule, mistake of fact or good faith
of the accused is a valid defense in a prosecution for a felony by dolo; such defense negates malice or
criminal intent. However, ignorance of the law is not an excuse because everyone is presumed to
know the law. Ignorantia legis neminem excusat. Where a spouse is absent for the requisite period,
the present spouse may contract a subsequent marriage only after securing a judgment declaring the
presumptive death of the absent spouse to avoid being charged and convicted of bigamy; the present
spouse will have to adduce evidence that he had a well-founded belief that the absent spouse was
already dead. Such judgment is proof of the good faith of the present spouse who contracted a
subsequent marriage; thus, even if the present spouse is later charged with bigamy if the absentee
spouse reappears, he cannot be convicted of the crime.
The court ruled against the petitioner.
2. The Court rules that the petitioner’s collective acts of fraud and deceit before, during and after his
marriage with the private complainant were willful, deliberate and with malice and caused injury to
the latter. The Court thus declares that the petitioner’s acts are against public policy as they
undermine and subvert the family as a social institution, good morals and the interest and general
welfare of society. Because the private complainant was an innocent victim of the petitioner’s perfidy,
she is not barred from claiming moral damages. Considering the attendant circumstances of the case,
the Court finds the award of P200,000.00 for moral damages to be just and reasonable.

PEOPLE V. PUNO
G.R. NO. 97471, 17 FEB. 1993, 219 SCRA 85

FACTS:
January 13, 1988 in QC, at around 5:00 pm: the accused Isabelo Puno, who is the personal driver of
Mrs. Sarmiento's husband (who was then away in Davao purportedly on account of local election
there) arrived at Mrs. Sarmiento's bakeshop in Araneta Ave, QC
He told Mrs. Sarmiento that her own driver Fred had to go to Pampanga on an emergency so Isabelo
will temporarily take his place. When it was time for Mrs. Sarmiento to go home to Valle Verde in
Pasig, she got into her husband's Mercedes Benz with Isabelo driving
After the car turned right on a corner of Araneta Ave, it stopped and a young man, accused Enrique
Amurao, boarded the car beside the driver Enrique pointed a gun at Mrs. Sarmiento as Isabelo told
her that he needs to "get money" from her Mrs. Sarmiento had P7,000 on her bag which she handed
to the accused but the accused said that they wanted P100,000 more the car sped off north towards
the North superhighway where Isabelo asked Mrs. Sarmiento to issue a check for P100,000 Mrs.
Sarmiento drafted 3 checks: two P30,000 checks and one P40,000 check Isabelo then turned the car
around towards Metro Manila; later, he changed his mind and turned the car again towards
CRIMINAL LAW 1
(Sunday, Section 52) 58

Pampanga. According to her, Mrs. Sarmiento jumped out of the car then, crossed to the other side of
the superhighway and was able to flag down a fish vendor's van, her dress had blood because
according to her, she fell down on the ground and was injured when she jumped out of the car. The
defense does not dispute the above narrative of the complainant except that according to Isabelo, he
stopped the car at North Diversion and freely allowed Mrs. Sarmiento to step out of the car. He said
he even slowed the car down as he drove away, until he saw that his employer had gotten a ride
He claimed that she fell down when she stubbed her toe while running across the highway.

ISSUES:
Whether or not the accused can be convicted of kidnapping for ransom as charged
Whether or not the said robbery can be classified as "highway robbery" under PD No. 532 (Anti-
Piracy and Anti-Highway Robbery Law of 1974)

RULING:
NO. There is no showing whatsoever that appellants had any motive, nurtured prior to or at the
time they committed the wrongful acts against complainant, other than the extortion of money
from her under the compulsion of threats or intimidation.
For this crime to exist, there must be indubitable proof that the actual intent of the malefactors was
to deprive the offended party of her liberty
In the case, the restraint of her freedom of action was merely an incident in the commission of
another offense primarily intended by the offenders
This does not constitute kidnapping or serious illegal detention
NO. Jurisprudence reveals that during the early part of the American occupation of our country,
roving bands were organized for robbery and pillage and since the then existing law against robbery
was inadequate to cope with such moving bands of outlaws, the Brigandage Law was passed (this is
the origin of the law on highway robbery)
PD No. 532 punishes as highway robbery only acts of robbery perpetrated by outlaws
indiscriminately against any person or persons on Philippine highways and not acts of robbery
committed against only a predetermined or particular victim
The mere fact that the robbery was committed inside a car which was casually operating on a
highway does not make PD No 532 applicable to the case
This is not justified by the accused's intention
Accused-appellants convicted of robbery (indeterminate sentence of 4 years and 2 months or prision
correccional, as minimum, to 10 years of prision mayor. Accused to pay Mrs. Sarmiento P7,000 as
actual damages and P20,000 as moral damages.)

PEOPLE V. DELIM
G.R. NO. 142773, 28 JANUARY 2003

FACTS:
It is due to the automatic review of the decision of the RTC Branch 46 (Urdaneta City) finding the
appellants, guilty beyond reasonable doubt and sentencing them to death for the murder of Modesto
Bantas.

Appellants pleaded not guilty to the charge. The appellants and victim are “related” for modesto is an
adopted son of their father. On January 23,1999 Marlon, Robert and Ronald Delim charged into the
house and poked a gun at modesto and herded him outside the house. Leon and Manuel Delim both
armed stayed put and made sure that randy and rita stayed put.
CRIMINAL LAW 1
(Sunday, Section 52) 59

Modesto's lifeless body was then found on January 25, 1999. Marlon, Ronald, and Leon used denial
and alibi as their evidence against the charge.
*alibis are the weakest of all defenses since it is easy to contrive and difficult to disprove

ISSUES:
Is conspiracy and treachery present in this case to ensure that murder can be the crime?

RULING:
Yes there is:
CONSPIRACY- is determined when two or more persons agree to commit a felony and decide to
commit it. Conspiracy must be proven with the same quantum of evidence as the felony itself, more
specifically by proof beyond reasonable doubt. It is not essential that there be proof as to the
existence of a previous agreement to commit a crime. It is sufficient if, at the time of commission of
the crime, the accused had the same purpose and were united in its executed.
appellants acted in unison when they abducted Modesto. So their acts were synchronized and
executed with precision evincing a preconceived plan to kill Modesto

There is no:
TREACHERY- there is treachery when the offender commits any of the crimes against person,
employing means, methods, or forms in the execution thereof which tend directly and especially to
insure its execution, without risk to himself arising from the defense which the offended party might
make.
For it to be appreciated prosecution needs to prove:
a. employment of means of execution which gives the person no opportunity to defend himself
b. the means of execution is deliberately and consciously adopted in the appellants case there are no
evidence to the particulars on how Modesto was assaulted and killed and this in fact does mean that
treachery cannot be proven since it cannot be presumed that modesto was defenseless during the
time that he was being attacked and shot at by the appellants.
Sheer numbers by the appellants when they attacked modesto does not constitute proof that the
three took advantage of their numerical superiority and their handguns when Modesto was shot and
stabbed.

HELD:
APPELLANTS ARE GUILTY BEYOND REASONABLE DOUBT OF THE FELONY OF HOMICIDE (THE
DECISION OF THE LOWER COURTS WERE MODIFIED TO LOWER THE CRIME FROM MURDER TO
HOMICIDE)

US VS. ELVINA, 24 PHIL. 230 (1913)

RECUERDO VS. PEOPLE


G.R. NO. 168217, 27 JUNE 2006

FACTS:
Petitioner Recuerdo, a dentist, was charged with the crime of Estafa under Art. 315 of Revised Penal
Code for, with intent to gain and by means of deceit, false pretenses and fraudulent manifestations,
and pretending to have sufficient funds with the Unitrust Makati Commercial Center Branch, PCI
Bank Makati-De La Rosa Branch, and Prudential Bank Legaspi Village Branch, did willfully, unlawfully
and feloniously prepare, draw, make and issue checks amounting to P132,000, P78,000, and
P600,000, to complaining witness Yolanda G. Floro, who is engaged in the business of buying and
selling of jewelry, as payment for jewelry she obtained from the said complainant, knowing fully well
CRIMINAL LAW 1
(Sunday, Section 52) 60

at the time the checks were issued that her representations were false for she had no sufficient funds
in the said bank, so much that upon presentment of the said checks with the said bank for
encashment, the same were dishonored and refused payment for having been drawn against an
“Account Closed”, and in spite of repeated demands to deposit with the said bank, the said accused
failed and refused to do so.
Recuerdo argued that her act of issuing the dishonored checks does not constitute the offense of
Estafa considering that the subject checks were not issued and delivered to Floro simultaneous to the
purchase of the pieces of jewelry, but only several days thereafter, when she had already thoroughly
examined the jewelry and is fully satisfied of its fine quality; that out of the 17 subject checks, nine
were honored by the drawee banks; that she made partial payments of the amounts of the subject
checks while the case was pending in the CA, contrary to the findings of the courts that she acted with
deceit when she drew and delivered the checks.

ISSUE:
Whether or not petitioner Recuerdo committed the crime of estafa.

RULING:
Yes, Recuerdo committed the crime of estafa.
Estafa through false pretense or fraudulent act under Paragraph 2(d) of Article 315 of the Revised
Penal Code, as amended by Republic Act No. 4885, is committed as follows:
By postdating a check, or issuing a check in payment of an obligation when the offender had
no funds in the bank, or his funds deposited therein were not sufficient to cover the amount
of the check. The failure of the drawer of the check to deposit the amount necessary to cover
his check within three (3) days from receipt of notice from the bank and/or the payee or
holder that said check has been dishonored for lack or insufficiency of funds shall be prima
facie evidence of deceit constituting false pretense or fraudulent act.
The essential elements of the felony are: (1) a check is postdated or issued in payment of an
obligation contracted at the time it is issued; (2) lack or insufficiency of funds to cover the
check; and (3) damage to the payee thereof. It is criminal fraud or deceit in the issuance of a check
which is made punishable under the Revised Penal Code, and not the non-payment of a debt. Deceit
is the false representation of a matter of fact whether by words or conduct by false or misleading
allegations or by concealment of that which should have been disclosed which deceives or is intended
to deceive another so that he shall act upon it to his legal injury. Concealment which the law denotes
as fraudulent implies a purpose or design to hide facts which the other party ought to have. The
postdating or issuing of a check in payment of an obligation when the offender had no funds in the
bank or his funds deposited therein are not sufficient to cover the amount of the check is a false
pretense or a fraudulent act.
Petitioner’s defense of good faith is even belied by the evidence of the prosecution and her own
evidence. When the postdated checks issued by petitioner were dishonored by the drawee banks and
the private complainant made demands for her to pay the amounts of the checks, she intransigently
refused to pay; she insisted that she issued and delivered the postdated checks to the private
complainant after the subject pieces of jewelry had been delivered to her. Petitioner never offered to
pay the amounts of the checks after she was informed by the private complainant that they had been
dishonored by the drawee banks. It was after the CA promulgated its decision affirming the decision
of the trial court, that petitioner made several payments to the private complainant; however, there
is no showing as to which checks they were made in payment for. In fine, it was the spectre of a long
prison term which jolted petitioner into making remittances to the private complainant, after the CA
affirmed the decision of the trial court and increased the penalty meted on her, and not because she
had acted in good faith in her transactions with the private complainant. To reiterate, petitioner
rejected the demands of the private complainant to pay the amounts of the dishonored checks.
CRIMINAL LAW 1
(Sunday, Section 52) 61

While it is true that nine of the 17 postdated checks petitioner issued and delivered to the private
complainant were honored by the drawee banks, such a circumstance is not a justification for her
acquittal of the charges relative to the dishonored checks. The reimbursement or restitution to the
offended party of the sums swindled by the petitioner does not extinguish the criminal liability of the
latter. Estafa is a public offense which must be prosecuted and punished by the State on its own
motion even though complete reparation had been made for the loss or damage suffered by the
offended party. The consent of the private complainant to petitioner’s payment of her civil liability
pendente lite does not entitle the latter to an acquittal. Subsequent payments does not obliterate the
criminal liability already incurred. Criminal liability for estafa is not affected by a compromise
between petitioner and the private complainant on the former’s civil liability.
IN LIGHT OF ALL THE FOREGOING, the petition is DENIED. The Decision and Resolution of the Court of
Appeals are AFFIRMED. No costs.

US VS. CATOLICO
18 PHIL. 504 (1911)

FACTS:
The justice of peace of Cagayan had before him 16 separate civil cases initiated by Juan Canillas for
damages resulting from breach of contract. All cases were decided in favour of Canillas and all
defendants appealed the decision and deposited Php 16 and a bond of Php 50 as required by law. It
appears that the sureties of the bond were insolvent and new bonds were not presented on the
extension given. Canillas appealed. The justice of peace dismissed the appeals and ordered the sm of
money attached and delivered to Canillas in satisfaction of the judgment. The judge was prosecuted
for malversation of funds.

ISSUE:
Whether or not the defendant is guilty of felony.

RULING:
Judgment of conviction is reversed and defendant ordered to be discharged from custody.
The judge decided in good faith under the belief that he was acting judiciously and correctly. It was a
result of erroneous exercise of judicial function and not an intention to deprive any person of his
property feloniously. He acted that debts might be paid t those who they are legally and justly due
and not to enrich himself or another by criminal misappropriation. It was a mistake not a crime.

US V. AH CHONG
G.R. NO. 5272, 19 MARCH 1910, 15 PHIL. 488

FACTS:
The defendant, Ah Chong, was employed as a cook in one of the Officers’ quarters at Fort McKinley,
Rizal Province. Together living with him in the said quarters was the deceased, Pascual Gualberto,
who was employed as a houseboy. There had been several robberies in Fort McKinley prior to the
incident thus prompting the defendant and his roommate to reinforce the flimsy hook used to lock
the door of their room by placing a chair against it. The defendant and the deceased had an
understanding that when either returned at night, he should knock on the door and say his name. On
the night of Aug. 14, 1908, Ah Chong, who was alone in his room, was awakened by someone trying
to force open the door of the room. The defendant called out twice, asking the identity of the person
but heard no answer. Fearing that the intruder was a robber or a thief, the defendant called out that
he would kill the intruder if he tried to enter. At that moment, the door was forced open and the
CRIMINAL LAW 1
(Sunday, Section 52) 62

defendant was struck first above the knee by the edge of the chair. Because of the darkness of the
room, the defendant thought he was being hit by the intruder and tried to defend himself by striking
wildly at the intruder using a common kitchen knife which he kept under his pillow. It turned out
that the said
intruder was actually the defendant’s roommate, Pascual Gualberto. The roommate was
brought to the military hospital where he died from the effects of the wound the following day.

ISSUE:
WON the defendant was criminally liable for committing a felony.

RULING:
Defendant was not criminally liable and exonerated. In order for mistake of fact to be held as a valid
defense, there has to be several requisites. One, that the act done would have been lawful had the
facts been as the accused believed them to be. Two, that the intention of the accused in performing
the act should be lawful, and lastly, that the mistake must be without fault or carelessness on the part
of the accused. In the case at bar, had the intruder been a robber as the defendant believed him to be,
then Ah Chong acted in good faith, without malice or criminal intent, and would have been wholly
exempt from criminal liability and that he cannot be said to have been guilty of negligence or
recklessness.

PEOPLE V. OANIS
G.R. NO. 47722, 27 JULY 1943, 74 PHIL. 257

FACTS:
Antonio Oanis and Alberto Galanta were instructed to arrest a notorious criminal and escaped
convict, Anselmo Balagtas, and if overpowered, to get him dead or alive. They went to the suspected
house then proceeded to the room where they saw the supposedly Balagtas sleeping with his back
towards the door. Oanis and Galanta simultaneously or successively fired at him which resulted to
the victim’s death. The supposedly Balagtas turned out to be Serepio Tecson, an innocent man.

ISSUE:
1. WON Oanis and Galanta incur no liability due to innocent mistake of fact in the honest performance
of their official duties.
2. WON Oanis and Galanta incur no criminal liability in the performance of their duty.

RULING:
1. No. Innocent mistake of fact does not apply to the case at bar. “Ignorance facti excusat” applies only
when the mistake is committed without fault or carelessness. The fact that the supposedly suspect
was sleeping, Oanis and Galanta could have checked whether it is the real Balagtas.
2. No. Oanis and Galanta are criminally liable. A person incurs no criminal liability when he acts in
the fulfillment of a duty or in the lawful exercise of a right or office. There are 2 requisites to justify
this: (1) the offender acted in teh perfomance of a duty or in the lawful exercise of a right or office,
(2) that the injury or offense committed be the necessary consequence of the due performance of
such duty or the lawful exercise of such right or office. In this case, only the first requisite is present.
CRIMINAL LAW 1
(Sunday, Section 52) 63

PADILLA V. DIZON
AC NO. 3086, 23 FEB. 1988, 158 SCRA 127

FACTS:
Respondent Baltazar R. Dizon acquitted, in his decision, the tourist and accused, Lo Chi Fai, saying
that Lo Chi Fai had no willful intention to violate the law. He also directed the release to Lo Chi Fai of
at least the amount of US$3,000.00 under Central Bank Circular No. 960.
Lo Chi Fai was caught by Customs guard at the Manila International Airport while attempting to
smuggle foreign currency and foreign exchange instruments out of the country.
An information was filed against Lo Chi Fai with the RTC for violation of Sec. 6, Central Bank Circular
No. 960 with a penal sanction provided by Sec. 1, PD NO. 1883.
Sec. 6, Central Bank Circular No. 960 provides that no person shall take out or transmit or attempt to
take out or transmit foreign exchange in any form out of the Philippines without an authorization by
the Central Bank. Tourists and non-resident visitors may take out or send out from the Philippine
foreign exchange in amounts not exceeding such amounts of foreign exchange brought in by them.
Tourists and non-resident temporary visitors bringing with them more than US$3,000.00 or its
equivalent in other foreign currencies shall declare their foreign exchange in the form prescribed by
the Central Bank at points of entries upon arrival in the Philippines.
Sec. 1, P.D. No. 1883 provides that any person who shall engage in the trading or purchase and sale
of foreign currency in violation of existing laws or rules and regulations of the Central Bank shall be
guilty of the crime of blackmarketing of foreign exchange and shall suffer the penalty of reclusion
temporal (minimum of 12 years and 1 day and maximum of 20 years) and a fine of no less than
P50,000.00.
At the trial, Lo Chi Fai tried to establish that he was a businessman from Hongkong, that he had come
to the Philippines 9 to 10 times to invest in business in the country with his business associates, and
that he and his business associates declared all the money they brought in and all declarations were
handed to and kept by him.
Because of the revolution taking place in Manila during that time, Lo Chi Fai was urged by his business
associates to come to Manila to bring the money out of the Philippines.
Commissioner of Customs, Alexander Padilla, then filed a complaint against Baltazar R. Dizon for
acquitting Lo Chi Fai.

ISSUE:
Whether or not respondent Baltazar R. Dizon is guilty of gross incompetence or gross ignorance of
the law in holding that the accused, Lo Chi Fai, for violation of Central Bank Circular No. 960, the
prosecution must establish that the accused had the criminal intent to violate the law.

RULING:
YES. Baltazar R. Dizon ignored the fact that the foreign currency and foreign currency instruments
found in the possession of Lo Chi Fai when he was apprehended at the airport and the amounts of
such foreign exchange did not correspond to the foreign currency declarations presented by Lo Chi
Fai at the trial, and that these currency declarations were declarations belonging to other people.
In invoking the provisions of the Central Bank Circular No. 960 to justify the release of US$3,000.00
to Lo Chi Fai, Baltazar R. Dizon again diplayed gross incompetence and gross ignorance of law. There
is nothing in the Central Bank Circular which could be taken as authority for the trial court to release
the said amount of US Currency to Lo Chi Fai.
CRIMINAL LAW 1
(Sunday, Section 52) 64

MAGNO V. CA
G.R. NO. 96132,26 JUNE 1992, 210 SCRA 475

FACTS:
Oriel Magno, lacking fund in acquiring complete set of equipment to make his car repair shop
operational, approached Corazon Teng, Vice President of Mancor Industries.
VP Teng referred Magno to LS Finance and Management Corporation, advising its Vice President, Joey
Gomez, that Mancor was willing to supply the pieces of equipment needed if LS Finance could
accommodate Magno and and provide him credit facilities.
The arrangement went on requiring Magno to pay 30% of the total amount of the equipment as
warranty deposit but Magno couldn't afford to pay so he requested VP Gomez to look for third party
who could lend him that amount.
Without Magno's knowledge, Corazon was the one who provided that amount.
As payment to the equipment, Magno issued six checks, two of them were cleared and the rest had
no sufficient fund.
Because of the unsuccessful venture, Magno failed to pay LS Finance which then pulled out the
equipment.
Magno was charged of violation of BP Blg. 2 (The Bouncing Checks Law) and found guilty.

ISSUE:
Whether or not Magno should be punished for the issuance of the checks in question.

RULING:
NO. To charge Magno for the refund of a warranty deposit which he did not withdraw as it was not
his own account, it having remained with LS Finance, is to even make him pay an unjust debt since
he did not receive the amount in question. All the while, said amount was in the safekeeping of the
financing company which is managed by the officials and employees of LS Finance.

GARCIA V. CA
G.R. NO. 157171 14 MAR. 2006, 484 SCRA 617

FACTS:
On May 11, 1995, within the canvassing period of 1995 senatiorial elections, Aquilino Pimintel, Jr.,
was informed that Arsenia Garcia (Arsenia), along with her co-conspirators, willfully and unlawfully
decreased the number of votes of the candidate from 6,998 to 1921 votes.
Pimintel filed a complaint against Asenia and her co-conspirators. All the accused was acquited due
to lack of evidence except for Arsenia who was found guilty of the crime defined under Republic Act
6646, Section 27 (b) for decreasing the votes of Senator Pimentel in the total of 5,034 and in relation
to BP Blg. 881.
Petitioner appealed to CA which also affirmed the decision of the RTC.
Arsenia appealed to SC, contending that the judgment of CA is erroneous and there was no motive on
her part to reduce the votes of private complainant.
Respondent on the other hand contends that good faith is not a defense in the violation of an election
law, which falls under the class of mala prohibita.

ISSUES:
(1) Whether or not a violation of Section 27(b) of Rep. Act No. 6646, classified under mala in se.
(2) Whether or not good faith and lack of criminal intent be valid defenses?
CRIMINAL LAW 1
(Sunday, Section 52) 65

HELD:
(1) YES. Section 27(b) of Republic Act No. 6646 provides: Any member of the board of election
inspectors or board of canvassers who tampers, increases, or decreases the votes received by a
candidate in any election or any member of the board who refuses, after proper verification and
hearing, to credit the correct votes or deduct such tampered votes.
Clearly, the acts prohibited in Section 27(b) are mala in se. For otherwise, even errors and mistakes
committed due to overwork and fatigue would be punishable.
(2) NO. Public policy dictates that extraordinary diligence should be exercised by the members of the
board of canvassers in canvassing the results of the elections. Any error on their part would result in
the disenfranchisement of the voters. The Certificate of Canvass for senatorial candidates and its
supporting statements of votes prepared by the municipal board of canvassers are sensitive election
documents whose entries must be thoroughly scrutinized.
The instant petition is DENIED. The assailed Decision of the Court of Appeals sustaining petitioners
conviction but increasing the minimum penalty in her sentence to one year instead of six months
is AFFIRMED.

PEOPLE VS. HASSAN


G.R. NO. L-68969, 22 JAN. 1998

Nature: Appeal from a decision of the RTC of Zamboanga City finding the accused guilty beyond
reasonable doubt of the crime of murder and sentenced to reclusion peretua.

FACTS: July 23, 1981, at around 7pm JOSE SAMSON was a backrider on the motorcycle of RAMON
PICHEL JR. when they went to buy mangoes at the Fruit Paradise near the Barter Trade Zone in
Zamboanga City. SAMSON saw a person stab PICHEL only once while he was parked 2-3 meters away.
After stabbing, the suspect fled to PNB. SAMSON rushed the victim to the General Hospital where the
latter died. In the hospital, the witness was interrogated as to the suspect’s description, who
according to him was had semi-long hair, wearing white polo short-sleeved shirt, maong pants,
standing 5’5’’ and with a dark complexion. According to SAMSON, he only knows the suspect by face
and not by name. At Funeraria La Merced, police brought the accused – alone, for identification where
the witness positively identified him as the killer.

RULING: Decision reversed; accused is acquitted.

Evidence of the prosecution does not satisfy quantum of proof—beyond reasonable doubt. Value
judgment must not be separated from the constitutionally guaranteed presumption of innocence.
Prosecution’s evidence is weak and unconvincing. Expert testimony of the medico-legal officer
(DR.VALENTIN BERNALEZ) contradicted on material points of the lone witness. He found two stab
wounds, (chest and at the left arm posterior), the nature of the wounds indicating they were inflicted
while the suspect was in front of the victim.

The investigation conducted by the police was not satisfactory. The lone presentation of HASSAN to
SAMSON at the funeral parlor violated the accused rights to counsel in all the stages of investigation
into the commission of a crime. Motive is essential when there is doubt as to the identity of the
culprit.
CRIMINAL LAW 1
(Sunday, Section 52) 66

PEOPLE VS. TEMBLOR


G.R. NO. L-66884, 28 MAY 1988

Nature: Appeal from the judgment of CFI of Agusan del Norte and Butuan City convicting VICENTE
TEMBLOR alias “Ronald” for murder sentencing him to the penalty of reclusion perpetua.

FACTS: On Dec. 30, 1980, at around 7:30 pm in Brgy. Talo-ao, Buenavista, Agusan del Sur, TEMBLOR
bought a half-pack of Hope cigarette from the store of JULIUS CAGAMPANG. While the latter was
opening the pack, a gun shot was heard and CAGAMPANG fell to the floor with a gunshot wound in
the head. The accused, together with another, barged into the room, demanding the firearms of the
victim. Before fleeing with the victim’s .38 caliber gun, TEMBLOR filed two more shots.

On Aug. 1981, during the mass surrender of dissidents, TEMBLOR surrendered to Mayor Dick
Carmona. On Nov. 26, 1981, he was arrested by the Buenavista Police at the public market and
detained at the municipal jail where he was seen by VICTORIA CAGAMPANG, the victim’s widow and
was positively identified.

TEMBLOR’s defense was an alibi: on the said date, he and his father had been in the house of
SILVERIO PEROL in Brgy. Camagong.

ISSUE: W o N motive is essential in convicting Temblor

HELD: Decision of lower court affirmed; civil indemnity increased to P30,000.

Motive is not essential when culprit has been identified. Fact that accused has knowledge of the
deceased’s firearm is sufficient enough for motive. The fleeing of the accused after killing and taking
CAGAMPANG’s firearm implies admission of guilt.

Accused’s alibi cannot prevail over the positive identification of the witness who had no base motive
to accuse him of the crime. In order for alibi to be acceptable as a defense, it is not enough for the
appellant to be elsewhere when the crime was committed, but it must be proven beyond reasonable
doubt that it was physically impossible for him to be at the scene of the crime. Nasipit is accessible to
Talo-ao by jeep or tricycle for 15 to 20 minutes.

AVECILLA VS. CA
G.R. NO. 46370, 2 JUNE 1992

This is a petition for review on certiorari of the decision dated December 20, 1976 of the Court of
Appeals in CA-G.R. No. 16628-CR entitled "People of the Philippines v. Antonio Avecilla" modifying
the decision of July 16, 1973 of the then Court of First Instance of Rizal, Branch 1 at Pasig finding the
accused-petitioner guilty of simple theft, by convicting the accused-petitioner instead, of qualified
theft and imposing on him accordingly, a higher penalty.
The accused-petitioner, Antonio Avecilla and one Juana Doe were charged before the said lower court
of the crime of theft, allegedly committed as follows:

That on or about the 16th day of November, 1971, in the municipality of Mandaluyong, province of
Rizal, Philippines, and within the jurisdiction of this Honorable Court, the above-named accused,
conspiring and mutually helping and aiding one another, with intent of gain and without the
knowledge and consent of the owner thereof, did then and there willfully, unlawfully and feloniously
CRIMINAL LAW 1
(Sunday, Section 52) 67

take, steal and carry away one Registered Letter number 247341, delivery number 3752, valued at
$500.00 belonging to one Lourdes Rodriguez de Lacson, to the damage and prejudice of the latter in
the aforementioned amount of $500.00, U.S. Dollar.
Contrary to law.

Upon arraignment, Antonio Avecilla entered a plea of not guilty to the crime charged. On the other
hand, to prove his guilt, the prosecution presented the following witnesses:
1. Lourdes Rodriguez de Lacson, an employee of Litton Mills, Inc., testified that her sister, Maria Paz,
a resident of Milwaukee, Wisconsin, U.S.A., sent her a registered letter containing a bank draft worth
$400.00 (not $500.00 as alleged in the information). She came to know about this registered letter
through another sister Carmencita Rodriguez, who, on December 17, 1971, also received another
letter from Maria Paz, inquiring whether she (witness) had received Registered Letter No. 247341,
which was addressed to her at Litton Mills, Inc. 3
Since she did not receive the registered letter with the bank draft, Lourdes went to the Mandaluyong
Post Office and inquired about it. She was informed by the Postmaster and the teller that Registered
Letter No. 247341 addressed to her was claimed on November 16, 1971 by one Antonio Avecilla,
whom they knew had been messenger of Litton Mills, Inc. for more than two years. That Avecilla got
the subject letter was shown by the registry notice duly signed by him.

Lourdes further testified that she filed a complaint in their office, as a result of which an investigation
was conducted by the personnel officer of Litton Knitting Mills. Antonio Avecilla admitted in her
presence that he took the said registered letter, but when she demanded its return, he refused to do
so. Hence, she filed a complaint with the police. 5
On cross-examination, Lourdes stated that on December 23, 1971, she made an overseas call to her
sister, Maria Paz, to request her to stop the payment of the bank draft. As a result, she was informed
by Maria Paz through their sister, Carmencita Rodriguez, that a "stop payment" order, had already
been made to the drawee bank. 6
2. Rosalinda Cervo, Clerk-in-charge of the registry section of Mandaluyong Post Office, declared that
on November 15, 1971, she received a letter for Mrs. Lourdes Lacson, with a given address at "Litton
Knitting Mills." She then issued the corresponding registry notice to the addressee which she sent
through the letter carrier. The following day, November 16, 1971, Antonio Avecilla went to the post
office to claim the registered letter. He presented the registry notice addressed to Lourdes Lacson
with her signature appearing thereon, signed it in her presence and introduced to her a woman as
Mrs. Lourdes Lacson who signed the control book.
She further testified that inasmuch as Antonio Avecilla had been, the authorized messenger of Litton
Knitting Mills since 1969, she entrusted Registered Letter No. 247341 to him. However, she got to
meet the lady who seemed to be the true Mrs. Lacson when the latter went to the post office to
complain about the letter that she (Mrs. Lacson) had not received. When shown the control book with
her alleged signatures, Mrs. Lacson repudiated the same as hers.
Subsequently, when Rosalinda Cervo informed Avecilla about Mrs. Lacson's complaint, he declared
that he had placed the letter on the table of Mrs. Lacson. 7
3. Federico Rivera, Sr., Postmaster of Mandaluyong, Rizal, testified that Litton Mills, through George
Litton, Sr., had previously written him a letter, authorizing one Antonio Avecilla to accept "registered
mails, i.e., checks, parcels and letters" for their company and employees. 8 He recalled that on
November 16, 1971, a registered letter addressed to Mrs. Lourdes Lacson c/o Litton Mills was
delivered to Mr. Avecilla. 9
The defense, before presenting the accused to testifying his own behalf, called Mrs. Lourdes R. Lacson
to the witness stand and asked her to introduce the letter of the cashier of Guardian State Bank,
Milwaukee, Wisconsin, dated October 27, 1972 addressed to Maria Paz R. Prado, stating that
CRIMINAL LAW 1
(Sunday, Section 52) 68

"Cashiers Check No. 27166, payable to Miss Carmencita S. Rodriguez, has not been paid as of this
date." 10
4. Antonio Avecilla declared that as messenger of Litton Hills, it was his duty to get all the incoming
and outgoing mails of both the Pasig and Mandaluyong branches of Litton Mills, Inc. He knew the
complainant, Mrs. Lacson, because the latter was his co-employee at Litton Mills. He often mailed the
letters of Mrs. Lacson for her and would also get her mail from the post office.
On November 16, 1971, he admitted having gotten from the Mandaluyong Post Office a registered
letter addressed to Mrs. Lacson by signing the name of Mrs. Lacson and his name on the registry
receipt. Once in the office, he left said letter on the table of Mrs. Lacson because at that time she was
already out as it was past 5:00 o'clock in the afternoon. He also said that when he left the letter on
the table, nobody was in the office. 11
On July 16, 1973, the trial court 12 rendered its decision finding accused-petitioner guilty beyond
reasonable doubt of simple theft. The dispositive part of the decision reads:
WHEREFORE, finding the evidence sufficient to prove the guilt of the accused beyond reasonable
doubt of the crime of Theft, pursuant to Article 308 and 309 of the Revised Penal Code, and applying
the Indeterminate Sentence Law, he is hereby sentenced to an indeterminate penalty of SIX (6)
MONTHS of arresto mayor as minimum to ONE (1) YEAR, EIGHT (8) MONTHS, TWENTY-ONE (21)
DAYS of prision correccional as maximum.
SO ORDERED. 13
Not satisfied with the decision, petitioner appealed to the Court of Appeals, which on December 20,
1976, promulgated a decision finding accused-petitioner guilty of qualified theft instead of simple
theft. The dispositive portion of the decision reads:
WHEREFORE, the appealed decision is hereby modified in the sense that the crime committed is
hereby designated as qualified theft; and that the appellant is hereby sentenced to suffer the
indeterminate penalty of from FOUR (4) YEARS, TWO (2) MONTHS AND ONE (1) DAY of prision
correccional,as minimum, to NINE (9) YEARS, FOUR (4) MONTHS AND ONE (1) DAY of prision mayor,
as maximum. In all other respects, the decision is affirmed, with costs against accused-appellant.
IT IS SO ORDERED. 14
The motion for reconsideration having been denied, 15 petitioner elevated the case to the Supreme
Court by way of the instant petition for review on certiorari.
Petitioner contends that his constitutional right to due process had been violated both substantially
and procedurally. He was convicted of qualified theft instead of simple theft and imposed a penalty
eight times longer than his original sentence, and his motion for reconsideration of the appellate
court's decision was "denied in one stereo-typed sentence." 16 He adds that his constitutional right
to be informed of the nature and cause of the accusation against him provided for in Art. IV, Sec. 19
of the 1971 Constitution and reiterated in Rule 115, Sec. 1, par. (c) of the Rules of Court had also been
violated.
Petitioner further argues that the Court of Appeals erred in convicting him of qualified theft just
because the information used the term "registered letter" when "not all registered letters is (sic) mail
matter." Because the information alleges that the registered letter belonged to Lourdes Rodriguez de
Lacson and considering Art. 723 of the Civil Code which provides that a letter becomes the personal
property of the addressee after it has been delivered, the crime charged is only simple theft.
Petitioner bewails the vagueness of the information which resulted in his "bewilderment" as to what
precisely he had allegedly stolen for a registered letter per se cannot be worth $500.00. He notes that
the information does not state that the registered letter contained a check. Moreover, he avers, the
essential elements of theft, whether simple or qualified, had not been substantiated by the facts
proven. Thus, petitioner adds, it had not been shown that he knew about the contents of the letter;
there was no unlawful taking because the delivery of the letter was made in the manner prescribed
by postal regulations; the allegedly stolen property had not been produced at all, and the prosecution
relied solely on the "sheer self-serving testimony" of the complaining witness. 17
CRIMINAL LAW 1
(Sunday, Section 52) 69

Petitioner's allegations necessitate a scrutiny of the information imputing to him the commission of
a crime. It need not be overly stressed that the averments in the complaint or information
characterize the crime to be prosecuted and determine the court before which the case must be
tried. 18 What controls is not the designation of the offense but the description thereof as alleged in
the information. 19
A thorough examination of the information reveals that it contains all the essential elements of the
crime of theft, to wit: (1) that there be taking of personal property; (2) that said property belongs to
another; (3) that the taking be done with intent to gain; (4) that the taking be done without the
consent of the owner; and (5) that the taking be accomplished without the use of violence or
intimidation against persons or force upon things. 20
While it is true that petitioner could not have been "bewildered" as to the nature of the charge against
him had the information been more accurately crafted, it nonetheless contains all the elements of the
crime of theft. Thus, it is alleged therein that petitioner, with the aid of and in conspiracy with an
unidentified woman, willfully took away Registered Letter No. 24341 belonging to Lourdes
Rodriguez de Lacson to her damage and prejudice. Although intent to gain is not explicitly alleged in
the information, it may be presumed from the allegation that the said mail matter was unlawfully
taken. 21 Since there is no allegation that the taking was accomplished with violence or intimidation
against persons or force upon things, it is apparent that the charge is for the crime of theft rather than
robbery.
The allegation that the subject of the taking is a registered letter categorizes the theft as a qualified
rather than a simple one. This is clear from the provision of Art. 310 of the Revised Penal Code which
states that qualified theft is committed if the property stolen is mail matter. In this regard, petitioner's
contention that not all registered letters are mail matter is incorrect. Under Sec. 1945 of the Revised
Administrative Code of 1917, first class mail matter includes letters. For the greater security of
valuable mail matter, Sec. 1962 of the same Code established a registry system "under which the
senders or owners of registered matter may be indemnified for losses thereof in the mails, the
indemnity to be paid out of postal revenues . . .
From the foregoing, it is clear that petitioner had not been deprived of his constitutional right to be
informed of the nature and cause of the accusation against him. Moreover, he may be convicted of a
crime and sentenced to the corresponding penalty as long as the facts alleged in the information and
proved at the trial constitute the crime for which he is convicted although different from the crime
designated and charged in the information. 22 The allegations in the complaint against petitioner had
been established beyond reasonable doubt at the trial. In this regard, it should be pointed out that
absolute certainty of guilt is not demanded by the law as basis for conviction of any criminal charge,
but moral certainty is required as to every proposition of proof requisite to constitute the
offense. 23 Moral certainty convinces and satisfies the reason and conscience that a crime has indeed
been committed. 24 This quantum of proof has been satisfied in this case.
In his defense, petitioner relied solely on his own unsupported testimony. His story that after taking
delivery of the registered letter addressed to complainant Mrs. Lacson, he left it on her desk after
office hours when no one else was in the office strains credulity. In his attempt to prove that he did
not benefit from the bank draft of $400.00, he presented Mrs. Lacson herself with a letter from the
cashier of the Guardian State Bank in Milwaukee, Wisconsin to the effect that the bank draft had not
been paid. However, as in the crime of robbery, the fact that the accused did not benefit from the
articles taken does not affect the nature of the crime because from the moment the offender gained
possession of the thing, the unlawful taking is complete. 25
Petitioner's assertion that he took Mrs. Lacson's registered letter by following the postal regulations
and hence, he may not be liable for its "misdelivery," falls flat in the face of the unrebutted proof that
he even used a woman to misrepresent herself as Mrs. Lacson. Although no one else witnessed the
deception and Rosalinda Cervo could no longer describe the woman, the undisputed fact remains
CRIMINAL LAW 1
(Sunday, Section 52) 70

that someone else other than Mrs. Lacson did sign the control book and that, thereafter, petitioner
took the letter with the bank draft of $400.00 which Mrs. Lacson never received.
Although proof as to motive for the crime is essential when the evidence of the theft is
circumstantial, 26 the intent to gain or animus lucrandi is the usual motive to be presumed from all
furtive taking of useful property appertaining to another, unless special circumstances reveals
different intent on the part of the perpetrator. 27 As earlier noted, the intent to gain may be presumed
from the proven unlawful taking.
The Court of Appeals considered P6,000.00 as the equivalent of $400.00. Under Art. 309 (2) of the
Revised Penal Code, the penalty for theft involving said amount is prision correccional in its minimum
and medium periods, but considering that qualified theft is punishable by a penalty two degrees
higher, 28 petitioner should be imposed in penalty of prision mayor in its medium and maximum
periods. In the absence of aggravating and mitigating circumstances, the penalty should be the
medium period of said penalty or nine (9) years, four (4) months and one (1) day of prision
mayor medium to ten (10) years, eight (8) months and one (1) day of prision mayor maximum. Hence,
the Court of Appeals correctly applied the Indeterminate Sentence Law and imposed the
indeterminate sentence of four (4) years, two (2) months and one (1) day of prision
correccional maximum as minimum penalty to nine (9) years, four (4) months and one (1) day
of prision mayor medium as maximum penalty.
WHEREFORE, the modification of the decision of the trial court by the Court of Appeals finding the
accused-appellant guilty, not only of simple but qualified theft, being in order, the above imposition
of the penalty prescribed by the Indeterminate Sentence Law is CORRECT. Costs against the
appellant.

PEOPLE VS. SARA


55 PHIL. 939 (1931)

FACTS: It is unnatural for aggrieved relatives to falsely accuse someone other than the actual culprit,
for their natural interest in securing the conviction of the guilty would deter them from implicating
any other. Paterno Morcillo was about to transfer his Carabao in front of their house in Cabatuan,
Iloilo when gunshots were fired against him which caused his death. Morcillo‘s two sons, Benjamin
and Felipe saw that the perpetrator was Bernardo Sara and Efren Robles. Upon investigation, the
police found Berning and Efren to be positive for gunpowder residue but they denied the allegation.
The Regional Trial Court (RTC) acquitted Efren and convicted Berning guilty of murder. The Court of
Appeals affirmed the decision of the RTC.

ISSUE:
Whether or not the evidence of the prosecution established the guilt of Bernardo Sara beyond
reasonable doubt

HELD:
Both Felipe and Benjamin testified that there was ample illumination from the moon to enable them
to see the face of there father‘s assailant. There was thus no possibility for both witnesses to be
mistaken in identifying their father‘s assailant, especially considering that they have known Bernardo
Sara, their neighbor, for a long time. There being no indication that Felipe and Benjamin
were actuated by any improper motive to falsely testify against appellant, their relationship with the
victim notwithstanding, there is no reason to doubt the veracity of their testimonies. Relationship
could in fact even strengthen the witnesses‘ credibility; it being unnatural for aggrieved relatives to
falsely accuse someone other than the actual culprit, for their natural interest in securing
the conviction of the guilty would deter them from implicating any other.
CRIMINAL LAW 1
(Sunday, Section 52) 71

PEOPLE VS. NANQUIL


43 PHIL. 232 (1922)

FACTS:
Juan Rosas’ cart and carabao disappeared, he requested the assistance of the Constabulary to recover
them and find the suspect. A sergeant and two soldiers were commissioned to investigate. During the
investigation, they called Severino Ramiscal, and one of them, surnamed Masiglat, examined
Ramiscal. He did not obtain any clear information from Ramiscal so he turned him over to the other
soldier, Antancio Nanquil for further examination. Sergeant who commanded that patrol was feeling
ill so he remained in a house in a neighborhood. As Atanasio Nanquil examined Ramiscal on a road,
Masiglat was 20 brazas from them. All of a sudden, Masiglat heard a blow and saw Ramsical fall to
the ground. Nanquil struck him with his gun which consequently killed Ramiscal. Atansio Nanquil
was prosecuted for the crime of homicide and sentence by the trial court to fourteen years, eight
months and one day of reclusion temporal. The defendant appealed.

ISSUE:
W/N Nanquil should be charged with homicide through reckless imprudence.

RULING:
No. The court finds a mitigating circumstance of the accused not having hadthe intention to cause the
death of the deceased. For this reason the penaltyof reclusion temporal must be imposed in its
minimum degree. The judgment appealed from is modified, and the appellant sentenced to twelve
years andone day of reclusion temporal, to the accessory penalties provided by article59 of the Penal
Code, to indemnify the heirs of Severino Ramiscal in theamount of one thousand pesos (P1,000), and
to pay the costs of bothinstances.
Ratio:Nanquil had no intention to commit so serious an evil as that which resulted,the crime
committed by him cannot be that of homicide through recklessimprudence, because he did have the
intention to do some evil unlawfully(maltreating the deceased), and this intention, although it was
not that of killing, is inconsistent with reckless imprudence.

PEOPLE VS. BINDOY


G.R. NO. L-34665 August 28, 1931

FACTS:
Appeal from a judgement of the CFI of Occidental Misamis, for appelant was stenced to 12 years and
1 day of reclusion temporal and to indemnify the heirs of the deceased with the amount of P1,000.
The crime charged against the accused is homicide.

In the afternoon of May 6, 1930, a disturbance arose in a tuba wineshop. Donato Bindoy offered some
tuba to Faustino Paca's wife Tibay. She refused because she already have one, but Bindoy threatened
to injure her if she did not accept. Pacas stepped in to defend his wife, attempting to take away from
Bindoy the bolo he carried. Emigdio Omamdam who came to the wine shop to see what;s happening,
instead got stabbed in the chest by Bindoy. This happened when Bindoy succeeded in disengaging
himself from Pacas, wrenching the bolo from the latter's hand towards the left behind the accused
and with such violence that the point of the bolo reached Omamdam's chest who was then behind
Bindoy.

ISSUE(S):
Whether or not Bindoy is criminally liable?
CRIMINAL LAW 1
(Sunday, Section 52) 72

HELD:
Corroborated by Gaudencio Cenas of the testimony of the accused, Pacas and Bindoy were actually
for the possession of the bolo. When Pacas let go of the bolo, Bindoy had pulled so violently that it
flew towards his left side, at the very moment when Emigdio Omamdam came up and who was
therefore hit in the chest without Bindoy seeing him. Bindoy alleges that it was caused accidentally
and without malicious intent because he was only defending his possession of the bolo which Pacas
was trying to wrench away from him and his conduct was perfectly lawful. The Court therefore
acquitted Bindoy based on the facts stated.
"In many criminal cases, one of the most important aids in completing the proof of the commission
of the crime by the accused is the introduction of evidence disclosing the motives which tempted the
mind of the guilty person to indulge the criminal act."

FACTS: On May 6, 1930, Donato Bindoy offered some tuba to Tibay, Faustino Pacas' wife. She refused
and Bindoy threatened to injure her if she did not accept. Pacas stepped in to defend his wife and
attempted to take away from Bindoy the bolo he carried. The disturbance attracted the attention of
Emigdio Omamdam. In the course of the struggle, Bindoy succeeded in disengaging himself from
Pacas, wrenching the bolo from the latter's hand, with such violence that the point of the bolo reached
Omamdam's chest, who was then behind Bindoy. The trial court held that Bindoy was guilty of the
crime of homicide. Bindoy appealed, alleging that the death of Omamdam was caused accidentally
and without malicious intent.

ISSUE: WON the crime of which Bindoy was found guilty of can be mitigated on the ground of
accident.

HELD: Yes. Decision is reversed. Bindoy is acquitted according to Article 8, No. 8 of the Revised Penal
Code

RATIO:
1. There is no evidence to show that Bindoy deliberately and intentionally klled Omamdam.
• No evidence that Onamdam took part in the fight between Bindoy and Pacas.
• No evidence that Bindoy was aware of Omamdam's presence.
• No evidenæ that there was disagreement or ill feelin$ between Bindoy & Omamdam. On the
contrary, they were nephew & uncle, & were on good terms with each other.
2. The witness for the defense corroborates the defendant to the effect that Pacas and Bindoy were
actually struggling for the possession of the bolo, and that when the latter let go, the former had
pulled so violently that it flew towards Omamdam, who was therefore hit in the chest, without
Bindoy's seeing him, because Omamdam had passed behind him. The testimony of this witness was
not contradicted by any rebuttal evidence adduced by the fiscal.
3. If, in the strugge, the defendant had atempted to wound his opponent, and intead of doing so, had
wounded Omamdam, he would be liable for his act, since whoever willfully commits a felony or a
misdemeanor incurs criminal liability, although the wrongful act done is different from that which he
intended.
This is not the case here. Bindoy did not try to wound Pacas. He was only trying to defend his
possession of the bolo, which Pacas was trying to wrench away from him. His conduct was perfectly
lawful.
CRIMINAL LAW 1
(Sunday, Section 52) 73

PEOPLE OF THE PHILIPPINES. plaintiff-appellee, vs. GUILLERMO NEPOMUCENO, JR, accused


appellant

DECISION
MELO, J.:
Accused-appellant Guillermo Nepomuceno, Jr. has interposed the instant appeal in regard to the
decision dated November 20, 1996 of the Regional Trial Court of the National Capital Judicial Region
(Manila, Branch 46) which decreed:
Wherefore, the court hereby renders judgment finding the accused guilty beyond reasonable doubt
of the crime of parricide as defined and penalized under Article 246 of the Revised Penal Code as
amended by Republic Act No. 7659 for the death of Grace Nepomuceno and hereby sentences him to
suffer imprisonment of Forty (40) years of reclusion perpetua and to pay the heirs of the deceased
the sum of P50,000.00 with costs against him.

Pursuant to Article 921, paragraph (1) of the Civil Code, the court declares the accused ineligible to
inherit from his wife. The entire estate should go to his son, Giordan Benitez Nepomuceno.
(pp. 20-21, Rollo)

The Information against accused-appellant charged:


That on or about May 2, 1994, in the City of Manila, Philippines, the said accused, did then and there
willfully, unlawfully and feloniously, with intent to kill and with treachery and evident premeditation,
attack, assault and use personal violence upon the person of one GRACE NEPOMUCENO Y BENITEZ,
his wife, with whom he was married in lawful wedlock, by then and there shooting her with a gun of
unknown caliber hitting her on the left hip, thereby inflicting upon the said GRACE NEPOMUCENO Y
BENITEZ gunshot wound which was necessarily fatal and which was the direct and immediate cause
of her death thereafter.

(p. 5, Rollo)
Upon arraignment, accused-appellant, entered a plea of not guilty and trial ensued in due course.
Thereafter, the trial court rendered the judgment of conviction now on appeal.
The prosecution presented seven witnesses, namely, Eden Ontog, SPO2 Rodolfo Rival, Forensic
Chemist Mary Ann Aranas, Medico-Legal Examiner Floresto Arizala, Monserrat De Leon, Ballistic
Expert Isabelo Silvestre, Jr. and Romeo Pabalan.

Eden Ontog declared that on May 2, 1994, she was the housemaid of the spouses Guillermo
Nepomuceno, Jr. and Grace Nepomuceno, having started as such since May 31, 1993. At around 11
o'clock on the evening of May 2, 1994, accused-appellant, who was drunk, arrived and went to their
bedroom where Eden and her ward Giordan the one-year old son of the couple, were sleeping. She
was awakened by the loud voices of the spouses who were arguing. She saw accused-appellant get a
gun from a drawer, so she went out of the room because of fear. After a few moments and while she
was outside the room, she heard Grace Nepomuceno say: "Sige patayin mo ako, patayin mo na kami
ng anak ko." Then Eden heard a gunshot. She was so scared that she went out of the house, reaching
the door of the house of Barangay Chairman Congen Leonardo which is 5 meters away. After ten
minutes, she saw accused-appellant coming out of the room. He told her to get a taxi so he could bring
the wounded Grace to the hospital. She was left behind in their room to take care of baby Giordan.
She tried to call up Monserrat de Leon, the sister of Grace in Pasig to inform her of the incident, but
she could not get any connection (tsn, July 27, 1994, pp. 4-17; 24-42).

Mary Ann T. Aranas, a chemist of the National Bureau of Investigation Chemistry Division, declared
that she conducted paraffin examination on both hands of the victim and those of accused-appellant.
CRIMINAL LAW 1
(Sunday, Section 52) 74

She found the victim's hands negative of nitrates, but found accused-appellant's right hand positive
thereof. She gave the opinion that in view of the absence of nitrates on the hands of the victim, it is
probable that she did not fire a gun and that accused-appellant, being positive of nitrates, did really
fire a gun (tsn, August 31, 1994, pp. 17, 22; tsn, July 8, 1996, p. 6).

Dr. Floresto Arizala, Jr., the Medico~Legal Officer of the NBI who conducted a second-post mortem
examination of the victim on May 7, 1994 at the Capitol Memorial Chapels, found that the victim died
due to a gunshot wound, with the slug hitting the left internal iliac artery and the small intestines and
thereafter resting between the uterus and the sacrum of the victim. He testified that taking into
consideration the location of the wound, if the victim were in a sitting or lying position, the trajectory
of the slug was upward coming from right to left; and if the victim were in a standing position, the
muzzle of the gun should have pointed up. The witness declared that the muzzle of the gun could not
have been less that one foot from the victim. He opined that grappling for possession of the gun was
impossible because the trajectory of the bullet was going upwards and there were no smudges or
signs of close firing. He believed that the victim could have survived if the surgeons had operated
immediately (tsn, July 23, 1996, pp. 15-30).

Monserrat de Leon, sister of the victim, declared that Grace would confide to her that accused-
appellant was jobless and that Grace had problems with the low income of the store she owned at
Zurbaran Mart as compared to her expenses. Accused-appellant would force sex on Grace especially
when he was drunk. Her sister had two miscarriages after their first child and it was during one of
these miscarriages that she saw accused-appellant carrying a gun in the Mary Chiles Hospital where
her sister was confined (tsn, August 12, 1996, pp. 3-28).

Upon the other hand, the defense presented accused~appellant himself as its lone witness. His story
was quoted by the trial court thus:
Two days before the incident on May 2, 1994, Grace, the deceased was very much worried about the
check (sic) she issued which was post dated May 2, 1994. She would have no funds for the checks.
She had been nagging him, displaying her tantrums (nagdadabog) pestering him to do something to
be able to fund the checks.

In the noon time of May 2, 1994, he left her spouse in their store and went to his mother's house in
San Andres Bukid, Manila. This day was the day after the accused-appellant and his wife, and in-laws
arrived from Batangas to attend a town fiesta.

He left the store to avoid further nagging, tantrums and pestering of his wife about his inability to
produce money to be able to fund the postdated checks (sic).
At about 11:00 P.M. after dropping at a friend's house, he decided to go home, thinking his wife has
already cooled off.

When his wife opened the door, she greeted the husband: 'You left in the store and you room (roam)
around, where you able to find money.' He replied, 'where would I get money, do you expect me to
hold up people?' (tsn, October 5, 1994, p. 18)

They had some arguments and Eden Ontog went out of their bedroom, and the arguments continued.
There was a point in the argument when the wife told the accused thus: 'Wala akong silbi, bakit pa
ako nag-asawa sa kanya.'
Because of these continued pestering and nagging of his wife he thought of separation. Perhaps it
would be better if he should end his life. He then took a gun from their child's drawer. He sat on the
CRIMINAL LAW 1
(Sunday, Section 52) 75

bed holding that gun, engrossed in his thinking what to do. The gun was pointed towards the floor of
their room.

In that situation, his wife continued with his nagging and pestering. He just remained silent.
And then Guillermo asked her wife: 'How come you do not treat me as a husband, why do you treat
me like this.'

It was at this point that Guillermo decided to end his life. Perhaps seriously, perhaps just to scare his
wife to stop all the pestering and tantrums. Surely not only a few husbands would thought (sic) as
what that Guillermo was thinking then, he felt desperate. He wanted to finish his life. (p. 24, TSN,
October 5, 1994)

It was during that time that their son, Jordan woke up, walked to the space between them (husband
and wife) and Nepomuceno block his son's way with his right knee. In the process, he wanted to
totally force Grace from taking possession and control of the gun. He raised his arm holding the gun
passing over the left leg of Grace.

The gun went off.

(pp. 16-17, Rollo)


Aggrieved by the decision of the trial court, accused-appellant assigns the following errors:
I THE TRIAL COURT ERRED IN NOT FINDING THE KILLING WAS ACCIDENTAL, AND THAT THE
DECEASED WAS EXEMPT FROM CRIMINAL LIABILITY.
II ASSUMING THE ACCUSED IS CRIMINALLY LIABLE, THE TRIAL COURT ERRED IN NOT FINDING
THE KILLING WAS RESULT OF SIMPLE NEGLIGENCE.
III THE TRIAL COURT ERRED IN FINDING THAT THE GUILT OF THE ACCUSED WAS PROVEN
BEYOND REASONABLE DOUBT.

(p. 5, Appellant's Brief.)


In support of the first assigned error, accused-appellant contends that he did not have the least
intention of killing his wife. He urges the Court to consider the circumstances attendant to the killing,
which, according to him negate all inferences and deductions, that he would kill his wife. First, the
deceased was hit in the upper leg, not in any vital organ. If he had the intention of killing the deceased,
he would have shot her at the most vital part of her body.

Secondly, the reaction of the deceased after she was hit was contrary to ordinary and usual human
behavior, if her husband really intended to kill her. The deceased just uttered, "Masakit Papa", she
did not curse nor mouth evil and harsh language against accused-appellant to show hatred and anger.
Thirdly, if accused-appellant really intended to kill his wife, why did he call a taxi and bring her to the
hospital for immediate medical attention?

Fourthly, why should accused-appellant voluntarily surrender to the police, if the incident was not
accidental?

Accused-appellant claims exemption from criminal liability under Paragraph 4, Article 12 of the
Revised Penal Code because, according to him, the incident occurred when he tried to prevent his
wife from killing herself, and he and his wife grappled for possession of the gun.
After a painstaking review of the evidence and record of this case, the Court finds itself unable to
reach conclusions identical to those put forward by accused-appellant.
CRIMINAL LAW 1
(Sunday, Section 52) 76

First, accused-appellant cannot invoke the benevolent provisions of Paragraph 4, Article 12 of the
Revised Penal Code in order to be exempted from criminal liability arising from the death of his wife,
Grace Nepomuceno. Said provision pertinently states:

Art. 12. Circumstances which exempt from criminal liability. The following are exempt from criminal
liability:

4. Any person who, while performing a lawful act with due care, causes an injury by mere
accident without fault or intention of causing it.
At all events, accident to be exempting, presupposes that the act done is lawful. Here, however, the
act of accused-appellant of drawing a weapon in the course of a quarrel, the same not being in self-
defense, is unlawful -- it at least constitutes light threats (Article 285, par. 1, Revised Penal Code).
There is thus no room for the invocation of accident as a ground for exemption (People vs. Reyta, Jr.,
13 CAR (25) 1190; 1195 [1968]).

The gun which accused-appellant took from his child's drawer was not even licensed or registered in
his name as shown by the Certification of the Firearms and Explosives Office of the Philippine
National Police, hence, he could have been charged with illegal possession of a firearm.
Secondly, accused~appellant's claim that the shooting happened when he tried to prevent his wife
from killing herself and he and his wife grappled for the possession of the gun is belied by the expert
testimony of Dr. Floresto Arizala, Jr. of the NBI who conducted a second post mortem examination on
the cadaver of Grace Nepomuceno. He declared:
Q. Now, is it possible Doctor, considering the location of the wound, the entrance wound and the
trajectory of the bullet upwards, would you say Doctor, that both parties, I mean the victim and the
assailant were grappling for the possession of said gun and it went off accidentally, is that possible,
Mr. Witness?
A. Well, I have to be convinced as to the grappling between the victim and the assailant, because
if we were to be re-construct of the scenario that the gun have been fired, the muzzle of the gun could
not have been closer than twelve (12) inches and considering that the gun was held by a hand, it
farther places the assailant farther from the victim and farther the victim is, from the assailant, then
the more impossible for the grappling for the gun. (tsn, July 23, 1996, pp. 19~20).
Thirdly, accused-appellant, testifying on the relative positions of the victim and himself when the gun
discharged, stated:
Q. Please tell the court your relative position and the victim when the gun actually went off?
A. When I was in the act of trying to dispossess Grace with that gun and I was trying to let my
right hand pass through my right side but because Grace was struggling, the butt of the gun hit a part
of her upper leg and it exploded.
Q. So when the gun actually fired, you were holding that gun, what part of your arm being held
by Grace?
A. Witness pointing the upper forearm and the lower portion of her upper arm."
(tsn, Oct. 5, 1994, p. 28.)
If Grace were holding the upper forearm and lower portion of the upper arm of accused-appellant
when the gun fired, then at least the hand of Grace that held the upper forearm of appellant would
have traces of nitrate considering its nearness to the exploding gun. However, in the paraffin test
conducted by the Forensic Chemistry Division of the National Bureau of Investigation on Grace
Nepomuceno's both hands, no traces of nitrates were found- while accused-appellant's right hand
was positive of nitrates. The absence of nitrates on the victim's hands is convincing proof that she did
not grapple with accused-appellant for the possession of the gun. It also proves that she was shot at
a distance.
CRIMINAL LAW 1
(Sunday, Section 52) 77

The fact that the victim was not shot in the head, or in any vital part of her body does not negate
intent to kill. The Post Mortem findings on the cadaver of Grace Nepomuceno by Dr. Arizala shows
that the bullet entered "the left thigh, lateral aspect, upper third . . . directed slight forwards, slightly
upwards and from left to right initially involving the skin and subcutaneous tissue, then taking an
intramascular route into the pelvic cavity thru the left obturator foramen, partially transecting the
left internal iliac artery and the small intestines with the slug lodging just underneath the uterus in
front of the sacrum where it was recovered." The extent of the physical injury inflicted on Grace, as
above proved, manifests intention to extinguish life (People vs. Dawandawan, 184 SCRA 264 [1994]).
Moreover, Dr. Arizala likewise declared that the bullet injured a vital organ of the victim (tsn, July 23,
1996, p. 9).
The fact that Grace, upon being shot, uttered, "Masakit, Papa" and did not use harsh language against
accused-appellant does not, in any way, negate intent to kill. The utterance of a victim made
immediately after sustaining serious injuries may be considered as pure emanations of the incident
or the incident speaking through the victim (People vs. Morin, 241 SCRA 709; 710 [1995]). Thus, by
the word "Papa", Grace was in effect, saying that it was accused-appellant who shot her.
We agree with the Solicitor General that the act of accused-appellant ordering Eden Ontog to call a
taxi in which he brought the wounded Grace to the hospital is "merely an indication or act of
repentance or contrition on the part of appellant" (Appellee's Brief, p. 71, Rollo).
Accused-appellant's voluntary surrender is not sufficient ground to exculpate him from criminal
liability. The law does not find unusual the voluntary surrender of criminal offenders; it merely
considers such act as a mitigating circumstance. Non-flight is not proof of innocence (People vs.
Quijada, 259 SCRA 191 [1996]).
Under the second assigned error, accused-appellant claims that even assuming that the killing was
not totally accidental, his acts would constitute only simple negligence. He asserts that he had
established that the gun went off while he was grappling with his wife for its possession. He was
preventing his wife from taking her own life. He might not have exercised the necessary due care in
wrestling for the gun that resulted in the injury of his wife, but he could be charged only with
parricide through simple negligence. So he says.
It has been held that a deliberate intent to do an unlawful act is essentially inconsistent with the idea
of reckless imprudence (People vs. Oanis, et al., 74 Phil 257 (1943); People vs. Nanquil, 43 Phil 232
[1922]). What qualifies an act of reckless or simple negligence or imprudence is the lack of malice or
criminal intent in the execution thereof (United States vs. Maleza, 14 Phil 468, 471 [1909]). Otherwise
stated, in criminal negligence, the injury caused to another should be unintentional, it being simply
the incident of another act done without malice but with lack of foresight, or with carelessness or
negligence, and which has harmed society or an individual (People vs. Castillo, Jr., (275 SCRA 752
[1997]).
The argument of accused-appellant finds no support in the physical evidence. As already discussed,
if the version of grappling for the gun were to be believed, there should have been nitrates on both
hands of Grace. And if it was when accused-appellant placed the barrel of the gun at the base of his
head that Grace grabbed his hand holding the gun and in the struggle for its possession his hand
holding the gun was pushed down so that its butt hit the upper leg of Grace causing it to fire, then the
trajectory of the slug should be downwards, through the upper thigh of Grace where it entered. Yet,
the autopsy report of Dr. Arizala, Jr. showed the bullet entered the left thigh of Grace directed slightly
upwards and from left to right, taking an intramascular route into the pelvic cavity, instead of a
downward direction if accused-appellant's version were to be believed.
Thus, over and above the testimony of accused-appellant, these physical evidence, the lack of powder
burns or nitrates on the hands of Grace and the trajectory of the bullet that entered her left thigh
being slightly upwards and from left to right instead of downwards, repudiate accused-appellant's
claim of simple negligence. Physical evidence is mute but an eloquent manifestation of truth and rates
high in our hierarchy of trustworthy evidence (People vs. Uycoqua, 246 SCRA 769 [1995]).
CRIMINAL LAW 1
(Sunday, Section 52) 78

Lastly, accused-appellant, perhaps in desperation, resorts to the shotgun type of argument that his
guilt has not been proved beyond reasonable doubt. The argument is bereft of merit.
The prosecution has sufficiently established the elements of parricide by its evidence. These elements
are: (1) the death of the deceased; (2) that she was killed by the accused; and (3) that the deceased
was a legitimate ascendant or descendant, or the legitimate spouse of the accused (Article 246,
Revised Penal Code; People vs. Embalido, 58 Phil 154 (1933)].
The first and third elements were stipulated during the pre~trial stage of the case, thus:
1. that the victim and the accused are legally married. Said civil marriage took place on July 5,
1990;
5. that immediately after the shooting, the accused voluntarily and bodily carried the victim into
a taxicab and proceeded to UERM Hospital where she died on the operating table." (Pre-Trial Order
of July 11, 1994, Record, p. 6)
The only issue then is whether accused-appellant intentionally killed Grace Nepomuceno, his legally
wedded wife.
In convicting accused-appellant, the trial court relied heavily on the testimony of the prosecution
witnesses. This Court finds no reason to do otherwise. It is a fundamental and settled rule that the
trial court's assessment in regard to the credibility of witnesses is entitled to the highest degree of
respect and will not be disturbed on appeal, as the trial court was in a better position to examine real
evidence as well as to observe the demeanor of the witnesses (People vs. Dominguez, 217 SCRA 170
[1993]; People vs. Camaddo, 217 SCRA 162 [1993]; People vs. Vallena (244 SCRA 685 [1995]).
The Court agrees with the conclusions of the trial court as they are founded on the dictum that
evidence to be believed must not only proceed from the mouth of a credible witness, but must be
credible in itself - such as the common experience of mankind can approve as probable under the
circumstances. We have no test of the truth of human testimony, except in conformity with our
knowledge, observation, and experience. Whatever is repugnant to these belongs to the miraculous
and is outside of judicial cognizance (People vs. Escalante, 238 SCRA 554 [1994]).
Further, accused-appellant having admitted that he shot his wife, he has the burden of proof of
establishing the presence of any circumstance which may relieve him of responsibility, and to prove
justification he must rely on the strength of his own evidence and not on the weakness of that of the
prosecution, for even if this be weak, it can not be disbelieved after the accused has admitted the
killing (People vs. Bautista, 254 SCRA 621 [1996). Unfortunately for accused-appellant, he has
miserably failed to discharge this task.
The trial court correctly appreciated the voluntary surrender of accused-appellant as a mitigating
circumstance, this fact having been stipulated by the parties at the pre-trial stage of this case
(Pre~Trial Order, Stipulation No. 10 Record, p. 17)
The penalty for the crime of parricide is reclusion perpetua to death; however, there being one
mitigating circumstance but no aggravating circumstance, the lower of the two indivisible penalties
should be imposed. The penalty cannot be further reduced by one degree as the Indeterminate
Sentence Law does not find application, the penalties involved being indivisible.
WHEREFORE, the assailed decision convicting accused-appellant GUILLERMO NEPOMUCENO, JR. of
the crime of Parricide is hereby AFFIRMED with the slight modification that his sentence shall be
simply reclusion perpetua not "imprisonment of Forty (40) Years of reclusion perpetua" as stated by
the trial court.
SO ORDERED.
CRIMINAL LAW 1
(Sunday, Section 52) 79

PEOPLE VS PUGAY
No. L-74324 17November1988

FACTS:
The accused are pronounced by the RTC of Cavite guilty beyond reasonable doubt for the crime of
murder of Bayani Miranda and sentencing them to a prison term ranging from 12 years (prison
mayor) as mimimum to 20 years (prison temporal) as maximum and for samson to be sentenced to
reclusion perpetua.

Miranda and the accused Pugay are friends. Miranda used to run errands for Pugay and they used to
sleep together. On the evening of May 19, 1982 a town fiesta was held in the public plaza of Rosario
Cavite. Sometime after midnight accused Pugay and Samson with several companions arrived (they
were drunk), and they started making fun of Bayani Miranda. Pugay after making fun of the Bayani,
took a can of gasoline and poured its contents on the latter, Gabion (principal witness) told Pugay not
to do the deed. Then Samson set Miranda on fire making a human torch out of him. They were
arrested the same night and barely a few hours after the incident gave their written statements.

ISSUES:
Is conspiracy present in this case to ensure that murder can be the crime? If not what are the criminal
responsibilities of the accused?

There is no: CONSPIRACY- is determined when two or more persons agree to commit a felony and
decide to commit it. Conspiracy must be proven with the same quantum of evidence as the felony
itself, more specifically by proof beyond reasonable doubt. It is not essential that there be proof as to
the existence of a previous agreement to commit a crime. It is sufficient if, at the time of commission
of the crime, the accused had the same purpose and were united in its executed.
Since there was no animosity between miranda and the accused, and add to the that that the meeting
at the scene of the incident was purely coincidental, and the main intent of the accused is to make fun
of miranda.

Since there is no conspiracy that was proven, the respective criminal responsibility of Pugay and
Samson arising from different acts directed against miranda is individual NOT collective and each of
them is liable only for the act that was committed by him.
**Conspiracy may be implied from concerted action of the assailants in confronting the victim.

Criminal Responsibilities:
PUGAY: Having failed to exercise diligence necessary to avoid every undesirable consequence arising
from any act committed by his companions who at the same time were making fun of the deceased. -
GUILTY OF RECKLESS IMPRUDENCE RESULTING TO HOMICIDE
SAMSON:Since there are NO sufficient evidence that appears in the record establishing qualifying
circumstances (treachery, conspiracy). And granted the mitigating circumstance that he never
INTENDED to commit so grave a wrong. - GUILTY OF HOMICIDE

HELD:
JUDGEMENT OF THE LOWER COURT WAS AFFIRMED WITH MODIFICATIONS. JUDGEMENT FOR
GUILTY BEYOND REASONABLE DOUBT FOR MURDER WAS LOWERED TO THE ABOVE
JUDGEMENTS.
CRIMINAL LAW 1
(Sunday, Section 52) 80

FACTS: May 19, 1982, a town fiesta was held in the public plaza of Rosario, Cavite. Sometime after
midnight, Eduardo Gabion was sitting in the ferris wheel and reading a comic book. Later, Pugay and
Samson with several companions arrived at the scene seemingly drunk. The group saw Bayani
Miranda and started making fun of him by tickling him with a piece of wood. Pugay suddenly took a
can of gasoline and poured its contents on Miranda. Gabion asked Pugay to stop during the process
of pouring the gasoline. Then Samson set Miranda on fire.

ISSUE/HELD:
WON Pugay and Samson are guilty of the crime murder. (NO)

RATIO: There was no evidence found that Pugay and Samson planned to kill Miranda. Their meeting
was accidental and the accused were merely making fun of the deceased- victim. Criminal
responsibility of Pugay and Samson are counted as individual acts and they are held liable only for
the acts they committed individually. Pugay should have known that what he was pouring on Miranda
was gasoline because of its smell. He failed to exercise diligence necessary to avoid the consequences
of his actions and exposed Miranda to danger and injury. Pugay is guilty of homicide through reckless
imprudence. Samson just wanted to set Miranda's clothes on fire but this doesn't relieve him of
criminal liability (Art. 4). Samson is guilty of homicide credited with ordinary mitigating
circumstance of no intention to commit so grave a wrong. Gabion testified that accused were stunned
when they noticed Miranda burning.

JASON IVLER y AGUILAR vs. HON. MARIA ROWENA MODESTO-SAN PEDRO, Judge of the
Metropolitan Trial Court G.R. No. 172716

FACTS:
Following a vehicular collision in August 2004, petitioner Jason Ivler (petitioner) was charged before
the Metropolitan Trial Court of Pasig City (MT C), with two separate offenses: (1) Reckless
Imprudence Resulting in Slight Physical Injuries for injuries sustained by respondent Evangeline L.
Ponce (respondent Ponce); and (2) Reckless Imprudence Resulting in Homicide and Damage to
Property for the death of respondent Ponce's husband Nestor C. Ponce and damage to the spouses
Ponce's vehicle.

Petitioner posted bail for his temporary release in both cases. On 2004, petitioner pleaded guilty to
the charge on the first delict and was meted out the penalty of public censure. Invoking this
conviction, petitioner moved to quash the Information for the second delict for placing him in
jeopardy of second punishment for the same offense of reckless imprudence.
The MTC refused quashal, finding no identity of offenses in the two cases. The petitioner elevated the
matter to the Regional Trial Court of Pasig City (RTC), in a petition for certiorari while Ivler sought
from the M TC the suspension of proceedings in criminal case, including the arraignment his
arraignment as a prejudicial question. Without acting on petitioner's motion, the MTC proceeded
with the arraignment and, because of petitioner's absence, cancelled his bail and ordered his arrest.
Seven days later, the M TC issued a resolution denying petitioner's motion to suspend proceedings
and postponing his arraignment until after his arrest. Petitioner sought reconsideration but as of the
filing of this petition, the motion remained unresolved.

ISSUES:
CRIMINAL LAW 1
(Sunday, Section 52) 81

1. Whether petitioner forfeited his standing to seek relief from his petition for certiorari when the
MTC ordered his arrest following his non-appearance at the arraignment in Reckless Imprudence
Resulting in Slight Physical Injuries for injuries sustained by respondent; and
2. Whether petitioner's constitutional right under the Double Jeopardy Clause bars further
proceedings in Reckless Imprudence Resulting in Homicide and Damage to Property for the death of
respondent Ponce's husband.

HELD:
(1) Petitioner's non-appearance at the arraignment in Criminal Case No. 82366 did not divest him of
personality to maintain the petition in S.C.A. 2803; and
(2) The protection afforded by the Constitution shielding petitioner from prosecutions placing him
in jeopardy of second punishment for the same offense bars further proceedings in Criminal Case No.
82366

PEOPLE VS GUILLEN
GR No. 1--1477, January 18, 1950

FACTS: The accused Julio Guillen, was found guilty beyond reasonable doubt of the crime of murder
and multiple frustrated murder after his attempt to assassinate the President of the Philippines,
Manuel Roxas on March 10, 1947. During the 1946 Presidential Elections, Guillen voted for the
opposing candidate of Manuel Roxas. According to the accused, he was disappointed with the latter
for failing to redeem and fulfill promises made by President Roxas during the elections. Consequently,
the accused determined to assassinate the President and found the opportunity to do so on the night
of March 10, 1947 when the President attended a popular meeting by the Liberal Party at Plaza de
Miranda, Quiapo, Manila. Guillen first intended to use a revolver to accomplish goal but he had
previously lost his licensed firearm, so he thought of using two hand grenades which were given to
him by an American soldier in exchange for two bottles of whisky. The accused stood on the chair he
had been sitting on and hurled the grenade at President when the latter had just closed his speech. A
general who was on the platform sav the smoking grenade and kicked it away from the platform
towards an open space where he thought the grenade was likely to do the least harm. The grenade
exploded in the middle of group of persons standing close to the platform and grenade fragments
seriously injured Simeon Varela, who died the next day due to the mortal wounds caused, and several
other persons. Guillen was arrested and he readily admitted his responsibility.

ISSUE: WON the accused was guilty only of homicide through reckless imprudence in regard to the
death of Simeon Varela and of less serious physical injuries in regard to the other injured persons.

HELD: The facts do not support the contention of the counsel for the appellant. In throwing the hand
grenade at the President with the intention of killing him, the appellant acted with malice and is
therefore liable for all the consequences of his wrongful act. As provided by Art. 4 of the Revised
Penal Code, criminal liability is incurred by any person committing a felony although the wrongful
act done be different from that which he intended. In criminal negligence, the injury caused to
another should be unintentional, it being simply the incident of another act performed without
malice. As held by the Court, a deliberate intent to do an unlawful act is essentially inconsistent with
the idea of reckless imprudence. Where such unlawful act is willfully done, a mistake in the identity
of the intended victim cannot be considered reckless imprudence. The sentence of the trial court is
affirmed by unanimous vote and death sentence shall be executed in accordance with article 81 of
the Revised Penal Code.
CRIMINAL LAW 1
(Sunday, Section 52) 82

PEOPLE V. HERMOGENES FLORA


G.R. NO. 125909

The 2 accused (Hermogenes and Edwin) were convicted for the murder of Emerita and Ireneo and
the attempted murder of Flor. The 2 were found to have conspired to kill Ireneo. However, during
the commission of the crime, Emerita was also killed and Flor hit by a bullet.

HELD:
Co-conspirators are liable only for acts done pursuant to the conspiracy. For other acts done outside
the contemplation of the co-conspirators or which are not the necessary and logical consequence of
the intended crime, only the actual perpetrators are liable. Evidence only shows conspiracy to kill
Ireneo and no one else. Hence, both can be convicted for the murder of Ireneo. However, only
Hermogenes who fired at Emerita and Flor can be convicted for the murder of Emerita and Flor
respectively.

FACTS:
On the evening of January 9, 1993, a dance party was held to celebrate the birthday of Jeng-jeng
Malubago in Sitio Silab, Barangay Longos, Kalayaan, Laguna. Appellant Hermogenes Flora, allegedly
a suitor of Jeng-jeng Malubago, attended the party with his brother and co-appellant Edwin Flora,
alias "Boboy". Also in attendance were Rosalie Roma, then a high school student; her mother, Emerita
Roma, and her aunt, Flor Espinas. Ireneo Gallarte, a neighbor of the Romas, was there too.
The dancing went on past midnight but at about 1:30, violence erupted. On signal by Edwin Flora,
Hermogenes Flora fired his .38 caliber revolver twice. The first shot grazed the right shoulder of Flor
Espinas, then hit Emerita Roma, below her shoulder. The second shot hit Ireneo Gallarte who
slumped onto the floor. Rosalie, was shocked and could only utter, "si Bodoy, si Bodoy", referring to
Hermogenes Flora. Edwin Flora approached her and, poking a knife at her neck, threatened to kill
her before he and his brother, Hermogenes, fled the scene.
The victims of the gunfire were transported to the Rural Health Unit in Longos, Kalayaan, Laguna,
where Emerita and Ireneo died. Early that same morning of January 10, 1993, the police arrested
Edwin Flora at his rented house in Barangay Bagumbayan, Paete, Laguna. Hermogenes Flora, after
learning of the arrest of his brother, proceeded first to the house of his aunt, Erlinda Pangan, in Pangil,
Laguna but later that day, he fled to his hometown in Pipian, San Fernando, Camarines Sur. The
autopsy conducted by the medico-legal officer, Dr. Ricardo R. Yambot, Jr., revealed the following fatal
wounds sustained by the deceased: On February 26, 1993, Prosecution Attorney Joselito D.R. Obejas
filed three separate informations charging appellants as follows: That on or about January 10, 1993,
at around 1:30 0'clock in the morning thereof, in Sitio Silab, Barangay Longos, municipality of
Kalayaan, province of Laguna, and within the jurisdiction of this Honorable Court,
1. Accused Hermogenes Flora conspiring and confederating with accused Edwin Flora , and mutually
helping one another, while conveniently armed then with a caliber .38 handgun, with intent to kill,
by means of treachery and with evident premeditation, did then and there wilfully, unlawfully and
feloniously attack, assault and shoot with the said firearm one EMERITA ROMA y DELOS REYES,
thereby inflicting upon the latter gunshot wounds on her chest which caused her immediate death,
to the damage and prejudice of her surviving heirs.
2. Accused HERMOGENES FLORA conspiring and confederating with accused Erwin [Edwin] Flora,
and mutually helping one another, while conveniently armed then with a caliber .38 handgun, with
intent to kill, by means of treachery and with evident premeditation, did then and there willfully,
unlawfully and feloniously attack, assault and shoot with the said firearm one IRENEO GALLARTE y
VALERA, thereby inflicting upon the latter gunshot wounds on his chest which caused his immediate
death, to the damage and prejudice of his surviving heirs.
CRIMINAL LAW 1
(Sunday, Section 52) 83

3. Accused Hermogenes Flora, conspiring and confederating with accused Erwin [Edwin] Flora, and
mutually helping one another, while conveniently armed then with a caliber .38 handgun, with intent
to kill, by means of treachery and with evident premeditation, did then and there willfully, unlawfully
and feloniously attack, assault and shoot with the said firearm one FLOR ESPINAS y ROMA, hitting
the latter on her shoulder, and inflicting upon her injuries which, ordinarily, would have caused her
death, thus, accused performed all the acts of execution which could have produced the crime of
Murder as a consequence hut which nevertheless did not produce it by reason of a cause independent
of their will. that is, by the timely and able medical attendance given the said Flor Espinas y Roma,
which prevented her death, to her damage and prejudice." During arraignment, both appellants
pleaded not guilty. Trial thereafter ensued. Resolving jointly Criminal Cases Nos. SC-4810, SC-4811
and SC- 4812, the trial court convicted both appellants for the murder of Emerita Roma and Ireneo
Gallarte, and the attempted murder of Flor Espinas.

ISSUE:
WON the trial court erred in convicting the two accused-appellants despite the failure of the
prosecution to morally ascertain their identities and guilt for the crimes charged.

HELD: When Hermogenes Flora suddenly shot Emerita and Ireneo, both were helpless to defend
themselves. Their deaths were murders, not simply homicides since the acts were qualified by
treachery. Thus, we are compelled to conclude that appellant Hermogenes Flora is guilty beyond
reasonable doubt of double murder for the deaths of Emerita Roma and Ireneo Gallarte, and guilty of
attempted murder of Flor Espinas.
Edwin Flora, equally guilty as his brother, in conspiracy between appellants for murder of Ireneo
Gallarte. For conspiracy to exist, it is not required that there be an agreement for an appreciable
period prior to the occurrence. It is sufficient that at the time of the commission of the offense, the
accused and co-accused had the same purpose and were united in execution. Edwin Flora
demonstrated not mere passive presence at the scene of the crime. He stayed beside his brother
Hermogenes, right behind the victims while the dance party drifted late into the night till the early
hours of the morning the following day. All the while, he and his brother gazed ominously at Ireneo
Gallarte, like hawks waiting for their prey. And then Edwin's flick of that lighted cigarette to the
ground signaled Hermogenes to commence shooting at the hapless victims. If ever Edwin appeared
acquiescent during the carnage, it was because no similar weapon was available for him. And he fled
from the crime scene together with his brother but not after violently neutralizing any obstacle on
their way. While getting away, Edwin grabbed Rosalie Roma and poked a knife at her neck when the
latter hysterically shouted "si Bodoy, Si Bodoy," in allusion to Hermogenes Flora, whom she saw as
the gunwielder. All told, Edwin, by his conduct, demonstrated unity of purpose and design with his
brother Hermogenes in committing the crimes charged. He is thus liable as co-conspirator. However,
we cannot find Edwin Flora similarly responsible for the death of Emerita Roma and the injury of
Flor Espinas. The evidence only shows conspiracy to kill Ireneo Gallarte and no one else. For acts
done outside the contemplation of the conspirators only the actual perpetrators are liable. And the
rule has always been that co-conspirators are liable only for acts done pursuant to the conspiracy.
For other acts done outside the contemplation of the co-conspirators or which are not the necessary
and logical consequence of the intended crime, only the actual perpetrators are liable.
Therefore, Appellants Hermogenes Flora and Edwin Flora are found GUILTY beyond reasonable
doubt of the MURDER Hermogenes Flora is found GUILTY beyond reasonable doubt of the MURDER
of Emerita Roma and the ATTEMPTED MURDER of Flor Espinas. Appellant Edwin Flora is
ACQUITTED of the murder of Emerita Roma and the attempted murder of Flor Espinas.
CRIMINAL LAW 1
(Sunday, Section 52) 84

PEOPLE VS SABALONES

FACTS:
On June 1, 1985 at 11:45 PM, respondents including Rolusape Sabalones, armed with firearms,
attacked and ambushed individuals riding in two vehicles resulting to the death of two persons and
injury to three others.

According to a witness presented, Sabalones was implicated in the killing of Nabing Velez because of
the slapping incident involving her father-in-law, Federico Sabalones, Sr. and Nabing Velez, which
took place prior to the death of Junior Sabalones (whose wake was during time of the commission of
the crime).

The conclusion of the trial court and the Court of Appeals that the appellants killed the wrong persons
was based on the extrajudicial statement of Appellant Beronga and the testimony of Jennifer Binghoy.
These pieces of evidence sufficiently show that appellants believed that they were suspected of
having killed the recently slain Nabing Velez, and that they expected his group to retaliate against
them.

The Trial Court observed that "they went to their grisly destination amidst the dark and positioned
themselves in defense of his turf against the invasion of a revengeful gang of supporters of the
recently slain Nabing Velez."

ISSUE:
WON the case is „one of aberratio ictus

HELD:
NO. The case is not one of aberration ictus but one of error in personae or mistake in identity, as
observed by the OSG.

Transferred intent is used when a defendant intends to harm one victim, but then unintentionally
harms a second victim instead. In this case, the defendant's intent transfers from the intended victim
to the actual victim and can be used to satisfy the mens rea element of the crime that the defendant
is being charged with. The transferred intent doctrine is only used for completed crimes, and is not
used for attempted crimes.

Aberratio ictus means mistake in the blow, characterized by aiming at one but hitting the other due
to imprecision of the blow. In the case at bar, the appellants opened fire because they mistook the
vehicles to be carrying the avenging men of Nabing Velez. The fact that they were mistaken does not
diminish their culpability. The Court has held that "mistake in identity of the victim carries the same
gravity as when the accused zeroes in on his intended victim."

Facts:
Beronga, Sabalones, Alegarbes, and Cabanero were convicted after a shooting incident in Cebu in
1985 which led to the death of Glenn Tiempo and Alfredo Nardo, and fatal injuries of Nelson Tiempo,
Rey Bolo and Rogelio Presores. The victims were asked to bring the car of a certain Stephen Lim who
also attended a wedding party. Nelson Tiempo drove the car with Rogelio Presores. Alfredo Nardo
drove the owner-type jeep along with Glenn Tiempo and Rey Bolo to aid the group back to the party
after parking the car at Lim’s house. When they reached the gate, they were met with a sudden burst
of gunfire. The accused were identified as the gunmen. The Court of Appeals affirmed the decision of
the trial court. Sabalones and Beronga appealed.
CRIMINAL LAW 1
(Sunday, Section 52) 85

Crime Committed: Two counts of murder, and three counts of frustrated murder

Contention of the People: Prosecution witnesses Edwin Santos and Rogelio Presores testified about
the shooting and identified the faces of the accused. Presores was riding in the car that is behind the
jeep. He positively identified Sabalones as one of the gunmen. When the gunmen fired at the car,
driver Nelson Tiempo immediately maneuvered and arrived at Major Juan Tiempo’s house from
which they have escaped death.

Contention of the Accused: Accused-appellants Sabalones and Beronga denied their presence during
the commission of the crime. Sabalones presented numerous witnesses who stated that he was sound
asleep when the incident took place [since he got tired watching over his brother’s wake]. While
Beronga testified that he attended a cock-derby in Cebu, and was fetched by his wife at 7 pm, arrived
home by 10:30 pm to sleep. Sabalones even escaped from place to place to flee from the wrath of Maj.
Juan Tiempo, the father of the two victims. The defense even pointed out errors from the testimonies
of the witnesses arguing that the place where the incident happened is dim and not lighted.

RULING:
The appeal is DENIED. Costs against appellants

Issue 1: Whether the prosecution witnesses and evidences are credible?


Yes. RTC findings were binding to court with appreciated testimonies of two witnesses. There was
positive identification by survivors who saw them when they peered during lulls in gunfire. The place
was well-lit, whether from post of car’s headlights. The extrajudicial confession has no bearing
because the conviction was based on positive identification. It is binding though to the co-accused
because it is used as cirmustancial evidence corroborated by one witness. The inconcistencies are
minor and inconsequential which strengthen credibility of testimony. Furthermore, in aberratio ictus
[mistake in blow], mistake does not diminish culpability; same gravity applies, more proper to use
error in personae. Alibi cannot prevail over positive identification by the prosecution witnesses.

Issue 2: Whether the alibis are acceptable?


No. It was still quite near the crime scene. It is overruled by positive identification. Using the case of
People v. Nescio, Alibi is not credible when the accused-appellant is only a short distance from the
scene of the crime. Furthermore, flight indicates guilt.
Issue 3:Whether the correct penalty is imposed?
No. Under Article 248 of the RPC, the imposable penalty is reclusion temporal in its maximum period,
to death. There being no aggravating or mitigating circumstance, aside from the qualifying
circumstance of treachery, the appellate court correctly imposed reclusion perpetua for murder. The
CA erred in computing the penalty for each of the three counts of frustrated murder. Under Article
50 of the RPC, the penalty for frustrated felony is next lower in degree than that prescribed by law
for the consummated felony. Because there are no mitigating or aggravating conspiracy between the
two accused. It does not matter that the prosecution has failed to show who was between the two
who actually pulled the trigger that killed the child. They are liable as co-conspirators since the act of
a conspirator becomes the act of another regardless of the precise degree of participation in the act.
Also there was a presence of treachery, because of the circumstances that the crime was done at night
time and that the accused hid themselves among the bamboo. Evident premeditation is also an
aggravating circumstance [the accused had planned to kill the victim some days before].
CRIMINAL LAW 1
(Sunday, Section 52) 86

PEOPLE v PLATEROS
83 SCRA 401 AQUINO; May 30 1978

FACTS:
One night, Pedro Candel together with other pedicab drivers and Tomas Metucua, a second year
college student drank beer in the kitchenette. Seated at another table were Warlito Plateros and
Murillo Lahoy who were also drinking beer. - Metucua and Plateros were rivals for the affection of
Estrella Silamro, the cashier in the kitchenette. When Metucua was talking with Estrella, his alleged
sweetheart, Plateros went near them and refused to leave them, thereby annoying Metucua. - At
about midnight. Piquero, Candel and Añora, accompanied by Metucua, left the kitchenette and went
to their pedicab. Candel was seated in the sidecar of the tricycle. Metucua sat on the driver's seat.
Lahoy and Plateros came out of the kitchenette. Lahoy appeared to be angry, hostile and menacing.
Without any warning, he stabbed Candel (maybe thinking that it was Metucua who was inside the
pedicab because Candel is the driver – abberatio personae) two times. Plateros also stabbed Candel.
Moved by the instinct of selfpreservation, Candel jumped out of the sidecar. He fell on the ground face
down. Lahoy allegedly stabbed Metucua. Then, Plateros and Lahoy fled from the scene of the assault,
- Candel was brought to the hospital but he died on that same morning. Procedure - Two informations
were filed in the Court of First Instance of Bohol accusing Plateros and Lahoy of (1) Murder of Candel
and (2) Attempted Murder of Metucua. - The trial court tried the two cases jointly and rendered only
one decision. Plateros and Lahoy were found guilty of murder, sentencing each of them of reclusion
perpetua. In that same decision, the trial court convicted Lahoy of attempted murder (Plateros, his
co-accused, was acquitted) of Metucua. - Lahoy appealed to the Court of Appeals and the CA acquitted
him. - The murder case was elevated to SC for review. Together with it, the Solicitor General elevated
the attempted murder case be he believed that the decision of CA is void because Lahoy’s appeal
ought to have been certified to the Supreme Court by the CA because the attempted murder imputed
to Lahoy was committed on the same occasion and arose out of the same occurrence as the murder
imputed to him and Plateros in this case, as contemplated in section 17(1), formerly section 17(4) of
the Judiciary Law, which reads: "SEC. 17, Jurisdiction of the Supreme Court. -x x x " x x xx x x x x x
"The Supreme Court shall have exclusive jurisdiction to review, revise, reverse, modify or affirm on
appeal, as the law or rules of court may provide, final judgments and decrees of inferior courts herein
provided, in "(1) All criminal cases involving offenses for which the penalty imposed is death or life
imprisonment; and those involving other offenses which, although not so punished, arose out of the
same occurrence or which may have been committed by the accused on the same occasion, as that
giving rice to the more serious offense, regardless of whether the accused are charged as principals,
accomplices or accessories, or whether have been tried jointly or separately; x x x." - In other words,
the attempted murder case like the instant murder case, comes within the exclusive appellate
jurisdiction of the SCt and should have been decided together with the instant murder case.

ISSUES:
On Attempted Murder Case 1. WON the decision of the Court of Appeals acquitting Lahoy of
attempted murder should be set aside for lack of appellate jurisdiction or as a “lawless thing” On
Murder case 2. WON the guilt of Lahoy and Plateros was proven beyond reasonable doubt 3. WON
there was conspiracy between Lahoy and Plateros 4. WON the crime should be categorized as simple
homicide only and not murder

HELD:
1. NO. Ratio: The rule in section 17(1) is designed to avoid conflicts between the decisions of this
Court and the Court of Appeals in cases involving offenses which arose from the same occurrence or
which were committed on the same occasion usually by the same accused. However, that general rule
has an exception. Where, by allowing the Court of Appeals to decide a can involving an offense, which
CRIMINAL LAW 1
(Sunday, Section 52) 87

is not punishable by death or reclusion perpetua but which arose out of the same occurrence or was
committed on the same occasion, as the case involving an offense punishable by death or reclusion
perpetua pending in this Court, there will be no conflict between the decisions of this Court and the
Court of Appeals, the former case need not be elevated to this Court (People vs. Cariño, 101 Phil.
1206). The rationale of that exception to the general rule is found in the maxim: Cessanie ratione
legis, cessat et ipsa lex. (The reason for the law ceasing, the law itself also ceases.) Reasoning: - The
doctrine of the Cariño case may be applied in this case because here there can be no conflict between
the decision of the Court of Appeals and this Court's decision in the instant murder case inasmuch as
the victims in the two cases are different. The attempted murder case decided by the Court of Appeals
involved the wounding of a certain Tomas Metucua whereas, in the instant murder case the victim
was Pedro Candel. The acquittal of Lahoy in connection with the wounding of Metucua would not
affect the determination of his guilt or innocence in connection with the death of Pedro Candel. - This
holding does not in anyway emasculate the rule in section 17(1) that criminal cases appealed to the
Court of Appeals, involving offenses which arose out of the same occurrence, or which were
committed on the same occasion as the offense punished by death or reclusion perpetua should be
certified to this Court by the Court of Appeals. It is this Court that would determine whether or not
the cases appealed to the Court of Appeals should be decided together with the case appealed to this
Court. 2. YES The feeble denials of Plateros and Lahoy (who admittedly were near the owns of the
crime, when it was perpetrated) cannot prevail over the positive and unequivocal declarations of the
eyewitnesses, Añora and Piquero, that the appellants were the authors of the stab wounds which
caused Candal's death. Their guilt was proven beyond reasonable doubt. 3. YES. There was a
conspiracy between Plateros and Lahoy as shown in their concerted efforts to injure Candel. Plateros
and Lahoy, as boon companions, had been together since four o'clock in the afternoon. They had gone
to different places and repaired twice to the kitchenette. They were together when they left the scene
of the stabbing. 4. NO Lahoy and Plateros, who could have stabbed Candel or Metucua inside the
kitchenette, did not do so. They waited for Metucua and the pedicab drivers to leave the kitchenette.
Their intention was to make a surprise attack without any risk to themselves. The assault was
deliberate, sudden and unexpected. That is the characteristics manifestation of treachery (alevosia).
Hence, the killing was properly categorized as murder by the trial court (Art. 14(16), Revised Penal
Code). Dispositive WHEREFORE, the trial court's judgment is affirmed with costs against the
appellants. They are entitled to credit for their preventive

PEOPLE vs. ALBUQUERQUE


Criminal Liability: How incurred – Wrongful act done be different from what was intended Date:
December 19, 1933

ISSUE:
1. Whether or not the appellant acted in self-defense or is guilty of the crime of homicide?

FACTS:
1. Gines Albuquerque y Sanchez is a widower of 55 years of age and a father of 9 living children. He
has been suffering from partial paralysis for some time and as a result, he has lost control of
movement of his right arm.
2. Appellant lives with daughter Maria along with other children including one named Pilar who
became acquainted and had intimate relations with the deceased Manuel Osma
3. Appellant’s daughter Pilar, hid her pregnancy from her father; the latter only finding out about said
pregnancy after she had given birth already.
4. Appellant wrote letters to the deceased which were hostile and threatening at times and other
times entreating the deceased to legitimize his union with Pilar by marrying her, or at least support
her and his child.
CRIMINAL LAW 1
(Sunday, Section 52) 88

5. Deceased agreed to give the child monthly allowance by way of support but he never complied
with promise
6. Appellant presented himself at the office of deceased one day and on that occasion, appellant
inflicted a wound at the base of the neck of the deceased causing his death.
7. Appellant testified that he proposed to said deceased to marry his daughter and that upon hearing
the latter refuse to do so, the appellant whipped out his penknife.
8. Upon seeing attitude of appellant, the deceased seized appellant by the neck whereupon the
appellant stabbed him on the face with the penknife. BUT due to his lack of control of the movementt
of his arm, the weapon landed on the base of the neck of the deceased
9. Appellant alleged that he did not intend to cause so grave an injury as the death of the deceased.

HOLDING:
The court held appellant did not act in legitimate self-defense inasmuch as he provoked and
commenced the aggression by drawing his penknife but that the (1) mitigating circumstances of lack
of intention to cause so grave an injury as to the death of the deceased, (2) appellant’s act of
voluntarily surrendering himself to the authorities, and (3) him having acted under the influence of
passion and obfuscation should accord the appellant a lowering in the degree of penalty imposed
upon him.

RULING:
The court ruled the case as one of homicide but the degree of penalization lowered.

PEOPLE VS. ORTEGA JR.


276 SCRA 166 G.R. NO. 116736, JULY 24 1997

Facts:
October 15, 1992 at about 5:30 p.m. Andre Mar Masangkay (victim), together with Ariel Caranto,
Romeo Ortega, and Roberto San Andres were having a drinking spree in Daangbakal, Valenzuela.
While they were drinking, Benjamin Ortega, Jr. (accused) and Manuel Garcia who were already drunk
arrived and joined them. When Masangkay went to the back portion of the house to pee, Ortega, Jr.
followed him. Later on, the drinking spree participants heard the Masangkay shouted, “Don’t, Help
me! Caranto and Quitlong went to the back and they saw Ortega, Jr. on top of Masangkay stabbing
him. Caranto ran and fetched Benjamin Ortega Sr., (father). Quitlong fetched Romeo Ortega to pacify
the accused.

Romeo Ortega, Benjamin Ortega Jr., and Manuel Garcia lifted Masangkay from the canal and dropped
him inside the well. Then, they dropped stones in the well to the victim’s body.

Romeo Ortega warned Quitlong not to tell anyone of what he saw. The latter agreed and was allowed
to go home. When Quitlong reached his house, his conscience bothered him, told his mother, and
went to Col. Orig’s house to report the matter. The accused Ortega Jr. and Garcia were apprehended.

The accused Garcia’s defense was that his wife came and asked him to go home because their
daughter was still sick. To alleviate his daughters illness, he fetched his mother-inlaw who performed
a ritual called tawas. After the ritual, he remained at home and attended to his sick daughter. He then
fell asleep but was awakened by police officers at six o clock in the morning of the following day.

While according to Ortega Jr, when he followed Masangkay in the back, he caught him peeping
through the room his sister Racquel. He ignored him and went to pee. After he was through,
CRIMINAL LAW 1
(Sunday, Section 52) 89

Masangkay approached him and asked where his sister was. He answered that he did not know.
Without warning, Masangkay allegedly boxed him in the mouth, an attack that induced bleeding and
caused him to fall on his back. When he was about to stand up, Masangkay drew a knife and stabbed
him, hitting him on the left arm, thereby immobilizing him. Masangkay then gripped his neck with
his left arm and threatened to kill him.Unable to move, Ortega shouted for help. Quitlong came and,
to avoid being stabbed, grabbed Masangkays right hand which was holding the knife. Quitlong was
able to wrest the knife from Masangkay and, with it, he stabbed Masangkay ten (10) times
successively, in the left chest and in the middle of the stomach. When the stabbing started, Ortega
moved to the left side of Masangkay to avoid being hit. Quitlong chased Masangkay who ran towards
the direction of the well. Thereafter, Ortega went home and treated his injured left armpit and lips.
Then, he slept.

The RTC charged Ortega Jr. and Manuel through conspiracy and the taking advantage of superior
strength committed murder

Issue:
Whether Manuel Garcia is criminally liable for the death of Masangkay.

Held:
Yes. Garcia criminally liable deserves an acquittal. The SC applied Article 4, par. 1, of the Revised
Penal Code states that criminal liability shall be incurred by any person committing a felony (delito)
although the wrongful act done be different from that which he intended. The essential requisites for
the application of this provision are that (a) the intended act is felonious; (b) the resulting act is
likewise a felony; and (c) the unintended albeit graver wrong was primarily caused by the actors
wrongful acts. In assisting Appellant Ortega, Jr. carry the body of Masangkay to the well, Appellant
Garcia was committing a felony. The offense was that of concealing the body of the crime to prevent
its discovery, i.e. that of being an accessory in the crime of homicide. Although Appellant Garcia may
have been unaware that the victim was still alive when he assisted Ortega in throwing the body into
the well, he is still liable for the direct and natural consequence of his felonious act, even if the
resulting offense is worse than that intended.

The drowning was the direct, natural and logical consequence of the felony that Appellant Garcia had
intended to commit; it exemplifies praeter intentionem covered by Article 4, par. 1, of the Revised
Penal Code. Under this paragraph, a person may be convicted of homicide although he had no original
intent to kill.

Verdict:
Ortega was guilty of homicide while Garcia was acquitted due to the exempting provision of Article
20 of the Revised Penal code since he is a relative of the principal accused.

BATACLAN V. MEDINA
L-10126, October 22, 1957, 102 Phil. 181

TOPIC/S: Elements of Criminal Liability: Causation (Proximate Cause)


PETITIONER/S: Salud Villanueva Vda. Bataclan and the minors Norma, Luzviminda, Elenita, Oscar
and Alfredo Bataclan, represented by their Natural guardian, Salud Villanueva Vda. Bataclan
RESPONDENT/S: Mariano Medina – defendant-appellant
CRIMINAL LAW 1
(Sunday, Section 52) 90

FACTS:
Summary: While running, one of the front tires of bus no. 30 burst and the vehicle fell into a canal
and overturned. Calls for help were made. Several men, one of them carrying a lighted torch, came to
help. Almost immediately, a fire started and consumed both the bus and the four passengers trapped
inside. It would appear that as the bus overturned, gasoline began to leak. The Court held that it was
she overturning of the bus and not the fire, which was the proximate cause of the death of the victims.

● September 13, 1952, around 2:00 AM: While bus no. 30 of the Medina Transportation owned by
Mariano Medina and driven by Conrado Saylon was running within the jurisdiction of Imus, Cavite,
one of the front tires burst and the vehicle began to zigzag until it fell into a canal or ditch on the right
side of the road and turned turtle. Bataclan, Lara, the Visayan and Natalia Villanueva, could not get
out of the overturned bus.
● Calls for help were made to the houses in the neighborhood. After half an hour, about ten men came,
one of them carrying a lighted torch made of bamboo with a wick on one end, evidently fueled with
petroleum. These men presumably approached the overturned bus, and almost immediately, a fierce
fire started, consuming the bus, including the four passengers trapped inside it.
● It would appear that as the bus overturned, gasoline began to leak and escape from the gasoline
tank on the side of the chassis, spreading over and permeating the body of the bus and the ground
under and around it, and that the lighted torch brought by one of the men who answered the call for
help set it on fire.
● Bataclan’s widow, Salud Villanueva, in her name and in behalf of her five minor children, brought
the present suit to recover from Mariano Medina compensatory, moral, and exemplary damages and
attorney's fees in the total amount of P87,150.
● The Court of First Instance of Cavite awarded P1,000 to the plaintiffs, plus P600 as attorney's fee,
plus P100, the value of the merchandise being carried by Bataclan to Pasay City for sale.

The plaintiffs and the defendants appealed the decision to the Court of Appeals, but the latter court
endorsed the appeal to the SC because of the value involved in the claim in the complaint.

ISSUE:
Whether it was the overturning of the bus, and not the fire that burned it, was the proximate cause
of Juan Bataclan’s death

HELD:
YES, the proximate cause was the overturning of the bus
● Definition of proximate cause:
That cause, which, in natural and continuous sequence, unbroken by any efficient
Intervening cause, produces the injury, and without which the result would not have occurred.
The proximate legal cause is that acting first and producing the injury, either immediately or by
setting other events in motion, all constituting a natural and continuous chain of events, each
having a close causal connection with its immediate predecessor, the final event in the chain
immediately effecting the injury as a natural and probable result of the cause which first acted,
under such circumstances that the person responsible for the first event should, as an ordinarily
prudent and intelligent person, have reasonable ground to expect at the moment of his act
or default that an injury to some person might probably result therefrom
● The proximate cause of the death of Bataclan was the overturning of the bus, this for the reason
that when the vehicle turned not only on its side but completely on its back, the leaking of the gasoline
from the tank was not unnatural or unexpected; that the coming of the men with a lighted torch was
in response to the call for help, made not only by the passengers, but most probably, by the driver
and the conductor themselves, and that because it was very dark (about 2:30 in the morning), the
CRIMINAL LAW 1
(Sunday, Section 52) 91

rescuers had to carry a light with them; and coming as they did from a rural area where lanterns and
flashlights were not available, they had to use a torch, the most handy and available; and what was
more natural than that said rescuers should innocently approach the overturned vehicle to extend
the aid and effect the rescue requested from them.
● The coming of the men with the torch was to be expected and was a natural sequence of the
overturning of the bus, the trapping of some of its passengers and the call for outside help.
● The burning of the bus can also in part be attributed to the negligence of the carrier, through its
driver and its conductor. The driver and the conductor
were on the road walking back and forth. They, or at least, the driver should and must have known
that in the position in which the overturned bus was, gasoline could and must have leaked from the
gasoline tank. Yet neither the driver nor the conductor would appear to have cautioned or taken steps
to warn the rescuers not to bring the lighted torch too near the bus.
● Moreover, a victim overheard Medina speaking to one of his bus inspectors, telling said inspector
to have the tires of the bus changed immediately because they were already old. If this be true, it goes
to prove that the driver had not been diligent and had not taken the necessary precautions to insure
the safety of his passengers. Had he changed the tires, especially those in front, with new ones, as he
had been instructed to do, probably, despite his speeding, the blow out would not have occurred.

DECISION:
In view of the foregoing, with the modification that the damages awarded by the trial court are
increased from ONE THOUSAND (P1,000) PESOS to Six THOUSAND (P6,000) PESOS, and from Six
HUNDRED PESOS TO EIGHT HUNDRED (P800) PESOS, for the death of Bataclan and for attorney's
fees, respectively, the decision appealed from is hereby affirmed, with costs.

PEOPLE vs ILIGAN

FACTS:
At around 2 in the morning Esmeraldo Quinones and his companions Zaldy Asis and Felix Lukban
were walking home from barangay Sto. Domingo after attending a barrio fiesta. On the way they met
the accused Fernando Iligan and his nephew Edmundo Asis and Juan Macandog. Edmundo Asis
pushed them aside prompting Zaldy Asis to box him. Felix quickly said that they had no desire to
fight. Upon seeing his nephew fall, Fernando Iligan drew from his back a bolo and hacked Zaldy but
missed.

Terrified the trio ran, pursued by the three accused. They ran for a good while and even passed the
house of Quinones, when they noticed that they were no longer being chased the three decided to
head back to Quinones house. On the way back the three accused suddenly emerged from the road
side, Fernando Iligan then hacked Quinones Jr. on the forehead with his bolo causing him to fall down.
Felix and Zaldy ran. Upon returning they saw that Quinones Jr. was already dead with his head busted.

The postmortem examination report and the death certificate indicates that the victim died of “ shock
and massive cerebral hemorrhages due to vehicular accident.”

ISSUE:
Whether or not the accused are liable for the victim’s death given that it was due to a vehicular
accident and not the hacking.

HELD:
YES. We are convinced beyond peradventure that indeed after Quinones, Jr. had fallen from the bolo
hacking perpetrated by Iligan, he was run over by a vehicle. This finding, however, does not in any
CRIMINAL LAW 1
(Sunday, Section 52) 92

way exonerate Iligan from liability for the death of Quinones Jr. This being under ART 4 of the RPC
which states that criminal liability shall be incurred by any person committing a felony although the
wrongful act done be different from that which he intended.

The essential requisites of Art 4 are: that an intentional felony has been committed and that the
wrong done to the aggrieved party be the direct natural and logical consequence of the felony
committed by the offender.

It is held that the essential elements are present in this case. The intentional felony committed was
the hacking of the head of Quinones the fact that it was considered superficial by the physician is
immaterial. The location of the wound intended to do away with him.
The hacking incident happened on the national highway where vehicles pass any moment, the
hacking blow received by Quinones weakened him and was run over by a vehicle. The hacking by
Iligan is thus deemed as the proximate cause of the victim’s death.

Iligan is held liable for homicide absent any qualifying circumstances

URBANO VS. INTERMEDIATE APPELLATE COURT


G.R. No. 72964, 7 January 1988

FACTS:
On October 23, 1980, petitioner Filomeno Urbano (Urbano) was on his way to his ricefield when he
discovered that the place where he stored palay was flooded with water coming from an irrigation
canal. When he investigated the area, he saw Marcelino Javier (Javier) and Emilio Efre (Efre). Javier
admitted that he opened the irrigation canal. A quarrel ensued, and Urbano started to hack Javier
with a bolo. Javier was wounded at the right palm of his hand.

Upon intervention, Urbano and Javier had an amicable settlement. Urbano agreed to shoulder all the
expenses for the medication of the wound of Javier, as well as to pay also whatever loss of income
Javier may have suffered. Javier, on the other hand, signed a statement of his forgiveness towards
Urbano and on that condition, he withdrew the complaint that he filed against Urbano.

After several weeks of treatments and medication, the doctor pronounced that the wound of Javier
was already healed. However, on November 14, 1980, Javier was rushed to the hospital when he had
sudden lockjaw and convulsions. The doctor found the condition to be caused by tetanus toxin which
infected from the healing wound in his right palm of his hand. The following day, on November 15,
1980, Javier died.

The heirs of Javier filed a case of homicide against Urbano. Urbano was charged with homicide and
was found guilty both by the trial court and on appeal by the Intermediate Appellate Court. Urbano
then filed a motion for a new trial based on the affidavit sworn by the Barangay Captain who stated
that he saw the deceased catching fish in the shallow irrigation canals on November 5. The motion
was denied by the respondent court. Hence, this petition.

ISSUES:
Whether or not the wound inflicted by Urbano to Javier may be considered as the proximate cause of
the latter’s death.
CRIMINAL LAW 1
(Sunday, Section 52) 93

HOLDING:
No, the wound inflicted by Urbano cannot be considered as the proximate cause of Javier’s death.

The Court defined proximate cause as “that cause which, in natural and continuous sequence,
unbroken by any efficient intervening cause, produces the injury, and without which the result would
not have occurred.” In this case, the death of the victim must be the direct, natural, and logical
consequence of the wounds inflicted upon him by the accused. And since we are dealing with a
criminal conviction, the proof that the accused caused the victim’ s death must convince a rational
mind beyond reasonable doubt.

The Court ruled that Urbano is not liable for the death of Javier. Urbano is only liable for the physical
injuries inflicted to Javier through the wound on the right palm of his hand. The Court took into
account the average incubation period of tetanus toxin, and medical evidence indicated that patients
affected with tetanus experience its symptoms within 14 days. If, indeed, Javier had incurred tetanus
poisoning out of the wound inflicted by Urbano, he would not have experienced the symptoms on the
23rd day after the hacking incident.

The medical findings lead to a distinct possibility that the infection of the wound by tetanus was an
efficient intervening cause later or between the time Javier was wounded to the time of his death.
The infection was, therefore, distinct and foreign to the crime. However, the act of Javier working in
his farm where the soil is filthy, using his own hands, is an efficient supervening cause which relieves
Urbano of any liability for the death of Javier. There is a likelihood that the wound was but the remote
cause and its subsequent infection, for failure to take necessary precautions, with tetanus may have
been the proximate cause of Javier's death with which the petitioner had nothing to do.

SULPICIO INTOD, petitioner, vs. HONORABLE COURT OF APPEALS and PEOPLE OF THE
PHILIPPINES, respondents.

FACTS:
Sulpicio Intod and 3 other men went to Salvador Mandaya’s house to ask him to come with them to
the house of Bernardina Palangpangan. The group of Sulpicio had a meeting with Manday and told
Mandaya that he wanted Palangpangan to be killed because of a land dispute between them. Mandaya
was required to accompany the 4 men otherwise he would also be killed. At that same day, Intod and
companions, all armed with firearms arrived at Bernardina Palangpangan’s house. The petitioner
fired at the said room. It turned out the Palangpangan was in another city and her home was then
occupied by her son-in-law and his family. No one was in the room when the accused fired. No one
was hit by the gunfire.

Petitioner and his companions were positively identified by witnesses. The Regional Trial Court
convicted Intod of attempted murder. Petitioner Intod seeks a modification of the judgment on the
ground that he is only liable for an impossible crime. Petitioner contends that, Palangpangan's
absence from her room on the night he and his companions fired their guns made the crime
inherently impossible.

The prosecutor though argues that the crime was not impossible instead the facts were sufficient to
constitute an attempt and to convict Intod for attempted murder. Respondent likewise alleged that
there was intent.The crime of murder was not consummated, not because of the inherent
impossibility of its accomplishment, but due to a cause of accident other that petitioner’s and his co-
CRIMINAL LAW 1
(Sunday, Section 52) 94

accused’s own spontaneous desistance. Palangpangan did not sleep at her house at that time. Had it
not been for this fact, the crime would have been possible.

ISSUE:
Is petitioner is liable only for an impossible crime?

HELD:
Under Article 4 of the Revise Penal Code, the act performed by the offender cannot produce an offense
against person or property because:
1) The commission of the offense is inherently impossible to be accomplishment
2) The means employed is either inadequate or ineffectual.
To be impossible under this clause, there must be either legal impossibility or physical
impossibility of accomplishing the intended act in order to qualify the act as an impossible crime.
Legal impossibility occurs where the intended act, even if complete would not amount to a crime.
Factual impossibility occurs when extraneous circumstances unknown to the actor or beyond his
control prevent the consummation of the intended crime. The case at bar belongs to this category.
Petitioner shoots the place where he thought his
victim would be, although in reality, the victim was not present in said place and thus, the petitioner
failed to accomplish his end.

PEOPLE V. SALADINO
L-3634, 30 MAY 1951, 89 PHIL. 807

FACTS:
In the night of June 23, 1948 Corporal Saladino and Private Alejo were sleeping in a house in Paoay,
Ilocos Norte together with three policemen of the municipality. They're on patrol duty to apprehend
those who fired upon their dwelling on a previous night. They were awakened by cries for help by
Felix Pasion saying he was robbed by Luis Bernabe at around midnight. The next morning Saladino
and Alejo together with the policemen went to Luis Bernabe's house. They brought the latter to
Pasion's house for questioning. Bernabe denied the accusation. Saladino repeatedly boxed and kicked
Bernabe in different parts of his body but he continued to deny. Saladino hit Bernabe with a piece of
wood and the former called Alejo to take his turn. Alejo reluctantly hit Bernabe four times and left.
Saladino continued to question and beat Bernabe. Saladino then tied Bernabe's wrists together and
made him hang on the ceiling. He continued to hit Bernabe. One of the policemen told Saladino to
stop and just bring Bernabe to their HQ to continue their investigation. Saladino refused and
continued beating Bernabe. Bernabe was untied and was made to sit on a chair. He continued to deny
so Saladino kicked the chair causing the former to fall on the floor. Saladino kicked Bernabe until the
latter remained motionless on the floor. The former stepped on the latter's throat and chest to show
that the latter was faking death. Bernabe was left alone for 15 minutes but he didn't move nor
breathe. An old man checked Bernabe's pulse and said that the latter was dead. Saladino ordered
Alejo to shoot Bernabe's dead body and just say that he ran away. Alejo complied and shot the body
four times.

ISSUES:
WON Alejo has criminal liability.
WON there was due process given to Bernabe.
WON there was conspiracy between Saladino and Alejo.

Ratio:
Bernabe died as a consequence of Saladino's violent mauling. The latter must be declared guilty of
CRIMINAL LAW 1
(Sunday, Section 52) 95

assassination. Alejo did not appear to have conspired with Saladino and is not liable for the death of
Bernabe but Alejo is guilty as accessory after the fact because by following the orders of Saladino, the
former tried to conceal the crime.

HELD:
Saladino was convicted of the murder of Bernabe and given the penalty that will be in accordance
with the law. Alejo was declared an accessory after the fact and given a penalty of imprisonment not
less than 3 years of prision correctional nor more than 6 years and 2 months of prision mayor.
Attempted and frustrated felonies in general- Arts.6 and 7

GEMMA T. JACINTO, Petitioner vs. PEOPLE OF THE PHILIPPINES, Respondent

Facts:
Baby Aquino handed petitioner Gemma Jacinto a Banco De Oro (BDO) Check in the amount of
P10,000.00. The check was payment for Baby Aquino's purchases from Mega Foam Int'l., Inc., and
petitioner was then the collector of Mega Foam. Somehow, the check was deposited in the Land Bank
account of Generoso Capitle, the husband of Jacqueline Capitle; the latter is the sister of petitioner
and the former pricing, merchandising and inventory clerk of Mega Foam.
Later, Rowena Ricablanca, another employee of Mega Foam, received a phone call from an employee
of Land Bank, who was looking for Generoso Capitle. The reason for the call was to inform Capitle
that the subject BDO check deposited in his account had been dishonored. Ricablanca then called and
relayed the message through accused Anita Valencia, a former employee/collector of Mega Foam,
because the Capitles did not have a phone; but they could be reached through Valencia, a neighbor
and former co-employee of Jacqueline Capitle at Mega Foam.
Valencia then told Ricablanca that the check came from Baby Aquino, and instructed Ricablanca to
ask Baby Aquino to replace the check with cash. Valencia also told Ricablanca of a plan to take the
cash and divide it equally into four: for herself, Ricablanca, petitioner Jacinto and Jacqueline Capitle.
Ricablanca, upon the advise of Mega Foam's accountant, reported the matter to the owner of Mega
Foam, Joseph Dyhengco.int
Thereafter, Joseph Dyhengco talked to Baby Aquino and was able to confirm that the latter indeed
handed petitioner a BDO check for P10,000.00 as payment for her purchases from Mega Foam. Baby
Aquino further testified that petitioner Jacinto also called her on the phone to tell her that the BDO
check bounced. Verification from company records showed that petitioner never remitted the
subject check to Mega Foam. However, Baby Aquino said that she had already paid Mega Foam
P10,000.00 cash as replacement for the dishonored check.
Dyhengco filed a Complaint with the National Bureau of Investigation (NBI) and worked out an
entrapment operation with its agents. Ten pieces of P1,000.00 bills provided by Dyhengco were
marked and dusted with fluorescent powder by the NBI. Thereafter, the bills were given to
Ricablanca, who was tasked to pretend that she was going along with Valencia's plan.
Ricablanca, petitioner, her husband, and Valencia then boarded petitioner's jeep and went on to Baby
Aquino's factory. Only Ricablanca alighted from the jeep and entered the premises of Baby Aquino,
pretending that she was getting cash from Baby Aquino. However, the cash she actually brought out
from the premises was the P10,000.00 marked money previously given to her by Dyhengco.
Ricablanca divided the money and upon returning to the jeep, gave P5,000.00 each to Valencia and
petitioner. Thereafter, petitioner and Valencia were arrested by NBI agents, who had been watching
the whole time.A case was filed against the three accused, Jacinto, Valencia and Capitle. RTC rendered
its Decision finding them GUILTY beyond reasonable doubt of the crime of QUALIFIED THEFT and
sentenced each imprisonment of FIVE (5) YEARS, FIVE (5) MONTHS AND ELEVEN (11) DAYS, as
minimum, to SIX (6) YEARS, EIGHT (8) MONTHS AND TWENTY (20) DAYS, as maximum.
CRIMINAL LAW 1
(Sunday, Section 52) 96

The three appealed to the CA and the decision of the trial court was MODIFIED, in that:(a) the
sentence against accused Gemma Jacinto stands; (b) the sentence against accused Anita Valencia
isreduced to 4 months arresto mayor medium, and (c) The accused Jacqueline Capitle is acquitted.
Hence, the present Petition for Review on Certiorari filed by petitioner alone,

Issue:
Whether or not a worthless check can be the object of theft.

Held:
As may be gleaned from the aforementioned Articles of the Revised Penal Code, the personal property
subject of the theft must have some value, as the intention of the accused is to gain from the thing
stolen. This is further bolstered by Article 309, where the law provides that the penalty to be imposed
on the accused is dependent on the value of the thing stolen.
In this case, petitioner unlawfully took the postdated check belonging to Mega Foam, but the same
was apparently without value, as it was subsequently dishonored. Thus, the question arises on
whether the crime of qualified theft was actually produced. The Court must resolve the issue in the
negative.
Intod v. Court of Appeals is highly instructive and applicable to the present case. In Intod (see
doctrines laid out in Intod), the Court went on to give an example of an offense that involved factual
impossibility, i.e., a man puts his hand in the coat pocket of another with the intention to steal the
latter's wallet, but gets nothing since the pocket is empty.
Herein petitioner's case is closely akin to the above example of factual impossibility given in Intod.
In this case, petitioner performed all the acts to consummate the crime of qualified theft, which is a
crime against property. Petitioner's evil intent cannot be denied, as the mere act of unlawfully taking
the check meant for Mega Foam showed her intent to gain or be unjustly enriched. Were it not for
the fact that the check bounced, she would have received the face value thereof, which was not
rightfully hers. Therefore, it was only due to the extraneous circumstance of the check being
unfunded, a fact unknown to petitioner at the time, that prevented the crime from being produced.
The thing unlawfully taken by petitioner turned out to be absolutely worthless, because the check
was eventually dishonored, and Mega Foam had received the cash to replace the value of said
dishonored check.
The fact that petitioner was later entrapped receiving the P5,000.00 marked money, which she
thought was the cash replacement for the dishonored check, is of no moment. The Court held in
Valenzuela v. People that under the definition of theft in Article 308 of the Revised Penal Code there
is only one operative act of execution by the actor involved in theft ─ the taking of personal property
of another.” As of the time that petitioner took possession of the check meant for Mega Foam, she
had performed all the acts to consummate the crime of theft, had it not been impossible of
accomplishment in this case. Obviously, the plan to convince Baby Aquino to give cash as replacement
for the check was hatched only after the check had been dishonored by the drawee bank. Since the
crime of theft is not a continuing offense, petitioner's act of receiving the cash replacement should
not be considered as a continuation of the theft. At most, the fact that petitioner was caught receiving
the marked money was merely corroborating evidence to strengthen proof of her intent to gain.
Moreover, the fact that petitioner further planned to have the dishonored check replaced with cash
by its issuer is a different and separate fraudulent scheme. Unfortunately, since said scheme was not
included or covered by the allegations in the Information, the Court cannot pronounce judgment on
the accused; otherwise, it would violate the due process clause of the Constitution. If at all, that
fraudulent scheme could have been another possible source of criminal liability.
CRIMINAL LAW 1
(Sunday, Section 52) 97

ARISTOTEL VALENZUELA y NATIVIDAD, petitioner, vs.


PEOPLE OF THE PHILIPPINES and HON. COURT OF APPEALS NACHURA,
respondents.

FACTS:
On 19 May 1994, at around 4:30 p.m., petitioner and Calderon were sighted outside the Super Sale
Club, asupermarket within the ShoeMart (SM) complex along North EDSA, by Lorenzo Lago (Lago), a
security guardwho was then manning his post at the open parking area of the supermarket. Lago
saw petitioner, who waswearing an identification card with the mark "Receiving Dispatching Unit
(RDU)," hauling a push cart with cases of detergent of the well-known "Tide" brand. Petitioner
unloaded these cases in an open parking space, whereCalderon was waiting. Petitioner then returned
inside the supermarket, and after five (5) minutes, emerged withmore cartons of Tide Ultramatic and
again unloaded these boxes to the same area in the open parking space.

Thereafter, petitioner left the parking area and haled a taxi. He boarded the cab and directed it
towards theparking space where Calderon was waiting. Calderon loaded the cartons of Tide
Ultramatic inside the taxi, thenboarded the vehicle. All these acts were eyed by Lago, who proceeded
to stop the taxi as it was leaving the openparking area. When Lago asked petitioner for a receipt of
the merchandise, petitioner and Calderon reacted byfleeing on foot, but Lago fired a warning shot to
alert his fellow security guards of the incident. Petitioner andCalderon were apprehended at the
scene, and the stolen merchandise recovered.

The filched items seized fromthe duo were four (4) cases of Tide Ultramatic, one (1) case of Ultra
25 grams, and three (3) additional cases of detergent, the goods with an aggregate value of
P12,090.00.

In a Decision promulgated on 1 February 2000, the Regional Trial Court (RTC) of Quezon City, Branch
90,convicted both petitioner and Calderon of the crime of consummated theft. They were sentenced
to anindeterminate prison term of two (2) years of prision correccional as minimum to seven (7)
years of prision mayor as maximum.

Valenzuela appealed before the Court of Appeals, arguing that he should only be convicted of
frustrated theftsince he was not able to freely dispose of the articles stolen.

Decision dated 19 June 2003,the Court of Appeals rejected this contention and affirmed petitioner’s
conviction,thus the Petition for Review was filed before the Supreme Court.

ISSUE:
Whether or not the crime committed has a frustrated stage.

HELD:
NO.The petition was DENIED. Article 6 of the Revised Penal Code provides that a felony is
consummatedwhen all the elementsnecessary for its execution and accomplishment are present.

Article 308 states that, in the crime of theft, the following elements should be present: (1) that
there betaking of personal property; (2) that said property belongs to another; (3) that the taking be
done with intentto gain; (4) that the taking be done without the consent of the owner; and (5) that
the taking be accomplishedwithout the use of violence against or intimidation of persons or force
upon things.
CRIMINAL LAW 1
(Sunday, Section 52) 98

The Court held that theft is produced when there is deprivation of personal property by one with
intent togain. Thus, it is immaterial that the offender is able or unable to freely dispose the property
stolen since hehas already committed all the acts of execution and the deprivation from the owner
has already ensued fromsuch acts. Therefore, theft cannot have a frustrated stage, and can only be
attempted or consummated.

RIVERA v PEOPLE GR No 166326,, 25 Jan 2006, 480 SCRA 188


ESMERALDO RIVERA, ISMAEL RIVERA, EDGARDO RIVERA, Petitioners,
vs.
PEOPLE OF THE PHILIPPINES, Respondent.
Subject Matter: Attempted v. Frustrated Murder, Art. 6 of the Revised Penal Code

Facts:
As Ruben Rodil went to a nearby store to buy food, Edgardo Rivera mocked him for being jobless and
dependent on his wife for support. Ruben resented the rebuke and thereafter, a heated exchange of
words ensued. In the following day, when Ruben and his three year old daughter went to the store to
buy food, Edgardo together with his brother Esmeraldo and Ismael Rivera emerged from their house
and ganged up on him. Esmeraldo and Ismael mauled Ruben with fist blows. And as he fell to the
ground, Edgardo hit him three times with a hollow block on the parietal area. The Rivera brothers
fled when policemen came. The doctor declared that the wounds were slight and superficial, though
the victim could have been killed had the police not promptly intervened.

Issues:
(1) WON there was intent to kill.
(2) WON the Court of Appeals was correct in modifying the crime from frustrated to attempted
murder.
(3) WON the aggravating circumstance of treachery was properly applied.

Held:
(1) Yes. The Court held that there was intent to kill as Esmeraldo and Ismael pummeled the victim
with fist blows, while Edgardo hit him three times with a hollow block. Even though the wounds
sustained by the victim were merely superficial and could not have produced his death, intent to kill
is presumed.
(2) Yes. The Court of Appeals was correct since based on Art. 6 of the RPC, there is an attempt when
the offender commences the commission of the felony directly by overt acts and does not perform all
the acts of execution which should produce the felony by reason of some cause or accident other than
his own spontaneous desistance.
(3) Yes. The essence of treachery is the sudden and unexpected attack, which gives no opportunity
for the victim to repel it or defend himself. In the present case, the sudden attack to the victim caused
him to be overwhelmed and had no chance to defend himself and retaliate. Thus, there was treachery.
CRIMINAL LAW 1
(Sunday, Section 52) 99

US vs EDUAVE, GR. No. 12155, 2 Feb. 1917, 36 Phil. 2019


THE UNITED STATES, plaintiff-appellee,
vs.
PROTASIO EDUAVE, defendant-appellant.

Valenzuela vs People, G.R. No. 160188, 21 June 2007


ARISTOTEL VALENZUELA y NATIVIDAD, petitioner,
vs.
PEOPLE OF THE PHILIPPINES and HON. COURT OF APPEALS NACHURA, respondents.

FACTS:
Sometime in May 1994, petitioner and Jovy Calderon were sighted outside SM North EDSA by
security guard Lorenzo Lago, unloading cases of detergent Tide Ultramatic on an open parking space.
Minutes later, petitioner loaded the cartons of detergent while Calderon was looking into a taxi and
procedeed to leave the parking area. Lago stopped the cab, checked the cartons, and asked for a
receipt but Valezuela and Calderon reacted a fled on foot. The same were apprehended on the scene
and the stolen merchandise recovered was worth P12,090.
CRIMINAL LAW 1
(Sunday, Section 52) 100

The two pleaded not guilty. Valenzuela said that he was with a friend to buy snacks when they heard
a gunshot fired by Lago. Calderon, on the other hand, said that he was with his cousin when he heard
a gunshot fired by Lago that caused everyone to flee on the viscinity.
RTC convicted the appellants guilty of consumated theft. CA affirmed. Petitioner contends that he was
only guilty of frustrated theft since at the time he was apprefended, he was never placed in a position
to freely dispose the articles stolen.

ISSUE:
WON Valenzuela is guilty of consumated theft.

RULING:
YES. Article 6 defines those three stages, namely the consummated, frustrated and attempted
felonies. A felony is consummated when all the elements necessary for its execution and
accomplishment are present. It is frustrated when the offender performs all the acts of execution
which would produce the felony as a consequence but which, nevertheless, do not produce it by
reason of causes independent of the will of the perpetrator. Finally, it is attempted when the offender
commences the commission of a felony directly by overt acts, and does not perform all the acts of
execution which should produce the felony by reason of some cause or accident other than his own
spontaneous desistance.

Each felony under the Revised Penal Code has a subjective phase, or that portion of the acts
constituting the crime included between the act which begins the commission of the crime and the
last act performed by the offender which, with prior acts, should result in the consummated crime.
After that point has been breached, the subjective phase ends and the objective phase begins. It has
been held that if the offender never passes the subjective phase of the offense, the crime is merely
attempted On the other hand, the subjective phase is completely passed in case of frustrated crimes,
for in such instances, [s]ubjectively the crime is complete.
The determination of whether a crime is frustrated or consummated necessitates an initial
concession that all of the acts of execution have been performed by the offender.
The long-standing Latin maxim actus non facit reum, nisi mens sit rea supplies an important
characteristic of a crime, that ordinarily, evil intent must unite with an unlawful act for there to be a
crime, and accordingly, there can be no crime when the criminal mind is wanting.
We thus conclude that under the Revised Penal Code, there is no crime of frustrated theft. That it has
taken all these years for us to recognize that there can be no frustrated theft under the Revised Penal
Code does not detract from the correctness of this conclusion.
Hence, the petition is DENIED

BALEROS vs PEOPLE, G.R. No 138033, 22 Feb. 2006, 483 SCRA 10


RENATO BALEROS, JR., Petitioner,
vs.
PEOPLE OF THE PHILIPPINES, Respondent.

FACTS:
• Martina Lourdes Albano (Malou), a medical student of the University of Sto. Tomas, stayed at
Room 307 with her maid Marvilou.
• December 12 10:30 pm: Malou slept. Her maid Marvilou slept on a folding bed right in front of
her bedroom door.
• December 13, 1991 1:00 am: Chito left the fraternity party with Robert Chan and Alberto
wearing a barong tagalog, with t-shirt inside, with short pants with stripes lent by Perla Duran and
leather shoes.
CRIMINAL LAW 1
(Sunday, Section 52) 101

• December 13, 1991 1:30 am: Chito arrived at the Building wearing a white t-shirt with fraternity
symbols and black shorts with the brand name “Adidas” from a party. He requested permission
from S/G Ferolin to go up to Room 306 leased by Ansbert Co but at that time only Joseph Bernard
Africa was there. Although Chito could not produce the required written authorization, he let him
in because he will be a tenant in the coming summer break. Joseph was awaken by Chito’s knock so
he glanced the alarm clock and let him. He saw him wearing dark-colored shorts and white T-shirt.
• December 13, 1991 1:50 am: Renato Baleros, Jr. y David (Chito) forcefully covered the face of
Martina Lourdes T. Albano with a piece of cloth soaked in chemical with dizzying effects. This
awakened Malou. She struggled but could not move because she was tightly held and pinned down
on the bed. She kicked him and got her right hand free to squeeze his sex organ causing him to let
her go. She went for the bedroom door and woke up Marvilou. She also intercommed S/G Ferolin
saying: "may pumasok sa kuarto ko pinagtangkaan ako". Malou proceed to Room 310 where her
classmates Christian Alcala, Bernard Baptista, Lutgardo Acosta and Rommel Montes were staying
and seeked help. She saw her bed in a mess and noticed that her nightdress was stained with
blue. Aside from the window with grills which she had originally left opened, another window
inside her bedroom which leads to Room 306 was now open.
• December 13, 1991 3:30 pm: Christian and his roommates, Bernard and Lutgardo were asked by
the CIS people to look for anything not belonging to them in their Unit when Rommel Montes went
inside and found a grey bag.
o Christian knew right away that it belonged to Chito. It contained white t-shirt with fraternity
symbol, a Black Adidas short pants, a handkerchief , 3 white T-shirts, an underwear and socks.
• Chito pleaded NOT Guilty
• 13 witnesses including Malou and her classmates, Joseph Bernard Africa, Rommel Montes,
Renato Alagadan and Christian Alcala
o Malou: Chito was her classmate whom he rejected a week before
o Chito: He only slept and at about 6 to 6:30, Joseph told him that something had happened and
asked him to follow him to Room 310 carrying his gray bag and since no one was there they went to
Room 401 where Renato Alagadan was. He left his grey bag at Room 306 the day before.
• handkerchief and Malou’s night dress both contained chloroform, a volatile poison which causes
first degree burn exactly like what Malou sustained on that part of her face where the chemical-
soaked cloth had been pressed
• RTC: guilty of attempted rape
• CA: Affirmed

ISSUE:
W/N Chito is guilty of attempted rape

HELD:
NO. REVERSED and SET ASIDE. ACQUITTING Renato D. Baleros, Jr. of the charge for attempted rape.
GUILTY of light coercion and is accordingly sentenced to 30 days of arresto menor and to pay a fine
of P200.00, with the accessory penalties thereof and to pay the costs.

• Under Article 335 of the Revised Penal Code, rape is committed by a man who has carnal
knowledge or intercourse with a woman under any of the following circumstances: (1) By using
force or intimidation; (2) When the woman is deprived of reason or otherwise unconscious; and (3)
When the woman is under twelve years of age or is demented.
• Under Article 6, in relation to the aforementioned article of the same code, rape is attempted
when the offender commences the commission of rape directly by overt acts and does not perform
all the acts of execution which should produce the crime of rape by reason of some cause or
accident other than his own spontaneous desistance.
CRIMINAL LAW 1
(Sunday, Section 52) 102

o whether or not the act of the petitioner, i.e., the pressing of a chemical-soaked cloth while on top
of Malou, constitutes an overt act of rape.
o Overt or external act has been defined as some physical activity or deed, indicating the intention
to commit a particular crime, more than a mere planning or preparation, which if carried out to its
complete termination following its natural course, without being frustrated by external obstacles
nor by the voluntary desistance of the perpetrator, will logically and necessarily ripen into a
concrete offense
• Chito was fully clothed and that there was no attempt on his part to undress Malou, let alone
touch her private part
• Verily, while the series of acts committed by the petitioner do not determine attempted rape,
they constitute unjust vexation punishable as light coercion under the second paragraph of Article
287 of the Revised Penal Code.
o As it were, unjust vexation exists even without the element of restraint or compulsion for the
reason that this term is broad enough to include any human conduct which, although not
productive of some physical or material harm, would unjustly annoy or irritate an innocent person
o That Malou, after the incident in question, cried while relating to her classmates what she
perceived to be a sexual attack and the fact that she filed a case for attempted rape proved beyond
cavil that she was disturbed, if not distressed

THE UNITED STATES, plaintiff-appellee, vs


TOMAS ADIAO, defendant-appellant
CRIMINAL LAW 1
(Sunday, Section 52) 103

VALENZUELA vs. PEOPLE


G.R. No. 160188 June 21, 2007

FACTS:
On May 19, 1994, at around 4:30 p.m., petitioner and Calderon were sighted outside the Super Sale
Club, a supermarket within the ShoeMart (SM) complex along North EDSA, by Lorenzo Lago (Lago),
a security guard who was then manning his post at the open parking area of the supermarket. Lago
saw petitioner, who was wearing an identification card with the mark “Receiving Dispatching Unit
(RDU),” hauling a push cart with cases of detergent of the well-known “Tide” brand . Petitioner
unloaded these cases in an open parking space, where Calderon was waiting. Petitioner then returned
inside the supermarket, and after five (5) minutes, emerged with more cartons of Tide Ultramatic
and again unloaded these boxes to the same area in the open parking space.
Thereafter, petitioner left the parking area and haled a taxi. He boarded the cab and directed it
towards the parking space where Calderon was waiting. Calderon loaded the cartons of Tide
Ultramatic inside the taxi, then boarded the vehicle . All these acts were eyed by Lago, who proceeded
to stop the taxi as it was leaving the open parking area. When Lago asked petitioner for a receipt of
the merchandise, petitioner and Calderon reacted by fleeing on foot, but Lago fired a warning shot to
alert his fellow security guards of the incident. Petitioner and Calderon were apprehended at the
scene, and the stolen merchandise recovered . The filched items seized from the duo were four (4)
cases of Tide Ultramatic, one (1) case of Ultra 25 grams, and three (3) additional cases of detergent,
the goods with an aggregate value of P12,090.00.

In arguing that he should only be convicted of frustrated theft , petitioner cites two decisions
rendered many years ago by the Court of Appeals: People vs. Diño and People vs. Flores. Both
decisions elicit the interest of the Court, as they modified trial court convictions from consummated
to frustrated theft and involve a factual milieu that bears similarity to the present case. Petitioner
CRIMINAL LAW 1
(Sunday, Section 52) 104

invoked the same rulings in his appeal to the Court of Appeals, yet the appellate court did not
expressly consider the import of the rulings when it affirmed the conviction.

ISSUE
Whether or not petitioner is guilty of frustrated theft only (NO, GUILTY OF CONSUMMATED THEFT)

HELD:
Article 6 defines those three stages, namely the consummated, frustrated and attempted felonies. A
felony is consummated “when all the elements necessary for its execution and accomplishment are
present.” It is frustrated “when the offender performs all the acts of execution which would produce
the felony as a consequence but which, nevertheless, do not produce it by reason of causes
independent of the will of the perpetrator.” Finally, it is attempted “when the offender commences
the commission of a felony directly by overt acts, and does not perform all the acts of execution which
should produce the felony by reason of some cause or accident other than his own spontaneous
desistance.”

Court has long recognized the following elements of theft as provided for in Article 308 of the Revised
Penal Code, namely: 1) that there be taking of personal property; 2) that said property belongs to
another; 3) that the taking be done with intent to gain; 4) that the taking be done without the consent
of the owner; and 5) that the taking be accomplished without the use of violence against or
intimidation of persons or force upon things.

So long as the “descriptive” circumstances that qualify the taking are present, including animo
lucrandi and apoderamiento, the completion of the operative act that is the taking of personal
property of another establishes, at least, that the transgression went beyond the attempted stage. As
applied to the present case, the moment petitioner obtained physical possession of the cases of
detergent and loaded them in the pushcart, such seizure motivated by intent to gain, completed
without need to inflict violence or intimidation against persons nor force upon things, and
accomplished without the consent of the SM Super Sales Club, petitioner forfeited the extenuating
benefit a conviction for only attempted theft would have afforded him.

Now, in order to ascertain whether the theft is consummated or frustrated, it is necessary to inquire
as to how exactly is the felony of theft “produced .” Parsing through the statutory definition of theft
under Article 308, there is one apparent answer provided in the language of the law — that theft is
already “produced” upon the “taking of personal property of another without the latter’s consent.”

For the purpose of ascertaining whether theft is susceptible of commission in the frustrated stage,
the question is again, when is the crime of theft produced? There would be all but certain unanimity
in the position that theft is produced when there is deprivation of personal property due to its taking
by one with intent to gain. Viewed from that perspective, it is immaterial to the product of the felony
that the offender, once having committed all the acts of execution for theft , is able or unable to freely
dispose of the property stolen since the deprivation from the owner alone has already ensued from
such acts of execution. This conclusion is reflected in Chief Justice Aquino’s commentaries, as earlier
cited, that “in theft or robbery the crime is consummated after the accused had material possession
of the thing with intent to appropriate the same , although his act of making use of the thing was
frustrated.”

It might be argued, that the ability of the offender to freely dispose of the property stolen delves into
the concept of “taking” itself, in that there could be no true taking until the actor obtains such degree
of control over the stolen item. But even if this were correct, the effect would be to downgrade the
CRIMINAL LAW 1
(Sunday, Section 52) 105

crime to its attempted , and not frustrated stage, for it would mean that not all the acts of execution
have not been completed, the “taking not having been accomplished.” Perhaps this point could serve
as fertile ground for future discussion, but the concern now is whether there is indeed a crime of
frustrated theft, and such consideration proves ultimately immaterial to that question. Moreover,
such issue will not apply to the facts of this particular case. The Court is satisfied beyond reasonable
doubt that the taking by the petitioner was completed in this case. With intent to gain, he acquired
physical possession of the stolen cases of detergent for a considerable period of time that he was able
to drop these off at a spot in the parking lot, and long enough to load these onto a taxicab .
Again, there is no language in Article 308 that expressly or impliedly allows that the “free disposition
of the items stolen” is in any way determinative of whether the crime of theft has been produced. The
Court thus concludes that under the Revised Penal Code, there is no crime of frustrated theft. That it
has taken all these years for the Court to recognize that there can be no frustrated theft under the
Revised Penal Code, does not detract from the correctness of this conclusion. It will take considerable
amendments to our Revised Penal Code in order that frustrated theft may be recognized.

PEOPLE vs. CAMPUHAN

Facts:
Campuhan was a helper in the business of the family of the victim, a 4-year-old girl. One time, the
mother of the victim heard the latter cry, “Ayoko!”, prompting her to rush upstairs. There, she saw
Campuhan kneeling before the victim, whose pajamas and pany were already removed, while his
short pants were down to his knees. Campuhan was apprehended. Physical examination of the victim
yielded negative results. No evident sign of extra-genital physical injury was noted. Her hymen was
intact and its orifice was only .5 cm in diameter.
Trial court found him guilty of statutory rape and sentenced him to death.

Issue:
Whether or not Campuhan is guilty of statutory rape.

Held: NO.
The gravamen of the offense of statutory rape is carnal knowledge of woman below 12 as provided
in RPC 335(3). The victim was only 4 years old when the molestation took place, thus raising the
penalty from “reclusion perpetua to death” to the single indivisible penalty of death under RA 7659
Sec. 11, the offended party being below 7 years old. In concluding that carnal knowledge took place,
full penetration of the vaginal orifice is not an essential ingredient, nor is the rupture of hymen
necessary; the mere touching of external genitalia by the penis capable of consummating the sexual
act is sufficient to constitute carnal knowledge. But the act of touching should be understood as
inherently part of the entry of penis into the labias of the female organ, and not mere touching alone
of the mons pubis or the pudendum (the part instantly visible within the surface).
Absent any showing of the slightest penetration of the female organ, i.e., touching of either labia by
the penis, there can be no consummated rape; at most, it can only be attempted rape, if not acts of
lasciviousness.
Here, the prosecution failed to discharge its onus of proving that Campuhan’s penis was able to
penetrate the victim’s vagina however slight. Also, there were no external signs of physical injuries
on the victim’s body to conclude that penetration had taken place.

Issue #2:
What crime did Campuhan commit?
CRIMINAL LAW 1
(Sunday, Section 52) 106

Held #2:
ATTEMPTED RAPE.
Under RPC 6 in relation to RPC 335, rape is attempted when the offender commences the commission
of rape directly by overt acts, and does not perform all acts of execution which should produce the
crime of rape by reason of some cause or accident other than his own spontaneous desistance. All the
elements of attempted rape are present in this case.
The penalty of attempted rape is 2 degrees lower than the imposable penalty of death for the crime
of statutory rape of minor below 7 years. Two degrees lower is reclusion temporal, which is 12 years
1 day to 20 years.
Applying ISLAW, and in the absence of aggravating or mitigating circumstance, the maximum penalty
shall be medium period of reclusion temporal (14 years 8 months 1 day to 17 years 4 months), while
the minimum is the penalty next lower in degree – prision mayor (6 years 1 day to 12 years).

Issue #3:
May there be a crime of frustrated rape?

Held #3:
NO.
In People vs Orita, SC finally did away with frustrated rape. Rape was consummated from the moment
the offender had carnal knowledge of the victim. All elements of the offense were already present
and nothing more was left for the offender to do. Perfect penetration was not essential; any
penetration of the female organ by the male organ, however slight, was sufficient.
For attempted rape, there was no penetration of the female organ because not all acts of execution
were performed or the offender merely commenced the commission of the felony directly by overt
acts.
PEOPLE V. HERNANDEZ
99 Phil. 515

FACTS:
Amado HERNANDEZ (member of the CPP and President of the Congress of Labor Organizations) re-
filed for bail (previous one denied) for his conviction of rebellion complexed with murders, arsons
and robberies. The prosecution said to deny this again because the capital punishment may be
imposed. The defense however contends that rebellion cannot be complexed with murder, arson, or
robbery. The information states that the “…murders, arsons and robberies allegedly perpetrated by
the accused “as a necessary means to commit the crime of rebellion, in connection therewith and in
furtherance thereof.”

ISSUE:
W/N rebellion can be complexed with murder, arson, or robbery.

Held: NO!

RATIO:
Under the allegations of the amended information, the murders, arsons and robberies described
therein are mere ingredients of the crime of rebellion allegedly committed by HERNANDEZ, as means
“necessary” for the perpetration of said offense of rebellion and that the crime charged in the
amended information is, therefore, simple rebellion, not the complex crime of rebellion with multiple
murder, arsons and robberies. Under Article 1346 and 1357, these five (5) classes of acts constitute
only one offense, and no more, and are, altogether, subject to only one penalty. One of the means by
which rebellion may be committed, in the words of said Article 135, is by “engaging in war against
CRIMINAL LAW 1
(Sunday, Section 52) 107

the forces of the government” and “committing serious violence” in the prosecution of said “war”.
These expressions imply everything that war connotes. Since Article 135 constitute only 1 crime,
Article 48 doesn’t apply since it requires the commission of at least 2 crimes.

PEOPLE V CABALLERO

Facts:
(According to Boholst)
The couple had a rough marriage. Soon after, Caballero left, and Boholst and her daughter was left to
the support of her parents.

One night, after carolling, Boholst met Caballero who upon seeing her, manhandled her. There were
an exchange of words and later on, Caballero was already holding her by the hair and slapping her
face until her nose bled. Caballero pushed her to the grounds, and to stop herself from falling, she
held on to his waist. As she did so, she grasped the knife tucked by the left side of his body. She fell to
the ground then Caballero knelt over her and chocked her saying that he will kill her. Because she
had no other recourse, she pulled out the knife of her husband and thrust it at him, hitting the left
side of his body near the belt line.

When she was finally free, she ran home and on the way, she threw the knife. In the morning, she
surrendered to the police and presented the torn and blood-stained dress she wore that night. The
police officer accompanied her to look for the weapon but when it can no longer be found, she was
advised to just give any knife and she did (now marked Exhibit C). (According to the Prosecution’s
witness, Caballero’s friend) On the night of the incident, Boholst was already waiting for Caballero,
and when he approached her, she suddenly stabbed Francisco her with the knife marked by the
prosecution as Exhibit C. His friends brought him to the hospital where he was later interviewed by
the police officer confirming that his wife stabbed him. But because he needs blood transfusion, he
needs to be transferred to another hospital. He died on the way.

Issue:
Did Boholst act in legitimate defense of her person?

Held:
Yes.

Ratio decidendi:
The RTC held that Boholst’s evidence was not clear and convincing:
Testimony improbable as brought out by her demonstration during the trial. No wound or injury on
her body treated by the physician that the knife used was a Moro knife and not exhibit C is incredible
Contradictory statements

Has motive: husband’s abandonment The court departs from the general rule that appellate court
will not disturb the findings of the trial court on facts testified by the witnesses
The trial court judge overlooked an important piece of evidence that could confirm the narration of
the appellant: location of the wound inflicted on the victim.

As she was flat on her back and and her husband choking her, she had no other recourse but to pull
out the knife inserted at the left side of her husband’s belt and stabbed him hitting the left back
portion just below the waist, as also described by the attending physician as the left lumbar region.
CRIMINAL LAW 1
(Sunday, Section 52) 108

The fact that the blow landed in the vicinity from where the knife was drawn is a strong indication of
the truth of her testimony, for as she lay on the ground with her husband bent over her it was quite
natural for her right hand to get hold of the knife tucked in the left side of the man’s belt and thrust
it at that section of the body nearest to her hand at the moment.

This particular location of the wound negates the credibility of the prosecution witness that is if it
was true, then the wound should have been directed towards the front of the body of the victim rather
than at his back. The Court finds the location of the wound as a valuable circumstance which confirms
the plea of self-defense. Appellant also lacks motive. She declared that she still loved her husband and
for several months prior to the incident, she appeared resigned to her fate.

She also surrendered herself immediately the morning after. The court also believed that the knife
must be a blade of six inches as stated by Boholst for it to penetrate through the left lumbar region to
the victim’s large intestine and cause the discharge of fecal matter. All the elements of self-defense
are present: unlawful aggression as pointed out above reasonable necessity for means employed:
woman strangled and chocked by a furious aggressor, rendered almost unconcious by the strong
pressure on her throat. What is vital is the imminent peril to Boholst’s life. The knife afforded
appellant the only reasonable means with which she could free and save herself. Necessity knows no
law.

Lack of sufficient provocation: Boholst did not provoke Caballero. She gave a valid excuse that she
went carolling to earn money for their child.

PEOPLE VS FELIPE KALALO, et.al.

Facts:
On November 10, 1932, the appellants namely, Felipe Kalalo, Marcelo Kalalo, Juan Kalalo, and
Gregorio Ramos, were tried in the Batangas jointly with Alejandro Garcia, Fausta Abrenica and Alipia
Abrenica in criminal cases Nos. 6858, 6859 and 6860, the first two for murder, and the last for
frustrated murder. Upon agreement of the parties said three cases were tried together and after the
presentation of their respective evidence, the said court acquitted Alejandro Garcia, Fausta Abrenica
and Alipia Abrenica, and sentenced the other appellants.

Issue:
W/O accused-appellants are liable of the crimes of murder and discharge of firearms?

Held:
The first case is, for the alleged murder of Marcelino Panaligan, to seventeen years, four months and
one day of reclusion temporal, with the corresponding accessory penalties, and to indemnify the
heirs of the said deceased Marcelino Panaligan in the sum of P1,000, with the costs.

The second case is, for the alleged murder of Arcadio Holgado, to seventeen years, four months and
one day of reclusion temporal, with the corresponding accessory penalties, and to indemnify the
heirs of the aforesaid victim, the deceased Arcadio Holgado, in the sum of P1,000, with the costs.

In the third case, that is, the court held that the crime committed was simply that of discharge of
firearm, not frustrated murder, the appellant Marcelo Kalalo was sentenced to one year, eight months
and twenty-one days of prision correccional and to pay the proportionate part of the costs of the
proceedings. Felipe Kalalo and Juan Kalalo, as well as their co-accused Fausta and Alipia Abrenica,
Gregorio Ramos and Alejandro Garcia, were acquitted of the charges therein.
CRIMINAL LAW 1
(Sunday, Section 52) 109

In all other respects, the appealed sentences in the said three cases are hereby affirmed without
prejudice to crediting the appellants therein with one-half of the time during which they have
undergone preventive imprisonment, in accordance with article 29 of the Revised Penal Code. So
ordered.

PEOPLE VS. SALVILLA


184 SCRA 671 (1990)

Facts:
The accused Bienvenido Salvilla together with his co-accused armed with homemade guns and hand
grenade robbed Rodita Habiero in the latter’s office. In the office of Rodita; her two daughters Mary
and Mimmie were also inside. One of the accused asks Mary to get the paper bag which
contained money. All accused held victims as hostage when the police and military authorities had
surrounded the lumber yard. After the negotiation fails to proceed, the police makes their move in
assaulting the robbers thus Mary and Mimmie are injured as well the accused also got an injury.

Issue:
Whether or not the crime of robbery was consummated

Held:
Yes.

Ratio:
From the moment the offender gained possession of the thing, even if the culprit had no opportunity
to dispose of the same, the unlawful taking is complete.

PEOPLE OF THE PHILIPPINES V. ROMEO GONZALES


G.R. Nos. 113255-56 July 19, 2001

Lessons Applicable: Pro reo doctrine, indeterminate sentence law, buy-bust operation, buy-bust
operation

FACTS:
Early February 1991: the police received an information that Romeo Gonzales was selling large
quantities of marijuana.

February 13, 1991: After 4 days of surveillance, they conducted a buy-bust entrapment operation.
Their informant introduced Sgt. Ortiz to Gonzales as a buyer (poseur-buyer) of 1 kg. marijuana for
P1,200. Then, Ortiz took out his handkerchief as a pre-arranged signal so the team immediately
rushed to the scene introducing themselves as Narcom agents and arrested Gonzales. Sgt. Ortiz
handed over the bag of marijuana to Pfc. Danilo Cruz.

The team confiscated 1 more bag containing 2 blocks of marijuana weighing about 1.5 kg and 10
medium size plastic bags containing 300 grams of marijuana. The tests yielded positive indications
for the presence of tetrahydrocannabinol, or THC Gonzales orally admitted that he was selling
CRIMINAL LAW 1
(Sunday, Section 52) 110

marijuana to different buyers, but claimed that somebody else owned the marijuana he sold. When
asked to identify the owner, he kept silent.

2 informations charging Gonzales with violation of RA 6425:


o Crim. Case No. 91-180: possession, custody and control of 2 block size of marijuana weighing (1.5
kilos) and 10 medium size plastic bags of dry marijuana weighing (300 grams)
o Crim. Case No. 91-181: selling more or less 1 kilo of high-grade marijuana
RTC: Romeo Gonzales guilty for Violation of Sections 8 and 4, Art. II., RA 6425 and imposes penalty
of imprisonment of 6 years and 1 day and a fine of P6,000 for Criminal Case No. 91-180 life
imprisonment and a fine of P20,000 for Criminal Case No. 91-181.
Gonzales: Victim of a frame-up since he was inside the comfort room of a neighbor from whom he
borrowed P100 to buy medicines for his sick mother and he was just wearing underwear when he
was brought out of the house. – NOT proven

A buy-bust operation, normally preceded by surveillance, is an effective mode of apprehending drug


pushers and, “if carried out with due regard to constitutional and legal safeguards, it deserves judicial
sanction.” A warrant of arrest is not essential because the violator is caught in flagrante delicto.
Searches made incidental thereto are valid.

ISSUE:
W/N the Indeterminate Sentence Law should apply to Crim. Case No. 91-180

HELD:
YES. AFFIRMED with MODIFICATION. In Criminal Case No. 91-181,life imprisonment and fine of
P20,000. In Criminal Case No. 91-180, indeterminate penalty of 2 years and 4 months of prision
correccional, as minimum, to 8 years and 1 day of prision mayor, as maximum, and to pay a fine of
P6,000.

The Dangerous Drugs Act, Sec. 8 (special law) prescribes as penalty for possession of Indian hemp
(marijuana), regardless of amount, an imprisonment of 6 years and 1 day to 12 years, and a fine of
P6,000 to P12,000. Applying the pro reo doctrine in criminal law (when in doubt favour the accused),
we hold that the penalty prescribed in R. A. No. 6425, Section 8 while not using the nomenclature of
the penalties under the RPC is actually prision mayor. Consequently, it is the first part of Section 1 of
the Indeterminate Sentence Law, which shall apply in imposing the indeterminate sentence.

PEOPLE of the PHILIPPINES vs LAMAHANG


G.R. No. L-43530, August 03, 1935

FACTS:
The defendant Aurelio Lamahang is on appeal from a decision finding him guilty of attempted robb
ery.
At early dawn on March 2, 1935, policeman Jose Tomambing, who was patrolling his beat on Delgad
o and C.R. Fuentes streets of the City of Iloilo, caughtthe accused in the act of making an opening wit
h an iron bar on the wall of a store of cheap goods located on the last named street.
At that time the owner of the store, Tan Yu, was sleeping inside with another Chinaman.
The accusedhad only succeeded in breaking one board and in unfastening another from the wall, w
hen the policeman showed up, who instantly arrested him and placed him under custody.
CRIMINAL LAW 1
(Sunday, Section 52) 111

ISSUE:
WON the accused was erroneously declared guilty of attempted robbery

RULING:
YES, he was erroneously declared guilty of attempted robbery. The accused is then held guilty of att
empted trespass to dwelling, committed by means of force, with the aforesaid aggravating and mitig
ating circumstances and sentenced to three months and one day of arresto mayor.

RATIONALE:
It is necessary to prove that said beginning of execution, if carried to its complete termination follo
wing its natural course, without being frustrated by external obstacles nor by the voluntary desista
nce of the perpetrator, will logically and necessarily ripen into a concrete offense. In the case of rob
bery, it must be shown that the offender clearly intended to take possession, for the purpose of gain
of some personal property belonging to another. In the instant case, it may only be inferred as a logi
cal conclusion that his evident intention was to enter by means of force said store against the will of
its owner. That his final objective, once he succeeded in entering the store, was to rob, to cause phy
sical injury to the inmates, or to commit any other offense, there is nothing in the record to justify a
concrete finding.

It must be borne in mind (I Groizard, p. 99) that in offenses not consummated, as the material dama
ge is wanting, the nature of the action intended (accion fin) cannot exactly be ascertained, but the sa
me must be inferred from the nature of the acts executed (accion medio). The relation existing betw
een the facts submitted for appreciation and the offense which said facts are supposed to produce
must be direct; the intention must be ascertained from the facts and therefore it is necessary, in ord
er to avoid regrettable instances of injustice.

Under article 280 of the Revised Penal Code, the Court is of the opinion that the fact under consider
ation does not constitute attempted robbery but attempted trespass to dwelling. Against the accuse
d must be taken into consideration the aggravating circumstances of nighttime and former convicti
ons, — inasmuch as the record shows that several final judgments for robbery and theft have been r
endered against him — and in his favor, the mitigating circumstance of lack of instruction.

PEOPLE VS BORINAGA
GR 33463 18 Dec 193

Facts:
The victim Harry Mooney, an American who resided in Calubian Leyte,contracted with Juan Lawaan
for the construction of a fish corral. Lawaanattempted to collect the whole amount of the contract
even though the corralis not yet finished. Upon Mooney‘s refusal to pay, Lawaan warned and
threatened him that something would happen to him.On that evening, Mooney was in the store of
his neighbor, sitting with his back towards a window when suddenly Basilio Borinaga struck him
with a knife. Theknife imbedded on the back of the seat though. Mooney fell off from theimpact but
was not injured. Borinaga left the scene but after ten minutes, hereturned to have another attempt at
Mooney but was warded off by Mooneyand his neighbor frightening him by turning a flashlight on
him.

Issue:
Whether or not the crime is frustrated murder.
CRIMINAL LAW 1
(Sunday, Section 52) 112

Held:
YES. As an essential condition of a frustrated crime, Borinaga performed all theacts of execution,
attending the attack. There was nothing left that he could dofurther to accomplish the work. The
cause resulting in the failure of the attack arose by reason of forces independent of his will. Borinaga
also voluntarilydesisted from further acts. The subjective phase of the criminal act was passed.
Dissenting opinion, J. Villa-Real:

“The acts of execution perfomed by [Borinaga] did not produce the death of
Mooney as a consequence not could they have produced it because the blowdid not reach his body;
therefore, the culprit did not perform all the acts of execution which should produce the felony. There
was lacking the infliction of the deadly wound upon a vital spot of the body of Mooney.”
What the back of the chair prevented was the wounding of Mooney, not hisdeath. It is the preventing of
death by causes independent of the will of theperpetrator, after all the acts of execution which should
produce the felony as aconsequence had been performed, that constitutes a frustrated felony,according
to the law, and not the preventing of the performances of all theacts of execution which constitute a
felony, as in the present case. Attempted murder only.

VALENZUELA VS PEOPLE
GR 160188 21 Jun 2007

Facts:
While a security guard was manning his post the open parking area of a supermarket, he saw the
accused, Aristotel Valenzuela, hauling a push cart loaded with cases of detergent and unloaded them
where his co-accused, Jovy Calderon, was waiting. Valenzuela then returned inside the supermarket,
and later emerged with more cartons of detergent. Thereafter, Valenzuela hailed a taxi and started
loading the boxes of detergent inside. As the taxi was about to leave the security guard asked
Valenzuela for the receipt of the merchandise. The accused reacted by fleeing on foot, but were
subsequently apprehended at the scene. The trial court convicted both Valenzuela and Calderon of
the crime of consummated theft. Valenzuela appealed before the Court of Appeals, arguing that he
should only be convicted of frustrated theft since he was not able to freely dispose of the articles
stolen. The CA affirmed the trial court’s decision, thus the Petition for Review was filed before the
Supreme Court.

Issue:
Whether or not petitioner Valenzuela is guilty of frustrated theft.

Held:
No. Article 6 of the RPC provides that a felony is consummated when all the elements necessary for
its execution and accomplishment are present. In the crime of theft, the following elements should
be present – (1) that there be taking of personal property; (2) that said property belongs to another;
(3) that the taking be done with intent to gain; (4) that the taking be done without the consent of the
owner; and (5) that the taking be accomplished without the use of violence against or intimidating of
persons or force upon things. The court held that theft is produced when there is deprivation of
personal property by one with intent to gain. Thus, it is immaterial that the offender is able or unable
to freely dispose the property stolen since he has already committed all the acts of execution and the
deprivation from the owner has already ensued from such acts. Therefore, theft cannot have a
frustrated stage, and can only be attempted or consummated.
CRIMINAL LAW 1
(Sunday, Section 52) 113

PEOPLE VS LAMAHANG
L-43530 3 Aug 1935

Facts:
Aurelio Lamahang was caught opening with an iron bar a wall of a store of cheap goods inFuentes St.
Iloilo. He broke one board and was unfastening another when a patrolling police caughthim. Owners
of the store were sleeping inside store as it was early dawn. Convicted of attempt of robbery

Issue:
WON crime is attempted robbery?

Held:
No. Attempted trespass to dwelling. Attempt should have logical relation to a particular and concrete
offense which would lead directly to consummation. Necessary to establish unavoidable connection
& logical & natural relation of cause and effect. Important to show clear intent to commit crime. In
case at bar, we can only infer that his intent was to enter by force, other inferences are not justified
by facts. Groizard: infer only from nature of acts executed. Acts susceptible of double interpretation
can’t furnish ground for themselves. Mind should not directly infer intent. Spain SC necessary that
objectives established or acts themselves obviously disclose criminal objective.

PEOPLE VS BORINAGA
GR 33463 18 Dec 1930

Facts:
The victim Harry Mooney, an American who resided in Calubian Leyte,contracted with Juan Lawaan
for the construction of a fish corral. Lawaanattempted to collect the whole amount of the contract
even though the corralis not yet finished. Upon Mooney‘s refusal to pay, Lawaan warned and
threatened him that something would happen to him.On that evening, Mooney was in the store of
his neighbor, sitting with his back towards a window when suddenly Basilio Borinaga struck him
with a knife. Theknife imbedded on the back of the seat though. Mooney fell off from theimpact but
was not injured. Borinaga left the scene but after ten minutes, hereturned to have another attempt at
Mooney but was warded off by Mooneyand his neighbor frightening him by turning a flashlight on
him.

Issue:
Whether or not the crime is frustrated murder.

Held:
YES. As an essential condition of a frustrated crime, Borinaga performed all theacts of execution,
attending the attack. There was nothing left that he could dofurther to accomplish the work. The
cause resulting in the failure of the attack arose by reason of forces independent of his will. Borinaga
also voluntarilydesisted from further acts. The subjective phase of the criminal act was passed.

Dissenting opinion, J. Villa-Real: “The acts of execution perfomed by [Borinaga] did not produce the
death of Mooney as a consequence not could they have produced it because the blowdid not reach his
body; therefore, the culprit did not perform all the acts of execution which should produce the felony.
There was lacking the infliction of the deadly wound upon a vital spot of the body of Mooney.” What
the back of the chair prevented was the wounding of Mooney, not hisdeath. It is the preventing of death
by causes independent of the will of theperpetrator, after all the acts of execution which should produce
CRIMINAL LAW 1
(Sunday, Section 52) 114

the felony as aconsequence had been performed, that constitutes a frustrated felony,according to the
law, and not the preventing of the performances of all theacts of execution which constitute a felony, as
in the present case. Attempted murder only.

PEOPLE VS KALALO
GR 39303-05 17 Mar 1934

FACTS::
On November 10, 1932, the appellants, namely, Felipe Kalalo, Marcelo Kalalo, JuanKalalo and
Gregorio Ramos, were tried in the Court of First Instance of Batangas, together with Alejandro Garcia,
Fausta Abrenica and Alipia Abrenica in criminal cases Nos. 6858, 6859 and 6860,the first two for
murder, and the last for frustrated murder. Upon agreement of the parties said threecases were tried
together and after the presentation of their respective evidence, the said courtacquitted Alejandro
Garcia, Fausta Abrenica and Alipia Abrenica, and sentenced the other appellants.Prior to the
commission of the three crimes, the appellant Marcelo Kalalo and IsabelHolgado, the latter being the
sister of one of the deceased, had a litigation over a parcel of landsituated in the barrio of Calumpang
in the municipality of San Luis, Batangas. Kalalo filed twocomplaints against the said woman in the Court of
First Instance of Batangas, alleging that he, Kalalocultivated the land in question during 1931 and 1932 but
that, when harvest time came IsabelaHolgado reaped all that had been planted thereon. Both
complaints were dismissed.On October 1, 1932, Isabela Holgado and her brother Arcadio Holgado, one of
thedeceased, ordered the plowing of the disputed land and employed several laborers for that
purpose.Marcelo Kalalo, upon learning about it, went to the place accompanied by his brothers and
Felipa andJuan, his brother-in-law Gregorio Ramos and by Alejandro Garcia. They were all armed with bolos
andupon arriving at the place, they ordered the workers to stop. Having been informed of the cause
of thesuspension of the work, Marcelino Panaligan, one of the deceased, ordered the laborers
to continuethe work. At this point, Marcelo Kalalo approached Arcadio and the other appellants
approachedMarcelino Panaligan and they all simultaneously struck with their bolos. Arcadio
Holgado andMarcelino Panaligan died instantly from the wounds received. After the two had fallen,
Marcelo Kalalotook the revolver that Marcelino Panaligan carried, and fired four shots at Hilario Holgado who
wasthen fleeing from the scene in order to save his own life.

ISSUE:
WON the appellants are guilty of murder or of simple homicide in each of the cases.

HELD:
It is true that under article 248 of the Revised Penal Code, which defines murder, the circumstance
of “abuse of superior strength”, if proven to have been presented, raises homicide to the category
of murder;but it is also to be borne in mind that the deceased were also armed, one of them with a
bolo,and the other with a revolver. The risk was even for the contending parties and their strength
wasalmost balanced because there is no doubt but that, under circumstances similar to those of
thepresent case, a revolver is as effective as, if not more than three bolos. For this reason, this court is
of the opinion that the acts established in cases Nos. 6858 and 6859 (G.R. Nos. L-39303 and
39304,respectively), merely constitute two homicides. As to the third case, the evidence shows
that Marcelo Kalalo fired four successive shots at HilarionHolgado while the latter was fleeing from
the scene of the crime in order to be out of reach of theappellants and their companions and save
his own life. The fact that the said appellant, not havingcontended himself with firing only once, fired said
successive shots at Hilarion Holgado, added to thecircumstance that immediately before doing so he and his
co-appellants had already killed ArcadioHolgado and Marcelino Panaligan, cousin and brother-in-law,
respectively, of the former, shows thathe was then bent on killing said Hilarion Holgado. He performed
everything necessary on his pat tocommit the crime that he determined to commit but he failed by
CRIMINAL LAW 1
(Sunday, Section 52) 115

reason of causes independent of hiswill, either because of his poor aim or because his intended victim
succeeded in dodging the shots,none of which found its mark. The acts thus committed by the said appellant
Marcelo Kalalo constituteattempted homicide.

PEOPLE VS TRINIDAD
GR 79123-25 9 Jan 1989

Facts:
Lolito Soriano is a fish dealer. His helpers were Ricardo Tan and Marcial Laroa. Whilethe three were
driving on their way to Davao City to sell fish, accused !eliano Trinidad as"edfor a ride to #gusan
del Norte. Trinidad, a !e!&er of the ntegrated $ational (olice, was inunifor! and had two
firear!s, a car&ine and a .- cali&er revolver.Tan was driving the car at that ti!e, and he was instructed &y
Trinidad to slow down&ecause they were treading dangerous territory. Tan suddenly heard two
gunshots %% Sorianoand Laroa slu!ped dead for &oth were hit on the head. Trinidad had used his
car&ine in "illingthe two victi!s. Tan was a&le to get off the car and hail a 'eepney passing
&y. However, henoticed that Trinidad was also seated at the &ac" of the said 'eepney. Tan
i!!ediately got off the 'eepney, followed &y Trinidad. When the 'eepney started to drive away, Tan suddenly clungto
its side, &ut Trinidad fired two shots, one of which hit Tan on his right thigh. Tan 'u!ped fro!the 'eep and fortunately
a (hilippine Consta&ulary !e!&er chanced upon hi! and helped hi!&oard a &us for )utuan.Trinidad was
charged with *R+STR#T D !urder in relation to the shooting of Tan, andhe was found guilty &y the RTC. /n appeal,
Trinidad clai!s that the RTC erred in convicting hi!of the cri!e of frustrated !urder.

Issues:
W0$ Trinidad is correct in contending that he can only &e convicted of atte!pted!urder1 2 S

Ruling:
Trinidad should only &e held cri!inally lia&le for atte!pted !urder.

Ratio:
Trinidad had co!!enced the co!!ission of the felony directly &y overt acts &ut wasuna&le to perfor! all
the acts of e3ecution which would have produced it &y reason of causes.

MARTINEZ VS CA
GR 168827 13 Apr 2007

Facts:
Dean Dongui... was a teacher at the Tubao National High School, La Union. Petitioner Benjamin
Martinez was the husband of Dean's co-teacher, Lilibeth Martinez. Petitioner eked out a living as a
tricycle driver.

On October 28, 1998, Dean and his wife Freda filed a complaint for damages against the spouses
Martinez in the Municipal Circuit Trial Court (MCTC) of Tubao, La Union. They alleged that in March
1998, petitioner, a suitor of Elvisa Basallo, had been peddling false reports that
Dean and Elvisa had illicit relations... he even told Freda that Elvisa was Dean's mistress. This led to
a quarrel between Dean and Freda, and the latter was hospitalized for her hear... t ailment.
Dean requested Lilibeth to stop her husband from spreading lies, and she replied that
Elvisa had been her husband's mistress. They prayed that they be awarded moral and exemplary
damages and litigation fees in the total amount of P100,000.00.[3] The case was docketed as Civil
Case No. 226.
CRIMINAL LAW 1
(Sunday, Section 52) 116

For her part, Elvisa also filed a complaint against the spouses Martinez in the MCTC of Tubao for
damages anchored on Article 26 of the New Civil Code.

Dean went to the Tubao Credit Cooperative (TCC) office to pick up the dividend certificate of his wife
who was a member of the cooperative. He left the building and walked to his car which was parked
in front.

Dean was about a step away from an L-300 van which was parked in front of the building when
petitioner, armed with a bolo, suddenly emerged from behind the vehicle and stabbed him on the left
breast. Dean instantly moved backward and saw his assailant.

Dean fled to the bank office and was able to gain entry into the bank. Petitioner ran after him and
upon cornering him, tried to stab him again. Dean was able to parry the blow with his right hand, and
the bolo hit him on the right elbow. Dean fell to the floor and tried to... stand up, but petitioner stabbed
him anew on his left breast.

Meantime, SPO1 Henry Sulatre was at the Tubao Police Station, about 100 meters away.
He was informed that a fight was going on in the bank. He rushed to the place on board the police car.
He... brought petitioner to the police station. On the way, they passed by the loading area of tricycles,
about 40 meters away from the... police station.

Petitioner shouted:
I stabbed him, he is just a visitor so he should not act like a king here in Tubao
SPO1 Sulatre placed Benjamin in jail. Benjamin kept on shouting:
"Napatay kon, napatay kon

In the meantime, PO3 Valenzuela brought Dean to the Doña Gregoria Memorial Hospital in Agoo, La
Union. The victim was transferred to the Ilocos Regional Hospital (IRH) in San Fernando, La Union
where Dean was examined and operated Dean sustained two stab wounds On February 7, 1999, Dean
gave a sworn statement to SPO1 Sulatre. On March 10, 1999, SPO1 Sulatre filed a criminal complaint
for frustrated murder against petitioner in the MCTC. On September 13, 2000 the Provincial
Prosecutor of La Union indicted Benjamin for frustrated murder before the Regional Trial Court
Petitioner declared that he merely defended himself against Dean's assault. Dean was so jealous of
him because his mistress, Elvisa, had also been his mistress. Petitioner declared that the criminal
charge against him was Dean's concoction and intended solely to harass him. He parked his tricycle
in front of the building on the left side of the railing going to the entrance of the cooperative. On his...
way, he met his 82-year-old uncle, who was also on his way to the cooperative to update his passbook
because he was intending to apply for a loan.

Dean thought that he was blocking his way and shouted invectives at him and his uncle
Dean kept attacking him, forcing him to move backward through the railing and towards his tricycle
He stabbed Dean on his right elbow.

He entered the office and stood by the entrance door to see if Dean would get a weapon. Dean
continued hurling invectives at him but was later pacified by Patricio Alterado, an... employee of the
cooperative. When Barangay Captain... he surrendered, along with his bolo. He never boasted on the
way to the police station that he had killed Dean.
CRIMINAL LAW 1
(Sunday, Section 52) 117

Issues:
Petitioner's argument that he should be acquitted because the criminal complaint against him was
not supported by the victim's sworn statement or by an affidavit. Petitioner next argues that should
he be convicted of any crime, it should be of less serious physical injuries only, absence the element
of intent to kill.

Ruling:
The petition is denied for lack of merit. Petitioner is guilty of frustrated murder under Article 248 in
relation to Article 6, first paragraph of the Revised Penal Code which reads, For voluntary surrender
to be appreciated, the following requisites should be present: (1) the offender has not been actually
arrested; (2) the offender surrendered himself to a person in authority or the latter's agent; and (3)
the surrender was voluntary. The surrender must be... spontaneous, made in such a manner that it
shows the interest of the accused to surrender unconditionally to the authorities, either because he
acknowledged his guilt or he wishes to save them the trouble and expenses that would necessarily
be incurred in the search and... capture. In the case at bar, SPO1 Salutre testified that petitioner did
not voluntarily surrender but was forcibly apprehended by Barangay Captain. We agree with the trial
court that the qualifying circumstance of evident premeditation has not been adequately shown. To
properly appreciate the same. The trial court awarded Dean the amount of P92,000.00 representing
his hospitalization and medical expenses which was increased by the CA to P92,715.68. IN LIGHT OF
ALL THE FOREGOING, the assailed Decision is hereby AFFIRMED WITH MODIFICATION. Petitioner
is hereby found guilty beyond reasonable doubt of Frustrated Murder under Article 248 in relation
to Article 6... prision mayor... to... eclusion temporal in its medium period, as maximum.

MONDRAGON VS PEOPLE
L-17666 30 Jun 1966

FACTS:
The petitioner, Isidoro Mondragon, was prosecuted in the Court of First Instance of Iloilo of the crime
of frustrated homicide. After trial the Court of First Instance of Iloilo found him guilty of the crime of
attemptedhomicide and sentenced him to an indeterminate prison term of from 4 months and !
days of arresto mayor to years, 4 months and ! day of prision correccional, with the accessory
penalties of the law and the costs.Mondragon appealed to the Court of Appeals, and the latter court
affirmed the decision of the Court of FirstInstance of Iloilo in all its parts, with costs. At a"out #:$$ in
the afternoon of %uly !!, !&#4, while complainant Serapion 'acionales was opening thedi(e of his
ricefield situated in Antandan, Miagao, Iloilo, to drain the water therein and prepare the ground
for planting the ne)t day, he heard a shout from afar telling him not to open.

Issue:
Whether Mondragon is guilty of the crime of frustrated homicide or less serious physical injury?

Held:
The element of intent to (ill not ha*ing "een duly esta"lished, and considering that the in2uries
suffered "ythe offended party were not necessarily fatal and could "e healed in less than
8$ days, SC hold that theoffense that was committed "y the petitioner is only that of less serious
physical in2uries.
CRIMINAL LAW 1
(Sunday, Section 52) 118

PEOPLE VS SY PIO
L-5848 30 Apr 1954

This is an appeal from a judgment of the Court of First Instance of Manila finding the defendant-
appellant herein Sy Pio, alias Policarpio de la Cruz, guilty of frustrated murder against the person of
Tan Siong Kiap, and sentencing him to suffer an indeterminate sentence of 6 years, 1 month, and 11
days of prision mayor, to 14 years, 8 months, and 1 day of reclusion temporal, to indemnify the
offended party Tan Siong Kiap in the sum of P350, without subsidiary imprisonment in case of
insolvency, and to pay the costs. The case was appealed to the Court of Appeals, but that court
certified it to this Court under the provisions of section 17 (4) of Republic Act No. 296, on the ground
that the crime charged was committed on the same occasion that the defendant-appellant had
committed crime of murder, with which the defendant-appellant was also charged.

The evidence for the prosecution shows that early in the morning of September 3, 1949, the
defendant-appellant entered the store at 511 Misericordia, Sta Cruz, Manila. Once inside he started
firing a .45 caliber pistol that he had in his hand. The first one shot was Jose Sy. Tan Siong Kiap, who
was in the store and saw the accused enter and afterwards fire a shot at Jose Sy, asked the defendant-
appellant, "What is the idea?" Thereupon defendant-appellant turned around and fired at him also.
The bullet fired from defendant-appellant's pistol entered the right shoulder of Tan Siong Kiap
immediately ran to a room behind the store to hide. From there he still heard gunshot fired from
defendant-appellant's pistol, but afterwards defendant-appellant ran away.

Tan Siong Kiap was brought to the Chinese General Hospital, where his wound was treated. He stayed
there from September 3 to September 12, 1949, when he was released upon his request and against
the physician's advice. He was asked to return to the hospital for further treatment, and he did so five
times for a period of more than ten days. Thereafter his wound was completely healed. He spent the
sum of P300 for hospital and doctor's fees.

The defendant-appellant shot two other persons in the morning of September 3, 1949, before
shooting and wounding Tan Siong Kiap; one was Ong Pian and the other Jose Sy. On September 5
information was received by the Manila Police Department that defendant-appellant was in custody
of the Constabulary in Tarlac, so a captain of the Manila police by the name of Daniel V. Lomotan
proceeded to Tarlac. There he saw the defendant-appellant and had a conversation with him. On this
occasion defendant-appellant and had a conversation with him. On this occasion defendant-appellant
admitted to Lomotan that his victims were Tan Siong Kiap, Ong Pian, and Jose Sy. The Constabulary
in Tarlac also delivered to Lomotan the pistol used by the defendant-appellant, marked Exhibit C, and
its magazine, Exhibit C-1, both of which the Constabulary had confiscated from the defendant-
appellant. The defendant-appellant was thereupon delivered to the custody of Lomotan, and the
latter brought him to Manila, where his statement was taken down in writing. This declaration was
submitted at the time of the trial as Exhibit D, and it contains all the details of the assaults that
defendant-appellant 3 against the persons of Tan Siong Kiap, Ong Pian, and Jose Sy. This written
statement was taken down on a typewriter and afterwards signed by the defendant-appellant in both
his Chinese and Filipino names, the latter being Policarpio de la Cruz.

According to the declaration of the defendant-appellant, some months prior to September 3, 1949,
he was employed as an attendant in a restaurant belonging to Ong Pian. Defendant-appellant's wife
by the name of Vicenta was also employed by Ong Pian's partner, Eng Cheng Suy. Prior to September
3 the relatives of his wife had been asking the latter for help, because her father was sick. Defendant-
appellant asked money from Ong Pian, but the latter could only give him P1. His wife was able to
CRIMINAL LAW 1
(Sunday, Section 52) 119

borrow P20 from her employer, and this was sent to his wife's parents in Cebu. Afterwards
defendant-appellant was dismissed from his work at the restaurant of Ong Pian, and he became a
peddler. Ong Pian presented a list of the sums that defendant-appellant had borrowed from him, and
these sums were deducted from the salary of his wife. Defendant-appellant did not recognize these
sums as his indebtedness, and so he resented Ong Pian's conduct.

As to Tan Siong Kiap, the confession states that a few days before September 3, 1949, defendant-
appellant had been able to realize the sum of P70 from the sales of medicine that he peddled. He laid
his money in a place in his room, but the following morning he found that it had disappeared from
the place in which he had placed it. Tan Siong Kiap and Jose Sy, upon the discovery of the loss of
money, told defendant-appellant that he must have given the money to his wife, and that nobody had
stolen it. After this incident of the loss, the defendant-appellant used to hear Tan Siong Kiap and Jose
Sy and other Chinamen say that the money had not been actually stolen, but that he lost it in gambling.
Because of these accusations against him, he nurtured resentment against both Tan Siong Kiap and
Jose Sy.

So early in the morning of September 3, while a Chinaman by the name of Ngo Cho, who the possessor
of a caliber .45 pistol, was away from his room, defendant-appellant got his pistol and tucked it in his
belt. With this pistol he went to the restaurant at 822 Ongpin, and there shot Ong Pian. After shooting
him, he proceeded to 511 Misericordia, in store where Jose Sy and Tan Siong Kiap were, and there he
fired at them. Then he escaped to Legarda Street, in Sampaloc, where he borrowed P1 from his
relatives. From there he went to Malabon, to the house of his mother, to whom he told he had killed
two persons and from he asked money.

The foregoing is the substance of the written declaration made by the defendant-appellant in Exhibit
D on September 6, 1949. At the time of the trial, however, he disowned the confession and explained
that he signed it without having read its contents. He declared that it was not he who shot the three
victims, but it was one by the name of Chua Tone, with whom he had previously connived to kill the
three other victims. He introduced no witnesses, however, to support his denial. Neither did he deny
that he admitted before Captain Lomotan having killed the three persons, or having been found in
Tarlac in possession of the caliber .45 pistol, Exhibit C, and its magazine, Exhibit C-1. In his cross-
examination he admitted many of the incidents mentioned in the confession, especially the cause of
his resentment against his victims Ong Pian, Jose Sy, and Tan Siong Kiap.

The trial court refused to believed his testimony, and therefore, found him guilty of the crime charged.
On this appeal counsel for the defendant-appellant claims that the trial court erred in not finding that
Tan Siong Kiap received the shot accidentally from the same bullet that had been fired at Jose Sy, and
in finding that defendant-appellant has committed a crime distinct and separate from that of murder
for the slaying of Jose Sy. We find no merit in this contention. According to the uncontradicted
testimony of the offended party Tan Siong Kiap, when the latters saw defendant-appellant firing
shots he asked him why he was doing so, and the defendant-appellant, instead of answering him,
turned around and fired at him also. It is not true, therefore, that the shot which hit him was fired at
Sy.

It is also contended that the evidence is not sufficient to sustain the judgment of conviction. We also
find no merit in this contention. The evidence submitted to prove the charge consists of: the
uncontradicted testimony of the victim himself; the admissions made verbally by the defendant-
appellant before Captain Lomotan in Tarlac; the fact that the defendant-appellant had escaped and
was found in Tarlac; his possession of the .45 caliber pistol coupled with the fact, attested to by the
testimony of the physician who examined and treated the wounds of Tan Siong Kiap, that the wounds
CRIMINAL LAW 1
(Sunday, Section 52) 120

found in his person must have been caused by the caliber .45 bullet; and, lastly, the confession of the
defendant-appellant himself, Exhibit D, which he was not able to impugn. As against this mass of
evidence, defendant-appellant has only made a very unbelievable story that it was not he but another
that had committed the crime charged. His admissions at the time of the trial regarding the incidents,
as well as the cause of his having assaulted his victims, coincide exactly with the reasons given in his
written confession. This shows that he had made the confession himself, for nobody but himself could
have known the facts therein stated. The claim that the offense has not been proved beyond
reasonable doubt must be dismissed.

The defendant-appellant lastly claims that the lower court also erred in sentencing him to pay an
indemnity of P350. The offended party testified that he actually spent P300 for hospital and doctor's
fees, and that he was confined in the hospital for nine days. The above facts stand uncontradicted.
This assignment of error must also be dismissed.

It is lastly contended that the defendant-appellant should be found guilty only of less serious physical
injuries instead of the crime of frustrated murder as defendant-appellant admitted in his confession
in the open court that he had a grudge against the offended party, and that he connived with another
to kill the latter. The intent to kill is also evident from his conduct in firing the shot directly at the
body of the offended party.

But while intent to kill is conclusively proved the wound inflicted was not necessarily fatal, because
it did not touch any of the vital organs of the body. As a matter of fact, the medical certification issued
by the physician who examined the wound of the offended party at the time he went to the hospital,
states that the wound was to heal within a period of fourteen days, while the offended party actually
stayed in the hospital for nine days and continued receiving treatment thereafter five time for the
period of more than ten days, or a total of not more than thirty days. The question that needs to be
determined, therefore, is: Did the defendant-appellant perform all the acts of execution necessary to
produce the death of his victim?

In the cases of U.S. vs. Eduave, 36 Phil., 209, People vs. Dagman, 47 Phil., 768 and People vs.
Borinaga, 55 Phil., 433, this Court has held that it is not necessary that the accused actually commit
all the acts of execution necessary to produce the death of his victim, but that it is sufficient that he
believes that he has committed all said acts. In the case of People vs. Dagman, supra, the victim was
first knocked down by a stone thrown at him, then attacked with a lance, and then wounded by bolos
and clubs wielded by the accused, but the victim upon falling down feigned death, and the accused
desisted from further continuing in the assault in the belief that their victim was dead. And in the
case of People vs. Borinaga, supra, the accused stabbed his intended victim, but the knife with which
he committed the aggression instead of hitting the body of the victim, lodged in the back of the chair
in which he was seated, although the accused believed that he had already harmed him. In both these
cases this Court held that of the crime committed was that of frustrated murder, because the
subjective phase of the acts necessary to commit the offense had already passed; there was full and
complete belief on the part of the assailant that he had committed all the acts of execution necessary
to produce the death of the intended victim.

In the case at bar, however, the defendant-appellant fired at his victim, and the latter was hit, but he
was able to escape and hide in another room. The fact that he was able to escape, which appellant
must have seen, must have produced in the mind of the defendant-appellant that he was not able to
his his victim at a vital part of the body. In other words, the defendant-appellant knew that he had
not actually all the acts of execution necessary to kill his victim. Under these circumstances, it can not
be said that the subjective phase of the acts of execution had been completed. And as it does not
CRIMINAL LAW 1
(Sunday, Section 52) 121

appear that the defendant-appellant continued in the pursuit, and as a matter of fact, he ran away
afterwards a reasonable doubt exist in our mind that the defendant-appellant had actually believed
that he has committed all the acts of execution or passed the subjective phase of the said acts. This
doubt must be resolved in favor of the defendant-appellant.

We are, therefore, not prepared to find the defendant-appellant guilty of frustrated murder, as
charged in the information. We only find him guilty of attempted murder, because he did not perform
all the acts of execution, actual and subjective, in order that the purpose and intention that he had to
kill his victim might be carried out.

Therefore, the judgment appealed from should be, as it is hereby, modified and the defendant-
appellant is found guilty of the crime of attempted murder, and the sentence imposed upon him
reduced to an indeterminate penalty of from 4 years, 2 months, and 1 day of prision correccional to
10 years of prision mayor. In all other respects the judgment is affirmed. With costs against the
defendant-appellant.

PEOPLE V. DIO
L-3646129 June 1984, 130 SCRA 151
CRIMINAL LAW 1
(Sunday, Section 52) 122

PEOPLE V. SALVILLA
G.R. No. 86163, 26 April 1990

Facts:
The accused Bienvenido Salvilla together with his co-accused armed with homemade guns and hand
grenade robbed Rodita Habiero in the latter’s office. In the office of Rodita; her two daughters Mary
and Mimmie were also inside. One of the accused asks Mary to get the paper bag which
contained money. All accused held victims as hostage when the police and military authorities had
surrounded the lumber yard. After the negotiation fails to proceed, the police makes their move in
assaulting the robbers thus Mary and Mimmie are injured as well the accused also got an injury.

Issue:
Whether or not the crime of robbery was consummated

Held:
Yes. From the moment the offender gained possession of the thing, even if the culprit had no
opportunity to dispose of the same, the unlawful taking is complete.
CRIMINAL LAW 1
(Sunday, Section 52) 123

PEOPLE V. ORITA
G.R. No. 88724, 3 April 1990, 184 SCRA 105

Facts:
• March 20, 1983 Early Morning: Cristina S. Abayan, 19-year old freshman student at the St.
Joseph's College, arrived at her boarding house after her classmates brought her home from
a party. She knocked at the door of her boarding house when a frequent visitor of another
boarder held her and poked a knife to her neck. Despite pleading for her release, he ordered
her to go upstairs with him. Since the door which led to the 1st floor was locked from the
inside, they used the back door to the second floor. With his left arm wrapped around her
neck and his right hand poking a "balisong" to her neck, he dragged her up the stairs. When
they reached the second floor, he commanded herwith the knife poked at her neck, to look
for a room. They entered Abayan's room. He then pushed her hitting her head on the
wall. With one hand holding the knife, he undressed himself. He then ordered her to take off
her clothes. Scared, she took off her T-shirt, bra, pants and panty. He ordered her to lie down
on the floor and then mounted her. He made her hold his penis and insert it in her vagina. Still
poked with a knife, she did as told but since she kept moving, only a portion of his penis
entered her. He then laid down on his back and commanded her to mount him. Still only a
small part of his penis was inserted into her vagina. When he had both his hands flat on the
floor. She dashed out to the next room and locked herself in. When he pursued her and
climbed the partition, she ran to another room then another then she jumped out through a
window.
• Still naked, she darted to the municipal building, 18 meters in front of the boarding house
and knocked on the door. When there was no answer, she ran around the building and
knocked on the back door. When the policemen who were inside the building opened the
door, they found her naked sitting on the stairs crying. Pat. Donceras, took off his jacket and
wrapped it around her. Pat. Donceras and two other policemen rushed to the boarding
house where they heard and saw somebody running away but failed to apprehend him due
to darkness. She was taken to Eastern Samar Provincial Hospital where she was physically
examined.
• Her vulva had no abrasions or discharges.
• RTC: frustrated rape

ISSUE:
W/N there is frustrated rape.

HELD:
NO. RTC MODIFIED. Guilty beyond reasonable doubt of the crime of rape and sentenced to reclusion
perpetua as well as to indemnify the victim in the amount of P30, 000.

• Correlating Art. 335 and Art. 6, there is no debate that the attempted and consummated stages
apply to the crime of rape.
• Requisites of a frustrated felony are:
o (1) that the offender has performed all the acts of execution which would produce the felony
o (2) that the felony is not produced due to causes independent of the perpetrator's will
• attempted crime the purpose of the offender must be thwarted by a foreign force or agency which
intervenes and compels him to stop prior to the moment when he has performed all of the acts which
should produce the crime as a consequence, which acts it is his intention to perform
o If he has performed all of the acts which should result in the consummation of the crime and
CRIMINAL LAW 1
(Sunday, Section 52) 124

voluntarily desists from proceeding further, it cannot be an attempt.


• in the crime of rape, from the moment the offender has carnal knowledge of his victim he actually
attains his purpose and, from that moment also all the essential elements of the offense have been
accomplished. Any penetration of the female organ by the male organ is sufficient. Entry of the labia
or lips of the female organ, without rupture of the hymen or laceration of the vagina is sufficient to
warrant conviction. Necessarily, rape is attempted if there is no penetration of the female organ
• The fact is that in a prosecution for rape, the accused may be convicted even on the sole basis of
the victim's testimony if credible. Dr. Zamora did not rule out penetration of the genital organ of the
victim.

PEOPLE V. CAMPUHAN
G.R. No. 129433, 30 March 2000

FACTS:
• April 25, 1996 4 pm: Ma. Corazon P. Pamintuan, mother of 4-year old Crysthel Pamintuan, went
to the ground floor of their house to prepare Milo chocolate drinks for her 2 children. There she
met Primo Campuhan, helper of Conrado Plata Jr., brother of Corazon, who was then busy filling
small plastic bags with water to be frozen into ice in the freezer located at the second floor.
• Then she heard Crysthel cry, "Ayo'ko, ayo'ko!" so she went upstairs and saw Primo Campuhan
inside her children's room kneeling before Crysthel whose pajamas or "jogging pants" and panty
were already removed, while his short pants were down to his knees and his hands holding his
penis with his right hand
• Horrified, she cursed "P - t - ng ina mo, anak ko iyan!" and boxed him several times. He evaded
her blows and pulled up his pants. He pushed Corazon aside who she tried to block his path.
Corazon then ran out and shouted for help thus prompting Vicente, her brother, a cousin and an
uncle who were living within their compound, to chase the Campuhan who was apprehended. They
called the barangay officials who detained.
• Physical examination yielded negative results as Crysthel ‘s hymen was intact
• Campuhan: Crysthel was in a playing mood and wanted to ride on his back when she suddenly
pulled him down causing both of them to fall down on the floor.
• RTC: guilty of statutory rape, sentenced him to the extreme penalty of death
• Thus, subject to automatic review

ISSUE:
W/N it was a consummated statutory rape

HELD:
NO. MODIFIED. guilty of ATTEMPTED RAPE and sentenced to an indeterminate prison term of eight
(8) years four (4) months and ten (10) days of prision mayor medium as minimum, to fourteen (14)
years ten (10) months and twenty (20) days of reclusion temporal medium as maximum. Costs de
oficio.

• People v. De la Peña: labia majora must be entered for rape to be consummated


• Primo's kneeling position rendered an unbridled observation impossible
• Crysthel made a categorical statement denying penetration but her vocabulary is yet as
underdeveloped
• Corazon narrated that Primo had to hold his penis with his right hand, thus showing that he had
yet to attain an erection to be able to penetrate his victim
• the possibility of Primo's penis having breached Crysthel's vagina is belied by the child's own
assertion that she resisted Primo's advances by putting her legs close together and that she did not
CRIMINAL LAW 1
(Sunday, Section 52) 125

feel any intense pain but just felt "not happy" about what Primo did to her. Thus, she only shouted
"Ayoko, ayoko!" not "Aray ko, aray ko!
• no medical basis to hold that there was sexual contact between the accused and the victim

US V. VALDES, L-14128
10 December 1918

PEOPLE V. AGUILOS
G.R. No. 121828, 27 June 2003

FACTS:
February 5, 1988 11:30 pm: Elisa Rolan was inside their store waiting for her husband to arrive.
Joselito Capa and Julian Azul, Jr. were drinking beer. Although already drunk, Edmar Aguilos and
Odilon Lagliba joined them. Edmar had a heated argument with Julian. Elisa pacified Edmar and
advised them to go home as she was already going to close up. Edmar and Odilon left then returned
to block Joselito and Julian. Edmar took off his eyeglasses and punched Julian in the face. Elisa
shouted: “Tama na. Tama na” but she was ignored as they continued until they reached the end of
the street. Odilon positioned himself on top of a pile of hollow blocks and watched as Edmar and
Julian swapped punches. As Joselito tried to stop the fight, Odilon pulled out his knife with his right
hand and stepped down from his perch. He placed his left arm around Joselito’s neck, and stabbed
him. Ronnie and Rene Gayot Pilola, who were across the street, saw their gangmate Odilon stabbing
the victim and decided to join the fray. Ronnie took a knife from the kitchen of Teresita and rushed
together with Pilola to the scene and stabbed Joselito. As Joeslito was stabbed 11 times (6 fatal stab
wounds), he fell in the canal. Odilon and Pilola fled while Ronnie went after Julian who ran dear life.
When Julian noticed that Ronnie was no longer running after him, he looked back and saw Ronnie
CRIMINAL LAW 1
(Sunday, Section 52) 126

pick up a piece of hollow block and bashed Joselito’s head. Then, Ronnie got a piece of broken bottle
and struck Joselito once more before fleing from the scene. Joselito died on the spot. Elisa rushed to
Joselito’s house and informed his wife and brother of the incident.

Agripina Gloria, a female security guard, saw Ronnie repeatedly stabbed Joselito and fled towards the
direction of the mental hospital. She did not see Odilon.
Elisa cross-examination had an inconsistency, she stated that it was Edmar who struck the victim
(before it was Ronnie)
RTC: Pilola GUILTY beyond reasonable doubt of Murder qualified by treachery and sentenced to
reclusion perpetua

ISSUE:
W/N Pilola is guilty of murder

HELD:
YES. Rene Gayot Pilola GUILTY beyond reasonable doubt of the crime of murder is AFFIRMED WITH
MODIFICATION

The identity of the person who hit the victim with a hollow block is of de minimis importance. Elisa’s
testimony is corroborated by the autopsy report of Dr. Bienvenido Muñoz. No showing of any
improper motive on the part of a witness to testify falsely against the accused or to falsely implicate
the latter in the commission of the crime. The trial court gave credence and full probative weight to
Elisa’s testimony.

There is conspiracy when two or more persons agree to commit a felony and decide to commit it.
Conspiracy as a mode of incurring criminal liability must be proved separately from and with the
same quantum of proof as the crime itself. Conspiracy need not be proven by direct evidence. After
all, secrecy and concealment are essential features of a successful conspiracy. It may be inferred from
the conduct of the accused before, during and after the commission of the crime, showing that they
had acted with a common purpose and design. Conspiracy may be implied if it is proved that two or
more persons aimed by their acts towards the accomplishment of the same unlawful object, each
doing a part so that their combined acts, though apparently independent of each other, were, in fact,
connected and cooperative, indicating a closeness of personal association and a concurrence of
sentiment. There may be conspiracy even if an offender does not know the identities of the other
offenders, and even though he is not aware of all the details of the plan of operation or was not in on
the scheme from the beginning. One need only to knowingly contribute his efforts in furtherance of
it. One who joins a criminal conspiracy in effect adopts as his own the criminal designs of his co-
conspirators. If conspiracy is established, all the conspirators are liable as co-principals regardless
of the manner and extent of their participation since in contemplation of law, the act of one would be
the act of all. Each of the conspirators is the agent of all the others.

The mere presence of an accused at the suits of the crime will not suffice. There must be intentional
participation in the transaction with a view to the furtherance of the common design and purpose.

Even if two or more offenders do not conspire to commit homicide or murder, they may be held
criminally liable as principals by direct participation if they perform overt acts which immediately or
immediately cause or accelerate the death of the victim. Art. 4. Criminal liability. – Criminal liability
shall be incurred by any person committing a felony (delito) although the wrongful act done be
different from that which he intended.
CRIMINAL LAW 1
(Sunday, Section 52) 127

PEOPLE VS. TULIN


G.R. No. 111709, 30 Aug. 2001

In the evening of March 2, 1991, "M/T Tabangao," a cargo vessel owned by the PNOC Shipping and
Transport Corporation, loaded with barrels of kerosene, regular gasoline, and diesel oil, was boarded
by 7 fully armed pirates. The pirates including the accused Roger P. Tulin, Virgilio Loyola, and Andres
Infante Jr. detained the crew and completely took over the vessel.

The vessel was directed to proceed to Singapore where the cargoes were unloaded transferred and
sold under the direct supervision of accused Cheong San Hiong. Thereafter, the captive vessel
returned to the Philippines.

A series of arrests was thereafter effected and all the accused were charged with qualified piracy or
violation of Presidential Decree No. 532 (Piracy in Philippine Waters). They were subsequently
convicted of the crime charged. Hence, this appeal. Meanwhile accused Cheong argues that the trial
court erred in convicting and punishing him as an accomplice when the acts allegedly committed by
him were done or executed outside of Philippine waters and territory, stripping the Philippine courts
of jurisdiction to hold him for trial, to convict, and sentence.

ISSUE:
WON the Philippines is without jurisdiction to try a crime committed outside the Philippine waters
and territory?

RULING:
We affirm the conviction of all the accused-appellants.
Article 122 of the Revised Penal Code, before its amendment, provided that piracy must be committed
on the high seas by any person not a member of its complement nor a passenger thereof. Upon its
amendment by Republic Act No. 7659, the coverage of the pertinent provision was widened to
include offenses committed "in Philippine waters." On the other hand, under Presidential Decree No.
532 (issued in 1974), the coverage of the law on piracy embraces any person including "a passenger
or member of the complement of said vessel in Philippine waters." Hence, passenger or not, a
member of the complement or not, any person is covered by the law.
Republic Act No. 7659 neither superseded nor amended the provisions on piracy under Presidential
Decree No. 532. There is no contradiction between the two laws. There is likewise no ambiguity and
hence, there is no need to construe or interpret the law. All the presidential decree did was to widen
the coverage of the law, in keeping with the intent to protect the citizenry as well as neighboring
states from crimes against the law of nations. As expressed in one of the "whereas" clauses of
Presidential Decree No. 532, piracy is "among the highest forms of lawlessness condemned by the
penal statutes of all countries." For this reason, piracy under the Article 122, as amended, and piracy
under Presidential Decree No. 532 exist harmoniously as separate laws.

As regards the contention that the trial court did not acquire jurisdiction over the person of accused-
appellant
Hiong since the crime was committed outside Philippine waters, suffice it to state that
unquestionably, the attack on and seizure of "M/T Tabangao" (renamed "M/T Galilee" by the
pirates) and its cargo were committed in
CRIMINAL LAW 1
(Sunday, Section 52) 128

Philippine waters, although the captive vessel was later brought by the pirates to Singapore
where its cargo was off-loaded, transferred, and sold. And such transfer was done under accused-
appellant Hiong's direct supervision.

Although Presidential Decree No. 532 requires that the attack and seizure of the vessel and its
cargo be committed in Philippine waters, the disposition by the pirates of the vessel and its cargo
is still deemed part of the act of piracy, hence, the same need not be committed in Philippine
waters. Moreover, piracy falls under Title One of Book Two of the Revised Penal Code. As such, it
is an exception to the rule on territoriality in criminal law. The same principle applies even if
Hiong, in the instant case, were charged, not with a violation of qualified piracy under the penal
code but under a special law, Presidential Decree No. 532 which penalizes piracy in Philippine
waters. Verily, Presidential Decree No. 532 should be applied with more force here since its
purpose is precisely to discourage and prevent piracy in Philippine waters (People v. Catantan,
278 SCRA 761 [1997]). It is likewise, well-settled that regardless of the law penalizing the same,
piracy is a reprehensible crime against the whole world (People v. Lol-lo, 43 Phil. 19 [1922]).

PEOPLE VS. PUGAY


G.R. No. L-74324, 17 Nov. 1998

Samson and Pugay were charged with the crime of murder. Samson and Pugay, committed the crime
with the qualifying circumstance of treachery and the aggravating circumstance of evident
premeditation and superior strength.

FACTS:
May 19, 1982, a town fiesta was held in the public plaza of Rosario, Cavite. Sometime after midnight,
Eduardo Gabion was sitting in the ferris wheel and reading a comic book.

Later, Pugay and Samson with several companions arrived at the scene seemingly drunk.
The group saw Bayani Miranda and started making fun of him by tickling him with a piece of wood.
Pugay suddenly took a can of gasoline and poured its contents on Miranda. Gabion asked Pugay to
stop during the process of pouring the gasoline.

Then Samson set Miranda on fire.

ISSUE/HELD:
WON Pugay and Samson are guilty of the crime murder. (NO)

RATIO:
There was no evidence found that Pugay and Samson planned to kill Miranda. Their meeting was
accidental and the accused were merely making fun of the deceased-victim. Criminal responsibility
of Pugay and Samson are counted as individual acts and they are held liable only for the acts they
committed individually. Pugay should have known that what he was pouring on Miranda was
gasoline because of its smell. He failed to exercise diligence necessary to avoid the consequences of
his actions and exposed Miranda to danger and injury. Pugay is guilty of homicide through reckless
imprudence. Samson just wanted to set Miranda’s clothes on fire but this doesn’t relieve him of
criminal liability (Art. 4). Samson is guilty of homicide credited with ordinary mitigating
circumstance of no intention to commit so grave a wrong. Gabion testified that accused were stunned
when they noticed Miranda burning.
CRIMINAL LAW 1
(Sunday, Section 52) 129

PEOPLE v. CABALLERO
GR NO. 149028-30

FACTS:
In an afternoon of August 03, 1994, Caballero brothers Armando, Marcianoand Robito were in the
house of their other brother Ricardo having drinking sessionsin the Mondragon compound. By 7pm
of that same day, Eugene Tayactac and ArnoldBacurna arrived in the sari-sari store of Wilma
Broce which was across theMondragon compound. Later on, Armando angrily approached
Eugene, and thelatter insisted that there is no quarrel between them. Armando left and
afterminutes, his brothers joined him, armed with knives. Armando grabbed Eugene andwhen the
latter resisted, the other Caballeros ganged up on him. Armando hit himwith the wooden support
of clothesline and Eugene was stabbed on the chest threetimes. Arnold tried to help but he was
also stabbed on the left side of his body andtwice on his forearm. Leonilo Broce, nephew of Wilma,
rushed to help but was alsostabbed on the chest by Robito. Eugene and Leonildo eventually
died from thesustained wounds. The Caballero borthers, except for Robito, were convicted
ofmurder for the deaths of Eugene and Leonildo, and frustrated murder for the injuriesof Arnold. The
appellants insists that the decision is incorrect.

ISSUE:
WON the appellants are guilty of frustrated murder for the injuries of Arnold?

HELD:
Basing the judgement in consideration of the Article 248, Article 6, par. 1 ofthe Revised Penal
Code, the court, beyond reasonable doubt, found the appellantsguilty of the crime frustrated
murder. With the fact that appellant Armando usedwooden pole and appellants Ricardo and
Robito used knives, it cannot be denied thatthere was an intent to kill Arnold. Moreover, the doctor
attended on Arnold testifiedthat the stab wound inflicted upon Arnold was mortal and may
have caused thelatter’s death, if not for the timely and effective medical intervention.
WHEREFORE, the acts committed by the appellants is indeed FRUSTRATED MURDER.

PEOPLE V AGUILOS
G.R. No. 121828. June 27, 2003

FACTS
Joselito Capa and Julian Azul was drinking in a store in Mandaluyong, while in the midst of the
drinking spree, Edmar Aguilos and Odilon Lagliba, arrived and they were invited to join the drinking.
During thecourse of the drinking, Edmar had an argument with the Julian that led to a commotion of
the store,Edmar punched Julian in the face, the latter and the former exchanged punches. Joselito
intervened to stopthe fight,. But the Odilon grabbed the Joselito the neck and stabbed the latter with
a knife, Ronnie and accused arrived at the scene and they saw their gang mate, Odilon, stabbing the
victim, they immediately pulled out their knives and subsequently stabbed the victim, Ronnie bashed
a hollow block and a bottle tothe head of the victim, the three accused fled from the scene of the
crime. Leaving the victim dead on the spot.

ISSUE
Whether or not the accused should be considered as a co- principal in the commission of the crime
and not merely an accomplice.
CRIMINAL LAW 1
(Sunday, Section 52) 130

HELD
The direct participation of the accused in stabbing the victim, resulting to the victim’s accelerated
death, constitutes to conspiracy. The overt act executed by accused qualifies the latter to be a co-
principal in the crime.

PEOPLE V CRUZ
GR 74048

FACTS:
In the afternoon of November 25, 1983, Jesus Baang, married, 36 years old, fisherman, succumbed to
three gunshot wounds on the head, chest and back. According to several eyewitnesses who were at
the scene of the crime, his alleged assailant was Rading (Radito) Sason, but Rolando Cruz was
implicated as a co-conspirator and principal by inducement.

"That on or about the 25th day of November 1983, in the Municipality of Malabon, Metro Manila,
Philippines and within the jurisdiction of this Honorable Court, the above-named accused,
conspiring, confederating and helping one another while armed with a gun, with intent to kill, with
treachery and evident premeditation, did, then and there, wilfully, unlawfully and feloniously attack,
assault and shot JESUS BAANG y VALDEZ, hitting the latter on the different vital parts of his body,
thereby inflicting upon him gunshot wounds which directly caused his death."
There were several eyewitnesses to the shooting of Jesus Baang. Their testimonies invariably pointed
to Rading Sason as the lone gunwielder. One witness, Danilo Soriano, 21 years old, laborer,
graphically described the incident. He recounted that at around four o’clock in the afternoon of
November 25, 1983, he was at the store of Lola Ida in Dampalit, Malabon, Metro Manila. He saw the
victim Jesus Baang walking along M. Sioson Street, followed by Rading Sason. Suddenly, Sason shot
Baang from behind. After the first gunshot, Baang turned around and faced his assailant who then
shot him for the second time. Baang tried to escape but he fell on the ground. It was at this point when
the gunman delivered the coup de grace and fatally shot Baang on the head. Sason then walked away
from the scene. 3

The witnesses’ narrations closely coincided with the findings of Dr. Ricardo Ibarrola, the NBI medico-
legal officer who autopsied the victim’s cadaver at nine o’clock in the evening of November 25, 1983.
His report disclosed that the deceased Baang was shot at the back, chest and head with a .45 caliber
pistol. He sustained three (3) wounds, two of which were fatal.8

The motive for the killing was apparently caused by the victim himself. It appears that one day before
the wedding of Laura, a sister of Rading Sason, a former girlfriend (Tiray) of the prospective groom
arrived from Palawan. Baang allowed her to sleep in his house and then brought her the following
morning to the church. The woman created a scene and as a result, the wedding was celebrated at
the side altar, instead of the main altar. The incident caused embarrassment and humiliation to the
bride’s family, particularly Rading Sason. 9

It was through the eyewitness account of Ismael "Doro" Rivera that the prosecution sought to
establish the existence of a conspiracy between Sason, the triggerman, and accused-appellant
Rolando Cruz to liquidate Baang.

Rivera, a 22-year old laborer and a cousin of the two accused, testified that at around two o’clock in
the afternoon of November 25, 1983, he was in the house of Aling Jusing, drinking with Rading Sason,
Danilo Soriano and Renato Ramirez. After a while, Soriano left, followed by Ramirez who said that he
was going to have a haircut. Then Sason followed suit and headed in the direction of the church. 10
CRIMINAL LAW 1
(Sunday, Section 52) 131

"The actuations of the latter (Rolando Cruz) in uttering the words ‘Andiyan na’ to Rading Sason of the
presence of Jesus Baang could not be given any meaning except that he is a party in the commission
of the crime in the killing of Jesus Baang. He (Rolando Cruz) certainly conspired and helped Rading
Sason in the consummation of the crime charged. There is a moral certainty that conspiracy existed
and therefore Rolando Cruz is as guilty as the principal by induction in the commission of the crime
of murder." 12

Rolando Cruz, on his part, maintains his innocence. He claims that at the time Jesus Baang was gunned
down by Rading Sason, he was at the back of his mother’s house near the fishpond, which is three (3)
electric posts away from the scene of the crime. He was spreading and laying the fishnet. Then Ismael
Rivera called out to him and told him of the violent incident at the "labasan." He quickly went outside
and saw many people looking at the sprawled body of the victim. On that same day, November 25,
1983, Cruz was arrested by the police and brought to the police headquarters for investigation. 13

ISSUE:
Could appellant be held liable as an accomplice since he cooperated in the execution of the offense
by signalling the arrival of the deceased.

HELD:
WHEREFORE, the appealed decision is REVERSED and SET ASIDE. Accused-appellant Rolando Cruz
alias "Tikboy" is hereby ACQUITTED on reasonable doubt.

RATIO:
It must be explained that the cooperation which the law punishes is the assistance which is knowingly
or intentionally given and which is not possible without previous knowledge of the criminal purpose.
20 In the case at bar, there is absence of conclusive proof indicating a prior plan or agreement
between appellant Cruz and Sason to kill the victim. That essential element cannot be automatically
deduced from the fact that the two accused had harbored a grudge against Baang.

All told, the criminal complicity of appellant Cruz, either as a principal by inducement or as an
accomplice, in the killing of Baang has not been established beyond reasonable doubt. It follows that
he must be acquitted.

Conspiracy, like any other ingredient of an offense, must be proved as sufficiently as the crime itself
through clear and convincing evidence, not by mere conjectures. It is also a well-entrenched rule that
proof beyond reasonable doubt is required to establish a finding of criminal conspiracy. 15 In the
case at bar, while the prosecution’s evidence against Sason appears overwhelming, its evidence
linking accused-appellant Cruz to the felonious acts of Sason is weak and inconclusive and utterly
insufficient to establish Cruz’s guilt.

Inducement exists if the command or advice is of such a nature that, without its concurrence, the
crime would not have materialized. 17 It taxes the imagination how the controversial phrase imputed
to the appellant could become the moving cause without which Sason would not have shot the victim
in cold blood. Murder is a serious charge. The slightest doubt must be resolved in favor of innocence.
CRIMINAL LAW 1
(Sunday, Section 52) 132

PEOPLE V CARBONELL
48 Phil. 868 (1926)

FACTS:
On the date of the commission of the crime there existed in the municipality of Hog, Province of
Occidental Negros, two rival societies, enemy to each other, known as Mainawaon and Kusug Sang
Imol,respectively. Eliseo Olmedo was a member of the Mainawaon and Jose Carbonel, Mamerto de
Leon, Fidel Arrojo, Catalino Matula, Silvino Bulahan, Susano Gualdrapa and Felipe Gualdrapa, were
members of the Kusug Sang Imol. Four days before the crime, Fidel Arrojo was stopped on his way
and then chased by four Mainawaons who tried to catch him, but he succeeded in escaping. In the
month of October, Fidel Arrojo went to the club of the Kusug Sang Imol and told the vice-president,
Ramon Larracas, and the secretary, Francisco Gemora, both of the said club, that
the Mainawaons of the barrio of Gosy, headed by Eliseo Olmedo, were after him. Upon hearing
such notice, Ramon Larracas struck the table with his fist and said: "Why did you not kill
him? Kill him." Francisco Gemora, also striking the table with his fist, seconded the proposition,
saying: "You try to kill him; you must kill him, because if you kill him there, where there are
many Mainawaons, they will become afraid, and if you kill him, do not be afraid because in Bacolod
there are good lawyers. I am a procurador here in Bacolod and our club has a good lawyer in
Bacolod." After the said interview, on the morning of December 26, 1924, Fidel Arrojo was telling
everybody the following: "Tonight, I am going to kill three Mainawaons; if not F. Bello, then Juan
Catalan; if not Juan Catalan, then Eliseo Olmedo," On the afternoon of the. same day, Eliseo Olmedo,
Mamerto de Leon, Fidel Arrojo, Jose Carbonel, Catalino Matula, Silvino Bulahan, Susano Gualdrapa,
Felipe Gualdrapa, Santiago Helboligaya, Andres de Leon and Vicente Genito met in the house of one
Basilio Salinas where there was a little celebration of the birthday of one of the children of the owner
of the house. During their stay in the said house and while drinking tuba and eating some viands,
Fidel Arrojo and his co-accused were seen talking in a low voice and so behaving as to arouse
the suspicion of the other guests that something wrong was being planned. Fidel Arrojo was seen
looking intently at Eliseo Olmedo from head to foot. When Andres de Leon asked him why he looked
at Eliseo Olmedo in such a manner, Fidel Arrojo whispered to him in the ear "You shut up, a lightning
will strike that fellow, I am going to kill him." When the departure of the guests began, Mamerto de
Leon and Fidel Arrojo were the first to leave, followed by Vicente Genito and Santiago
Helboligaya. Afterwards Eliseo Olmedo left the house accompanied by Catalino Matula, Jose
Carbonel, Felipe Gualdrapa, Susano Gualdrapa and Silvino Bulahan. When they were about
30 meters from the house of Basilio Salinas, the voice of Fidel Arrojo was heard, saying "Go ahead,
strike him now." Catalino Matula then placed his right hand upon the shoulder of Eliseo Olmedo,
whereupon a fight ensued between the two. Silvino Bulahan, Susano Gualdrapa, Mamerto de
Leon and Felipe Gualdrapa immediately also seized Eliseo Olmedo, two upon each arm, pulling his
two arms crosswise; two holding both legs; one holding his waist and Mamerto de Leon holding his
nape with his left hand. When Eliseo Olmedo had become weakened, his assailants took of his shirt
and lowered his pantaloons until the knees. On that instant the voice of Fidel Arrojo was again heard
saying "strike him now with the bolo." Upon hearing this, in the twinkling of an eye everybody
let Eliseo Olmedo and Mamerto de Leon strike him with his bolo on the nape and everybody run
away. The witness Vicente Genito who saw with Andres de Leon all of this from behind some
shrubbery, attempted to interfere, but Mamerto de Leon met him and said "You also want to help
him?," at the same time giving him a bolo blow which hit him on the right hand, inflicting a wound
2 inches long. When the body of Eliseo Olmedo was examined by Dr. Fortunato
Angeles on December 28, 1924, the following wounds and contusions were found: One on the base
of the cranium, 15 inches long by 1 inch wide and 1½ inches deep, which ran from the posterior part
of the right ear downward almost to the back part of the lobe of the left ear cutting the scalp,
the occipital bone and the hard membrane which covers the brains, and penetrating to the arteries
CRIMINAL LAW 1
(Sunday, Section 52) 133

and the cerebellum, this wound is mortal of necessity; another wound on the left shoulder 1½ inches
long, having the same direction as the former one apparently inflicted by the same bolo blow; one
ecchymosis about 3 inches in diameter on the back; another on the buttock one upon each calf and
one on each omoplate.

It is a phenomenon now no longer a matter of speculation, but a psychological truth, that two persons
seeing the same accident or act do not perceive the details in the same manner nor receive the same
impression. Experience has shown that there can only be absolute coincidence in the details of
observation of the same accident or act by two or more persons when there has been a previous
concert.

ISSUE:
Does the facts proven at the trial constitute the crime of murder, qualified by the circumstances of
known premeditation and alevosia, as regards Fidel Arrojo and of alevosia as regards the other
defendants, each and everyone of them being criminally responsible as principal?

HELD:
Yes. Catalino Matula, Silvino Bulahan, Felipe Gualdrapa and Susano Gualdrapa are held guilty of the
crime of murder, as principals by direct participation, and each of them is sentenced to the
penalty of cadena perpetua, said judgment being affirmed in all other respects, with
proportional costs against the appellants and with credit of one-half of the preventive
imprisonment already suffered.

RATIO:
Direct proof is not essential to show conspiracy. "It need not be shown that the parties actually came
together and agreed in express terms to enter in and pursue a common design. The existence of the
assent of minds which is involved in a conspiracy may be, and, from the secrecy of the crime,
usually must be, inferred by the jury from proof of facts and circumstances which, taken together,
apparently indicate that they are merely parts of some complete whole. If it is proved that two or
more persons aimed by their acts towards the accomplishment of the same unlawful object, each
doing a part so that their acts, though apparently independent, were in fact connected
and cooperative, indicating a closeness of personal association and a concurrence of sentiment, a
conspiracy may be inferred though no actual meeting among them to concert means is proved. The
details of the consipiracy need not be proved. If a community of purpose among the parties
to do some criminal act or acts is shown, it is not necessary that the acts which are charged, or of
which evidence has been given, were specifically contemplated by them or included in the original
design.
It is not essential that each conspirator shall take part in every act, or that he shall know the
exact part to be performed by the other conspirators in execution of the conspiracy. Conspiracy
implies concert of design and not participation in every detail of execution. Nor is it necessary
that the plan of xi combination shall embrace in detail in its early stages the various means by which
it is to be executed, as it is sufficient that there is a general plan to accomplish the result sought by
such means as may from time to time be found expedient." (12 Corpus Juris, p. 545.) "Generally it
is not material that the plan which was carried out differs widely from the original plan, nor will it be
required to show the existence of any previous plan if, from the evidence, it seems clear that
there had been negotiations to the same end.”
CRIMINAL LAW 1
(Sunday, Section 52) 134

PEOPLE V TABUSO

Facts:
Prosecution witnesses narrated that the victim Robert Bugasin passed by the group
of Arnold Mendoza, accused Arquillos Tabuso, and some other companions. Tabuso allegedly
shouted "nandiyan na si dagul", and then gunshots were heard. They all left hurriedly thereafter.
Tabuso for his part consistently testified that he knew nothing of the killing when he was identified
by witnesses pointing to him as a relative of ArnoldMendoza. Arnold Mendoza was the one seen
shooting the victim.

Issue:
Whether Tabuso may be convicted for murder as a conspirator.

Held:
He may not be convicted. Settled is the rule that to establish conspiracy, evidence of actual
cooperation rather them mere cognition or approval of an illegal act is required. Conspiracy certainly
transcends companionship. Assumed intimacy between two persons of itself does not give that much
significance to the existence of criminal conspiracy.

PEOPLE VS. LARRAÑAGA


G.R. Nos. 138874-75, February 3, 2004

FACTS:
On the night of July 16, 1997, victims Marijoy and Jacqueline Chiong failed to come home on the
expected time. Twodays after, a young woman was found dead at the foot of a cliff. Her pants were
torn, her t-shirt was raised up to herbreast and her bra was pulled down. Her face and neck were
covered with masking tape and attached to her left wristwas a handcuff. The woman was identified
as Marijoy. After almost ten months, accused Davidson Rusia surfaced andadmitted before the police
having participated in the abduction of the sisters. He identified appellants Francisco Juan Larrañaga,
Josman Aznar, Rowen Adlawan, Alberto Caño, Ariel Balansag, James Anthony Uy, and James Andrew
Uy asco-perpetrators in the crime. Rusia provided the following before the trial court:1) That at
10:30 in the evening of July 16, 1997, he met Rowen and Josman and told him to ride with them in a
white car.Following them were Larrañaga, James Anthony and James Andrew who were in a red car.
Josman stopped the white carin front of the waiting shed where the sisters Marijoy and Jacqueline
were standing and forced them to ride the car.Rusia taped their mouths while Rowen handcuffed
them jointly.2) That after stopping by a safe house, the group thereafter headed to the South Bus
Terminal where they met Albertoand Ariel, and hired the white van driven by the former. They
traveled towards south of Cebu City, leaving the red car atthe South Bus Terminal.3) That after
parking their vehicles near a precipice, they drank and had a pot session. Later, they started to rape
Marijoyinside the vehicle, and thereafter raped Jaqueline.4) That Josman instructed Rowen and Ariel
to bring Marijoy to the cliff and push her into the ravine.The claims of Rusia were supported by other
witnesses. He was discharged as an accused and became a state witness.Still, the body of Jacqueline
was never found. The trial court found the other appellants guilty of two crimes ofkidnapping and
serious illegal detention and sentenced each of them to suffer the penalties of two (2)
reclusionperpetua. The appellants assailed the said decision, arguing inter alia, that court erred in
finding that there wasconspiracy. James Anthony was also claimed to be only 16 years old when the
crimes were committed.
CRIMINAL LAW 1
(Sunday, Section 52) 135

ISSUES:
1) Whether or not there was conspiracy.
2) Whether or not the trial court erred in characterizing the crime.
3) Whether or not the trial court erred imposing the correct penalty.

HELD:
1) Yes. Conspiracy may be deduced from the mode and manner by which the offense was perpetrated,
or may be inferred from the acts of the accused themselves, when such point to a joint design and
community of interest. The appellants’ actions showed that they had the same objective to kidnap
and detain the Chiong sisters. The Court affirmed the trial court’s finding that the appellants indeed
conspired in the commission of the crimes charged.

2) Yes. The rule is that when the law provides a single penalty for two or more component offenses,
the resulting crime is called a special complex crime. Article 267 of the Revised Penal Code, as
amended by Section 8 of R.A. 7659, provides that in the crime of kidnapping and serious illegal
detention, when the victim is killed or dies as a consequence of the detention, or is raped or is
subjected to torture or dehumanizing acts, the maximum penalty shall be imposed. Thus, the
resulting crime will change from complex crime to special complex crime. In the present case, the
victims were raped and subjected to dehumanizing acts. Thus, the Court held that all the appellants
were guilty of the special complex crime of kidnapping and serious illegal detention with homicide
and rape in the case where Marijoy is the victim; and simple kidnapping and serious illegal detention
in the case of Jacqueline.3) Yes. Article 68 of the Revised Penal Code provides that by reason of
minority, the imposable penalty to the offender is one degree lower than the statutory penalty. James
Anthony was only 16 years old when the crimes were committed. Aspenalty for the special complex
crime of kidnapping and serious illegal detention with homicide and rape is death, the correct penalty
to be imposed should be reclusion perpetua. On the other hand, the penalty for simple kidnapping
and serious illegal detention is reclusion perpetua to death. One degree lower from the said penalty
is reclusion temporal.There being no aggravating and mitigating circumstance, the penalty to be
imposed on him should be reclusiontemporal in its medium period. Applying the Indeterminate
Sentence Law, he should be sentenced to suffer the penaltyof twelve (12) years of prison mayor in
its maximum period, as minimum, to seventeen (17) years of reclusion temporal in its medium
period, as maximum. With regard to the rest of the appellants, the statutory penalty as provided
above should be imposed. Therefore, trial court erred in merely imposing “two (2) reclusion
perpetua”.

ESTRADA V. SANDIGANBAYAN
GR 148965 – Feb. 26, 2002

FACTS:
As an offshoot of the impeachment proceedings against Joseph Ejercito Estrada, then President of the
Republic of the Philippines, five criminal complaints against the former President and members of
his family, his associates, friends, and conspirators were filed with the respondent Office of the
Ombudsman.

The respondent Ombudsman issued a Joint Resolution finding probable cause warranting the filing
with the Sandiganbayan of several criminal Informations against the former President and the other
respondents therein. One of the Informations was for the crime of plunder under Republic Act No.
7080 and among the respondents was herein petitioner Jose Jinggoy Estrada, then mayor of San Juan,
Metro Manila.
CRIMINAL LAW 1
(Sunday, Section 52) 136

ISSUE:
Should the charge against petitioner be dismissed on the ground that the allegation of conspiracy in
the Information is too general?

HELD:
No. In the crime of plunder, different parties may be united by a common purpose. In the case at bar,
the different accused and their different criminal acts have a commonality to help the former
President amass, accumulate or acquire ill-gotten wealth. Sub-paragraphs (a) to (d) in the Amended
Information alleged the different participation of each accused in the conspiracy. The gravamen of
the conspiracy charge, therefore, is not that each accused agreed to receive protection money from
illegal gambling, that each misappropriated a portion of the tobacco excise tax, that each accused
ordered the GSIS and SSS to purchase shares of Belle Corporation and receive commissions from such
sale, nor that each unjustly enriched himself from commissions, gifts, and kickbacks; rather, it is that
each of them, by their individual acts, agreed to participate, directly or indirectly, in the amassing,
accumulation and acquisition of ill-gotten wealth of and/or for former President Estrada.
Under the Philippine law, conspiracy should be understood on two levels. As a general rule,
conspiracy is not a crime in our jurisdiction. It is punished as a crime only when the law fixes a penalty
for its commission such as in conspiracy to commit treason, rebellion, and sedition. In contrast, under
American criminal law, the agreement or conspiracy itself is the gravamen of the offense. When
conspiracy is charged as a crime, the act of conspiring and all the elements of the said crime must be
set forth in the complaint or information. The requirements on the sufficiency of allegations are
different when conspiracy is not charged as a crime in itself but only as the mode of committing the
crime as in the case at bar.

There is less necessity of reciting its particularities in the Information because conspiracy is not the
gravamen of the offense charged. The conspiracy is significant only because it changes the criminal
liability of all the accused in the conspiracy and makes them answerable as co-principals regardless
of the degree of their participation in the crime. The liability of the conspirators is collective and each
participant will be equally responsible for the acts of others, for the act of one is the act of all.
A conspiracy indictment need not, of course, aver all the components of conspiracy or allege all the
details thereof, like the part that each of the parties therein has performed, the evidence proving the
common design or the facts connecting all the accused with one another in the web of the conspiracy.
Neither is it necessary to describe conspiracy with the same degree of particularity required in
describing a substantive offense. It is enough that the indictment contains a statement of facts relied
upon to be constitutive of the offense in ordinary and concise language, with as much certainty as the
nature of the case will admit, in a manner that can enable a person of common understanding to know
what is intended, and with such precision that the accused may plead his acquittal or conviction to a
subsequent indictment based on the same facts.

Following the stream of our own jurisprudence, it is enough to allege conspiracy as a mode in the
commission of an offense in either of the following manner: (1) by use of the word conspire, or its
derivatives or synonyms, such as confederate, connive, collude, etc; or (2) by allegations of basic facts
constituting the conspiracy in a manner that a person of common understanding would know what
is intended, and with such precision as would enable the accused to competently enter a plea to a
subsequent indictment based on the same facts.
Thus, the petition is dismissed for failure to show that the respondent Sandiganbayan acted without
or in excess of jurisdiction or with grave abuse of discretion amounting to lack of jurisdiction.
CRIMINAL LAW 1
(Sunday, Section 52) 137

PEOPLE V DAGANI
GR No. 153875 – Aug. 16, 2005

Facts:
This is a murder case filed against Otello Sanitano and Rolando Dagani for the killing of Ernesto Javier.
At about 4:45 in the afternoon of September 11, 1989, a group composed of Ernesto Javier (Javier),
Lincoln Miran (Miran), and two other individuals had been drinking at the canteen located inside the
compound of the Philippine National Railways (PNR) along C.M. Recto Avenue, Tondo, Manila. All of
a sudden, appellants, who were security officers of the PNR and covered by the Civil Service Rules
and Regulations, entered the canteen and approached the group. Appellant Dagani shoved Miran,
causing the latter to fall from his chair. Dagani then held Javier while Santiano shot Javier twice at his
left side, killing the latter. The RTC ruled that Otello Santiano and Rolando Dagani are guilty beyond
reasonable doubt of the crime of Murder defined and punished under Art. 248, RPC, with the
presence of the mitigating circumstance of voluntary surrender. Appellants invoked the justifying
circumstances of self-defense and lawful performance of official duty as PNR security officers. They
also argued that the prosecution failed to establish treachery and conspiracy.

Issues:
1. Whether or not the lower erred in not appreciating self-defense on the part of the accused.
2. Whether or not it was a performance of lawful duty.
3. Whether or not there was a conspiracy.

Held:
1. The defense was unable to prove that there was unlawful aggression on the part of Javier. They
were unable to present evidence that the victim actually fired his gun. No spent shells from the .22
caliber pistol were found and no bullets were recovered from the scene of the incident. Javier also
tested negative for gunpowder residue.
Moreover, the trial court found appellant Dagani’s account of the incident to be incredible and self-
serving. In sum, the defense presented a bare claim of self defense without any proof of the existence
of its requisites.
2. The defense failed to prove that the security officers were in fact on duty at the time they were at
the canteen. The trial court gave weight to the fact that the appellants were unable to submit their
daily time records to show that they were on duty at the time. Appellants’ assertion that they were
ordered to go on 24-hour duty was belied by PNR Security Investigator Rolando Marinay’s testimony
that PNR security officers work in two 12-hour shifts, from 7:00 a.m. to 7:00 p.m. and from 7:00 p.m.
to 7:00 a.m. 3. The Supreme Court cannot agree with the findings of the courts a quo that the
appellants were in conspiracy simply because there is no evidence presented to show that the
appellants planned to kill Javier. Wherefore the decision of the Court of Appeals is modified.
Appellant Otello Santiano is found guilty beyond reasonable doubt of Homicide. Appellant Rolando
Dagani is hereby acquitted.

MANABAN V. COURT OF APPEALS


G.R. No. 150723, 11 July 2006

FACTS:
At around 1:00 am on Oct. 11, 1996, Joselito Bautista’s daughter was rushed to UP Health Center.
Needing money, Bautista (member of UP Police force) went to withdraw money from the ATM at BPI
Kalayaan Branch where accused Ramonito Manaban was on duty. It was alleged that had taken
alcoholic drinks before that. Bautista failed to withdraw and that his card was captured because he
CRIMINAL LAW 1
(Sunday, Section 52) 138

had entered a wrong pin. He then started kicking and pounding the machine. Manaban asked him the
problem and suggested that Bautista return the next morning. This angered Bautista all the more and
resumed pounding the machine. Manaban urged him to calm down but continued raging and striking
the machine. When Manaban could no longer pacify him, he fired a warning shot. That diverted the
attention of Bautista. Instead of venting his ire against the machine, he confronted Manaban. After
some exchange of words, a shot rang out fatally hitting Bautista. Several police officers arrived at the
crime scene where they saw Bautista lying on the ground, still breathing. They noticed that a .38
caliber pistol was tucked in Bautista’s waist. Bautista was shot at the back. Manaban allegedly
admitted the shooting. They brought Bautista to East Avenue Medical Center where he died. The NBI
medico-legal officer testified that Bautista was shot at the back, the muzzle of the gun at about more
than 24inches away from the entry point, and that the wound incurred by him was fatal as the bullet
hit the right lung and lacerated parts of the liver, stomach and the pancreas. He deduced that assailant
must have been behind the victim. Manaban testified that after he fired the warning shot, Bautista
allegedly raised his shirt and showed his gun which was tucked in his waist. Manaban stepped back
and told Bautista not to draw his gun, otherwise he would shoot. However, Bautista allegedly kept on
moving toward Manaban, who again warned Bautista not to come near him or he would be forced to
shoot him. Bautista suddenly turned his back and was allegedly about to draw his gun. Fearing that
he would be shot first, Manaban pulled the trigger and shot Bautista. Manaban declared that it did
not occur to him to simply disable the victim for fear that Bautista would shoot him first.

ISSUE:
Whether there was unlawful aggression on the part of the victim, Bautista.

HELD:
NO. Unlawful aggression is an actual physical assault or at least a threat to attack or inflict physical
injury upon a person. A mere threatening or intimidating attitude is not considered unlawful
aggression, unless the threat is offensive and menacing, manifestly showing the wrongful intent to
cause injury. There must be an actual, sudden, unexpected attack or imminent danger thereof,
which puts the defendant’s life in real peril.
In this case, there was no unlawful aggression on the part of the victim. First, Bautista was shot at
the back as evidencedb y the point of entry of the bullet. Second, when Bautista was shot, his gun was
still inside a locked holster and tucked in his right waist. Third, when Bautista turned his back at
Manaban, Manaban was already pointing his service firearm at
Bautista. These circumstances clearly belie Manaban’s claim of unlawful aggression on Bautista's
part. The allegation of Manaban that Bautista was about to draw his gun when he turned his back at
Manaban is mere speculation. Besides, Manaban was already aiming his loaded firearm at Bautista
when the latter turned his back. In that situation, it was Bautista whose life was in danger considering
that Manaban, who had already fired a warning shot, was pointing his firearm at Bautista. Bautista,
who was a policeman, would have realized this danger to his life and would nothave attempted to
draw his gun which was still inside a locked holster tucked in his waist. Furthermore, if Manaban
really feared that Bautista was about to draw his gun to shoot him, Manaban could have easily
disabled Bautista by shooting his arm or leg considering that Manaban’s firearm was already aimed
at Bautista. Aggression presupposes that the person attacked must face a real threat to his life and
the peril sought to be avoided is imminent and actual, not imaginary. Absent such actual or imminent
peril to one’s life or limb, there is nothing to repel and there is no justification for taking the life or
inflicting injuries on another

SENOJA V. PEOPLE
G.R. No. 160341, 19 October 2004
CRIMINAL LAW 1
(Sunday, Section 52) 139

Facts:
Exequiel Senoja, Fidel Senoja (they were brothers), Jose Calica and Miguel Lumasac were drinking
gin in the hut of Crisanto Reguyal. Leon Lumasac suddenly barged in, holding a bolo and was looking
for his brother Miguel whom he suspected of drying up the ricefield he was plowing. However, when
Senoja (Exequiel) approached Leon, the latter tried to hack him so he embraced Leon and Jose took
Leon’s bolo. After the confrontation, Leon wanted to get his bolo back because he wanted to go home.
After getting it back, Leon walked out of the place followed by Senoja. Suddenly, Senoja stabbled Leon
at the back. When Leon turned around, Senoja continued stabbing him until he fell to the ground.
Then petitioner ran towards the barangay road and threw away the knife he used to stab Leon.
Petitioner admitted killing the victim but invoked the affirmative defense of self-defense. His version
said that after the commotion inside the house, Leon left but with a threat that something will happen
to Senoja. Senoja followed Leon as the latter was making his way home. When Leon realized that
Senoja was following him, Leon walked back towards him and suddenly hacked Senoja at the left side
of his head and right thigh. Unable to evade the treacherous attack by Leon, Senoja drew his colonial
knife and stabbed Leon in self-defense, inflicting upon him multiple wounds which caused his death.

Issue:
W/N Senoja merely acted in self-defense

Held:
No, Senoja is guilty of HOMICIDE. The affirmative defense of self-defense may be complete or
incomplete. It is complete when all the three essential requisites are present; it is incomplete if only
unlawful aggression on the part of the victim and any of the two essential requisites were present.
Unlawful aggression on the part of the victim is a condition sine qua non to self-defense, complete or
incomplete. The right of self-defense proceeds from necessity and limited by it. The right begins
where necessity does, and ends where it ends. There is however, a perceptible difference between
necessity and self-defense. Self- defense excuses the repulse of a wrong; necessity justifies the
invasion of a right. Hence, it is essential to self-defense that it should be a defense against a present
unlawful attack. Self-defense is an act to save life; hence, it is right and not a crime. x x x It is a settled
rule that to constitute aggression, the person attacked must be confronted by a real threat on his life
and limb; and the peril sought to be avoided in imminent and actual, not merely imaginary. But what
then is the standard? We rule that the test should be: does the person invoking the defense believe,
in due exercise of his reason, his life or limb is in danger? Hence, when an inceptual/unlawful
aggression ceases to exist, the one making a defense has no right to kill or injure the former aggressor.
After the danger has passed, one is not justified in following up his adversary to take his life.
IN THIS CASE, there were two events concerned: 1) The arrival of Leon who was armed with a bolo
and 2) When Leon demanded for his bolo because he wanted to go home already after the commotion
inside the house, and then eventually left with a threat. Quoting the appellate court, the SC said that
the victim had already left the hut. At that point in time, the victim was simply walking toward his
home; he had stopped being an aggressor. It was Senoja who wanted a confrontation this time. It was
Senoja who was now the unlawful aggressor in this second phase of their confrontation.
CRIMINAL LAW 1
(Sunday, Section 52) 140

PEOPLE V. DECENA
G.R. No. 107874, 4 Aug. 1994, 235 SCRA 67

Facts:
On Christmas day, around 4pm, Luzviminda (14 y.o., daughter of the Jaime Ballesteros, victim), saw
Decena rushing towards her father with a long bladed weapon prompting her to warn her father.
Decena, however, stabbed him on the right chest causing his death.
Narration of the defense:
At about 4pm, the victim was drunk and for no apparent reason, he held the appellant by the neck
and poked a fork against it. A barangay tanod intervened and advised the appellant to go home.
Appellant left but was later followed by Jaime (victim).
Biala, uncle of the appellant, testified that he saw Jaime attacking the appellant with a balisong.
Appellant was able to parry the blow, and overpowering Jaime, thruste the knife into his body.
Issue: W/N the appellant acted in complete self-defense that in killing Jaime Ballesteros absolving
him from criminal liability.

Held:
No.

Ratio:
In criminal cases, the burden of proof is on the prosecution which may rely on the strength of its
evidence and not on the weakness of the defense. However, upon invoking self-defense, the accused
admits that he killed the victim and the burden of proof is upon him in proving that he really acted in
self-defense. Basic requirement for self-defense as a justifying circumstance is unlawful aggression
against the person defending himself. It must be shown that there was a previous unlawful and
unprovoked attack that placed the life of the accused in danger forcing him to inflict wounds upon
his assailant. According to the defense, the unlawful aggression started when the victim started
poking the appellant with a fork. Elementary rule: when the aggressor leaves, the aggression ceases.
It follows that when appellant and Jaime heeded the advice of the barangay tanod, the unlawful
aggression had ended. Since the aggression no longer existed, appellant had no right to kill or even
wound the former aggressor. The defense failed to establish that the victim persisted in his design to
attack the appellant Defense: continuing aggression. Whenever the victim was drunk, he would look
for trouble (refuted by the testimony of the wife). Witnesses: Jaime was staggering or wobbling as he
walked – the victim could not have persisted in attacking the appellant with his current state.
Testimony of the uncle: imaginative or coached witness

PEOPLE V. DELA CRUZ


G.R. No. 128359, 6 Dec. 2000, 347 SCRA 100

Facts:
Appellant Mark Dela Cruz was found guilty of violation of Section 5, Article II of Republic Act (R.A.)
No. 9165 after he allegedly sold prohibited drugs to the poseur-buyer. The prohibited drugs were
handed to appellant by companions identified to be an alias Amay and an alias Tabo. Appellant denied
the charge and said that he was arrested after refusing to give information about Amay, whom the
police were after. His testimony was corroborated by other witnesses.

Lower court gave weight to the testimony by the poseur-buyer and upheld the presumption of
regularity in the operation conducted by the officers.
CRIMINAL LAW 1
(Sunday, Section 52) 141

Appellant appealed, questioning the identity of the shabu allegedly confiscated from him in view of
Section 21 (1) of RA No. 9165 (inventory of seized drugs) and Section 21 (3) of the same
law(certification of the forensic laboratory examination results).

Ruling:
The elements necessary for the prosecution of illegal sale of drugs are: (1) the identities of the buyer
and the seller, the object, and consideration; and (2) the delivery of the thing sold and the payment
therefor. What is material to the prosecution for illegal sale of dangerous drugs is the proof that the
transaction or sale actually took place, coupled with the presentation in court of evidence of corpus
delicti.

Citing jurisprudence, the failure of the police to comply with the procedure in the custody of the
seized drugs raised doubt as to its origins.

The chain of custody rule requires that the admission of an exhibit be preceded by evidence sufficient
to support a finding that the matter in question is what the proponent claims it to be. TheCourt
believed that the prosecution failed to clearly establish the chain of custody of the seized plastic
sachets, containing shabu from the time they were first allegedly received until they were brought to
the police investigator. There were no records to show that the procedural requirements in Section
21 were complied with.

The presumption of regularity cannot prevail over the constitutional right of presumption of
evidence in view of the circumstances. “The presumption of regularity is merely just that--a mere
presumption disputable by contrary proof and which when challenged by the evidence cannot be
regarded as binding truth.”

The appellant was acquitted.

PEOPLE V. JAURIGUE
G.R. No. 384, 21 February 1946, 76 Phil. 174

Topic. Commission of crime in a place of worship

Case. Petition to review lower court decision finding Avelina guilty of homicide with ACs

Facts:
Victim Amado was courting Avelina. In one occasion, while Avelina’s cousing washed the latter’s
hankerchief, Amado stole it. Amado proposed her love to Avelina. Avelina refused. Amado grabbed
her, kissed her, and touched her chest. To this, Avelina slapped him, threw fist blows, and kicked him.
Avelina told her mother the next day. Her mother gave her a fan knife for protection.

Not long after, Amado intruded in Avelina’s house while she was asleep. He kissed her hand which
woke her up. She screamed for help, he hid under the bed. Her parents arrive in the room. He
apologized.

Days after, Avelina and her family attended services in the Seventh Day Adventists chapel. Avelina’s
father got there first, Avelina followed and sat on one of the benches. Amado saw Avelina, sat beside
her and put his hand in her right thigh. Avelina tried to get her knife with right hand. Amado held her
right hand to stop her. Avelina quickly grabbed the knife with her left hand and stabbed Amado in
the neck.
CRIMINAL LAW 1
(Sunday, Section 52) 142

Amado staggered. Avelina’s father saw Amado bleeding. Her father asked why she did it, she said she
could not endure it anymore. CFI found Avelina guilty of homicide wit the aggravating circumstance
of having committed the act in a place of worship, inter alia. Avelina appealed.

Issue:
Is the AC of offending a place of worship applicable? -No

Ratio:
While it is true that the crime occurred in a chapel, this does not necessarily denote the attachment
of said AC. As per law, for a proper appreciation of AC of having committed the crime in a place of
worship, there should be premeditation that the offender intended to do it in the place of worship.
As the facts show, Avelina did not plan to kill Amado nor to kill him in a chapel. Avelina had in her
case the fan knife just for protection whenever and wherever she might need it. It just so happened
that Amado provoked her in the chapel and that was where Avelina could not bear it anymore. With
this in mind, the present court ruled that the lower court erred in assigning the AC in its judgment
against Avelina.

PEOPLE V. NARVAEZ
L-33466-67, 20 April 1983, 121 SCRA 389

FACTS:
Mamerto Narvaez has been convicted of murder (qualified by treachery) of David Fleischer and
Flaviano Rubia. On August 22, 1968, Narvaez shot Fleischer and Rubia during
the time the two were constructing a fence that would prevent Narvaez from getting into his house
and rice mill. The defendant was taking a nap when he heard sounds of construction and
found fence being made. He addressed the group and asked them to stop destroying his house and
asking if they could talk things over. Fleischer responded with "No, gadamit, proceed, go
ahead." Defendant lost his "equilibrium," and shot Fleisher with his shotgun. He also shot Rubia who
was running towards the jeep where the deceased's gun was placed. Prior to the
shooting, Fleischer and Co. (the company of Fleischer's family) was involved in a legal battle with the
defendant and other land settlers of Cotabato over certain pieces of property. At the time
of the shooting, the civil case was still pending for annulment (settlers wanted granting of property
to Fleisher and Co. to be annulled). At time of the shooting, defendant had leased his
property from Fleisher (though case pending and ownership uncertain) to avoid trouble. On June 25,
defendant received letter terminating contract because he allegedly didn't pay rent.
He was given 6 months to remove his house from the land. Shooting was barely 2 months after letter.
Defendant claims he killed in defense of his person and property. CFI ruled that
Narvaez was guilty. Aggravating circumstances of evident premeditation offset by the mitigating
circumstance of voluntary surrender. For both murders, CFI sentenced him to reclusion perpetua, to
indemnify the heirs, and to pay for moral damages.

ISSUES:
1. Whether or not CFI erred in convicting defendant-appellant despite the fact that he acted in defense
of his person.

No. The courts concurred that the fencing and chiselling of the walls of the house of the defendant
was indeed a form of aggression on the part of the victim. However, this
CRIMINAL LAW 1
(Sunday, Section 52) 143

aggression was not done on the person of the victim but rather on his rights to property. On the first
issue, the courts did not err. However, in consideration of the violation of property rights, the courts
referred to Art. 30 of the civil code recognizing the right of owners to close and fence their land.

Although is not in dispute, the victim was not in the position to subscribe to the article because his
ownership of the land being awarded by the government was still pending, therefore putting
ownership into question. It is accepted that the victim was the original aggressor.

2. WON the court erred in convicting defendant-appellant although he acted in defence of his rights.

Yes. However, the argument of the justifying circumstance of self-defense is applicable only if the 3
requirements are fulfilled. Art. 11(1) RPC enumerates these requisites:
Unlawful aggression. In the case at bar, there was unlawful aggression towards appellant's property
rights. Fleisher had given Narvaez 6 months and he should have left him in peace before time was up,
instead of chiseling Narvaez's house and putting up fence. Art. 536 of the Civil Code also provides
that possession may not be acquired through force or intimidation; while Art. 539 provides that every
possessor has the right to be respected in his possession
Reasonable necessity of means employed to prevent or repel attack. In the case, killing was
disproportionate to the attack.
Lack of sufficient provocation on part of person defending himself. Here, there was no provocation
at all since he was asleep
Since not all requisites present, defendant is credited with the special mitigating circumstance of
incomplete defense, pursuant to Art. 13(6) RPC. These mitigating circumstances are: voluntary
surrender and passion and obfuscation (read p. 405 explanation) Crime is homicide (2 counts) not
murder because treachery is not applicable on account of provocation by the deceased. Also, assault
was not deliberately chosen with view to kill since slayer acted instantaneously. There was also no
direct evidence of planning or preparation to kill. Art. 249 RPC: Penalty for homicide is reclusion
temporal. However, due to mitigating circumstances and incomplete defense, it can be lowered three
degrees (Art. 64) to arrestomayor.

3. WON he should be liable for subsidiary imprisonment since he is unable to pay the civil indemnity
due to the offended party.

No. He is not liable to be subsidiarily imprisoned for nonpayment of civil indemnity. RA 5465 made
the provisions of Art. 39 applicable to fines only and not to reparation of damage caused,
indemnification of consequential damages and costs of proceedings. Although it was enacted only
after its conviction, considering that RA 5465 is favorable to the accused who is not a habitual
delinquent, it may be given retroactive effect pursuant to Art. 22 of the RPC.

Judgment: Defendant guilty of homicide but w/ mitigating circumstances and extenuating


circumstance of incomplete self defense. Penalty is 4 months arresto mayor and to indemnify
each group of heirs 4,000 w/o subsidiary imprisonment and w/o award for moral damages.
Appellant has already been detained 14 years so his immediate release is ordered.

Gutierrez, dissenting. Defense of property can only be invoked when coupled with form of attack on
person defending property. In the case at bar, this was not so. Appellant should then be sentenced to
prision mayor. However, since he has served more than that, he should be released.
CRIMINAL LAW 1
(Sunday, Section 52) 144

SABANG VS PEOPLE
PEOPLE VS DAGANI

RUJJERIC Z. PALAGANAS, petitioner,vs PEOPLE OF THE PHILIPPINES, respondent


(G.R. No. 165483 September 12, 2006)

FACT:
On January 16, 1998, around 8:00 in the evening, brothers Servillano, Melton, and Michael Ferrer
were having their drinking spree at their house but later decided to proceed to Tidbits Videoke Bar
to continue their drinking spree and to sing. Thereafter, Jaime Palaganas arrived together
withFerdinand Palaganas (nephew) and Virgilio Bautista. Later, when Jaime was singing, Melton
Ferrer sang along with him as he was familiar with the song “ My Way”. Jaime got irritated and
insulted. He felt he was being mocked by Melton Ferrer that caused him to go to Ferrer’s table and
utter statements which began the fight. With this, Ferdinand, Jaime’s nephew, sought help to his
brother, Rujjeric Palaganas, who was now the petitioner in this case. They went to the bar and upon
seeing the Ferrer’s outside, Ferdinand, pointing at the Ferrer’s, instructed Rujjeric to shoot them.
Rujjeric Palaganas shot Servillano, Melton, and Michael with the use of unlicensed firearm. As a result,
Melton was killed, Servillano was fatally wounded, and Michael was shot in his right shoulder. On
October 28, 1998, RTC of Pangasinan decided in 3 Criminal Cases finding the petitioner guilty beyond
reasonable doubt of the crime of homicide and two (2) counts of frustrated homicide of Article
249 of the Revised Penal Code. On September 30, 2004, Court of Appeals AFFIRMED the decision of
the lower court with modifications considering his voluntary surrender as ordinary mitigating
circumstance. However, on November 16, 2004, petitioner prayed for the reversal of the decision
holding that the CA erred in affirming the judgment of conviction of RTC and in not acquitting him on
the ground of self-defense. Hence, this certiorari.

ISSUE:
Whether or not the use of the unlicensed firearm is a special aggravating circumstance which should
be appreciated by the court at the case at bar?

HELD:
YES. An aggravating circumstance was provided for under PD No. 1866 as amended by RA8294 which
is a special law that was passed on June 6, 1997. Its pertinent provision states: If homicide or murder
is committed with the use of an unlicensed firearm, such use of an unlicensed firearm shall be
considered as an aggravating circumstance. This special aggravating circumstance cannot be offset
by an ordinary mitigating circumstance. Voluntary surrender of petitioner in this case is merely
an ordinary mitigating circumstance. Thus, it cannot off set the special aggravating circumstance of
use of unlicensed firearm. In accordance with Article 64, paragraph 3 of the Revised Penal Code, the
penalty imposable on petitioner should be in its maximum period. Therefore, decision was
AFFIRMED by SC with certain modifications appreciating the special aggravating circumstance in the
case at bar.
CRIMINAL LAW 1
(Sunday, Section 52) 145

PEOPLE OF THE PHILIPPINES vs. CARMELO CATBAGAN


(G.R. Nos. 149430-32. February 23, 2004)

FACTS:
Carmelo Catbagan, an investigator of Criminal Investigation Service of PNP, went to the residence of
Danilo Lapidante to respond to a citizen complaint about sounds of multiple gunshots. When he
arrived at the said place, a piece of stone was hurled from the direction of the Lapidante’s house
which hit him. At the same time, Sgt. Celso Suico, the one responsible for firing the shots, extended
his hand towards Catbagan as he introduced himself. Completely ignoring the gesture of the latter,
Catbagan drew out his pistol and fired at Suico which later killed him. Almost simultaneously,
Catbagan directed his attention to Lapidante and later shot him dead. In his defense, Catbagan argued
that he was justified in shooting the victims, as he was merely defending himself and fulfilling his
sworn duties. He claimed that the victims were rushing and encircling him. The lower court,
nonetheless, convicted him with the crime of murder.

ISSUE:
W/N Catbagan should be guilty of Murder which is a grave misconduct of a police officer.

RULING:
Yes. Catbagan should be guilty of Murder, the court ruled that the appellant was not performing his
duties at the time of the shooting as there was no proof that he had personal knowledge on who had
been firing the Armalite, nor he was there to effect an arrest.

PEOPLE OF THE PHILIPPINES VS. RICOHERMOSO


(L-30527-28, March 29, 1974)
Avoidance of Greater Evil or Injury

FACT:
Geminiano de Leon, together with his common-law wife, son Marianito de Leon and one Rizal
Rosales, chanced upon Pio Ricohermoso. Owning a parcel of land, which Ricohermoso cultivated as
kaingin, Geminiano asked about his share of palay harvest and added that she should be allowed to
taste the palay harvested from his land. Ricohermoso said Geminiano could collect the palay anytime.
Upon returning from his trip to Barrio Bagobasin, Geminiano dropped by Ricohermoso’s house and
asked him about the palay, to which the latter answered defiantly that he will not give him the palay,
whatever happens. Geminiano remonstrated and that point (as if by prearrangement), Ricohermoso
unsheathed his bolo, while his father-in-law Severo Padernal got an axe, and attacked Geminiano. At
the same time and place, Ricohermoso’s brother-in-law Juan Padernal suddenly embraced Marianito.
They grappled and rolled down the hill, at which point Marianito passed out. When he regained
consciousness, he discovered that the rifle he carried beforehand was gone and that his father was
mortally wounded. The defendants shifted the responsibility of killing in their version of the case.

ISSUE:
W/N appellant Juan Padernal can invoke the justifying circumstance of avoidance of a greater evil or
injury

HELD:
No. Juan Padernal’s reliance on the justifying circumstance is erroneous because his act in preventing
Marianito from shooting Ricohermoso and Severo Padernal, the aggressors in this case, was designed
CRIMINAL LAW 1
(Sunday, Section 52) 146

to insure the killing of Geminiano de Leon without any risk to the assailants and not an act to prevent
infliction of greater evil or injury. His intention was to forestall any interference in the assault.
Treachery was also appreciated in the case. The trial court convicted the appellants with lesiones
leves, from an attempted murder charge with respect to Marianito de Leon. Judgment as to Juan
Padernal affirmed.

(Note: Severo Padernal withdrew his appeal, thus, in effect, accepted the prosecution’s version of the
case and trial court’s finding of guilt.)

PEOPLE OF THE PHILIPPINES vs. NORMA HERNANDEZ


(CA-GR No. 22553-R April 14, 1959, 55 OG 8465)

FACTS:
- Vivencio Lascano, and Maria Norma Hernandez have a boyfriend- girlfriend relationship. The two
talked about getting married.
- Vivencio’s parents went to the house of Norma to have a “pamamanhikan”. They brought chickens
and goats.
- The marriage was set on March 19, 1955. The preparation went on but on the day of wedding,
Norma is did not show up, causing Vivencio and his family great same and humiliation.
- Norma Hernandez confessed that she was not really in love with him, and that she accepted the
proposal because she was convinced by her parents. That she decided to leave home as last recourse
to prevent the marriage. Norma’s parents also corroborated her testimony.
- RTC convicted her of serious slander by deed because she purposely and deliberately fled to prevent
celebration of marriage. Thus, she appealed.

ISSUE:
Whether Or Not Norma should be convicted on the ground of serious slander by deed

HELD:
NO, SC reversed the RTC judgment and acquitted the appellant. Agreement to marry cannot be held
liable for Slander by Deed. — A party to an agreement to marry who backs out cannot be held liable
for the crime of slander by deed , for then that would be an inherent way of compelling said party to
go into a marriage without his or her free consent, and this would contravene the principle in law
that what could not be done directly could not be done indirectly; and said party has the right to avoid
to himself or herself the evil of going through a loveless marriage pursuant to Article 11, paragraph
4 of the Revised Penal Code. One of the essential requisites of slander hasn’t been proven. There is
no malice in the act of the appellant changing her mind. She was merely exercising her right not to
give her consent to the marriage after mature consideration. Furthermore, there were no strained
relations existing between the complainant & appellant before the incident. There always existed
good relations between them for they were neighbors so it cannot be sustained that appellant was
motivated by spite or ill-will in deliberately frustrating the marriage. Appellant has the privilege to
reconsider her previous commitment to marry and it would be utterly inconsistent to convict her for
slander by deed simply because she desisted in continuing with the marriage. If she would be liable,
then that would be tantamount to compelling her to go into a marriage without her free consent.
Appellant had the right to avoid to herself the evil of going through a loveless marriage. Justifying
Circumstances (Art. 11 par.4, RPC) Any person who, in order to avoid an evil or injury, does not act
which causes damage to another, provided that the following requisites are present; First. That the
evil sought to be avoided exist; Second. That the injury feared be greater than that done to avoid it;
Third. That there be no other practical and less harmful means of preventing it.
CRIMINAL LAW 1
(Sunday, Section 52) 147

PEOPLE OF THE PHILIPPINES VS. DELIMA


(G.R. No. L-18660, December 22, 1922)

FACTS:
Lorenzo Napilon escaped from the jail. Some days afterwards, policeman Felipe Delima found him in
the house of Jorge Alegria, armed with a pointed piece of bamboo in the shape of a lance, and
demanded his surrender. Napilon answered with a stroke of his lance. Delima dodged, it, and to
impose his authority fired his revolver, but the bullet did not hit him. Napilon ran away, without
parting with his weapon. Delima went after him and fired again his revolver, this time hitting and
killing him. Delima was tried and convicted by the lower court for homicide and sentenced to
reclusion temporal and the accessory penalties.

ISSUE:
W/N police officer Felipe Delima should be guilty of homicide which is a grave misconduct of a police
officer when he shot and killed Napilon on the ground of performance of a duty.

RULING:
No, the killing was done in the performance of a duty. The deceased was under the obligation to
surrender, and had no right, after evading service of his sentence, to commit assault and disobedience
with a weapon in the hand, which compelled the policeman to resort to such an extreme means,
which, although it proved to be fatal, was justified by the circumstances.

THE PEOPLE OF THE PHILIPPINES, plaintiff-appellee, vs. IGNACIO LAGATA, defendant-


appellant. (G.R. Nos. L-1940-42 March 24, 1949)

FACTS:
The accused, Ignacio Lagata, a provincial guard of Catbalogan, Samar, was in charge of 6 prisoners
(Jesus, Tipace, Eusebio, Mariano, Labong & Abria) assigned to work in the capitol plaza of Samar.
Lagata ordered the prisoners to go to the nursery to pick up gabi. Not long afterwards, they were
called to assemble. Epifanio Labong was missing so Lagata ordered the 5 remaining prisoners to go
look for him. Eusebio Abria said that while they were gathering gabi, he heard 3 shots.He was
wounded by the 2nd one. They were already assembled by the 1st shot and that he did not see Tipace
being shot. He said he ran away because he was afraid that he might be shot again and that his
companions were also probably scared and that is why they ran. Another prisoner, Mariano Ibañez
stated that Epifanio Labong did not answer their call so Ignacio Lagata ordered to go look for him in
the mountain. He said that Abria went to the camote plantation and found footprints and called on
Lagata to inform him about the footprints. When Abria told Lagata of the flattened grass and that he
was unable to look for Labong, Ignacio Lagata fired at him and he was hit on his left arm. Abria told
Lagata he was wounded and in turn, Lagata told them to assemble. Once they were assembled, Lagata
cocked his gun and shot Ceferino Tipace. Mariano said that when he saw Tipace was shot, he ran
away because he also could have been shot. Eustaquio Galet, another detainee, received good
treatment from Lagata though his testimony corroborated those of the other prisoners. Pedro
Mayuga, chief of Samar Provincial Hospital & Gilberto Rosales, Sanitary Division president, verified
the gunshot wound and that the death of Tipace resulted therein. Ignacio Lagata, however, said that
he fired his gun because the prisoners were running far from him when he already ordered them to
stop. He saidthat he would be the one in jail if a prisoner escaped under his custody. Furthermore, he
would be discharged from duty like the others. He was hopeless already. Moreover, the picking up of
gabi was not part of the prisoner’s work.
CRIMINAL LAW 1
(Sunday, Section 52) 148

HELD:
Court ruled that Lagata should be sentenced for homicide and serious physical injuries.
Appellant was entitled to the benefit of mitigating circumstance of incomplete justifying
circumstance. (Art.11par.5, RPC)

RATIO:
It was clear that Lagata had absolutely no reason to fire at Tipace. The record does not show that
Tipace was bent on committing any act of aggression or that he attempted to escape.
According to Lagata himself, Tipace was running towards and around him. How could anyone
intending to escape run towards and around the very guard one was supposed to escape from? Even
if Lagata sincerely believed that he acted in the performance of his duties, the circumstances show
that there was no necessity for him to fire directly against the prisoners as to wound them seriously
and even kill one of them. While custodians should take care for prisoners not to escape, only
ABSOLUTE NECESSITY would authorize them to fire against them.

MAMANGUN vs PEOPLE OF THE PHILIPPINES


GR No. 149152 | February 2, 2007 | J. Garcia)

Fulfillment of Duty/Lawful Exercise of Right

FACT:
Policeman (PO2) Rufino Mamangun was responding to a robbery-holdup call, with his fellow police
officers, at Brgy. Calvario, Meycauayan, Bulacan. A certain Liberty Contreras was heard shouting,
which prompted residents to respond and chase the suspect, who entered the yard and proceeded to
the rooftop of Antonio Abacan. Mamangun, with PO2 Diaz and Cruz, each armed with a drawn
handgun, searched the rooftop and saw a man who they thought was the robbery suspect.
Mamangun, who was ahead of the group, fired his gun once and hit the man, who turned out to be
Gener Contreras (not the suspect) – Contreras died of the gunshot wound.
According to the lone witness Crisanto Ayson, he accompanied the policemen to the lighted rooftop.
He was beside Mamangun when he (Ayson) recognized the deceased. According to Ayson, Mamangun
pointed his gun at the man, who instantly exclaimed “Hindi ako, hindi ako!” to which Mamangun
replied, “Anong hindi ako?” and shot him.

The defense rejects this testimony, alleging that they were the only ones at the dark rooftop when
Mamangun noticed a crouching man who suddenly continued to run. Mamangun shouted “Pulis,
tigil!” whereupon the person stopped and raised a steel pipe towards Mamangun’s head. This
prompted Mamangun to shoot the person. The three police claim that Contreras only said “Hindi ako,
hindi ako” only when they approached him. Mamangun then asked “Why did you go to the rooftop?
You know there are policemen here.” Mamangun reported the incident to the desk officer who
directed investigator Hernando Banez to investigate the incident. Banez later found a steel pipe on
the roof.

ISSUE:
W/N the death of the victim was the necessary consequence of the petitioner’s fulfillment of his duty

HELD:
No. The Court denies the instant petition and affirms Sandiganbayan’s decision after finding the
petitioner’s testimony to be nothing, but a concocted story designed to evade criminal liability. Per
CRIMINAL LAW 1
(Sunday, Section 52) 149

Sandiganbayan’s observations, the defense was self-serving for the accused and biased with respect
to his co-policemen-witnesses because: After supposed introductions and forewarnings uttered
allegedly by Mamangun, it is contrary to human experience for a man (who is not the suspect) to
attack one of three policemen with drawn guns Mamangun’s admission that he did not ask the victim
“Why did you try to hit me, if you are not the one?” clearly belies their claim The location of the entry
of bullet belies their claim because it appears that the victim instinctively shielded himself instead.
Additionally, petitioner’s pretense that Contreras struck him was not initially reported to the desk
and was only conveniently remembered when the investigator found a pipe in the crime scene.
Acts in the fulfillment of duty and self-defense does not completely justify the petitioner’s firing the
fatal gunshot. The element of unlawful aggression on the part of the victim was absent, which leads
to the failure of the petitioner’s plea. Also, there can only be incomplete justification (a privileged
mitigating circumstance) in the absence of a necessary justifying circumstance the injury was caused
by necessary consequence of due performance of duty.

PEOPLE OF THE PHILIPPINES vs DAGANI


(G.R.no. 153675 August 16, 2006)

FACTS:
This is a murder case filed against Otello Sanitano and Rolando Dagani for the killing of Ernesto Javier.
At about 4:45 in the afternoon of September 11, 1989, a group composed of Ernesto Javier, Lincoln
Miran, and two other individuals had been drinking at the canteen located inside the compound of
the Philippine National Railways (PNR) along C.M. Recto Avenue, Tondo, Manila. All of a sudden,
appellants, who were security officers of the PNR and covered by the Civil Service Rules and
Regulations, entered the canteen and approached the group. Appellant Dagani shoved Miran, causing
the latter to fall from his chair. Dagani then held Javier while Santiano shot Javier twice at his left side,
killing the latter. The RTC ruled that Otello Santiano and Rolando Dagani are guilty beyond
reasonable doubt of the crime of Murder defined and punished under Art. 248, RPC, with the
presence of the mitigating circumstance of voluntary surrender. Appellants invoked the justifying
circumstances of self-defense and lawful performance of official duty as PNR security officers. They
also argued that the prosecution failed to establish treachery and conspiracy.

ISSUES:
1. Wether or not the lower erred in not appreciating self-defense on the part of the
accused
2. Wether or not it was a performance of lawful duty
3. wether or not there was a conspiracy

HELD:
1. The defense was unable to prove that there was unlawful aggression on the part of Javier. They
were unable to present evidence that the victim fired his gun. No spent shells from the .22 caliber
pistol were found and no bullets were recovered from the scene of the incident. Javier also tested
negative for gunpowder residue. Moreover, the trial court found appellant Dagani’s account of the
incident to be incredible and self-serving. In sum, the defense presented a bare claim of self-defense
without any proof of the existence of its requisites.

2. The defense failed to prove that the security officers were in fact on duty at the time they were at
the canteen. The trial court gave weight to the fact that the appellants were unable to submit their
daily time records to show that they were on duty at the time. Appellants’ assertion that they were
ordered to go on 24-hour duty was belied by PNR Security Investigator Rolando Marinay’s testimony
CRIMINAL LAW 1
(Sunday, Section 52) 150

that PNR security officers work in two 12-hour shifts, from 7:00 a.m. to 7:00 p.m. and from 7:00 p.m.
to 7:00 a.m.

3. The Supreme Court cannot agree with the findings of the courts a quo that the appellants were in
conspiracy simply because there is no evidence presented to show that the appellants planned to kill
Javier. Wherefore the decision of the Court of Appeals is modified. Appellant Otello Santiano is found
guilty beyond reasonable doubt of Homicide. Appellant Rolando Dagani is hereby acquitted.

PEOPLE V BERONILLA
L-4445, 28 February 1955, 96 Phil 566

Facts:
Manuel Beronilla, PolicarpioPaculdo, Filipino Velasco and Jacinto Adriatico file an appeal from the
judgement of the Abra CFI, which convicted them of murder for the execution of Arsenio Borjal, the
elected mayor of La, Paz, Abra (at the outbreak of war), which was found to be aiding the enemy.

Borjal moved to Bangued because of death threats was succeeded by Military Mayor Manuel
Beronilla, who was appointed by Lt. Col. Arbold, regimental commander of the 15th Infantry of the
Phil. Army, operating as guerilla unit in Abra. Simultaneously upon his appointment, Beronilla
received a memorandum which authorized him to appoint a jury of 12 bolo men to try persons
accused of treason, espionage and aiding or abetting the enemy.

Upon the return of Borjal and his family to Abra, to escape bombing in Bangued, he was placed under
custody and tried and sentenced to death by the jury based on various complaints made by the
residents. Beronilla reported this to Col. Arnold who replied, saying “…I can only compliment you for
your impartial but independent way of handling the whole case.”

Two years thereafter, Beronilla, along with the executioner, digger and jury, were indicted for the
murder of Borjal. Soon after, President Manuel Roxas issued Executive Proclamation 8, which granted
amnesty to persons who committed acts in furtherance of the resistance to the enemy against
persons aiding in the war efforts of the enemy.

The rest of defendants applied and were granted amnesty, but Beronilla and others were convicted
on the grounds that the crime was made on purely personal motives and that the crime was
committed after the expiration of time limit for amnesty proclamation.

Issue:
W/N the defendant-appellants’ actions are covered by justifying circumstances for obedience to
lawful order of superior

Held:
Yes. The accused acted upon orders of their superior officers, which as military subordinates, they
could not question and obeyed in good faith without the being aware of its illegality.

The evidence is sufficient to sustain the claim of the defense that arrest, prosecution and trial of Borjal
was done in pursuant to express orders of superiors. Additionally, it could not be established that
Beronilla received the radiogram from Colonel Volckmann, overall area commander, which called
attention to the illegality of Borjal’s conviction and sentence. Had Beronilla known the violation, he
would not have dared to report it to Arnold. The conduct of the accused also does not show malice
CRIMINAL LAW 1
(Sunday, Section 52) 151

on their part because of the conduct of the trial, defense through counsel given to Borjal, suspension
of trial based on doubts of illegality and death sentence review sent to the superior officers.

Criminal intent then could not be established. The maxim here is actus non facitreum, nisi mens rea
(Crime is not committed if the mind of the person performing the act complained of to be innocent).

Additionally, the lower court should not have denied their claim to the benefits of the Guerilla
Amnesty Proclamation No. 8 inspite of contradictory dates of liberation of La Paz, Abra. Even if the
dates were contradictory, the court should have found for the Beronila, et al because if there are “any
reasonable doubt as to whether a given case falls within the (amnesty) proclamation should be
resolved in favor of the accused.”

Judgement reversed, appellants acquitted.

TABUENA V SANDIGANBAYAN
G.R. No. 103501-03, 17 February 1997
268 SCRA 332

Facts:
Then President Marcos instructed Luis Tabuena over the phone to pay directly to the president’s
office and in cash what the Manila International Airport Authority (MIAA) owes the Philippine
National Construction Corporation (PNCC), pursuant to the 7 January 1985 memorandum of then
Minister Trade and Industry Roberto Ongpin. Tabuena agreed. About a week later, Tabuena received
from Mrs. Fe Roa-Gimenez, then private secretary of Marcos, a Presidential Memorandum dated 8
January 1986 reiterating in black and white such verbal instruction. In obedience to President
Marcos’ verbal instruction and memorandum, Tabuena, with the help of Gerardo G. Dabao and Adolfo
Peralta, caused the release of P55 Million of MIAA funds by means of three (3) withdrawals. On 10
January 1986, the first withdrawal was made for P25 Million, following a letter of even date signed
by Tabuena and Dabao requesting the PNB extension office at the MIAA the depository branch of
MIAA funds, to issue a manager’s check for said amount payable to Tabuena. The check was encashed,
however, at the PNB Villamor Branch. Dabao and the cashier of the PNB Villamor branch counted the
money after which, Tabuena took delivery thereof. The P25 Million in cash was delivered on the same
day to the office of Mrs. Gimenez. Mrs. Gimenez did not issue any receipt for the money received.
Similar circumstances surrounded the second withdrawal/encashment and delivery of another P25
Million, made on 16 January 1986. The third and last withdrawal was made on 31 January 1986 for
P5 Million. Peralta was Tabuena’s co-signatory to the letter- request for a manager’s check for this
amount. Peralta accompanied Tabuena to the PNB Villamor branch as Tabuena requested him to do
the counting of the P5 Million. After the counting, the money was loaded in the trunk of Tabuena’s
car. Peralta did not go with Tabuena to deliver the money to Mrs. Gimenez’ office. It was only upon
delivery of the P5 Million that Mrs. Gimenez issued a receipt for all the amounts she received from
Tabuena. The receipt was dated January 30,1986. Tabuena and Peralta were charged for
malversation of funds, while Dabao remained at large. One of the justices of the Sandiganbayan
actively took part in the questioning of a defense witness and of the accused themselves; the volume
of the questions asked were more the combined questions of the counsels. On 12 October 1990, they
were found guilty beyond reasonable doubt. Tabuena and Peralta filed separate petitions for review,
appealing the Sandiganbayan decision dated 12 October 19990 and the Resolution of 20 December
1991.

Issue:
Whether or not petitioners are guilty of the crime of malversation.
CRIMINAL LAW 1
(Sunday, Section 52) 152

Held:
Luis Tabuena and Adolfo Peralta are acquitted of the crime of malversation. Tabuena acted in strict
compliance with the MARCOS Memorandum. The order emanated from the Office of the President
and bears the signature of the President himself, the highest official of the land. It carries with it the
presumption that it was regularly issued. And on its face, the memorandum is patently lawful for no
law makes the payment of an obligation illegal. This fact, coupled with the urgent tenor for its
execution constrains one to act swiftly without question. Records show that the Sandiganbayan
actively took part in the questioning of a defense witness and of the accused themselves. The
questions of the court were in the nature of cross examinations characteristic of confrontation,
probing and insinuation. Tabuena and Peralta may not have raised the issue as an error, there is
nevertheless no impediment for the court to consider such matter as additional basis for a reversal
since the settled doctrine is that an appeal throws the whole case open to review, and it becomes the
duty of the appellate court to correct such errors as may be found in the judgment appealed from
whether they are made the subject of assignments of error or not.

PEOPLE V TANEO
L-37673, 31 March 1933
58 Phil 255

FACTS:
On January 16, 1932, in the house of PotencianoTaneo’s parents in Dolores, Ormoc, Leyte, because of
severe stomachache, Potenciano slept early. While sleeping, he suddenly got up, left the room with a
bolo in hand and upon meeting his wife who tried stop him, he wounded her in the abdomen. Several
others were also attacked, this includes his father, and his guests, Fred Tanner and Luis Malinao. It
was claimed that he was dreaming when the crime happened. The trial court found Potenciano guilty
of parricide and was sentenced to reclusion perpetua.

ISSUE:
WON the defendant is criminally liable.

HELD:
No. The defendant acted while in a dream and his acts with which he was charged were not voluntary
in the sense of entailing criminal liability. The expert witness claimed that the defendant was under
the influence of hallucination and not in his right mind. The defendant is not criminally liable
however, he was ordered to be confined in an insane asylum.

PEOPLE V BONOAN
L-45130, 17 February 1937
64 Phil 87

Facts:
Celestino Bonoan is charged with the crime of murder for stabbing Carlos Guison with a knife, which
caused his death three days afterwards. An arraignment was then called, but the defense objected on
the ground that the defendant was mentally deranged and was at the time confined at the Psychopatic
Hospital. After several months of summons for doctors, production of the defendant’s complete
record of mental condition from the hospital and defendant’s admission to the hospital for personal
observation, assistant alienist Dr. Jose Fernandez finally reported to the court that Bonoan may be
discharged for being a “recovered case”. After trial, the lower court found Bonoan guilty and
sentenced him to life imprisonment.
CRIMINAL LAW 1
(Sunday, Section 52) 153

The defense now appeals, claiming the lower court made errors in finding Bonoan suffered dementia
only occasionally and intermittently, did not show any kind of abnormality, that the defense did not
establish the defendant’s insanity and finding accused guilty.

Issue:
W/N the lower court erred in finding the accused guilty

Held:
Yes. The Court finds the accused demented at the time he perpetrated the crime, which consequently
exempts him from criminal liability, and orders for his confinement in San Lazaro Hospital or other
hospital for the insane. This ruling was based on the following evidence:

Uncontradicted evidence that accused was confined in the insane department of San Lazaro Hospital
and diagnosed with dementia praecox long before the commission of the offense and recurrence of
ailments were not entirely lacking of scientific foundation
Persons with dementia praecox are disqualified from legal responsibility because they have no
control of their acts; dementia praecox symptoms similar to manic depression psychosis
Accused had an insomnia attack, a symptom leading to dementia praecox, four days prior to act
according to Dr. Francisco. Accused was sent the Psychopatic hospital on the same day of crime and
arrest, indicating the police’s doubt of his mental normalcy. Defendant suffered from manic
depressive psychosis according to Dr. Joson

Dissenting (Justices Imperial, Diaz and Concepcion):

The dissenting opinions pose that the accused committed the crime when he was sane, or at least,
during a lucid interval. The legal presumption is always in favor of sanity; no positive evidence of
accused mental state was established. Based on expert testimonies, accused was cured of dementia
praecox and later manic depressive psychosis. Based on observance of arresting officer Damaso
Arnoco, corrobating statement of Benjamin Cruz, and other witnesses, accused appear sane at the
time immediately after commission. There is a motive of aggression on part of accused is real and
positive fact: deceased’s failure to pay borrowed money.

PEOPLE VS DUNGO
G.R. No. 89420, 31 July 1991
199 SCRA 860

FACTS
On March 16, 1987 between 2:00 and 3:00pm, the accused went to Mrs. Sigua's office at the
Department of Agrarian Reform, Apalit, Pampanga. After a brief talk, the accused drew a knife from
the envelope he was carrying and stabbed Mrs. Sigua several times. After which he departed from the
office with blood stained clothes, carrying a bloodied bladed weapon. The autopsy report revealed
that the victim sustained 14 wounds, 5 of which were fatal.

The accused, in defense of him, tried to show that he was insane at the time of the commission of the
offense. The defense first presented the testimony of Andrea Dungo, the wife of the accused.
According to her, her husband had been engaged in farming up to 1982 when he went to Lebanon for
six (6) months. Later, in December 1983, her husband again left for Saudi Arabia and worked as
welder. Her husband did not finish his two-year contract because he got sick. Upon his arrival, he
CRIMINAL LAW 1
(Sunday, Section 52) 154

underwent medical treatment. He was confined for one week at the Macabali Clinic. Thereafter he
had his monthly check-up. Because of his sickness, he was not able to resume his farming. Two weeks
prior to March 16, 1987, she noticed her husband to be in deep thought always; maltreating their
children when he was not used to it before; demanding another payment from his customers even if
the latter had paid; chasing any child when their children quarreled with other children. Thereafter,
he went back to the store. When Andrea followed him to the store, he was no longer there. She got
worried as he was not in his proper mind. She looked for him. She returned home only when she was
informed that her husband had arrived. While on her way home, she heard from people the words
"mesaksak" and "menaksak" (translated as "stabbing" and "has stabbed"). She saw her husband in
her parents-in-law's house with people milling around, including the barangay officials. She
instinctively asked her husband why he did such act, but he replied, "That is the only cure for my
ailment. I have a cancer in my heart." Her husband further said that if he would not be able to kill the
victim in a number of days, he would die, and that he chose to live longer even in jail. The testimony
on the statements of her husband was corroborated by their neighbor Thelma Santos who heard their
conversation. Turning to the barangay official, her husband exclaimed, "here is my wallet, you
surrender me." However, the barangay official did not bother to get the wallet from him. That same
day the accused went to Manila.

Dra. Sylvia Santiago and Dr. Nicanor Echavez of the National Center for Mental Health testified that
the accused was confined in the mental hospital, as per order of the trial court dated August 17, 1987,
on August 25, 1987. Based on the reports of their staff, they concluded that RosalinoDungo was
psychotic or insane long before, during and after the commission of the alleged crime and that his
insanity was classified under organic mental disorder secondary to cerebro-vascular accident or
stroke.

ISSUE
Whether or not the accused was insane during the commission of the crime charged.

RULING
No. For insanity to relieve the person of criminal liability, it is necessary that there be a complete
deprivation of intelligence in committing the act, that he acts without or the least discernment and
that there be complete absence or deprivation of the freedom of the will.

One who suffers from insanity at the time of the commission of the offense charged cannot in a legal
sense entertain a criminal intent and cannot be held criminally responsible for his acts. His unlawful
act is the product of a mental disease or a mental defect. In order that insanity may relieve a person
from criminal responsibility, it is necessary that there be a complete deprivation of intelligence in
committing the act, that is, that the accused be deprived of cognition; that he acts without the least
discernment; that there is complete absence or deprivation of the freedom of the will. (People v. Puno,
105 SCRA 151)

It is difficult to distinguish sanity from insanity. There is no definite defined border between sanity
and insanity. Under foreign jurisdiction, there are three major criteria in determining the existence
of insanity, namely: delusion test, irresistible impulse test, and the right and wrong test. Insane
delusion is manifested by a false belief for which there is no reasonable basis and which would be
incredible under the given circumstances to the same person if he is of compos mentis. Under the
delusion test, an insane person believes in a state of things, the existence of which no rational person
would believe. A person acts under an irresistible impulse when, by reason of duress or mental
disease, he has lost the power to choose between right and wrong, to avoid the act in question, his
free agency being at the time destroyed. Under the right and wrong test, a person is insane when he
CRIMINAL LAW 1
(Sunday, Section 52) 155

suffers from such perverted condition of the mental and moral faculties as to render him incapable
of distinguishing between right and wrong.

Under Philippine jurisdiction, there's no definite test or criterion for insanity. However, the definition
of insanity under Sec 1039 of the Revised Administrative Code can be applied. In essence, it states
that insanity is evinced by a deranged and perverted condition of the mental faculties, which is
manifested in language or conduct. An insane person has no full and clear understanding of the
nature and consequence of his act.
Evidence of insanity must refer to the mental condition at the very time of doing the act. However, it
is also permissible to receive evidence of his mental condition for a reasonable period before and
after the time of the act in question. The vagaries of the mind can only be known by outward acts.

It is not usual for an insane person to confront a specified person who may have wronged him. But in
the case at hand, the accused was able to Mrs. Sigua. From this, it can be inferred that the accused
was aware of his acts. This also established that the accused has lucid intervals. Moreover, Dr.
Echavez testified to the effect that the appellant could have been aware of the nature of his act at the
time he committed it when he shouted (during laboratory examination) that he killed Mrs. Sigua. This
statement makes it highly doubtful that the accused was insane when he committed the act.

The fact that the accused was carrying an envelope where he hid the fatal weapon, that he ran away
from the scene of the incident after he stabbed the victim several times, that he fled to Manila to evade
arrest, indicate that he was conscious and knew the consequences of his acts in stabbing the victim.

PEOPLE VS RAFANAN
L-54135, 21 November 1991
204 SCRA 65

FACTS:
Complainant EstelitaRonaya was only 14 years old when hired as a househelper by the mother of the
accused. The accused PolicarpioRafaran and his family lived with his mother in the same house.
Policarpio was married and has children. One evening, the mother of the accused called complainant
to help him close the door. When the complainant went near him, he pulled her inside the store and
raped her despite her resistance. After that, he warned the complainant not to tell anyone about it or
he will kill her. The next day, the family of the accused knew what happened. Appellant claimed that
he is suffering from schizophrenia when he inflicted violent intentions to Estelita. Trial court
suspended the tria; and ordered his confinement to National Mental Hospital in Mandaluyong. After
2 years, he was reported to be behaved and in improved condition and in mental condition to stand
court in trial. Trial of case resumed.

ISSUE:
W/N the reason of insanity is sufficient to relieve him from criminal liability

HELD:
No. The allegation of insanity or imbecility must be clearly proved. Without positive evidence that
the defendant had previously lost his reason or was demented, a few moments prior to or during the
perpetration of the crime, it will be presumed that he was in a normal condition.
CRIMINAL LAW 1
(Sunday, Section 52) 156

PEOPLE VS MADARANG
G.R. No. 132319, 12 May 2000
332 SCRA 99

FACTS:
Appellant was convicted of parricide for stabbing his wife, causing her death. Appellant alleges he
was in a state of insanity and claims he had no recollection of the stabbing incident. He insists that he
was deprived of intelligence, making his act involuntary. His psychiatric evaluation revealed he was
suffering from schizophrenia but after two years in the National Center for Mental Health his
condition improved thus, he was released.

ISSUE:
Whether or not evidence adduced by the defense is sufficient to establish appellant’
sinsanity, which would be a basis for being free from criminal liability.

HELD:
In the Philippines, the courts have established a more stringent criterion for insanity to be exempting
as it is required that there must be a complete deprivation of intelligence in committing the act,i.e.,
the accused is deprived of reason; he acted without the least discernment because there is a complete
absence of the power to discern, or that there is total deprivation of the will. Mere abnormality of the
mental faculties will not exclude imputability.The issue of insanity is a question of fact. The state or
condition of a man's mind can only be measured and judged by his behavior. Establishing one's
insanity requires testimony of an expert witness, such as a psychiatrist. The proof must relate to the
time preceding or coetaneous with the commission of the offense with which he is charged. None of
the witnesses declared that he exhibited any of the symptoms associated with schizophrenia
immediately before or simultaneous with the stabbing incident. Also schizophrenics have lucid
intervals during which they are capable of distinguishing right from wrong.

PEOPLE VS ROBIOS
G.R. No. 138453, 29 May 2002
382 SCRA 581

FACTS:
Melencio Robios was found guilty with the complex crime of parricide with unintentional abortion
and was sentenced of death. May 31, 1995, he was accused of killing his pregnant wife. March 25,
1995 Lorenzo Robios, son of Melencio heard his parents quarreling and saw Melencio stab her mom
Lorenza with an 8-inch double bladed knife on the right shoulder. On the same day, Benjamin, brother
of Lorenza reported that Melencio has also killed their uncle. Benjamin knowing what Melencio did
to her sister, went to her sister’s house and when he was 150m away, saw Melencio and the latter
shouted “Its good you would see how your sister would die.” Benjamin sought the help of the police.

SPO1 saw Melencio embracing her wife uttering the words “I will kill myself, I will kill myself”.
Lorenza, who was lying on her back and facing upward, was no longer breathing. She appeared to be
dead. Appellant dropped the knife which was taken by SPO3 Martin. Appellant tried to resist the
people who held him but was overpowered. The police, with the help of the barangay officials
present, tied his hands and feet with a plastic rope. However, before he was pulled away from the
body of his wife and restrained by the police, appellant admitted to Rolando Valdez, a neighbor of his
and a barangay kagawad, that he had killed his wife, showing him the bloodstained knife.
CRIMINAL LAW 1
(Sunday, Section 52) 157

Special report showed that Lorenza Robios was six (6) months pregnant. She suffered 41 stab
wounds on the different parts of her body and that the appellant was under the influence of alcohol
and also stabbed himself.

Melencio admitted that she killed his wife but wish to be exempted of his criminal liability invoking
insanity. His son testified that Melencio saw someone in their house that wanted to kill him. A nurse
said that Melencio “isolated himself, laging nakatingin sa malayo, rarely talked, just stared at her and
murmured alone”. A detention prisoner witnessed the appellant usually refusing to respond in the
counting of prisoners. Sometimes, he stayed in his cell even if they were required to fall in line in the
plaza of the penal colony. And another prisoner said that accused sometimes was lying down, sitting,
looking, or staring on space and without companion, laughing and sometimes crying. And Melencio
said that he did not know that he was charged for the crime of parricide with unintentional abortion.
He could not remember when he was informed by his children that he killed his wife. He could not
believe that he killed his wife

ISSUE:
Can he be exempted on the grounds of insanity?
What is the proper penalty for him?

HELD:
Testimonies from both prosecution and defense witnesses show no substantial evidence that
appellant was completely deprived of reason or discernment when he perpetrated the brutal killing
of his wife. The fact that appellant admitted to responding law enforcers how he had just killed his
wife may have been a manifestation of repentance and remorse -- a natural sentiment of a husband
who had realized the wrongfulness of his act. His behavior at the time of the killing and immediately
thereafter is inconsistent with his claim that he had no knowledge of what he had just done and he
was not insane during the commission of the crime.

Since appellant was convicted of the complex crime of parricide with unintentional abortion, the
penalty to be imposed on him should be that for the graver offense which is parricide and punishable
with reclusion perpetua to death.

PEOPLE V OPURAN
G.R. Nos. 147674-75. March 17, 2004

Facts:
Anacito Opuran was charged with two counts of murder before the Regional Trial Court of
Catbalogan, Samar, Branch 29, for the death of Demetrio Patrimonio, Jr., and Allan Dacles under
separate informations, the accusatory portions of which respectively read:
That on or about November 19, 1998, at nighttime, at Km. 1, South Road, Municipality of Catbalogan,
Province of Samar, Philippines, and within the jurisdiction of this Honorable Court, said accused, with
deliberate intent to kill and treachery, did, then and there willfully, unlawfully, and feloniously attack,
assault and stab Demetrio Patrimonio, Jr., with the use of a bladed weapon (5 long from tip to handle
with scabbard), thereby inflicting upon the victim fatal stab wounds on the back of his body, which
wounds resulted to his instantaneous death.All contrary to law, and with attendant qualifying
circumstance of treachery.

Criminal Case No. 4703


That on or about November 19, 1998, at nighttime, at Purok 3, Barangay 7, Municipality of
Catbalogan, Province of Samar, Philippines, and within the jurisdiction of this Honorable Court, said
CRIMINAL LAW 1
(Sunday, Section 52) 158

accused, with deliberate intent to kill, with treachery, did, then and there, willfully, unlawfully and
feloniously attack, assault and stab one Allan Dacles, who was lying on the bench, with the use of a
bladed weapon, locally known as pisao, thereby inflicting upon the victim fatal stab wounds on the
different parts of his body, which wounds resulted to his instantaneous death. All contrary to law,
and with attendant qualifying circumstance of treachery.

After Anacito entered a plea of not guilty at his arraignment, trial ensued. For its part, the defense
presented, as its first witness, the appellant himself, Anacito Opuran. He declared that on the evening
of 19 November 1998, he was resting in their house in Canlapwas, another barangay in Catbalogan,
Samar. He never went out that night. While he was sleeping at about 8:30 p.m., eight policemen
entered his house, pointed their guns at him, and arrested him. He was brought to the police station
and detained there until the following morning. He denied being present at the place and time of the
stabbing incidents. He admitted knowing Demetrio Jr. as a distant relative and friend whom he had
not quarreled with. As for Allan, he never knew him. He had no misunderstanding with prosecution
witness Bambi Herrera. He asserted that the accusations against him were fabricated because he was
envied and lowly regarded by his accusers.

The hearings were suspended upon the trial courts’s order authorizing the psychiatric examination
of defendant. He was issued a Medical Certificate stating a normal mental status but was suffering
from some degree of mental aberration which required further psychiatric evaluation.The trial court
thus ordered a deferment of the hearing and granted the motion for the psychiatric examination of
Anacito at the Eastern Visayas Regional Medical Center (EVRMC), Tacloban City. On 3 August 2000,
the trial court received the Medical Report of Dr. Lyn Verona, physician-psychiatrist of the EVRMC,
on the psychiatric examination she conducted on Anacito. Her diagnosis was that Anacito was
suffering from schizophrenia. In its decision on 23 January 2001, the trial court found Anacito guilty
of murder for the death of Demetrio Patrimonio, Jr., and homicide for the death of Allan Dacles.

Issue:
Whether or not accused can use the exempting circumstance of insanity as a defense

Held:
The Supreme Court affirmed the decision of the OSG and the trial court. In its decision, the court
stated: He who pleads the exempting circumstance of insanity bears the burden of proving it, for
insanity as a defense is in the nature of confession and avoidance. An accused invoking insanity
admits to have committed the crime but claims that he is not guilty because he is insane. The
testimony or proof of an accused's insanity must, however, relate to the time immediately preceding
or coetaneous with the commission of the offense with which he is charged. [ It is, therefore,
incumbent upon accuseds counsel to prove that his client was not in his right mind or was under the
influence of a sudden attack of insanity immediately before or at the time h executed the act
attributed to him. A careful scrutiny of the records, indicates that Anacito failed to prove by clear and
convincing evidence the defense of insanity. Tested against the stringent criterion for insanity to be
exempting, such deportment of Anacito, his occasional silence, and his acts of laughing, talking to
himself, staring sharply, and stabbing his victims within a 15-minute interval are not sufficient
proof that he was insane immediately before or at the time he committed the crimes. Such unusual
behavior may be considered as mere abnormality of the mental faculties, which will not exclude
imputability. Anacitos psychiatric history likewise fails to meet the stringent yardstick established
by case law. What it shows is that Anacito was prescribed thorazine and evadyne, and later an
injectable medicine to remedy his lack of sleep and noisiness. As the trial court noted, it was never
shown that these drugs were for a mental illness that deprived Anacito of reason. Further, Anacito
was just an out-patient at the NCMH, EVRMC, and Samar Provincial Hospital. While Remedios
CRIMINAL LAW 1
(Sunday, Section 52) 159

claimed that she requested the confinement of Anacito and that the doctors did not refuse her, the
fact remains that Anacito was never confined in a mental institution. Moreover, as found by the trial
court, the results of Dr. Veronas examinations on Anacito were based on incomplete or insufficient
facts.

It must be stressed that an inquiry into the mental state of an accused should relate to the period
immediately before or at the precise moment of the commission of the act which is the subject of the
inquiry. His mental condition after that crucial period or during the trial is inconsequential for
purposes of determining his criminal liability. We likewise reject the alternative plea of Anacito that
he be credited with the mitigating circumstance of diminished willpower. In the cases where we
credited this mitigating circumstance after rejecting a plea of insanity, it was clear from the records
that the accused had been suffering from a chronic mental disease that affected his intelligence and
willpower for quite a number of years prior to the commission of the act he was being held for. The
situation does not exist in the cases at bar. It was only in 2000 that Anacito was diagnosed as
psychotic with flight of ideas and auditory hallucinations and was found to be schizophrenic. We
agree with the trial court that treachery cannot be appreciated as far as the killing of Allan is
concerned because the sole eyewitness did not see the commencement of the assault. For treachery
to be considered, it must be present and seen by the witness right at the inception of the
attack. Where no particulars are known as to how the killing began, the perpetration with treachery
cannot be supposed. We do not find merit in appellants contention that he cannot be convicted of
murder for the death of Demetrio Jr. because treachery was not alleged with specificity as a qualifying
circumstance in the information. Such contention is belied by the information itself, which alleged:
All contrary to law, and with the attendant qualifying circumstance of treachery. In any event, even
after the recent amendments to the Rules of Criminal Procedure, qualifying circumstances need not
be preceded by descriptive words such as qualifying or qualified by to properly qualify an offense.

SOLOMON VERDADERO Y GALERA V POPLE

Facts:
On March 12, 2009, at the Bagao Police Station, a complaint was reported by Maynard Plata, together
with his father Romeo and his companion Ronnie Elaydo, against Solomon Verdadero. The complaint
was about Verdadero for stealing a fan belt of their irrigation pump. After a confrontation with
Verdadero, the three men made their way home but took a stop at a drugstore because Maynard
intended to buy some supplies there. Romeo proceeded to the drugstore, and after awhile Maynard
and Ronnie saw Romeo being stabbed by Verdadero. At that moment, Maynard tried to help his father
but Verdadero tried to attack him. As a response, Maynard tried to defend himself with as tool, which
he used to hit Verdadero in the chest. Meanwhile, Ronnie ran toward the police station to seek for
assistance. After arriving at the scene, the responding police officer arrested Verdadero while
Maynard and Ronnie rush Romeo to a clinic. After arriving at the clinic, they were advised to transfer
him to Cagayan Valley Medical Center (CVMC), where he soon died of cardiopulmonary arrest after
arriving there. During the trial, Verdadero pleaded guilty and invoke his defense of insanity. He
admitted that he was not in the proper state of mind during that faithful night. As early as 1999, he
was brought to the Psychiatric Department of CVMC for treatment. Then he was diagnosed of
depression and schizophrenia on the subsequent years. He also suffered a relapse in 2009, and
became an in and out patient from his confinement in 2009 until the stabbing events.

Issue:
Whether or not the exempting circumstance under Article 12- That an imbecile or insane person,
unless the latter has acted during a lucid interval can be appreciated in this case
CRIMINAL LAW 1
(Sunday, Section 52) 160

Held:
Yes, it can be appreciated in this case. According to Revised Penal Code Article 12 Paragraph 1- An
imbecile or insane person, unless the latter has acted during a lucid interval is exempted from
criminal liability. It must be pointed out that in order for this circumstance to be exempted from
criminal liability; there must be a complete deprivation of intelligence or that there be total
deprivation of the freedom of will at the commission of the felony. Thus, mere abnormality of mental
faculties is not enough, especially if the offender has not lost consciousness of his act. In the case
under consideration, Verdadero is already diagnosed of schizophrenia, a chronic mental disorder
characterized by inability to distinguish between fantasy and reality and often accompanied by
hallucination and delusion. Worth emphasizing that he already suffered from this mental disorder
before the commission of the felony, and suffered a relapse during that year. Thus he is afforded of
reservation if he did it in a lucid interval during the commission of the felony. From the facts given, it
can be said that this exempting circumstance can be appreciated in this case.

PEOPLE V GENOSA
G. R. No. 135981

Facts:
Marivic Genosa, the appellant, on November 15, 1995, attacked and wounded his husband which
ultimately led to his death. According to the appellant, she did not provoke her husband when she
got home that night and it was her husband who began the provocation. The appellant said she was
frightened that her husband would hurt her and she wanted to make sure she would deliver her baby
safely.

The appellant testified that during her marriage she had tried to leave her husband at least five times,
but that Ben would always follow her and they would reconcile. The appellant said that the reason
why Ben was violent and abusive towards her that night was because he was crazy about his recent
girlfriend, Lulu Rubillos. The appellant, after being interviewed by specialist, has been shown to be
suffering from Battered Woman Syndrome. The appellant with a plea of self-defense admitted the
killing of her husband. She was found guilty of the crime of parricide, with the aggravating
circumstance of treachery, for the husband was attacked while asleep.

Issues:
(1) Whether or not appellant acted in self-defense.
(2) Whether or not treachery attended the killing.

Held:
For the first issue, the SC held that the defense failed to establish all the elements of self-defense
arising from battered woman syndrome, to wit: (a) Each of the phases of the cycle of violence must
be proven to have characterized at least two battering episodes between the appellant and her
intimated partner; (b) The final acute battering episode preceding the killing of the batterer must
have produced in the battered person’s mind an actual fear of an imminent harm from her batterer
and an honest belief that she needed to use force in order to save her life, and; (c) At the time of the
killing, the batterer must have posed probable – not necessarily immediate and actual – grave harm
to the accused based on the history of violence perpetuated by the former against the latter.
For the second issue, the SC ruled out treachery as an aggravating circumstance because the quarrel
or argument that preceded the killing must have forewarned the victim of the assailant’s aggression.
CRIMINAL LAW 1
(Sunday, Section 52) 161

PEOPLE V DOQUEÑA

Facts:
The accused-appellant, who is a minor, was prosecuted for homicide in the Court of First Instance of
Pangasinan, for having killed Juan Ragojos by stabbing him in the breast with a knife on November
19, 1938, in the municipality of Sual, Pangasinan. The court, after trying the case, held that the
accused acted with discernment in committing the act imputed to him and, proceeding in accordance
with the provisions of article 80 of the Revised Penal Code, as amended by Commonwealth Act No.
99, ordered him to be sent to the Training School for Boys to remain therein until he reaches the age
of majority. From this order the accused interposed an appeal alleging that the court erred in holding
that he had acted with discernment and in not having dismissal the case.
On the date of the crime, the appellant was exactly thirteen years, nine months and five days old. The
incident that gave rise to the aggression committed by him on the deceased is narrated in the
appealed order as follows:
Between 1 and 2 o'clock in the afternoon of November 19, 1938, the now deceased Juan Ragojos and
one Epifanio Rarang were playing volleyball in the yard of the intermediate school of the municipality
of Sual, Province of Pangasinan. The herein accused, who was also in said yard, intervened and,
catching the ball, tossed it at Juan Ragojos, hitting him on the stomach. For this act of the accused,
Juan Ragojos chased him around the yard and, upon overtaking him, slapped him on the nape. Said
accused then turned against the deceased assuming a threatening attitude, for which the reason said
deceased struck him on the mouth with his fist, returning immediately to the place where Epifanio
Rarang was in order to continue playing with him. The accused, offended by what he considered an
abuse on the part of Juan Ragojos, who was taller and more robust than he, looked around the yard
for a stone with which to attack the now deceased Juan Ragojos, but finding none, he approached a
cousin of his named Romualdo Cocal, to ask the latter to lend him his knife. Epifanio Rarang, who had
heard what the accused had been asking his cousin, told the latter not to give the accused his knife
because he might attack Juan Ragojos with it. The accused, however, succeeded in taking possession
of the knife which was in a pocket of his cousin's pants. Once in possession of the knife, Valentin
Doqueña approached Juan Ragojos and challenged the latter to give him another blow with his fist,
to which the deceased answered that he did not want to do so because he (Juan Ragojos) was bigger
that the accused. Juan Ragojos, ignorant of the intentions of the accused, continued playing and, while
he was thus unprepared and in the act of stopping the ball with his two hands, the accused stabbed
him in the chest with the knife which he carried.

The order also contains the following conclusions and findings of fact which we are not at liberty to
alter, not being called upon or authorized to do so, in view of the nature of the appeal before us, by
section 138 of the Administrative Code, as amended by Commonwealth Act No. 3:
Taking into account the fact that when the accused Valentin Doqueña committed the crime in
question, he was a 7th grade pupil in the intermediate school of the municipality of Sual, Pangasinan,
and as such pupil, he was one of the brightest in said school and was a captain of a company of the
cadet corps thereof, and during the time he was studying therein he always obtained excellent marks,
this court is convinced that the accused, in committing the crime, acted with discernment and was
conscious of the nature and consequences of his act, and so also has this court observed at the time
said accused was testifying in his behalf during the trial of this case.

Issue:
Did the accused act with discernment when he committed the crime?
CRIMINAL LAW 1
(Sunday, Section 52) 162

Held:
Yes. The discernment that constitutes an exception to the exemption from criminal liability of a minor
under fifteen years of age but over nine, who commits an act prohibited by law, is his mental capacity
to understand the difference between right and wrong, and such capacity may be known and should
be determined by taking into consideration all the facts and circumstances afforded by the records
in each case, the very appearance, the very attitude, the very comportment and behaviour of said
minor, not only before and during the commission of the act, but also after and even during the trial.

JOEMAR ORTEGA V PEOPLE


G. R. No. 151085, August 20, 2008

Facts:
At the time of commission of rape, the accused was 13 years old while the victim was 6. The case was
pending when the Juvenile Justice and Welfare Act of 2006 (R.A. 9344) was enacted amending among
others the age of criminal irresponsibility being raised from 9 to 15 years old. At the time of the
promulgation of judgment, the accused already reached the age of majority.

Issue:
Whether or not the Juvenile Justice and Welfare Act of 2006 (R.A. 9344) should be applied, in the
resolution of the case.

Held:
The Juvenile Justice and Welfare Act of 2006 (R.A. 9344) should be applied. By virtue of R.A. No.
9344, the age of criminal irresponsibility has been raised from 9 to 15 years old, this law is evidently
favorable to the accused. Petitioner was only 13 years old at the time of the commission of the alleged
rape. This was duly proven by the certificate of live birth, by petitioner's own testimony, and by the
testimony of his mother. Furthermore, petitioner’s age was never assailed in any of the proceedings
before the RTC and the CA. Indubitably, petitioner, at the time of the commission of the crime, was
below 15 years of age. Under R.A. No. 9344, he is exempted from criminal liability.

PEOPLE OF THE PHILIPPINES VS ALLEN UDTOJAN MANTALABA


G.R. No. 186227, July 20, 2011

Facts:
Task Force Regional Anti-Crime Emergency Response (RACER) in Butuan City received a report that
Mantalaba who was 17 yrs old was selling shabu. After a buy-bust operation, two informations was
filed against Mantalaba which was later on consolidated. Mantalaba pleaded not guilty. RTC found
Mantalaba guilty beyond reasonable doubt and was penalized of reclusion perpetua to death and fine
of 500k for selling shabu and (2) for illegally possessing shabu, Mantalaba was penalized, in
application of the ISL, 6 yrs and 1 day as minimum and 8 yrs as maximum of prision mayor and fine
of 300k. CA affirmed in toto the decision of the RTC. Thus, the present appeal. Mantalaba: the lower
court gravely erred in convicting him and that there was no evidence of actual sale between him and
the poser-buyer during the buy-bust operation. He also claims that the chain of custody of the seized
shabu was not established.

Issue:
Whether Mantalaba is guilty of drug trafficking and possession.
CRIMINAL LAW 1
(Sunday, Section 52) 163

Held:
The petition is without merit. The buy-bust operation was valid, establishing the following: (1) the
identity of the buyer and the seller, the object, and the consideration; and (2) the delivery of the thing
sold and the payment therefore. From the above testimony of the prosecution witness, it was well
established that the elements have been satisfactorily met. The seller and the poseur-buyer were
properly identified. The subject dangerous drug, as well as the marked money used, was also
satisfactorily presented. The testimony was also clear as to the manner in which the buy-bust
operation was conducted. Non-compliance by the apprehending/buy-bust team with Section 21 is
not fatal as long as there is justifiable ground therefor, and as long as the integrity and the evidentiary
value of the confiscated/seized items are properly preserved by the apprehending officer/team. Its
non-compliance will not render an accused arrest illegal or the items seized/confiscated from him
inadmissible. As to his minority, Mantalaba was minor during the buy-bust operation but was of legal
age during the promulgation of the decision. It must be noted that RA 9344 took effect after the
promulgation of the RTC's decision against Mantalaba. The RTC did not suspend the sentence in
accordance with PD 603 (Child and Youth Welfare Code) and Rule on Juveniles in Conflict with the
Law that were applicable at the time of the promulgation of the judgment. However, as ruled in
People vs Sarcia, suspension of sentence can still be applied but NOT when the offender upon the
promulgation of judgment is 21 yrs old. or older. Mantalaba is now 21 yrs old, therefore his
suspension of sentence is already moot and academic. But as to the penalty, CA must have
appreciated Mantalaba's minority as privileged mitigating circumstance in fixing the penalty. Thus,
applying the rules stated above, the proper penalty should be one degree lower than reclusion
perpetua, which is reclusion temporal, the privileged mitigating circumstance of minority having
been appreciated. Necessarily, also applying the Indeterminate Sentence Law (ISLAW), the minimum
penalty should be taken from the penalty next lower in degree which is prision mayor and the
maximum penalty shall be taken from the medium period of reclusion temporal, there being no other
mitigating circumstance nor aggravating circumstance.

US V TANEDO
L-5481, 12 February 1940, 15 Phil 196

Facts:
Petenia, Castaneda and Lugon conspired to kill and rob Mrs. Guanlao of her valuables. Petenia
covered the mouth of Guanlao. Lugon hit her with adove stone. Petenia delivered blows. As the victim
fell to the ground, Lugon and Petenia hit her again twice and caused her death. Petenia and Castaneda
were arrested for robbery with homicide.

Issues:
1. Whether or not the lower court erred in not rejecting their extrajudicial confessions.
2. Whether or not the Court erred in finding the crime was committed in conspiracy.
3. Whether or not the lower court erred in not acquitting them cause of exempting circumstance
(uncontrollable fear)

Held:
1. No. Burden is on defense to prove that a confession was obtained as a result of violence,
intimidation, threat or promise of reward or leniency. No evidence to justify that the extrajudicial
confessions were by force.
2. No, Circumstances under the committed crime show a single pupose and in unison with each act
of the accused to the attainment of the purpose.
3. No. requisites must be present. (Existence of uncontrollable fear, fear must be real and imminent,
fear of injury is greater than or equal to the committed act) Petenia contended that he delivered fistic
CRIMINAL LAW 1
(Sunday, Section 52) 164

blows because he was afraid that Lugon and Castaneda will hit him with hollow blocks. Castaneda
claimed that he ransacked bthe bedroom of Guanlao because Lugon and Petania poked a knife at him.
The court said Castaneda could have easily escaped when he was already at the house of the victim.
The guilt of the accused has been proven beyond reasonable doubt.

PEOPLE OF THE PHILIPPINES VS. ISAIAS CASTILLO Y COMPLETO


G.R. No. 172695 29 June 2007

Facts:
In the evening of 5 November 1993, the accused-appellant came home drunk and angry. His father-
in-law (Guillermo) tried to subdue him but to no avail which caused the former to leave the house.
As he was leaving, Guillermo saw him take out his sling and arrow. Consorcia, the accused’s wife, was
heard crying and screaming. Thereafter, the accused-appellant was seen carrying the bloodied body
of Consorcia out of the house and was later taken to the hospital but to no avail. Cause of death was
the cut jugular vein caused by a fatal weapon which could have been a “pointed instrument like a
nail.” Appellant was charged with parricide for allegedly shooting his wife with a dart from a rubber
sling, hitting her at the neck and causing her instantaneous death. In his defense, the accused said
that he had no intention of killing his wife and that he was practicing the use of the weapon when his
wife was accidentally hit by the arrow. However, the trial court nonetheless found him guilty on the
ground that the evidence showed that the infliction of the fatal injury upon his wife was preceded by
a quarrel between her and the appellant, thus negating the latter’s defense. The same was affirmed
on appeal. In the present petition, the appellant contends that assuming that he was the one who
killed his wife the same was accidental and not intentional.

Issue:
Whether or not the exempting circumstance of accident applicable in the instant case?

Held:
No. Article 12, par. 4 of the Revised Penal Code, provides:

ART. 12. Circumstances which exempt from criminal liability. The following are exempt from criminal
liability: Any person who, while performing a lawful act with due care, causes an injury by mere
accident without fault or intention of causing it.

"Accident" is an affirmative defense which the accused is burdened to prove, with clear and
convincing evidence. The defense miserably failed to discharge its burden of proof. The essential
requisites for this exempting circumstance, are:

i. A person is performing a lawful act;


ii. With due care;
iii. He causes an injury to another by mere accident;
iv. Without fault or intention of causing it.

By no stretch of imagination could playing with or using a deadly sling and arrow be considered as
performing a "lawful act." Thus, on this ground alone, appellant's defense of accident must be struck
down because he was performing an unlawful act during the incident. Also, the fact that the accused-
appellant disappeared while his wife was in the hospital is unbecoming of a husband with a dying
wife. Accused was found guilty of the crime of Parricide wherein the court imposed the penalty of
reclusion perpetua.
CRIMINAL LAW 1
(Sunday, Section 52) 165

PEOPLE VS RETUBADO
GR NO. 124058, December 10, 2003
CRIMINAL LAW 1
(Sunday, Section 52) 166

POMOY v. PEOPLE
G.R. NO. 150647 SEPTEMBER 29, 2004 439 SCRA 439

Facts:
1990- Policemen arrested Tomas Balboa, a master teacher of the Concepcion College of Science
andFisheries in Concepcion, Iloilo, for he was allegedly connected with a robbery. He was brought to
theHeadquarters of the Philippine Constabulary Company at Camp Jalandoni in Iloilo where he was
detained.Roweno Pomoy, a member of the Iloilo Provincial Mobile Force Company, directed the latter
to come out, fortactical interrogation at the investigation room. Petitioner had a gun hanging from
his bolster. After that, 2gunshots were heard. Petitioner was seen holding his .45 caliber pistol facing
Balboa who was lying in a poolof blood. Balboa died.--Version of the Defense: (presented 2 witnesses
+ accused Pomoy)Self-defense: Balboa allegedly tried to grab the handle of Pomoy’s gun. Balboa was
not able to take actual holdof the gun because of his efforts in preventing him. He and Balboa grappled
in taking control of his gun.Balboa was accidentally shot.--RTC of Iloilo City found Pomoy guilty of the
crime of homicide.--CA modified the RTC decision (removed aggravating circumstance of abuse of
public position) stating that1) the victim was not successful in his attempts to grab the gun, since
petitioner had been in control of theweapon when the shots were fired; 2) the gun had been locked
prior to the alleged grabbing incident andimmediately before it went off; it was petitioner who
released the safety lock before he deliberately fired thefatal shots; and 3) the location of the wounds
found on the body of the deceased did not support the assertionof petitioner that there had been a
CRIMINAL LAW 1
(Sunday, Section 52) 167

grappling for the gun. Thus, there is no unlawful aggression on the part ofthe deceased to justify self-
defense.

Issue:
WON the shooting of Tomas Balboa was the result of an accident in his fulfillment of duty.

Held:
YES. It was in the lawful performance of his duty as a law enforcer that petitioner tried to defend his
possession of the weaponwhen the victim suddenly tried to remove it from his holster. Petition is
granted and the assailed decision REVERSED. Petitioner is ACQUITTED.

Article 12. Circumstances which exempt from criminal liability.

The following are exempt from criminal liability:4. Any person who, while performing a lawful act
with due care, causes an injury by mere accidentwithout fault or intent of causing it.Exemption from
criminal liability proceeds from a finding that the harm to the victim was not due to the faultor
negligence of the accused, but to circumstances that could not have been foreseen or controlled. Thus,
in determining whether an “accident” attended the incident, courts must take into account the dual
standards of lack of intent to kill and absence of fault or negligence. This determination inevitably
brings to the fore themain question in the present case: was petitioner in control of the .45 caliber
pistol at the very moment theshots were fired? YES. that petitioner did not have control of the gun
during the scuffle. The deceasedpersistently attempted to wrest the weapon from him, while he
resolutely tried to thwart those attempts.Elements of Accident:1) the accused was at the time
performing a lawful act with due care;2) the resulting injury was caused by mere accident; and3) on
the part of the accused, there was no fault or no intent to cause the injury. All these elements were
present in this case.

US V. CABALLEROS
G.R. NO. 1352, MARCH 29, 1905 4 PHIL 350

Facts:
Robert Baculi and Apolonio Caballeros were convicted as accessories to the crime of assassination or
murder of four American school-teachers, having buried the corpses of the victims to conceal the
crime. They were allegedly coerced.

Issue:
WON the defense under Art12(5) is tenable

Held:
Yes. Not only is Baculi’s confession that he only assisted in the burial of the corpses because he was
compelled by the murderers, but this was corroborated by the only eyewitness to the crime, Sabate.
Sabate said that he was present when the Americans were killed; that Baculi was not a member of
the group of murderers but he was in the banana plantation gathering some bananas; that when he
heard the shots he began to run; that he was, however, seen by Damaso and Isidro, the leaders of the
band; that the latter called to him and striking him with the butts of their guns forced him to bury the
corpses. As for Caballeros, there was no proof that he took any part in the execution of the crime;
there was conclusive proof to the contrary. Sabate and Baculi declared that Caballeros did not take
any part in the burial of the aforesaid corpses, nor was he even in the place of the occurrence when
the burial took place. Their failure to report the crime is not an offense punished by the Penal Code.
CRIMINAL LAW 1
(Sunday, Section 52) 168

PEOPLE V. LORENO
L-54414, JULY 9, 1984 130 SCRA 311
CRIMINAL LAW 1
(Sunday, Section 52) 169

PEOPLE VS. JOSELITO DEL ROSARIO


GR 127755, April 14, 1999 305 SCRA 740
CRIMINAL LAW 1
(Sunday, Section 52) 170

PEOPLE VS. BANDIAN


G.R. NO. 45186 SEPTEMBER 30, 1936 63 PHIL 530

FACTS:
At About 7 in the morning of January 31, 1936, Valentine Aguilar, the apellant’s neighbor, saw the
appelant go to the thicket about four or five brazas from her house, apparently to respond to a call of
nature because it was there that the people of the place used to go for that purpose. A few minutes
later, he then again saw her emerge from the thicket with her clothes stained with blood both in front
and back, stagerring and visibly showing signs of not being able to support herself. He ran to her aid
and having noted that she was very weak and dizzy, he supported and helped her go up to her house
and placed her in her bed.

Upon being asked before Aguilar brought her to her house, what had happened to her, the appellant
answered that she was very dizzy. Not wishing to be alone with the appellant in such circumstances,
Aguilar called Adriano Comcom who lived nearby to be there and help him and the appellant. He
asked Comcom to take bamboo leaves to stop the hemhorrage which had come upon the appellant.
Comcom had scarcely gone about five brazas when he saw the body of newborn baby near a path
adjoining the thicket where the appellant had gone a few moments before. Comcom informed Aguilar
of it and the latter told him to bring the body to the appellant’s house. Upon being askes whether the
babywhich was shown to her was hers or not, the appellant answered in the affirmative.

In the afternoon of the said day, Dr. Emilio Nepomuceno, president of the sanitary division went to
the appellant’s house and found her still lying in bed still bleeding. In his opinion, the physician
declared that the appellant gave birth in her house, and afterwhich, he threw the child into the thicket
to kill it for the purpose of concealing her dishonor from the man, Luis Kirol, with whom she was
married to, because the child was not his but with another man with whom she had previously has
amorous relations. Nepomuceno testified that the appellant admitted killing her child.

ISSUE:
WON Bandian is guilty of infanticide.

HELD:
No, Infanticide and abandonment of a minor, to be punishable must be committed willfully and
consciously, or at least it must be the result of a voluntary, conscious and free act or omission. Even
in cases where said crimes are committed through mere imprudence, the person who commits them,
under said circumstances, must be in the full enjoyment of his mental facilities, or must be conscious
of his acts, in order that he may be held liable.

The evidence certainly does not show that the appellant , in causing her child’s death in one way or
another, or in abandoning it in the thicket, did so willfully, consciously, or imprudently. She had no
cause to kill or abandon it, to expose it to death , because her affair with a former lover, Luis Kirol
took place three years before the incident. The husband of the appellant testified at the trial affirming
the belief that the child was his.

The law exempts from criminal liability any person who acts under the circumstances in which the
appellant acted in this case, by giving birth to a child in the thicket and later abandoning it, not
because of imprudence or any other cause than that she was overcome by severe dizziness and
extreme debility, with no fault or intention on her part. She has in her favor the fourth and the seventh
exempting circumstances.
CRIMINAL LAW 1
(Sunday, Section 52) 171

In conclusion, taking into account the foregoing facts and considerations, and granting that the
appellant was aware of her involuntary childbirth in the thicket and that she later failed to take her
child therefrom, having been so prevented by reason of causes entirely independent of her will, it
should be held that under such circumstances, appellant has the fourth and seventh exempting
circumstances of article 12 of the Revised Penal Code in her favor.

PEOPLE VS LUA CHU


G.R. No. 34917, September 7
1931 56 PHIL 44

Facts:
Lua Chu and Uy Se Tieng were convicted of the illegal importation of opium.On November, 1929,
Tieng wrote to his correspondent in Hongkong to send him a shipment of opium.Tieng went to Juan
Samson's house and told him that the opium shipment consisted of 3,000 tins for P2 atin, and that
opium is in the vessel Kolambugan, awaiting shipment direct to Cebu. When the vesselarrived, Tieng
was told that he must pay over the Php6000 before the opium be taken out.The next day, Samson
informed of what had taken place to Colonel Francisco, who then instructed theCaptain Buenconsejo,
to discuss the capture of the opium owners with Samson. Samson also went to theoffice of the
provincial fiscal, reported the same, and asked for a stenographer to note his conversationwith Tieng
that night and in the presence of Captain Buenconsejo.On December 17, 1929, Buenconsejo,
Fernando, and the stenographer went to Samson's house andconcealed themselves behind a curtain
made of strips of wood.Samson asked Tieng where the opium was, and the latter answered that it
was in the cases numbered 11 to18 – a total of 3,252 tins. Tieng returned later that night with Lua
Chu, who said he was not the soleowner of the opium.Samson then interrogated Chu on when the
former was going to get the opium, on whether Chu
had brought the money, on how he had come to bring in the opium; Chu would answer them, while
Buenconsejo listened in. As Tieng was handing certain papers over to his companion, Uy Ay;
Buenconsejo, who had been hiding, appeared and arrested the two men. After the two to the
Constabularyheadquarters, and notified the fiscal, Buenconsejo and Samson went to Chu's home to
search it and arresthim, and took him to the Constabulary headquarters, and then went to the
customhouse to examine thecases marked.Contention of the accused: The accused principal defense
was that they were induced by Samson toimport the opium in questionContention of the state: The
state contends that the defendants do not deny their participation in the act inquestion. Samson
denied his connection with the offense for purposes of gain; further contending that hesmoothed the
way for the introduction of the prohibited drug, but he did not do so to help them carry their plan to
a successful issue, rather to assure the seizure of the imported drug and the arrest of the smugglers.

Issue:
Whether or not the Samson instigated the accused to import opium

Held:
Samson neither induced nor instigated the herein accused to import the opium in question,
but pretended to have an understanding with the collector of customs, Natividad; not to gain the Ph
p2000 intended for him out of the transaction, but in order the better to assure the seizure of the
prohibited drugand the arrest of the surreptitious importers. There is certainly nothing immoral in
this or against the public good which should prevent the Government from prosecuting and
punishing the culprits, for this isnot a case where an innocent person is induced to commit a crime
merely to prosecute him, but it simply atrap set to catch a criminal.The mere fact that the Samson
pretended to agree a plan for smuggling illegally imported opium throughthe customhouse, in order
CRIMINAL LAW 1
(Sunday, Section 52) 172

the better to assure the seizure of said opium and the arrest of its importers, isno bar to the
prosecution and conviction of the latter.Decision against the accused is affirmed.

PEOPLE VS. DORIA


G.R. NO. 125299, JANUARY 22, 1999 301 SCRA 668

FACTS:
A buy-bust operation was conducted by the police which caught accused Doria red-handed of selling
prohibited drugs and during the operation the police officers searched for the marked bills that they
used in buying said drugs which happened to be in the house of Gaddao, according to Doria. When
they reached her house, the police officers came upon a box. He saw that one of the box's flaps was
open and inside the box was something wrapped in plastic. The plastic wrapper and its contents
appeared similar to the marijuana earlier "sold" to him by "Jun." His suspicion aroused, PO3
Manlangit entered "Neneth's" house and took hold of the box. He peeked inside the box and found
that it contained ten (10) bricks of what appeared to be dried marijuana leaves.
Both accused were convicted of the crime chared. Hence, this present petition.

ISSUE:
WON the warrantless arrest of Gaddao, the search of her person and house, and the admissibility of
the pieces of evidence obtained therefrom was valid.

HELD:
We hold that the warrantless arrest of accused-appellant Doria is not unlawful. Warrantless arrests
are allowed in three instances as provided by Section 5 of Rule 113. Under Section 5 (a), a person
may be arrested without a warrant if he "has committed, is actually committing, or is attempting to
commit an offense." Appellant Doria was caught in the act of committing an offense. When an accused
is apprehended in flagrante delicto as a result of a buy-bust operation, the police are not only
authorized but duty-bound to arrest him even without a warrant.
The warrantless arrest of appellant Gaddao, the search of her person and residence, and the seizure
of the box of marijuana and marked bills are different matters.
Our Constitution proscribes search and seizure without a judicial warrant and any evidence obtained
without such warrant is inadmissible for any purpose in any proceeding. 105 The rule is, however, not
absolute. Search and seizure may be made without a warrant and the evidence obtained there from
may be admissible in the following instances: (1) search incident to a lawful arrest; 2) search of a
moving motor vehicle; (3) search in violation of customs laws; (4) seizure of evidence in plain
view; (5) when the accused himself waives his right against unreasonable searches
and seizures. Accused-appellant Gaddao was not caught red-handed during the buy-bust operation
to give ground for her arrest under Section 5 (a) of Rule 113. She was not committing any crime.
Contrary to the finding of the trial court, there was no occasion at all for appellant Gaddao to flee
from the policemen to justify her arrest in "hot pursuit."114 In fact, she was going about her daily
chores when the policemen pounced on her.
This brings us to the question of whether the trial court correctly found that the box of marijuana
was in plain view, making its warrantless seizure valid.
The "plain view" doctrine applies when the following requisites concur: (a) the law enforcement
officer in search of the evidence has a prior justification for an intrusion or is in a position from which
he can view a particular area; (b) the discovery of the evidence in plain view is inadvertent; (c) it is
immediately apparent to the officer that the item he observes may be evidence of a crime, contraband
or otherwise subject to seizure. The law enforcement officer must lawfully make an initial intrusion
or properly be in a position from which he can particularly view the area. In the course of such lawful
CRIMINAL LAW 1
(Sunday, Section 52) 173

intrusion, he came inadvertently across a piece of evidence incriminating the accused. The object
must be open to eye and hand and its discovery inadvertent.
It is clear that an object is in plain view if the object itself is plainly exposed to sight. The difficulty
arises when the object is inside a closed container. Where the object seized was inside a closed
package, the object itself is not in plain view and therefore cannot be seized without a warrant.
However, if the package proclaims its contents, whether by its distinctive configuration, its
transparency, or if its contents are obvious to an observer, then the contents are in plain view and
may be seized. In other words, if the package is such that an experienced observer could infer from
its appearance that it contains the prohibited article, then the article is deemed in plain view. It must
be immediately apparent to the police that the items that they observe may be evidence of a crime,
contraband or otherwise subject to seizure. The marijuana was not in plain view and its seizure
without the requisite search warrant was in violation of the law and the Constitution. 135 It was fruit
of the poisonous tree and should have been excluded and never considered by the trial court.

INTESTATE ESTATE OF MANOLITA GONZALES VDA.


DE CARUNGCONG, - v e r s u s -
PEOPLE OF THE PHILIPPINES and WILLAM SATO,

FACTS:
The above-named accused (respondent), by means of deceit, did, then and there, willfully, unlawfully
and feloniously defraud the deceased mother in the following manner, to wit: the said accused
induced said deceased, who was already then blind and 79 years old, to sign and thumbmark a special
power of attorney the daughter of said accused, making her believe that said document involved only
her taxes, accused knowing fully well that said document authorizes his minor daughter to sell,
assign, transfer or otherwise dispose of to any person or entity of her properties all located at
Tagaytay City. The duly appointed administratrix of petitioner intestate estate of her deceased
mother, filed a complaint-affidavit for estafa against the accused (her brother-in-law, a Japanese
national). The accused moved for the quashal of the Information, claiming that under Article 332 of
the Revised Penal Code, his relationship to the person allegedly defrauded, the deceased who was his
mother-in-law, was an exempting circumstance. The trial court granted Sato’s motion and ordered
the dismissal of the criminal case. The petition for certiorari with CA was likewise dismissed. Hence,
this petition.

ISSUE:
W/N the death of accused’s wife and deceased mother’s daughter, Zenaida, extinguished the
relationship by affinity between the accused and deceased.
Whether or not accused should be exempt from criminal liability for reason of his relationship.

HELD:
No. Relationship by affinity between the surviving spouse and the kindred of the deceased spouse
continues even after the death of the deceased spouse, regardless of whether the marriage produced
children or not. Thus, the relationship by affinity between the accused and deceased is not
extinguished by the death of Zenaida.

No. The absolutory cause under Article 332 of the Revised Penal Code only applies to the felonies of
theft, swindling and malicious mischief. Under the said provision, the State condones the criminal
responsibility of the offender in cases of theft, swindling and malicious mischief. As an act of grace,
the State waives its right to prosecute the offender for the said crimes but leaves the private offended
party with the option to hold the offender civilly liable.
CRIMINAL LAW 1
(Sunday, Section 52) 174

However, the coverage of Article 332 is strictly limited to the felonies mentioned therein. The plain,
categorical and unmistakable language of the provision shows that it applies exclusively to the simple
crimes of theft, swindling and malicious mischief. It does not apply where any of the crimes
mentioned under Article 332 is complexed with another crime, such as theft through falsification or
estafa through falsification. Under Article 332 of the Revised Penal Code, the State waives its right to
hold the offender criminally liable for the simple crimes of theft, swindling and malicious mischief
and considers the violation of the juridical right to property committed by the offender against
certain family members as a private matter and therefore subject only to civil liability. The waiver
does not apply when the violation of the right to property is achieved through (and therefore
inseparably intertwined with) a breach of the public interest in the integrity and presumed
authenticity of public documents. For, in the latter instance, what is involved is no longer simply the
property right of a family relation but a paramount public interest.

THE PEOPLE OF THE PHILIPPINES, PLAINTIFF-APPELLEE VS NICOLAS JAURIGUE AND


AVELINA JAURIGUE, defendants. Avelina Jaurigue, appellant.

FACTS:
Nicolas Jaurigue and Avelina Jaurigue were prosecuted in the Court of First Instance of Tayabas, for
the crime of murder, of which Nicolas Jaurigue was acquitted, but defendant Avelina Jaurigue was
found guilty of homicide and sentenced to an indeterminate penalty ranging from seven years, four
months and one day of prision mayorto thirteen years, nine months and eleven days of reclusion
temporal, with the accessory penalties provided by law, to indemnify the heirs of the deceased,
Amando Capina, in the sum of P2,000, and to pay one-half of the costs. She was also credited with
one-half of the period of preventive imprisonment suffered by her. From said judgment of conviction,
defendant Avelina Jaurigue appealed to the Court of Appeals for Southern Luzon.

On September 20, 1942, at around 8o’clock in the evening, Nicolas Jaurigue went to the chapel of the
Seventh Day Adventists o attend religious services. Avelina Jaurigue entered the chapel shortly after
the arrival of her father, also for the purpose of attending religious services, upon observing the
presence of Avelina Jaurigue, Amado Capina went to the bench on which Avelina was sitting and sat
by her right side, and, without saying a word, Amado, with the greatest of impudence, placed his hand
on the upper part of her right thigh. Avelina Jaurigue, conscious of her personal dignity and honor,
pulled out with her right hand the fan knife which she had in a pocket of her dress, with the intention
of punishing Amado's offending hand. Amado seized Avelina's right hand, but she quickly grabbed
the knife with her left hand and stabbed Amado once at the base of the left side of the neck, inflicting
upon him a wound about 4 1/2 inches deep, which was necessarily mortal. Fearing that Amado's
relatives might retaliate, barrio lieutenant Lozada advised Nicolas Jaurigue and herein defendant and
appellant to go home immediately, to close their doors and windows and not to admit anybody into
the house, unless accompanied by him.

Then three policemen arrived in their house, at about 10 o'clock that night, and questioned them
about the incident, defendant and appellant immediately surrendered the knife marked as Exhibit B,
and informed said policemen briefly of what had actually happened

ISSUES:
Whether or not the lower court erred in (1) not holding said appellant had acted in the legitimate
defense of her honor, (2) in not finding in her favor additional mitigating circumstances, and (3) in
holding that the commission of the alleged offense attended by aggravating circumstance.
CRIMINAL LAW 1
(Sunday, Section 52) 175

HELD:
In the mind of the court, there is not the least doubt that, in stabbing to death the deceased Amado
Capina, in the manner and form and under the circumstances above indicated, the defendant and
appellant committed the crime of homicide, with no aggravating circumstance whatsoever, but with
at least three mitigating circumstances of a qualified character to be considered in her favor. Said
chapel where the incident took place was lighted with electric lights and there were several people
inside; under the circumstances, there was and there could be no possibility of her being raped. The
means employed by her in the defense of her honor was evidently excessive; and under the facts and
circumstances of the case, she cannot be legally declared completely exempt from criminal liability.

The facts that the defendant and appellant (1) immediately, voluntarily and unconditionally
surrendered and admitted having stabbed the deceased, (2) had acted in the immediate vindication
of grave offense committed against her, (3) had not intended to kill the deceased but merely wanted
to punish his offending hand, be considered as mitigating circumstances.

Defendant and appellant Avelina Jaurigue is hereby sentenced to an indeterminate penalty ranging
from two months and one day of arresto mayor, as minimum, to two years, four months, and one day
of prision correccional, as maximum, with the accessory penalties prescribed by law, to indemnify
the heirs of the deceased Amado Capina, in the sum of P2,000, and to suffer the corresponding
subsidiary imprisonment, not to exceed 1/3 of the principal penalty, in case of insolvency, and to pay
the costs. Defendant and appellant should also be given the benefit of 1/2 of her preventive
imprisonment, and the knife marked ordered confiscated.

PEOPLE VS. NARVAEZ


121 SCRA 389 (1983)

FACTS:
Mamerto Narvaez has been convicted of murder (qualified by treachery) of David Fleischer and
Flaviano Rubia. On August 22, 1968, Narvaez shot Fleischer and Rubia during the time the two were
constructing a fence that would prevent Narvaez from getting into his house and rice mill. The
defendant was taking a nap when he heard sounds of construction and found fence being made. He
addressed the group and asked them to stop destroying his house and asking if they could talk things
over. Fleischer responded with "No, gadamit, proceed, go ahead." Defendant lost his "equilibrium,"
and shot Fleisher with his shotgun. He also shot Rubia who was running towards the jeep where the
deceased's gun was placed. Prior to the shooting, Fleischer and Co. (the company of Fleischer's
family) was involved in a legal battle with the defendant and other land settlers of Cotabato over
certain pieces of property. At the time of the shooting, the civil case was still pending for annulment
(settlers wanted granting of property to Fleisher and Co. to be annulled). At time of the shooting,
defendant had leased his property from Fleisher (though case pending and ownership uncertain) to
avoid trouble. On June 25, defendant received letter terminating contract because he allegedly didn't
pay rent. He was given 6 months to remove his house from the land. Shooting was barely 2 months
after letter. Defendant claims he killed in defense of his person and property. CFI ruled that Narvaez
was guilty. Aggravating circumstances of evident premeditation offset by the mitigating circumstance
of voluntary surrender. For both murders, CFI sentenced him to reclusion perpetua, to indemnify the
heirs, and to pay for moral damages.

ISSUES/ HELD:
1. Whether or not CFI erred in convicting defendant-appellant despite the fact that he acted in defense
of his person.
CRIMINAL LAW 1
(Sunday, Section 52) 176

No. The courts concurred that the fencing and chiselling of the walls of the house of the defendant
was indeed a form of aggression on the part of the victim. However, this aggression was not done on
the person of the victim but rather on his rights to property. On the first issue, the courts did not err.
However, in consideration of the violation of property rights, the courts referred to Art. 30 of the civil
code recognizing the right of owners to close and fence their land. Although is not in dispute, the
victim was not in the position to subscribe to the article because his ownership of the land being
awarded by the government was still pending, therefore putting ownership into question. It is
accepted that the victim was the original aggressor.

2. WON the court erred in convicting defendant-appellant although he acted in defence of his rights.
Yes. However, the argument of the justifying circumstance of self-defense is applicable only if the 3
requirements are fulfilled. Art. 11(1) RPC enumerates these requisites:

Unlawful aggression. In the case at bar, there was unlawful aggression towards appellant's property
rights. Fleisher had given Narvaez 6 months and he should have left him in peace before time was up,
instead of chiseling Narvaez's house and putting up fence. Art. 536 of the Civil Code also provides
that possession may not be acquired through force or intimidation; while Art. 539 provides that every
possessor has the right to be respected in his possession Reasonable necessity of means employed to
prevent or repel attack. In the case, killing was disproportionate to the attack. Lack of sufficient
provocation on part of person defending himself. Here, there was no provocation at all since he was
asleep Since not all requisites present, defendant is credited with the special mitigating circumstance
of incomplete defense, pursuant to Art. 13(6) RPC. These mitigating circumstances are: voluntary
surrender and passion and obfuscation (read p. 405 explanation) Crime is homicide (2 counts) not
murder because treachery is not applicable on account of provocation by the deceased. Also, assault
was not deliberately chosen with view to kill since slayer acted instantaneously. There was also no
direct evidence of planning or preparation to kill. Art. 249 RPC: Penalty for homicide is reclusion
temporal. However, due to mitigating circumstances and incomplete defense, it can be lowered three
degrees (Art. 64) to arrestomayor.

WON he should be liable for subsidiary imprisonment since he is unable to pay the civil indemnity
due to the offended party.

No. He is not liable to be subsidiarily imprisoned for nonpayment of civil indemnity. RA 5465 made
the provisions of Art. 39 applicable to fines only and not to reparation of damage caused,
indemnification of consequential damages and costs of proceedings. Although it was enacted only
after its conviction, considering that RA 5465 is favorable to the accused who is not a habitual
delinquent, it may be given retroactive effect pursuant to Art. 22 of the RPC.
Judgment: Defendant guilty of homicide but w/ mitigating circumstances and extenuating
circumstance of incomplete self defense. Penalty is 4 months arresto mayor and to indemnify each
group of heirs 4,000 w/o subsidiary imprisonment and w/o award for moral damages. Appellant has
already been detained 14 years so his immediate release is ordered.
Gutierrez, dissenting. Defense of property can only be invoked when coupled with form of attack on
person defending property. In the case at bar, this was not so. Appellant should then be sentenced to
prision mayor. However, since he has served more than that, he should be released.
CRIMINAL LAW 1
(Sunday, Section 52) 177

ULEP VS PEOPLE

Facts:
- On Dec 22 1995, Buenaventura Wapili appeared to have gone crazy and kept on running without
any particular direction.
- SPO1 Ulep, together with Espadera and Pillo, arrived at the scene armed with M-16 rifles and saw
the naked Wapili approaching them.
- The police claimed that Wapili was armed with a bolo and a rattan stool, while Wapili’s relatives
and neighbours said he had no bolo, but only a rattan stool.
- SPO1 Ulep fired a warning shot in the air and told Wapili to put down his weapons ar they would
shoot him.
- When Wapili was only about 2-3 meters away from them, SPO1 Ulep shot the victim with his M-16
rifle, hitting him in various parts of his body. As the victim slumped to the ground, SPO1 Ulep came
closer and pumped another bullet into his head and literally blew his brains out.

Issue:
w/n accussed should be acquitted on the basis of his claim that the killing of the victim was in the
course of the performance of his official duty as a police officer, and in self-defense

Held:
It cannot be said that the fatal wound in the head of the victim was a necessary consequence of
accused-appellant’s due performance of a duty or the lawful exercise of a right or office.
The evidence does not favour his claim of self-defense.
Accused-appelant SPO1 ERNESTO ULEP is found guilty of Homicide, instead of murder.
The accused must prove the presence of 2 requisites: (1) that he acted in the performance of a duty
or in the lawful exercise of a right or an office, and (2) the injury caused or the offense committed be
the necessary consequence of the due performance of the duty or the lawful exercise of such right or
office.
There were two stages of the incident:
1. The victim threatened the safety of the police officers by menacingly advancing towards them. Up
to that point, his decision to respond with a barrage of gunfire to halt the victim’s further advance
was justified under the circumstances.
2.When he fatally shot the victim in the head, perhaps in his desire to take no chances, even after the
latter slumped to the ground due t multiple gunshot wounds sustained while charging at the police
officers. He cannot be exonerated from overdoing his duty.
-The aggression that was initially begun by the victim already ceased when accused-appellant
attacked him. From that moment, there was no longer any danger to his life.
No treachery, thus the offense is only murder. Victim was given more than sufficient warning before
he was shot.
–Art. 69 of RPC is applicable.
Incomplete justification is a special or privileged mitigating circumstance, which, not only cannot be
offset by aggravating circumstances but also reduces the penalty by one or two degrees than that
prescribed by law.
The instant case would have fallen under Art. 11, par 5 had the two conditions therefore concurred.
CRIMINAL LAW 1
(Sunday, Section 52) 178

GUILLERMO VS PEOPLE
GR 153287 June 30, 2008

FACTS:
In re: Article 13, Par 1 and Article 69
Facts: Noel Guillermo, Arnaldo Socias, and Joemar Palma was charged for the killing of Winnie Alon,
of the 3, only Guillermo, was found guilty of Homicide with the special or privileged mitigating
circumstance of incomplete justification, and sentenced to 6 years of prision correccional, as
minimum, to 10 years of prision mayor, as maximum, with indemnity of P50,000. Appellant contends
that self-defense should have been appreciated.

ISSUE:
WON self-defense should be appreciated.

HELD:
Yes. Self-defense must be appreciated as Alon was about to attack Guillermo with a broken bottle
when Guillermo realized he had a knife. Guillermo had attacked Alon in self-defense, albeit the
reasonable necessity of the means employed to repel it was not found as he stabbed Alon 3 times.
There was also a disproportionate difference between an actual blade and a broken beer bottle. All
this would lead to an incomplete self-defense, which is a mitigating circumstance only, and not a
justifying one. His sentence is modified accordingly.

THE PEOPLE OF THE PHILIPPINES (PLAINTIFF-APPELLEE) VS. DOMINGO URAL


(ACCUSED-APPELLANT) L-30801 MARCH 27 1974 J. AQUINO APPEAL FROM A JUDGMENT OF THE
CFI OF ZAMBOANGA DEL SUR

FACTS:
- Alberio went to the municipal building and saw Ural, a policeman inside the jail where he was boxing
prisoner Napola (who was imprisoned for being drunk). When Napola fell to the ground he U kicked
him and poured some liquid on N and then ignited N’s body. - Dr. Luzonia Bakil who treated the
victim, said that he sustained 2nd degree burns on the arms, neck, left side of the face and one half of
the body including the back. She also testified that without any medical intervention, the burns would
have caused death - Napola died on Aug 25 1966. Death certificate indicated burn as the cause of
death. - During the trial, the prosecutors failed to present the detention prisoners who saw the
burning of Napola as witnesses as well as the wife of the deceased - Nevertheless, Ural was convicted
of murder, was sentenced to reclusion perpetua and was ordered to pay for costs

ISSUE:
Whether the evidence of the prosecution was sufficient to prove his guilt beyond reasonable doubt.

Held: TC did not err in convicting Ural for murder. Ural had his own version of the story. According
to him he heard a scream for help from Napola whose shirt was in flames when found by him, he
removed the shirt, but did not summon the doctor because he thought that the burns were not
serious. o SC: this statement cannot prevail over the testimony of Alberio o This statement does not
prove that he was not the one who burned Napola, at most this could only mean that he was alarmed
by the consequences of his evil act - Ural assailed the credibility of Alberio as a witness, saying that
he was not listed as a prosecution witness and that he was convicted of murder in the past o Wouldn’t
preclude him from being a credible witness. o Since there was no police investigation (accused a
police officer), the investigation that ensued was done by a special counsel of the fiscal’s office. A
CRIMINAL LAW 1
(Sunday, Section 52) 179

possible explanation of alberio not being listed at first. The statements of the witnesses for the
defense were not inconsistent with that of Alberio’s.

Therefore, there is no reason to not believe in Alberio’s testimony. - The present case is covered by
article 4 (par.1-result greater than what was intended). Aggravating circumstance: art 14(1). o TC
erred in not appreciating the Mitigating circumstance “that the offender had no intention to commit
so grave a wrong as that committed” No intent to kill but only to maltreat the drunk napola who
might have been making a nuisance of himself He realized the fearful consequence of his felonious
act, he allowed Napola to secure medical treatment at the municipal dispensary. Since the mitigating
circumstance offset the aggravating circumstance, TC correctly imposed the penalty of reclusion
perpetua which is the medium period of the penalty for murder.

DECISION: TC decision AFFIRMED.

PEOPLE OF THE PHILIPPINES vs. INOCENCIO GONZALEZ, JR.


G.R. No. 139542 June 21, 2001

FACTS:
After their vehicles almost collided with each other, Andres and Appellant had an altercation.
Thereafter, Andres went back inside to his car when he was blocked by the appellant’s son who said,
"Anong problema mo sa erpat ko." Andres testified that he felt threatened and so he immediately
boarded his vehicle, sat at the driver’s seat, closed the door, and partially opened the car window just
wide enough to talk back to appellant’s son, Dino. In the meantime, appellant, thinking that Andres
was going to get something from his car, took a gun. However, he was pushed by his daughter-in-law
which made him lost his balance and accidentally fired the gun hitting Andres’ wife, and two
sons.Appellant was charged and convicted of Murder, Double Frustrated Murder and Attempted
Murder in the RTC. The appellant seeks a reversal and prays that judgment be rendered exempting
him from criminal and civil liabilities contending that he had no intention to shoot Noel Andres much
less his wife nor the children. He lost his balance when his daughter Trisha approached and pushed
him backwards to stop him from joining Dino and Noel Andres but the appellant tried to free his right
hand holding the gun and it accidentally fired.

ISSUE:
Whether or not there was treachery attendant in the crime.

RULING:
No. Treachery under par.16 of Article 14 of the Revised Penal Code is defined as the deliberate
employment of means, methods or forms in the execution of a crime against persons which tend
directly and specially to insure its execution, without risk to the offender arising from the defense
which the intended victim might raise. For treachery to be appreciated two elements must concur:
1) the employment of means of execution that would insure the safety of the accused from retaliatory
acts of the intended victim and leaving the latter without an opportunity to defend himself and 2) the
means employed were deliberately or consciously adopted by the offender.8 We affirm the
recommendation of the Solicitor-General that the shooting was not attended by treachery and
accordingly the crime committed for the death of Feliber Andres is homicide and not murder.
CRIMINAL LAW 1
(Sunday, Section 52) 180

PEOPLE VS PAGAL

Facts:
Pedro Pagal and Jose Torcelino (accused) were employees Gau Guan (victim). Allegedly Gau Guan
maltreated them during their employment with the latter. One night Pedro and Jose robbed and
stabbed their employer with an ice pick and took the valuables of the victim. They were both
convicted of murder and were sentenced to suffer the extreme penalty of death. Hence this automatic
review by the S.C., when the case was called for arraignment, counsel de oficio for the accused
informed said court of their intention to enter a plea of guilty provided that they be allowed
afterwards to prove the mitigating circumstances of sufficient provocation or threat on the part of
the offended party immediately preceding the act, and that of having acted upon an impulse so
powerful as to produce passion and obfuscation.

Issue:
Whether Pedro and Jose are entitled to the above mentioned mitigating circumstances?

Ruling:
No. The court held that the appellants 'contention is devoid of merit. Firstly, since the alleged
provocation which caused the obfuscation of the appellants arose from the same incident, that is, the
alleged maltreatment and/or ill-treatment of the appellants by the deceased, these two mitigating
circumstances cannot be considered as two distinct and separate circumstances but should be
treated as one. Secondly, the circumstance of passion and obfuscation cannot be mitigating in a crime
which as in the case at bar is planned and calmly meditated before its execution. Thirdly, the
maltreatment that appellants claim the victim to have committed against them occurred much earlier
than the date of the commission of the crime. Provocation in order to be a mitigating circumstance
must be sufficient and immediately preceding the act. We hold that the trial court did not commit any
error in not appreciating the said mitigating circumstances in favor of the appellants. The sentenced
of the accused is lowered to reclusion perpetua due to the mitigating circumstance of Plea of Guilty.

URBANO V PEOPLE

FACTS:
On September 28, 1993, at around 8:00 p.m., the victim Brigido Tomelden and petitioner were at the
compound of the Lingayen Water District (LIWAD) in Lingayen, Pangasinan, having just arrived from
a picnic in the nearby town of Bugallon, Pangasinan, where, with some other co-workers, they drunk
beer in a restaurant. While inside the compound, the two had a heated altercation in the course of
which Tomelden hurled insulting remarks at petitioner. Reacting, petitioner asked why Tomelden,
when drunk, has the penchant of insulting petitioner. The exchange of words led to an exchange of
blows. Cooler heads succeeded in breaking up the fight, but only for a brief moment as the
protagonists refused to be pacified and continued throwing fist blows at each other. Then petitioner
delivered a "lucky punch," as described by eyewitness Orje Salazar, on Tomelden’s face, which made
Tomelden topple down. Tomelden was on the verge of hitting his head on the ground had their
companions not caught him and prevented the fall. The blow, however, caused Tomelden’s nose to
bleed and rendered him unconscious. Tomelden allegedly slapped and hurled insults at petitioner,
calling him "sipsip" just to maintain his employment as Navarro’s tricycle driver. Tomelden allegedly
then delivered several fist and kick blows at petitioner, a couple of which hit him despite his evasive
actions. Petitioner maintained that he only boxed the victim in retaliation, landing that lucky punch
in the course of parrying the latter’s blows. Petitioner and his other co-workers brought Tomelden
to the office of the LIWAD general manager where he spent the night. He remained in the compound
the following day, September 29, 1993. Upon arriving home at around 6:00 p.m. of that day,
CRIMINAL LAW 1
(Sunday, Section 52) 181

Tomelden informed his wife, Rosario, of the fight the previous night and of his having been rendered
unconscious. He complained of pain in his nape, head, and ear which impelled Rosario to immediately
bring him to the Lingayen Community Hospital where Dr. Daisy Arellano examined him and treated
his lacerated left index finger, contusions, and hematoma at the right cerebrum. On October 2 and 7,
1993, Tomelden went back to the hospital complaining of dizziness, headache, and other pains. The
attending doctors observed the patient to be in a state of drowsiness and frequent vomiting. On
October 8, 1993, Rosario brought Tomelden to the Sison Memorial Provincial Hospital in Dagupan
City, where the attending physician, Dr. Ramon Ramos, diagnosed Tomelden suffering from "brain
injury, secondary to mauling to consider cerebral hemorrhage." Tomelden was confined in the
provincial hospital until 3:00 p.m. of October 10, 1993, and, due to financial constraints, was
thereafter discharged despite signs negating physical condition improvement. Upon reaching their
house, however, Tomelden again complained of extreme head pain, prompting his wife to bring him
back to the Lingayen Community Hospital where Dr. Arellano again attended to him. This time, things
turned for the worst, the doctor noting that Tomelden appeared to be semi-conscious, sleepy,
uncooperative, and not responding to any stimulant. Tomelden died at 9:00 p.m. of that day due, per
Dr. Arellano, to "cardio-respiratory arrest secondary to cerebral concussion with resultant cerebral
hemorrhage due to mauling incident." The defense presented petitioner who denied having any
intention to kill, asserting that hypertension, for which Tomelden was receiving treatment, was the
cause of the latter’s death. RTC rendered judgment finding petitioner guilty as charged for the crime
of homicide. Petitioner appealed, but CA affirmed the decision modifying the award of moral
damages. The appellate court held that the commission by petitioner of the crime of homicide, as
defined and penalized under Article 249 of the Revised Penal Code (RPC), had been proved beyond
moral certainty of doubt, pointing to the lucky punch as the proximate cause of Tomelden’s
hospitalization and ultimately his death. And like the RTC, the CA found no qualifying circumstance
to increase or lower the penalty.

ISSUE:
Did the Court err in not appreciating the mitigating circumstances of sufficient provocation on the
part of the victim and lack of intent to commit so grave a wrong in favor of the petitioner?

HELD:
Yes. Judgment is affirmed with modification considering the presence of mitigating circumstances
The prosecution witness, Salazar, testified about petitioner’s lucky punch hitting Tomelden right
smack on the face. And even if Tomelden’s head did not hit the ground as his co-workers averted that
actuality, that punch gave him a bleeding nose and rendered him unconscious right after the
September 28, 1993 fight. From then on, Tomelden was in and out of the hospital complaining of
headache, among other pains, until his demise on October 10, 1993, or 12 days after the blow that
made Tomelden unconscious. Significantly, Dr. Arellano testified conducting an autopsy on the body
of Tomelden and stressed that the "softened portion of the scalp over (R) occipito-temporal area
about 5 inches above and posterior to the (R) ear" of the victim could have been caused by a fist blow.
She also opined that the fist blow which landed on Tomelden’s head could have shaken his brain
which caused the cerebral concussion; and that the cause of the victim’s death was "cardio-
respiratory arrest secondary to cerebral concussion with resultant cerebral hemorrhage due to
mauling incident." The combined effects of the testimonies of Salazar and Dr. Arellano, buttressed by
that of Rosario who related about her husband’s post September 28, 1993 severe head pain, clearly
establish beyond cavil the cause of Tomelden’s death and who was liable for it.

Mitigating Circumstances Present


Petitioner next contends that the mitigating circumstances of no intention to commit so grave a
wrong and sufficient provocation on the part of the victim ought to be appreciated in petitioner’s
CRIMINAL LAW 1
(Sunday, Section 52) 182

favor. On this score, we agree with petitioner. Paragraphs 3 and 4 of Art. 13, RPC provide as follows:
Art. 13. Mitigating circumstances.––The following are mitigating circumstances: x x x x 3. That the
offender had no intention to commit so grave a wrong as that committed. 4. That sufficient
provocation or threat on the part of the offended party immediately preceded the act. When the law
speaks of provocation either as a mitigating circumstance or as an essential element of self-defense,
the reference is to an unjust or improper conduct of the offended party capable of exciting, inciting,
or irritating anyone; it is not enough that the provocative act be unreasonable or annoying; the
provocation must be sufficient to excite one to commit the wrongful act and should immediately
precede the act. This third requisite of self-defense is present: (1) when no provocation at all was
given to the aggressor; (2) when, even if provocation was given, it was not sufficient; (3) when even
if the provocation was sufficient, it was not given by the person defending himself; or (4) when even
if a provocation was given by the person defending himself, it was not proximate and immediate to
the act of aggression. In the instant case, Tomelden’s insulting remarks directed at petitioner and
uttered immediately before the fist fight constituted sufficient provocation. This is not to mention
other irritating statements made by the deceased while they were having beer in Bugallon. Petitioner
was the one provoked and challenged to a fist fight.

It is abundantly clear from the above transcript that the provocation came from Tomelden. In fact,
petitioner, being very much smaller in height and heft, had the good sense of trying to avoid a fight.
But as events turned out, a fisticuff still ensued, suddenly ending when petitioner’s lucky punch found
its mark. Moreover, the mitigating circumstance that petitioner had no intention to commit so grave
a wrong as that committed should also be appreciated in his favor. While intent to kill may be
presumed from the fact of the death of the victim, this mitigating factor may still be considered when
attendant facts and circumstances so warrant, as in the instant case. Consider: Petitioner tried to
avoid the fight, being very much smaller than Tomelden. He tried to parry the blows of Tomelden,
albeit he was able, during the scuffle, to connect a lucky punch that ended the fight. And lest it be
overlooked, petitioner helped carry his unconscious co-worker to the office of the LIWAD’s general
manager. Surely, such gesture cannot reasonably be expected from, and would be unbecoming of, one
intending to commit so grave a wrong as killing the victim. A bare-knuckle fight as a means to parry
the challenge issued by Tomelden was commensurate to the potential violence petitioner was facing.
It was just unfortunate that Tomelden died from that lucky punch, an eventuality that could have
possibly been averted had he had the financial means to get the proper medical attention. Thus, it is
clear that the mitigating circumstance of "no intention to commit so grave a wrong as that
committed" must also be appreciated in favor of petitioner while finding him guilty of homicide. That
petitioner landed a lucky punch at Tomelden’s face while their co-workers were trying to separate
them is a compelling indicium that he never intended so grave a wrong as to kill the victim. Withal,
with no aggravating circumstance and two mitigating circumstances appreciable in favor of
petitioner, we apply par. 5 of Art. 64, RPC, which pertinently provides: Art. 64. Rules for the
application of penalties which contain three periods.––In cases in which the penalties prescribed by
law contain three periods, whether it be a single divisible penalty or composed of three different
penalties, each one of which forms a period in accordance with the provisions of Articles 76 and 77,
the courts shall observe for the application of the penalty the following rules, according to whether
there are or are no mitigating or aggravating circumstances: 5. When there are two or more
mitigating circumstances and no aggravating circumstances are present, the court shall impose the
penalty next lower to that prescribed by law, in the period that it may deem applicable, according to
the number and nature of such circumstances.
CRIMINAL LAW 1
(Sunday, Section 52) 183

PEOPLE OF THE PHILIPPINES VS. BENITO


159 Phil. 408 [ GR No. L-32042, Feb 13, 1975 ]

ESGUERRA, J.:
This is a mandatory review of the judgment of the Circuit, Criminal Court of Manila in Criminal Case
No. CCC-VI-609, entitled "People of the Philippines vs. Alberto Benito y Restubog," imposing upon the
accused, Alberto Benito y Restubog, upon his plea of guilty to the charge of murder, the penalty of
"death; to indemnify the heirs of the deceased as follows: P12,000.00 for the death of the deceased;
P20,000.00 as indemnity for loss of earning capacity of the deceased who was then only 36 years of
age at the time of his death and earning P7,597.80 per annum; P20,000.00 for exemplary damages;
P25,000.00 for moral damages, all amounts to bear interest until they shall have been fully paid; and
to pay the costs".

The issues raised by the accused revolve around the alleged errors of the lower court in considering
the mitigating and aggravating circumstances attendant to the commission of the crime to determine
the proper penalty to be imposed on the accused.

It is not controverted that at about 5:30 p.m. of December 12, 1969, the victim Pedro Moncayo, Jr.,
Assistant Chief of Personnel Transaction and Acting Chief of the Administrative Division of the Civil
Service Commission, while driving his car on P. Paredes street in front of the Office of the Civil Service
Commission was followed by the accused, and when the car was about to turn at the intersection of
P. Paredes and Lepanto Streets, Manila, the accused shot him eight times with a .22 caliber revolver,
causing the victim's death. The accused was charged with murder and when the case was called for
trial, through counsel de parte, he manifested his desire to withdraw his previous plea of not guilty
and substitute it with a plea of guilty without prejudice to proving mitigating circumstances. The
prosecution manifested that it would controvert whatever mitigating circumstances the accused
would prove and also prove other aggravating circumstances. The trial court repeatedly explained
to the accused the nature and consequences of his plea of guilty to the offense charged and warned
him that the maximum penalty imposable is death. Notwithstanding the explanation and warning of
the trial court, the accused, assisted by his counsel de parte upon being re-arraigned, entered a plea
of guilty. The accused presented evidence to prove mitigating circumstances and the prosecution
subsequently introduced evidence to prove aggravating circumstances not mentioned in the
information. The Court sentenced the accused to death after finding him guilty as principal in the
crime of murder qualified by treachery, with the aggravating circumstances of evident premeditation
and disregard of the respect due to the offended party on account of his rank, offset by the mitigating
circumstance of accused's plea of guilty.

On the first assignment of error regarding the failure of the lower Court to consider the mitigating
circumstance of voluntary surrender, both the accused and the Solicitor General are agreed that the
said mitigating circumstance should be considered in his favor. The intention of the accused to
surrender could be clearly discerned from the fact that immediately after the shooting, the accused
having all the opportunity to escape, did not do so but instead called up the Manila Police
Department. When the policemen went to the scene of the crime to investigate, the accused
voluntarily approached them and, without revealing his identity, told them that he would help in
connection with the case as he knew the suspect as well as the latter's motive. While it may be true
that the accused did not immediately tell the police that he was the assassin, perhaps because he was
momentarily shocked by the enormity of his crime, nevertheless when brought to the police station
immediately thereafter as a possible witness (accused was with the police investigators all that time),
CRIMINAL LAW 1
(Sunday, Section 52) 184

he confided to the investigators that he was "voluntarily surrendering" and also surrendering the
fatal gun used in the shooting of the victim" (p. 9, t.s.n. December 26, 1969). We fully subscribe to
appellee's observation that all the aforementioned acts of the accused were strongly indicative of his
intent or desire to surrender voluntarily to the authorities. The accused must be credited with the
mitigating circumstance of voluntary surrender.

II

It is the contention of the accused that the criminal act of murder was committed in the immediate
vindication of a grave offense done by the victim against the accused and, therefore, this mitigating
circumstance must be credited in his favor. The supposed grave offense done by the victim was an
alleged remark made in the presence of the accused at about 11:00 a.m. of December 12, 1969, that
the Civil Service Commission is a hangout of thieves. The accused felt alluded to because he was
facing then criminal and administrative charges on several counts involving his honesty and integrity.

There is merit in appellee's argument that said victim's remark even if actually uttered in the
presence of the accused, cannot be considered a grave offense against the latter. The remark itself
was general in nature and not specifically directed to the accused. If he felt alluded to by a remark
which he personally considered insulting to him, that was his own individual reaction thereto. Other
people in the vicinity who might have heard the remark could not possibly know that the victim was
insulting the accused unless they were aware of the background of the criminal and administrative
charges involving moral turpitude pending against the accused. At most, said remark might be
considered a mere provocation and not a grave offense which might have impelled the accused to
commit a crime in immediate retaliation. As the provocation was not sufficient and did not
immediately precede the act, it may not be considered as a mitigating circumstance.

In this case, however, the provocation was the remark uttered at 11:00 a.m. of December 12, 1969,
while the crime of murder was committed by the accused at about 5:30 p.m. of the same day, giving
him several hours to reflect and hold his temper. Stated otherwise, the act of killing did not
immediately or proximately follow the supposed sufficiently insulting and provocative remark. The
juridical reason for appreciating this mitigating circumstance is the implied recognition by the law of
the weakness of human nature such that an ordinary human being if sufficiently provoked would
immediately retaliate in the unchristian spirit of vindictive retribution. But the circumstances of this
case are such that the act of murder committed by the accused could not reasonably be attributed to
an immediate or proximate retaliatory action on his part to vindicate what personally appeared to
him as sufficient provocation in the form of an insulting remark allegedly uttered by the victim. The
failure of the accused to immediately react to the supposed provocative insulting remark might even
be taken as his ignoring it altogether, or considering it unimportant at the moment he heard the
remark. In other words, the remark was inadequate to stir or drive the accused to violence at the
time it was uttered and he had more than sufficient time to suppress his emotion over said remark if
he ever did resent it. The trial Court did not commit any error when it rejected the aforementioned
incident as a basis for crediting a mitigating circumstance in favor of the accused.

III

The accused also claims that the lower Court should have considered the mitigating circumstance
that sufficient provocation or threat on the part of the deceased immediately preceded the act
because of the alleged statement of the deceased in Tagalog uttered at about 7:00 p.m. on the night
of December 11, 1969 (night preceding the day of the crime), to wit, "Umalis ka na nga diyan baka
may mangyari pa sa iyo at baka ipayari kita dito" (Get out of there, because something might happen
CRIMINAL LAW 1
(Sunday, Section 52) 185

to you and because I might have you finished here). That statement of the deceased was supposed
to have been uttered in the presence of other people almost twenty four (24) hours before the crime
was committed. It was not accompanied by any overt act against accused and nothing more
happened during that night, so that the accused by that utterance could not have felt sufficiently
provoked or threatened so as to immediately react in his defense or retaliate by committing a
crime. The provocation or threat, did not immediately precede the shooting. In other words, the
accused had almost a day to mull over the alleged threat or provocation before he reacted by shooting
the victim. The inevitable conclusion is that the accused did not feel sufficiently threatened or
provoked by the alleged utterance of the victim at the time it was uttered, or within a reasonable time
thereafter, and when he shot the victim the next day, it was a deliberate act of vengeance and not the
natural reaction of a human being to ward off a serious threat or to immediately retaliate when
provoked.

We agree with appellee's contention that "provocation or threat to constitute a mitigating


circumstance, must, in the language of the law, be "sufficient", that is, adequate to excite the person
to commit the wrong and must accordingly be proportionate to its gravity and must also immediately
precede the act."

The lower Court correctly rejected the claim of the accused to this mitigating circumstance.

IV

The generic aggravating circumstance of disregard of rank considered by the lower Court against the
accused is being assailed on the ground that at the time of the commission of the murder, the accused
was no longer connected with the Civil Service Commission as the decision in the administrative case
against him ordering his dismissal from the service became effective February 16, 1966.

There is no question, however, that accused was a clerk in the Civil Service Commission and the
victim was Assistant Chief of the Personnel Transaction of that Office. When the accused saw and
talked with the deceased regarding the former's administrative case that proved to be the motive for
the murder by his own admission, accused made it very obvious that he recognized the deceased as
his superior officer. The mere fact that the dismissal of the accused from office was made
immediately executory was of no moment since he appealed that decision and the case was still
pending and, by his own allegation, he was later completely exonerated by the Civil Service Board of
Appeals in its decision of February 17, 1971.

It may be true that this aggravating circumstance was considered against the accused even if it was
not alleged in the information, but this is a generic aggravating circumstance, and not a qualifying
circumstance that would change the nature or affect the gravity of the crime committed, but one
which is capable of being proven and taken into consideration even if it was not alleged in the
information. The lower Court in considering this generic aggravating circumstance against the
accused did not violate his constitutional right to be informed of the nature and cause of the
accusation against him for murder. This aggravating circumstance was correctly considered against
the accused.

We cannot see Our way clear to the argument of the accused that the aggravating circumstance of
CRIMINAL LAW 1
(Sunday, Section 52) 186

evident premeditation, although included in the information, should not be considered against the
accused because although he pleaded guilty to the charge unconditionally, the prosecution sought
and was allowed to adduce evidence to show the criminal participation of appellant in the
commission of the offense and the background of the crime imputed to him, and the evidence
submitted by the prosecution failed to establish the elements of the aggravating circumstance of
evident premeditation. It is further argued that the prosecution is deemed to have thereby waived
the effect of the unconditional plea of guilty by the accused in so far as the aggravating circumstance
of evident premeditation is concerned.

What upsets the entire argument of the accused is the fact that the prosecution successfully proved
the existence of evident premeditation because Exhibit "A", his own declaration, narrates fully the
several attempts of the accused to talk with the deceased; how he was rebuffed in those attempts and
even insulted, and that he was jobless after having been dismissed from the office on allegedly
fabricated charges made by the deceased. All of these admissions provided a strong motive for the
accused to plan on how to retaliate against the victim by taking the law into his hands.

We cannot disregard the fact that the accused unconditionally pleaded guilty to the offense charged
after the lower Court specifically called his attention to the aggravating circumstance of evident
premeditation before he was re-arraigned (p. 5, t.s.n. December 26, 1969), and after the Fiscal had
rejected his counsel's proposal to delete this aggravating circumstance from the information (p. 3,
t.s.n. December 26, 1969). The accused was fully aware of the consequences of his unconditional plea
of guilty to the offense of murder after it was explained to him, and the serious implication and
meaning of the aggravating circumstance of evident premeditation expressly mentioned in the
information.

The admission of the accused that he had with him a .22 caliber revolver on the afternoon of
December 12, 1969; that when he saw the victim driving his car on P. Paredes Street he followed him
up to the corner of P. Paredes and Lepanto Streets where he shot the victim eight times suddenly and
without any warning, speaks eloquently of his plan, generated by an all consuming hatred, to kill the
person whom he considered responsible for all his misfortunes.

The lower Court did not, therefore, err in considering the aggravating circumstance of evident
premeditation against the appellant.

For all the foregoing, the guilt of the appellant has been established beyond reasonable doubt, with
two mitigating circumstances in his favor, that of plea of guilty and voluntary surrender. However
these are offset by the aggravating circumstances of evident premeditation and disregard of respect
due to the deceased. The crime of murder being punishable with reclusion temporal in its maximum
period to death (Art. 248, Revised Penal Code), the penalty, pursuant to Article 248 in relation to
Article 64 of the Revised Penal Code, should be, as it is hereby, imposed in its medium period,
or reclusion perpetua.

The penalty of death imposed by the trial court is hereby modified and reduced, as above indicated,
to reclusion perpetua with accessories of the law. Costs against the accused.

SO ORDERED.
CRIMINAL LAW 1
(Sunday, Section 52) 187

PEOPLE OF THE PHILIPPINES VS. BENITO


December 17, 1976 | Aquino, J.
Mitigating Circumstances: Vindication of a Wrong JEAF
DOCTRINE: Grave offense must be directed to the accused.

CASE SUMMARY:
When the victim remarked “Nagiistambay pala dito and magnanakaw," the accused got offended. In
the afternoon of the same day, he shot the victim to death. He claims that there’s mitigating
circumstance of vindication of a wrong. The OSG, however, said that it wasn’t specifically directed
to him. The SC also said that the 6-hour interval should have been sufficient for him to regain his
serenity.

FACTS:
Benito was a former employee of the Civil Service Commission at its main office and was assigned as
Clerk 2 in the Administrative Division from Nov. 1963 continuously up to Nov. 1965 when he was
suspended for "DISHONESTY"

After two months, he was reinstated but was criminally charged for QUALIFIED THEFT,
MALVERSATION OF PUBLIC FUNDS, ESTAFA and FALSIFICATION OF PUBLIC DOCUMENTS and
administratively charged for "DISHONESTY" culminating in his dismissal from the Civil Service on
February 1966.

October 21, 1965 the victim Moncayo, as an administrative officer, reported to the Commissioner of
Civil Service that Benito admitted having malversed an amount between P4,000 and P5,000 from his
sales of examination fee stamps.

At eleven o'clock in the morning of December 12, 1969 Moncayo, allegedly made upon seeing Benito
in the compound of the Civil Service Commission near the canteen: "Nagiistambay pala dito and
magnanakaw."; or, as Benito testified, Moncayo said: "Hindi ko alam na itong Civil Service pala ay
istambayan ng magnanakaw."

At about 5:25 p.m. of that same day, Dec. 12, 1969, the suspect shot the victim eight (8) times on the
head and different parts of the body at closer range which consequently caused the latter's death on
the spot inside his car.

Benito contends that there’s mitigating circumstance of vindication of a grave offense since Moncayo
insulted him when he remarked that a thief was loitering in the premises of the Civil Service
Commission.

NOTE: Benito was later on acquitted of the crime that Moncayo alleged he had committed.

ISSUE/RULING:
W/N the defamatory remark by the victim may give rise to the mitigating circumstance of vindication
of a wrong? NO.

OSG said that the defamatory remark was not specifically directed at Benito. SC said that even
assuming that Moncayo's remark was directed at Benito, Benito "had more than sufficient time to
suppress his emotion over said remark if he ever did resent it.” The six-hour interval between the
alleged grave offense committed by Moncayo against Benito and the assassination was more than
sufficient to enable Benito to recover his serenity. But instead of using that time to regain his
CRIMINAL LAW 1
(Sunday, Section 52) 188

composure, he evolved the plan of liquidating Moncayo after office hours.

Benito literally ambushed Moncayo just a few minutes after the victim had left the office. He acted
with treachery and evident premeditation in perpetrating the cold-blooded murder. Also, SC said that
the facts of the case strongly suggest that what really impelled Benito to assassinate Moncayo was
not the latter's alleged defamatory remark but the refusal of Moncayo to change his report so as to
favor Benito. Benito did not act primarily to vindicate an alleged grave offense to himself but mainly
to chastise Moncayo for having exposed the alleged anomalies or defraudation committed by Benito
and for obstinately refusing to change his report. Because according also to Benito’s testimony, he
saw Moncayo three hours later after the remark or at two o'clock in the afternoon and inquired from
him about his case and Moncayo said that he had already submitted his report and he could not do
anything more about Benito's case

DISPOSITION: SC denied his petition.


BACABAC VS PEOPLE
Sept 11, 2007 | CARPIO-MORALES
P: Ricardo Bacabac
R: People of the PH
TOPIC: Mitigating Circumstance – VINDICATION OF A WRONG

FACTS
RTC Convicted policeman Bacabac, Jose, Edzel, Jonathan, and Jesus of murder qualified by treachery.

Evening of Dec 23, 1990: 2 groups of people were at a dance hall in Purok 4, San Joaquin, Iloilo City.
Group 1: Hernani Quidato (victim) and companions Eduardo and Melchor
Group 2: Jonathan and Edzel
Also at the dance hall: Jesus

On their way home, the 2 groups encountered each other and had a misunderstanding.

Jesus, also on his way home, witnessed the commotion


Saw that Melchor "hugging" Edzel, and "tying" Jonathan "with his hands."
Also saw the victim hit Edzel with a "stick."
He told Group 1 that Edzel is the son of Councilor Jose Talanquines, Jr.
Eduardo told Jesus to go away for they might shoot him
Jesus thus left and went to Edzel's residence to report to his father what he had witnessed.

Edzel and Jonathan managed to flee.

Group 1 headed back home but they again encountered Group 2. This time, Group 2 was with their
uncle Ricardo Bacabac, Edzel's father, Jose, mother, and 2 sisters
Bacabac and Jose were carrying M-16 armalites, while Jonathan and Edzel were carrying a piece of
wood and a revolver, respectively

Jesus pointed to Group 1 and told Group 2 companions that they were the ones who manhandled
Jonathan and Edzel. The victim apologized, explaining that he and his companions mistook Jonathan
and Edzel for other persons.

Jesus blurted out: "You are just bragging that you are brave. You are only bullying small children."
THEN SHOOTING STARTED!
CRIMINAL LAW 1
(Sunday, Section 52) 189

Bacabac fired his armalite into the air. Jose also fired his armalite, but directed at Group 1, "as if
spraying his rifle from right to left"
He hit Jonathan in the thigh as he moved to strike the victim with a piece of wood.
Eduardo and the victim fell. As the victim was raising his hands in surrender, Jose shot him again

Melchor escaped. The victim, Eduardo, and Jonathan were brought to the hospital.
Victim pronounced DOA
Eduardo died 2 hours later

RTC Convicted Bacabac, Jose, Edzel, Jonathan, and Jesus of murder qualified by treachery
But there is a mitigating circumstance
Immediate vindication for Jose and Jesus
Voluntary surrender for Bacabac

Bacabac argues that there is no conspiracy to kill


He merely fired a warning shot into the air to respond to a public disturbance, to avert further acts
of violence, in pursuance of his duty as police officer to keep peace in the community
There was no unity of purpose and execution. Witnesses said after Jose fired, Bacabac merely stood
there, doing and saying nothing. Bacabac said this is because he was stunned by the events
Bacabac immediately reported the incident to their office
His action were not something a co-conspirator would do
RTC gave credence to an improbable and unnatural scenario where the men let their daughters and
wife become exposed to danger

ISSUES
W/N there was conspiracy and treachery between Bacabac and Jose’s company –YES
W/N the mitigating circumstance of immediate vindication absolves him of liability –NO

HELD/RATIO
There was conspiracy because
Bacabac’s failure to assist the victims after the shooting reinforces the conspiracy between him and
his co-accused to harm the victims. That it was he who first officially reported the shooting to the
police station does not make him any less a conspirator.
A conspirator who wants to free himself from criminal liability usually performs an overt act to
dissociate or detach himself from the felony while the commission of the felony is in progress.
He only reported the shooting after it had taken place
Voluntary surrender and non-flight do not conclusively prove innocence.

Once conspiracy is established, the act of one is the act of all even if not all actually hit and killed the
victim

There was also treachery because victim and companions had no chance to defend themselves →
were not armed, the attack was unexpected, and the victim already surrendered

Bacabac’s invocation of the mitigating circumstance of immediate vindication of a grave offense fails
because for it to be credited, the act should be committed in the immediate vindication of a grave
offense to the one committing the felony, his spouse, ascendants, descendants, legitimate, natural or
adopted brothers or sisters, or relatives by affinity within the same degree [RPC Article 13 (5)]
CRIMINAL LAW 1
(Sunday, Section 52) 190

The offense committed on Edzel was "hitting" his ear with a stick (according to Jesus), a bamboo pole
(according to Edzel).
By Edzel's own clarification, "he was hit at his ear, not on his head" → NOT A GRAVE OFFENSE
Edzel is petitioner's nephew, hence, not a relative by affinity "within the same degree"

PETITION DISMISSED

THE UNITED STATES vs. AUGUSTUS HICKS


G.R. No. 4971
September 23, 1909
Ponente: TORRES, J.

FACTS:
CFI- Moro Province convicted Augustus Hicks of murder and sentenced him to the penalty of death,
to be executed according to the law, to indemnify the heirs of the deceased in the sum of P1,000, and
to pay the costs. The case has been submitted to SC for review.

From September, 1902, to November, 1907, Augustus Hicks, an Afro-American, and Agustina Sola, a
Christian Moro woman, illicitly lived together in the municipality of Parang, Cotabato, Moro Province.
Trouble arose between them in the last-mentioned month of 1907. Agustina left Hick's house and
went to live with her brother-in-law, Luis Corrales. A few days later she contracted new relations
with another negro (LOL) named Corporal Wallace Current.

December 21, 1907 at about 7:30 p. m., - Augustus Hicks together with a soldier named Lloyd Nickens
called at said house, and from the sala called out to his old mistress who was in her room with
Corporal Current, and after conversing with her in the Moro dialect for a few minutes, asked the
corporal to come out of said room; in response thereto the corporal appeared at the door of the room,
and after a short conversation, Current approached Hicks and they shook hands, when Hicks asked
him the following question: "Did I not tell you to leave this woman alone?," to which Current replied:
"That is all right, she told me that she did not want to live with you any longer, but if she wishes, she
may quit me, and you can live with her." The accused then replied: "God damn, I have made up my
mind;" and as Corporal Current saw that Hicks, when, he said this, was drawing a revolver from his
trousers' pocket, he caught him by the hand, but the latter, snatching his hand roughly away, said:
"Don't do that," whereupon Current jumped into the room, hiding himself behind the partition, just
as Hicks drew his revolver and fired at Agustina Sola who was close by in the sala of the house. The
bullet struck her in the left side of the breast; she fell to the ground, and died in a little more than an
hour later.

Upon hearing the shot Edward Robinson, who was also in the house, went to render assistance and
wrested the weapon from the hand of the accused. Hicks immediately fled from the house and gave
himself up to the chief of police of the town, H. L. Martin, asking him to lock him up in jail; and, when
a few minutes later a policeman came running in and reported that Hicks had fired a shot at Agustina,
the said chief of police caused Hicks to be arrested. The latter, when once in jail, threw eight revolver
cartridges out of the window; these were picked up by a policeman who reported the occurrence and
delivered the cartridges to his chief.

HICKS’ VERSION:
When he (Hicks) withdrew his hand from that of Current, who had seized him, he fell backward but
managed to support himself on his two hands, and when he got up again Current threatened him with
CRIMINAL LAW 1
(Sunday, Section 52) 191

a revolver thrust into his face; whereupon he also drew his revolver, just as Edward Robinson caught
him from behind, when his revolver went off, the bullet striking Augustina.

ISSUE:
Whether or not judgment of should be affirmed (?)

HELD:
Yes.

RATIO:
The above-stated facts, which have been fully proven in the present case, constitute the crime of
murder, with two aggravating circumstances: treachery and premeditation (plus the fact that srime
was committed in the home of the deceased).
Treachery. Augustina was suddenly and roughly attacked and unexpectedly fired upon with a 45-
caliber revolver, at close, if not point blank range, while the injured woman was unarmed and
unprepared. It is logically inferred that means, manners, and forms were employed in the attack
directly and specially insured the consummation of the crime without such risk to Hicks as might
have been offered by the Augustina who, owing to the suddenness of the attack, was doubtless unable
to flee from the place where she was standing, or even escape or divert the weapon.
Premeditation.

1. According to the testimony of Charles Gatchery and Eugenio R. Whited, Hicks asked leave from the
former to be absent from the canteen where he was working on the morning of the day when the
affray occurred, alleging that his mind was unsettled and that he feared getting into trouble.
2. It is also shown by the fact that Whited, who was in Hicks' house about noon upon the latter's
invitation, and while both where drinking gin, and while the revolver, the instrument of the crime,
was lying on the table on which were also several loaded cartridges, heard the accused repeatedly
say, referring to the deceased, that her time had come, adding that he would rather see her dead than
in the arms of another man.

3. On the day after the crime the police found on a table in the cuprit's house several loaded
cartridges, a bottle of oil and a piece of cloth used undoubtedly for cleaning the revolver.
No mitigating circumstances present, not even loss of reason and self-control produced by jealousy
as alleged by the defense, inasmuch as the only causes which mitigate the criminal responsibility for
the loss of self-control are such as originate from legitimate feelings, not those which arise from
vicious, unworthy, and immoral passions.
Judgment affirmed. with costs, provided, however, that the death penalty shall be executed according
to the law in force, and that in the event of a pardon being granted, the culprit shall suffer the
accessory penalties of article 53 of the Penal Code unless the same be expressly remitted in the
pardon.
CRIMINAL LAW 1
(Sunday, Section 52) 192

US V DE LA CRUZ
Mitigating Circumstances – Passion or Obfuscation
Date: 29 March 1912
Ponente: Carson, J.

SUMMARY:
Defendant, in the heat of passion, killed his querida (concubine or lover) upon catching her red-
handed in the arms of another. Trial court convicted defendant of homicide and sentenced him to
14 years 8 months 1 day of reclusion temporal. Supreme Court found extenuating/mitigating
circumstances in the commission of the act of homicide therefore reducing defendant’s sentence to
12 years 1 day of reclusion temporal.

FACTS:
1. Defendant (De la Cruz), in the heat of passion, killed his querida when he caught her redhanded
in carnal communication with a mutual acquaintance.
2. Trial court found defendant guilty of homicide without any extenuating circumstances present.
Defendant was sentenced to 14 years 8 months 1 day of reculsion temporal (medium degree of
penalty prescribed by the code).

ISSUES:
1. WoN there is an extenuating/mitigating circumstance present - YES

HOLDING:
1. YES. There is an extenuating circumstance present in the case. The Court is of the opinion that the
defendant acted upon an impulse so powerful as naturally to have produced passion and obfuscation
when he caught his querida in carnal communication with a mutual acquaintance. The Court
mentioned the view taken by the Supreme Court of Spain regarding a case with similar state of facts:
A man who kills a woman (his lover) for having caught her in her underclothes with another man and
afterwards shoots himself inflicting a serious wound should be responsible for the act but with
extenuating circumstance considered because he acted as such due to strong emotion which impelled
him to perform the criminal act. The situation presents a sufficient impulse in the natural and
ordinary course to produce the violent passion and obfuscation which the law regards as a special
reason for extenuation. The Court also differentiated this case from US v Hicks:

• In the Hicks case, the cause of the alleged passion and obfuscation of the defendant was his vexation,
disappointment, and deliberate anger due to the woman’s refusal to live with him. The act of killing
was done with premeditation and preparation. Prior to the accomplishment of his criminal design,
he acted properly in front of his victim in order to mask his true intentions.
• In this case, the impulse upon which defendant acted and which naturally produced passion and
obfuscation was because of the sudden revelation that she was untrue to him and his Related
Provisions: Subsection 7, Art. 9 (penal code imposed back then):

The following are extenuating or mitigating circumstances: Xxx That of having acted upon an impulse
so powerful as naturally to have produced passion and obfuscation discovery of her in flagrante in
the arms of another. RULING: Modified by the finding that the commission of the crime was marked
with the extenuating circumstance set out in subsection 7 of article 9, and by the reduction of the
penalty of fourteen years eight months and one day of reclusion temporal to twelve years and one
day of reclusion temporal, the judgment of conviction and the sentence imposed by the trial court
should be and are hereby affirmed, with the costs of this instance against the appellant.
CRIMINAL LAW 1
(Sunday, Section 52) 193

PEOPLE V. GELAVER
G.R. No. 95357 June 9, 1993
MITIGATING CIRCUMSTANCES – PASSION / OBFUSCATION

Facts:
Eduardo Gelaver was charged with Parricide, killing his lawfully wedded wife, Victoria Pacinabao.
The prosecution presented Randy Mamon, who testified that at 7:00 a.m. of March 24, 1988, he heard
shouts coming from the house of Tessie Lampedario in Barangay Poblacion, Municipality of Sto. Niño,
South Cotabato. He saw Gelaver and a woman having a heated argument. Thereafter, Gelaver held
the neck of the victim, dragged her and with a knife on his right hand, stabbed the latter three times
on the breast. Gelaver then went out of the gate and fled in the direction of the public market of Sto
Niño.

Gelaver testified that he was married to Victoria Pacinabao, with whom he had four children. They
lived together at their conjugal home until July 3, l987 when she abandoned her family to live with
her paramour. He did not know his wife’s paramour or their whereabouts. He further testified that
on March 24, 1988, after he was informed by his daughter that his wife and paramour were living at
a house in front of the Sto. Niño Catholic Church, he immediately went to that place. Upon entering
the house, he saw his wife lying on her back and her paramour on top of her, having sexual
intercourse.

Eduardo Gelaver admitted killing his wife but claimed that he did so after catching her having carnal
act with her paramour. He narrated that when his wife saw him, she pushed her paramour aside. Her
paramour immediately stood up, took a knife placed on top of the bedside table and attacked him.
The latter was able to wrest possession of the knife and then used it against the paramour, who
evaded his thrusts by hiding behind the victim. Thus, it was the victim who received the stab intended
for the paramour. He continued to stab his wife because his mind had been "dimmed" or
overpowered by passion and obfuscation by the sight of his wife having carnal act with her paramour.

The RTC found him guilty, imposing the penalty of reclusion perpetua for the crime of Parricide with
mitigating circumstances of voluntary surrender and obfuscation. In which he appealed to the
Supreme Court and contends that the trial court should have imposed the penalty of destierro for
killing under exceptional circumstances pursuant to Article 247 of the Revised Penal Code.

Issue:
Whether or not the mitigating circumstance of passion and obfuscation is present.

Held:
Before Article 247 of RPC may be operative, that the death caused must be the proximate result of
the outrage overwhelming the accused after chancing upon his spouse in the act of infidelity. In this
case, Gelaver wants the court to believe that he caught his wife and her paramour in sexual
intercourse. However, his testimony is tainted with inconsistencies, suggesting otherwise.

As for the mitigating circumstance of obfuscation, the SC found that the trial court erred in finding
the presence of the mitigating circumstance of passion or obfuscation "as a result of his wife leaving
their home and their children." Before this circumstance may be taken into consideration, it is
CRIMINAL LAW 1
(Sunday, Section 52) 194

necessary to establish the existence of an unlawful act sufficient to produce such a condition of mind.
The act producing the obfuscation must not be far removed from the commission of the crime by a
considerable length of time, during which the accused might have recovered his equanimity. In this
case, the crime was committed almost a year after the victim had abandoned the conjugal dwelling.

RTC’s judgment was affirmed with modification.

PEOPLE V. BELLO
G.R. No. L-18792 February 28, 1964
THE PEOPLE OF THE PHILIPPINES, Plaintiff-Appellee, vs. GUILLERMO BELLO, Defendant-Appellant.

Facts:
Guillermo and Alicia lived together as husband and wife without the benefit of marriage. Guillermo
was a 54 year old widower, and Alicia’s senior by 30 years. Prior to Alicia’s employment at Maring’s
Place, the couple led a ‘blissful’ life. Due to poverty, Alicia became an entertainer/public hostess at
the said bar, and Guillermo used to watch her there everyday; very much smitten by her beauty.

However, on May 16, Guillermo saw Alicia enter the Gumaca theater with a man, and surprised the
man caressing her inside the movie house. Guillermo dragged her outside.
Two weeks later, Guillermo visited Maring’s Place to ask Alicia for money, but the owner, Maring
(who was fantastically creative enough to name the bar after him), told him to go home and to leave
Alicia alone because he was an old invalid. He proceeded to walk home empty handed, but upon
passing Bonifacio Street, he came across the Marasigan brothers who mocked him with the above
stated remark. The self-loathing Guillermo proceeded to Paty’s place and downed give glasses of
Tuba.

By nighttime of the same day, Guillermo returned to Maring’s Place and did then and there stab Alicia
several times. Realizing what he had done, he ran to Gumarca and surrendered to the police there.
He was found guilty by the Court of First Instance of Quezon City of murder attended by the
aggravating circumstances of nighttime, alevosia, and abuse of confidence and ungratefulness; The
penalty for which is death; thus the automatic review of the Supreme Court.

ISSUES:
Whether or not the victim should be given the benefit of the mitigating circumstance of passion or
obfuscation, albeit his relationship with the victim being merely a common-law marriage?

HELD:
Yes. By stare decisis, passion or obfuscation on the part of the offender must arise from legitimate
and moral sentiments. Since common-law marriages are considered unlawful in the Philippines,
Obfuscation, when relationship is illegitimate, cannot be appreciated as a mitigating circumstance.

To answer this question, we must first differentiate the circumstances of this case with that of U.S. v
Hicks. In the said case, the common-law wife of Mr. Hicks terminated her relations with the American,
and contracted new relations with a certain corporal. Mr. Hicks shot his ex-common-law-wife when
she refused to go home with him and resume their relationship. Since they were not married, she was
entitled to do so. What she did – cruel as it may be – was legal in the eyes of law. Passion and
obfuscation were not appreciated in such a case, since:
The common-law wife had a right to leave her common-law husband, as they were not united in holy
matrimony. He had no right to compel her to go with him. Remember that the first requirement of
CRIMINAL LAW 1
(Sunday, Section 52) 195

passion or obfuscation is that there be an unlawful act, sufficient to produce diminution of self-
control or the exercise of will power.

Returning now to the case at hand, what Guillermo was asking from Alicia was that she (1) quit her
job as a hostess; an ill-reputed profession corroborated by her promiscuous relations with other men,
and (2) resume her job as a hostess.

The Supreme Court held that Alicia’s flat out refusal was an exhibition of immorality itself. A
monogamous illegitimate relationship is definitely of higher standing than illicit relationship for the
sake of gain – or gainful promiscuity as the court put it.

This, coupled with the cruel words against him were enough to constitute passion and obfuscation in
the old soul. Thus, he is entitled to a mitigating circumstance. It should be noted too that fortunately,
the couple did not have a child.

PEOPLE OF THE PHILIPPINES, plaintiff-appellee, vs. WILLIE AMAGUIN, GILDO AMAGUIN AND
CELSO AMAGUIN, accused.

January 10, 1994


(Accused) brothers Willie, Gildo and Celso, all surnamed Amaguin, being charged with the murder of the
Oro brothers Pacifico and Diosdado.
Facts: On their way to the plaza, Pacifico (deceased) was called by accused Celso Amaguin. After the
refusal of the deceased, the accused, Celso, with a butcher's knife in hand, rushed towards Pacifico.
Gildo, Celso's younger brother, followed with a knife and slingshot.
Celso hacked Pacifico. Gildo then stabbed Diosdado with a knife. Thereafter, Willie, the eldest of the
Amaguin brothers, appeared with a handgun and successively shot the brothers Pacifico, Diosdado
and the fleeing Danilo.

Diosdado, own kneeling, gasping for breath and pleading for his life, was again shot by Willie who
next fired anew at Pacifico. Meanwhile, Gildo and Celso repeatedly stabbed Pacifico who already lying
prostrate and defenseless.

*Other survivors also gave their respective versions.

The defense however maintains that it was the Oro brothers who started the fight. Accused Gildo
Amaguin recounted that Pacifico with five others went to their house and approached his brother
Celso and there deceased together with his companions initiated a fight.

Nenita Amaguin, mother of the accused brothers, affirmed that her son Celso was indeed
troublesome, but added that Willie had no prior violations against the law.

After a joint trial, and finding the version of the prosecution to be more credible, the CIF of Iloilo
found Gildo Amaguin, guilty of murder and Willie Amaguin as accomplice.
Issue:
Whether or not the mitigating circumstance of voluntary surrender be appreciated in favor of the
accused?

Ruling:
Yes. While it may have taken both Willie and Gildo a week before turning themselves in, the fact is,
they voluntarily surrendered to the police authorities before arrest could be effected. For voluntary
CRIMINAL LAW 1
(Sunday, Section 52) 196

surrender to be appreciated as a mitigating circumstance, the following elements must be present:


(a) the offender has not been actually arrested; (b) the offender surrendered himself to a person in
authority; and (c) the surrender must be voluntary. All these requisites appear to have attended their
surrender.

THE PEOPLE OF THE PHILIPPINE ISLANDS, plaintiff-appellee,


vs.
FRANCISCO DE LA CRUZ, ET AL., defendants.
FRANCISCO DE LA CRUZ, appellant.
Marciano Sayoc for appellant.
Undersecretary of Justice Melencio for appellee.
G.R. No. L-45284 December 29, 1936

AVANCEÑA, C.J.:
This case was prosecuted upon the following information:
That on or about the 30th day of May, 1936, in the City of Manila, Philippine Islands, the said accused
Francisco de la Cruz, Fernando Legaspi and three other persons whose identities are still unknown,
confederating together and helping one another, did then and there willfully, unlawfully and
feloniously, and with intent of gain, attack, assault and use personal violence upon one Yu Wan, by
then and there giving him blows with his fist on the face and other parts of the body, thereby inflicting
upon him physical injuries which have required and will require medical attendance for a period of
more than one but less than nine days and have prevented and will prevent the said Yu Wan from
engaging in his customary labor for the same period of time; and afterwards took, stole and carried
away with him without the consent of the owner thereof the following personal property, to wit:

Twenty-six (P26) pesos in cash, consisting of different denominations ................ P26.00


belonging to said Yu Wan, to the damage and prejudice of the said owner in the said sum of P26,
Philippine currency.
That the said accused Francisco de la Cruz is a habitual delinquent under the provisions of the
Revised Penal Code, he having been previously convicted once of the crime of theft and twice of the
crime of estafa, by virtue of final judgments rendered by competent courts, having been last convicted
on July 24, 1933.

Upon arraignment, the accused pleaded not guilty.

During the trial and after two witnesses for the prosecution had testified, the accused withdrew their
plea of not guilty, substituting it by that of guilty. The court sentenced Francisco de la Cruz to six
months and one day of prision correccional and, considering him a habitual delinquent, sentenced
him furthermore to the additional penalty of six years and one day of prision mayor. The other
accused Fernando Legaspi was sentenced to ten months of prision correccional. Francisco de la Cruz
appealed for this sentence.

The facts charged constitute the crime of robbery defined in article 294 of the Revised Penal Code
and punished by the penalty of prision correccional to prision mayor in its medium period.
The allegations of the information with respect to the appellant Francisco de la Cruz are not sufficient
to consider him a habitual delinquent (People vs. Venus, p. 435, ante). However, the facts alleged in
this respect constitute the aggravating circumstance of recidivism.lawphi1.net
On the other hand, the appellant's plea of guilty does not constitute a mitigating circumstance under
article 13, subsection 7, of the Revised Penal Code, which requires that this plea be spontaneous and
CRIMINAL LAW 1
(Sunday, Section 52) 197

that it be made prior to the presentation of evidence by the prosecution. The confession of guilt,
although subsequent to the consummation of the crime and entirely alien to its development,
constitutes a cause for the mitigation of the penalty, not because it is a circumstance modifying
criminal responsibility already incurred and in the evolution of which it has not intervened
absolutely, but because, as an act of repentance and respect for the law, it indicates a moral
disposition in the accused favorable to his reform. It is clear that these benefits are not deserved by
the accused who submits to the law only after the presentation of some evidence for the prosecution,
believing that in the end the trial will result in his conviction by virtue thereof.

Wherefore, eliminating the additional penalty by reason of habitual delinquency, considering the
presence of an aggravating circumstance in the commission of the crime without any mitigating
circumstance, and applying the Indeterminate Sentence Law, the appellant is sentenced to the
penalty of from six months of arresto mayor, as minimum, to six years, ten months and one day
of prision mayor, as maximum, affirming the appealed sentence in all other respects, with the costs.
So ordered.

EXUPERANCIO CANTA, petitioner, vs. PEOPLE OF THE PHILIPPINES, respondent.


G.R. No. 140937. February 28, 2001

DECISION
MENDOZA, J.:
This is a petition for review on certiorari of the decision, dated August 31, 1999, and resolution, dated
November 22, 1999, of the Court of Appeals,[1] which affirmed the decision of the Regional Trial
Court, Branch 25, Maasin, Southern Leyte,[2] finding petitioner Exuperancio Canta guilty of violation
of P.D. No. 533, otherwise known as the Anti-Cattle Rustling Law of 1974, and sentencing him to ten
(10) years and one (1) day of prision mayor, as minimum, to twelve (12) years, five (5) months, and
eleven (11) days of reclusion temporal medium, as maximum, and to pay the costs.
The information against petitioner alleged:

That on or about March 14, 1986, in the municipality of Malitbog, province of Southern Leyte,
Philippines, and within the jurisdiction of this Honorable Court, the above-named accused with intent
to gain, did then and there, willfully, unlawfully and feloniously, take, steal and carry away one (1)
black female cow belonging to Narciso Gabriel valued at Three Thousand Pesos (P3,000.00) without
the knowledge and consent of the aforesaid owner, to his damage and prejudice in the amount
aforestated.

CONTRARY TO LAW.
The prosecution established the following facts:
Narciso Gabriel acquired from his half-sister Erlinda Monter a cow, subject of the case, upon its birth
on March 10, 1984. The cow remained under the care of Erlinda Monter for sometime. Subsequently,
Narciso gave the care and custody of the animal, first, to Generoso Cabonce, from October 24, 1984
to March 17, 1985; then to Maria Tura, from May 17, 1985 to March 2, 1986; and lastly, to Gardenio
Agapay, from March 3, 1986 until March 14, 1986 when it was lost.[4] It appears that at 5 oclock in
the afternoon of March 13, 1986, Agapay took the cow to graze in the mountain of Pilipogan in
Barangay Candatag, about 40 meters from his hut. However, when he came back for it at past 9 oclock
in the morning of March 14, 1986, Agapay found the cow gone. He found hoof prints which led to the
house of Filomeno Vallejos. He was told that petitioner Exuperancio Canta had taken the animal.
CRIMINAL LAW 1
(Sunday, Section 52) 198

Upon instructions of the owner, Gardenio and Maria Tura went to recover the animal from
petitioners wife, but they were informed that petitioner had delivered the cow to his father,
Florentino Canta, who was at that time barangay captain of Laca, Padre Burgos, Southern
Leyte. Accordingly, the two went to Florentinos house. On their way, they met petitioner who told
them that if Narciso was the owner, he should claim the cow himself. Nevertheless, petitioner
accompanied the two to his fathers house, where Maria recognized the cow. As petitioners father was
not in the house, petitioner told Gardenio and Maria he would call them the next day so that they
could talk the matter over with his father.

However, petitioner never called them. Hence, Narciso Gabriel reported the matter to the police of
Malitbog, Southern Leyte.[6] As a result, Narciso and petitioner Exuperancio were called to an
investigation. Petitioner admitted taking the cow but claimed that it was his and that it was lost on
December 3, 1985. He presented two certificates of ownership, one dated March 17, 1986 and
another dated February 27, 1985, to support his claim (Exh. B).

Narciso presented a certificate of ownership issued on March 9, 1986, signed by the municipal
treasurer, in which the cow was described as two years old and female. On the reverse side of the
certificate is the drawing of a cow with cowlicks in the middle of the forehead, between the ears, on
the right and left back, and at the base of the forelegs and hindlegs (Exhs. C, C-1 to 4).[8] All four
caretakers of the cow identified the cow as the same one they had taken care of, based on the location
of its cowlicks, its sex, and its color. Gardenio described the cow as black in color, with a small portion
of its abdomen containing a brownish cowlick, a cowlick in the middle of the forehead, another at the
back portion between the two ears, and four cowlicks located near the base of its forelegs and the
hindlegs.

On the other hand, petitioner claimed he acquired the animal under an agreement which he had with
Pat. Diosdado Villanueva, that petitioner take care of a female cow of Pat. Villanueva in consideration
for which petitioner would get a calf if the cow produced two offsprings. Petitioner claimed that the
cow in question was his share and that it was born on December 5, 1984.This cow, however, was lost
on December 2, 1985. Petitioner said he reported the loss to the police of Macrohon, Padre Burgos,
and Malitbog, on December 3, 1985 (Exh. A and Exh. 1).

Petitioner said that on March 14, 1986, his uncle Meno told him that he had seen the cow at Pilipogan,
under the care of Gardenio Agapay. He, therefore, went to Pilipogan with the mother cow on March
14, 1986 to see whether the cow would suckle the mother cow. As the cow did, petitioner took it with
him and brought it, together with the mother cow, to his father Florentino Canta.[11] Maria Tura tried
to get the cow, but Florentino refused to give it to her and instead told her to call Narciso so that they
could determine the ownership of the cow.[12] As Narciso did not come the following day, although
Maria did, Florentino said he told his son to take the cow to the Municipal Hall of Padre
Burgos. Petitioner did as he was told. Three days later, Florentino and Exuperancio were called to the
police station for investigation.

Petitioner presented a Certificate of Ownership of Large Cattle dated February 27, 1985[14] and a
statement executed by Franklin Telen, janitor at the treasurers office of the municipality of Padre
Burgos, to the effect that he issued a Certificate of Ownership of Large Cattle in the name of petitioner
Exuperancio Canta on February 27, 1985 (Exh. 5).[15] The statement was executed at the preliminary
investigation of the complaint filed by petitioner against Narciso.

Petitioners Certificate of Ownership was, however, denied by the municipal treasurer, who stated
that petitioner Exuperancio Canta had no Certificate of Ownership of Large Cattle in the municipality
CRIMINAL LAW 1
(Sunday, Section 52) 199

of Padre Burgos (Exhs. E, E-1 and 2).[17] On the other hand, Telen testified that he issued the
Certificate of Ownership of Large Cattle to petitioner on March 24, 1986 but, at the instance of
petitioner, he (Telen) antedated it to February 27, 1985.

On January 24, 1997, the trial court rendered its decision finding petitioner guilty of the offense
charged. In giving credence to the evidence for the prosecution, the trial court stated:

From the affidavits and testimonies of the complainant and his witnesses, it is indubitable that it was
accused Exuperancio Canta who actually took the cow away without the knowledge and consent of
either the owner/raiser/caretaker Gardenio Agapay.

That the taking of the cow by the accused was done with strategy and stealth considering that it was
made at the time when Gardenio Agapay was at his shelter-hut forty (40) meters away tethered to a
coconut tree but separated by a hill.

The accused in his defense tried to justify his taking away of the cow by claiming ownership. He,
however, failed to prove such ownership. Accused alleged that on February 27, 1985 he was issued
a Certificate of Ownership of Large Cattle (Exh. 2-A) for his cow by Franklin Telen, a janitor at the
Office of the Municipal Treasurer of Padre Burgos, a neighboring town. On rebuttal Franklin Telen
denied in Court the testimony of the accused and even categorically declared that it was only on
March 24, 1986 that the accused brought the cow to the Municipal Hall of Padre Burgos, when he
issued a Certificate of Ownership of Large Cattle for the cow, and not on February 27, 1985. Franklin
Telen testified thus:

Q. According to the defense, this Certificate of Ownership of Large Cattle was issued by you on
February 27, 1985. Is that correct?
A. Based on the request of Exuperancio, I antedated this.
(TSN, June 3, 1992, p. 7)
The testimony of Franklin Telen was confirmed in open court by no less than the Municipal Treasurer
of Padre Burgos, Mr. Feliciano Salva. (TSN, September 29, 1992, pp. 5-8).
If accused Exuperancio Canta were the owner of the cow in question, why would he lie on its
registration? And why would he have to ask Mr. Franklin Telen to antedate its registry? It is clear that
accused secured a Certificate of Ownership of Large Cattle (Exh. 2-A) by feigning and manipulation
(Exhs. A & B) only after the act complained of in the instant case was committed on March 14,
1986. His claim of ownership upon which he justifies his taking away of the cow has no leg to stand
on. Upon the other hand, the complainant has shown all the regular and necessary proofs of
ownership of the cow in question.
The Court of Appeals affirmed the trial courts decision and denied petitioners motion for
reconsideration. Hence, this petition. It is contended that the prosecution failed to prove beyond
reasonable doubt his criminal intent in taking the disputed cow.

First. Petitioner claims good faith and honest belief in taking the cow. He cites the following
circumstances to prove his claim:
1. He brought the mother cow to Pilipogan to see if the cow in question would suckle to the mother
cow, thus proving his ownership of it;
2. He compared the cowlicks of the subject cow to that indicated in the Certificate of Ownership of
Large Cattle issued on February 27, 1985 in his name, and found that they tally;
3. He immediately turned over the cow to the barangay captain, after taking it, and later to the police
authorities, after a dispute arose as to its ownership; and
4. He filed a criminal complaint against Narciso Gabriel for violation of P. D. No. 533.
CRIMINAL LAW 1
(Sunday, Section 52) 200

These contentions are without merit.


P.D. No. 533, 2(c) defines cattle-rustling as
. . . the taking away by any means, methods or scheme, without the consent of the owner/raiser, of
any of the abovementioned animals whether or not for profit or gain, or whether committed with or
without violence against or intimidation of any person or force upon things.
The crime is committed if the following elements concur: (1) a large cattle is taken; (2) it belongs to
another; (3) the taking is done without the consent of the owner; (4) the taking is done by any means,
methods or scheme; (5) the taking is with or without intent to gain; and (6) the taking is accomplished
with or without violence or intimidation against person or force upon things.[20]
These requisites are present in this case. First, there is no question that the cow belongs to Narciso
Gabriel. Petitioners only defense is that in taking the animal he acted in good faith and in the honest
belief that it was the cow which he had lost. Second, petitioner, without the consent of the owner,
took the cow from the custody of the caretaker, Gardenio Agapay, despite the fact that he knew all
along that the latter was holding the animal for the owner, Narciso. Third, petitioner falsified his
Certificate of Ownership of Large Cattle by asking Telen to antedate it prior to the taking to make it
appear that he owned the cow in question. Fourth, petitioner adopted means, methods, or schemes
to deprive Narciso of his possession of his cow, thus manifesting his intent to gain. Fifth, no violence
or intimidation against persons or force upon things attended the commission of the crime.

Indeed, the evidence shows that the Certificate of Ownership of Large Cattle which petitioner
presented to prove his ownership was falsified. Franklin Telen, the janitor in the municipal
treasurers office, admitted that he issued the certificate to petitioner 10 days after Narcisos cow had
been stolen. Although Telen has previously executed a sworn statement claiming that he issued the
certificate on February 27, 1985, he later admitted that he antedated it at the instance of petitioner
Exuperancio Canta, his friend, who assured him that the cow was his.

Telens testimony was corroborated by the certification of the municipal treasurer of Padre Burgos
that no registration in the name of petitioner was recorded in the municipal records. Thus,
petitioners claim that the cowlicks found on the cow tally with that indicated on the Certificate of
Ownership of Large Cattle has no value, as this same certificate was issued after the cow had been
taken by petitioner from Gardenio Agapay. Obviously, he had every opportunity to make sure that
the drawings on the certificate would tally with that existing on the cow in question.

The fact that petitioner took the cow to the barangay captain and later to the police authorities does
not prove his good faith. He had already committed the crime, and the barangay captain to whom he
delivered the cow after taking it from its owner is his own father. While the records show that he
filed on April 30, 1986 a criminal complaint against Narciso Gabriel, the complaint was dismissed
after it was shown that it was filed as a countercharge to a complaint earlier filed on April 16, 1986
against him by Narciso Gabriel.

Petitioner says that he brought a mother cow to see if the cow in question would suckle to the mother
cow. But cows frequently attempt to suckle to alien cows.[22] Hence, the fact that the cow suckled to
the mother cow brought by petitioner is not conclusive proof that it was the offspring of the mother
cow.

Second. Petitioner contends that even assuming that his Certificate of Ownership is not in order, it
does not necessarily follow that he did not believe in good faith that the cow was his. If it turned out
later that he was mistaken, he argues that he committed only a mistake of fact but he is not criminally
liable.
CRIMINAL LAW 1
(Sunday, Section 52) 201

Petitioners Certificate of Ownership is not only not in order. It is fraudulent, having been antedated
to make it appear it had been issued to him before he allegedly took the cow in question. That he
obtained such fraudulent certificate and made use of it negates his claim of good faith and honest
mistake. That he took the cow despite the fact that he knew it was in the custody of its caretaker
cannot save him from the consequences of his act.[23] As the Solicitor General states in his Comment:

If petitioner had been responsible and careful he would have first verified the identity and/or
ownership of the cow from either Narciso Gabriel or Gardenio Agapay, who is petitioners cousin
(TSN, 9/12/91, p. 26). Petitioner, however, did not do so despite the opportunity and instead rushed
to take the cow. Thus, even if petitioner had committed a mistake of fact he is not exempted from
criminal liability due to his negligence.

In any event, petitioner was not justified in taking the cow without the knowledge and permission of
its owner. If he thought it was the cow he had allegedly lost, he should have resorted to the court for
the settlement of his claim. Art. 433 of the Civil Code provides that The true owner must resort to
judicial process for the recovery of the property. What petitioner did in this case was to take the law
in his own hands.[25] He surreptitiously took the cow from the custody of the caretaker, Gardenio
Agapay, which act belies his claim of good faith.

For the foregoing reasons, we hold that the evidence fully supports the finding of both the trial court
and the Court of Appeals that accused-appellant is guilty as charged. There is therefore no reason to
disturb their findings.

However, the decision of the Court of Appeals should be modified in two respects.
First, accused-appellant should be given the benefit of the mitigating circumstance analogous to
voluntary surrender. The circumstance of voluntary surrender has the following elements:(1) the
offender has not actually been arrested; (2) the offender surrenders to a person in authority or to the
latters agent; and (3) the surrender is voluntary.[26] In the present case, petitioner Exuperancio Canta
had not actually been arrested. In fact, no complaint had yet been filed against him when he
surrendered the cow to the authorities. It has been repeatedly held that for surrender to be voluntary,
there must be an intent to submit oneself unconditionally to the authorities, showing an intention to
save the authorities the trouble and expense that his search and capture would require.[27] In
petitioners case, he voluntarily took the cow to the municipal hall of Padre Burgos to place it
unconditionally in the custody of the authorities and thus saved them the trouble of having to recover
the cow from him. This circumstance can be considered analogous to voluntary surrender and should
be considered in favor of petitioner.

Second, the trial court correctly found petitioner guilty of violation of 2(c) of P. D. No. 533, otherwise
known as the Anti-Cattle Rustling Law of 1974. However, it erred in imposing the penalty of 10 years
and 1 day of prision mayor, as minimum, to 12 years, 5 months and 11 days of reclusion
temporal medium, as maximum. The trial court apparently considered P. D. No. 533 as a special law
and applied 1 of the Indeterminate Sentence Law, which provides that if the offense is punished by
any other law, the court shall sentence the accused to an indeterminate sentence, the maximum term
of which shall not exceed the maximum fixed by said law and the minimum shall not be less than the
minimum term prescribed by the same. However, as held in People v. Macatanda,[28] P. D. No. 533 is
not a special law. The penalty for its violation is in terms of the classification and duration of penalties
prescribed in the Revised Penal Code, thus indicating that the intent of the lawmaker was to amend
the Revised Penal Code with respect to the offense of theft of large cattle. In fact, 10 of the law
provides:
CRIMINAL LAW 1
(Sunday, Section 52) 202

The provisions of Articles 309 and 310 of Act No. 3815, otherwise known as the Revised Penal Code,
as amended, pertinent provisions of the Revised Administrative Code, as amended, all laws, decrees,
orders, instructions, rules and regulations which are inconsistent with this Decree are hereby
repealed or modified accordingly.

There being one mitigating circumstance and no aggravating circumstance in the commission of the
crime, the penalty to be imposed in this case should be fixed in its minimum period.Applying the
Indeterminate Sentence Law, in relation to Art. 64 of the Revised Penal Code, petitioner should be
sentenced to an indeterminate penalty, the minimum of which is within the range of the penalty next
lower in degree, i. e., prision correccional maximum to prision mayor medium, and the maximum of
which is prision mayor in its maximum period.
WHEREFORE, the decision of the Court of Appeals is AFFIRMED, with the modification that petitioner
Exuperancio Canta is hereby SENTENCED to suffer a prison term of four (4) years and two (2) months
of prision correccional maximum, as minimum, to ten (10) years and one (1) day of prision
mayor maximum, as maximum. SO ORDERED.

PEOPLE VS LEGASPI
PEOPLE VS CAPALAC
PEOPLE VS GAPASIN
PEOPLE VS TIONGSON

PEOPLE VS. MAGDUENO


Topic. Crime committed with treachery
Case. Review of RTC decision charging Magdueño of murder with AC of treachery

Facts:
The facts have been established following the testimonies of three eyewitnesses. According to them,
Fiscal Dilig had just sat in the driver seat of his car, parked outside his house. They heard a shout
saying “Fiscal” then two successive gun shots from a guy. As soon as the shooting ended, the guy ran,
still holding his gun. At trial, the witnesses identified the shooter as Magdueño who was a stranger in
the area.

Magdueño said that he acted because of the reward posed by two other people. It was also found that
he carried an unlicensed gun. RTC found Magdueño guilty of murder with AC of treachery.

Issue:
Is AC of treachery present? -Yes

Ratio:
Treachery is present because the situation met the two requisites of treachery. As per law, for there
to be treachery, the attacker must: (1) deliberately and consciously adopt means and ways of
committing the crime, and; (2) have insured its execution without risk to himself. In the instant case,
Magdueño deliberately adopted means of carrying out the crime as he had an unlicensed gun with
him. This is proof that the crime was planned. More than that, Magdueño shot Dilig when Dilig had
no chance to protect himself or make ilag. There was no risk on Magdueño because he shot Dilig many
times. These said, the AC of treachery applies.
CRIMINAL LAW 1
(Sunday, Section 52) 203

PEOPLE VS TAC-AN
February 26, 1990
G.R. 76338 -39

The last paragraph of Article 152 shows that while a teacher or professor of a public or
recognized private school is deemed to be a "person in authority," such teacher or professor is
so deemed only for purposes of application of Articles 148 (direct assault upon a person in
authority), and 151 (resistance and disobedience to a person in authority or the agents of such
person) of the Revised Penal Code. A teacher or professor of a public or recognized private
school cannot be regarded as a "public authority" within the meaning of paragraph 2 of Article
14 of the Revised Penal Code.

PEOPLE VS. DIAZ


G.R. No. 130210, December 8, 1999
PEOPLE OF THE PHILIPPINES, plaintiff-appellee,
vs.
RALPH VELEZ DIAZ alias “JIMBOY,” accused-appellant.

Facts:
On 11 April 1997, the Regional Trial Court found accused-appellant Ralph Velez Diaz guilty beyond
reasonable doubt of murder in relation to sexual abuse (sodomy) of an 11-year-old child by the name
of Francis Bart Fulache, attended by treachery (RA 7610). He was sentenced to death and ordered to
pay the heirs of the victim P50,000.00 as death indemnity, P250,000.00 as moral damages,
P100,000.00 as exemplary damages and P40,000.00 as reimbursement for funeral expenses.
The trial court was convinced that notwithstanding the exclusion of the extrajudicial confession of
accused-appellant and the absence of any eyewitness to the crime, there were enough pieces of
circumstantial evidence to support his conviction, to wit:
(a) the testimony of 10-year old Felbart that he saw his brother last alive in the company of accused-
appellant;
(b) the physical evidence of sexual abuse through sodomy committed against the victim;
(c) the exculpatory plea of insanity which only tended to negate liability but was an admission of
guilt;
(d) the reenactment of the crime by accused-appellant the details of which could not have been
known to anybody but himself; and,
(e) the fact that accused-appellant voluntarily confessed to the crime without any evidence of
coercion, duress or intimidation exerted upon him.

Issues:
1. Whether the trial court is correct in finding the accused-appellant guilty of murder.
2. Whether the trial court is correct in imposing upon him the supreme penalty of death.
3. Whether the accused-appellant is guilty of the crime on the grounds that he might have been insane
upon its commission and insanity constitutes a valid exempting circumstance.

Held:
1. Yes. The Supreme Court agrees with the RTC that the crime committed by the accused-appelant
was murder even in the absence of the qualifying circumstance of evident premeditation because
treachery and abuse of superior strength were present – either of which qualified the crime to
murder. The killing of Francis Bart must be deemed ipso facto qualified by treachery by reason of his
inherent defenselessness. Likewise, there is a clear case of abuse of superior strength although it is
understood to be absorbed in treachery.
CRIMINAL LAW 1
(Sunday, Section 52) 204

2. No. The Supreme Court agrees with accused-appellant that he should not be meted the supreme
penalty of death.
Records showed that the Information charged him only with murder qualified by treachery, abuse of
superior strength and evident premeditation. It failed to mention the commission of sexual abuse or
“sodomy” on the victim. The real nature of the criminal charge should be determined by the actual
recital of facts in the complaint or information, not by a mere reference to a law. Thus, even if there
is positive proof of sexual abuse accused-appellant cannot be convicted therefor as it was not so
alleged in the information.
For ignominy to be appreciated as an aggravating circumstance in the instant case, it must be shown
that the sexual assault on Francis Bart was done by accused-appellant to put the former to shame
before killing him. This is clearly not the case here for accused-appellant’s intention was shown to be
the commission of sexual abuse on the victim as an act of revenge for his similar experience as a child.
Surely, the killing was done to eliminate the only witness to his crime.
3. Yes.Parens patriae is Latin for “parent of the nation.” In law, it refers to the public policy power of
the state to intervene against an abusive or negligent parent, legal guardian or informal caretaker,
and to act as the parent of any child or individual who is in need of protection e.g., someone whose
mental capacity is impared.

The defense, invoking the doctrine of parens patriae, also appeals to this Court for the psychiatric
examination and evaluation of accused-appellant, emphasizing the fact that accused-appellant was
institutionalized twice within a considerable period in the National Center for Mental Health;
consequently, there is no certainty that he was sane when he committed the crime imputed to him.
Supreme Court did not grant the request. When accused-appellant was committed to the National
Center for Mental Health, he was not diagnosed as insane but was suffering from pedophilia. Thus,
there is no doubt in our mind that he was sane during his two-year confinement in the center,
pedophilia being dissimilar to insanity.

A defendant in a criminal case who interposes the defense of mental incapacity has the burden of
establishing that fact, i.e., he was insane at the very moment when the crime was committed. In the
instant case, the defense of insanity as an exempting circumstance was not established and did not
overcome the legal presumption that a person’s acts are of his own free will and intelligence. Thus,
the conviction of accused-appellant no doubt is in order.

WHEREFORE, the 11 April 1997 Decision of the RTC-Br. 15, Cebu City, is MODIFIED. Accused-appellant
Ralph Velez Diaz is found guilty beyond reasonable doubt of murder and sentenced to reclusion
perpetua instead of death. He is also ordered to pay the legal heirs of Francis Bart Fulache the amount
of P50,000.00 as death indemnity, P100,000.00 as moral damages, P25,000.00 as exemplary damages
and P40,000.00 as reimbursement for funeral expenses. Costs de oficio.

PEOPLE VS ARIZOBAL
PEOPLE VS DANIEL
CRIMINAL LAW 1
(Sunday, Section 52) 205

PEOPLE OF THE PHILIPPINES VS APOLONIO APDUHAN JR., ET AL


Citation: G.R. No. L-19491, August 30, 1968

Procedural History:

This is a review on the judgment convicting respondent to death and indemnifying the heirs of the
victim.

Facts:
Respondent along with five other persons entered the house of the spouses Miano, shooting
Geronimo Miano and Norberto Aton that killed both and took money amounting to Php 322.00
belonging to Geronimo Miano. Respondent pleaded not guilty initially and later after advise from
counsel Tirol, pleaded guilty. Judge Hipolito Alo informed respondent that the penalty imposed might
be death and respondent insisted on pleading guilty with the condition that he be sentenced to life
imprisonment instead of death. Respondent then desisted from his plea of guilt and having made it
on record, counsel Tirol conferred with him and later manifested that respondent will enter the plea
of guilty with the trial courts ascertainment that he was not forced into pleading guilty. The mitigating
circumstances alleged by respondent were 1) intoxication that was not corroborated; 2) voluntary
plea of guilty; and 3) Lack of intent to commit a grave so wrong that was withdrawn after prosecution
withdrew the fourth aggravating circumstance abuse of superior strength. The aggravating
circumstances alleged by the prosecution were 1) band; 2) dwelling; 3) nighttime; and 4) abuse of
superior strength that was withdrawn.

Issue:
Whether or not respondent’s voluntary plea of guilty is spontaneous and insistent.

Ratio Decidendi:
No. Respondents initial plea was not one of guilty and changed it with the condition that he be
sentenced to life imprisonment and not death since he will plead guilty. Respondent then desisted
from the plea of guilt and stated his plea of not guilty made in record, going back to plea of guilty only
after conferring with his counsel. The virtue of Judge Alo's efforts in ascertaining whether Apduhan
pleaded guilty with full knowledge of the significance and consequences of his act, recommends itself
to all trial judges who must refrain from accepting with alacrity an accused's plea of guilty, for while
justice demands a speedy administration, judges are duty bound to be extra solicitous in seeing to it
that when an accused pleads guilty he understands fully the meaning of his plea and the import of an
inevitable conviction. For failure to secure the required number of votes, the penalty of death cannot
be legally imposed. The penalty next lower in degree - reclusion Perpetua - should consequently be
imposed on the accused.

Holding:
The death sentence imposed upon Apolonio Apduhan, Jr. by the court a quo is reduced to reclusion
Perpetua, the judgment a quo is affirmed in all other respects.
CRIMINAL LAW 1
(Sunday, Section 52) 206

PEOPLE VS. MANDOLADO


G.R. No. L-51304-05 (1983)
Ponente: Guerrero

FACTS:

Martin Mandolado was sentenced to death for the shooting Nolasco Mendoza at different parts of his
body, causing the latter's instantaneous death. The incident happened on October 3, 1977 in Sultan
Kudarat, Maguindanao. In another criminal case, he was charged for the murder of Herminigildo
Fajardo Tenorio. Both criminal cases had the aggravating circumstances of treachery, evident
premeditation and use of superior strength.

• On October 3, 1977, Julian Ortillano, Martin Mandolado, Conrado Erinada, and Anacleto Simon,
draftees of the AFP, were passengers of a bus bound for Midsayap, South Cotabato. In a bus
terminal, they all met each other and decided to drink rum. Mandolado got drunk and went
inside a public market. When he returned, he grabbed his .30 caliber machine gun and started
firing. His companions tried to stop him, but he continued firing his gun.

• Conrado and Anacleto hailed and boarded a passing Ford Fiera. The others also followed and
boarded, and they forced the driver to being them to the crossing. Mandolado grabbed his
knife and tried to attack the driver, then he fired his gun at a speeding vehicle, hitting the right
side of the back of the driver's sister who was onboard the vehicle. At the crossing, a privately-
owned jeep, driven by Herminigildo, passed by. Nolasco Mendoza was on board that jeep
bound for Cotabato City. Conrado and Anacleto boarded the jeep, while the others ran after it
at shouted at Herminigildo to stop the vehicle. Thereafter, Mandolado and Ortillano also
boarded the jeep, but kept firing their guns. Herminigildo said that if they did not stop firing
their guns, he will "ram the jeep" into something.

• Upon learning that the jeep was bound of Cotabato City and not Pikit, North Cotabato, Mandolado
got mad and pointed his gun at Herminigildo. After alighting the vehicle, Mandolado fired his
machine gun at the jeep and hit Nolasco and Herminigildo. Mandolado and Ortillano were
able to escape, and they even watched a movie. It was only when they met a certain Sgt.
Villanueva that they were informed of being suspects in the Herminigildo's and Nolasco's
deaths. As such, Mandolado purchased 2 passenger tickets for Manila. Before they could
board the ship, however, they were apprehended by a team led by Lt. Licas, and were brought
to Pikit, North Cotabato for investigation.

All evidence pointed to the guilt of Ortillano and Mandolado. The sole assignment of error in the case
at bar is that trial court erred because their guilt was not proven beyond reasonable doubt in terms
of being actual perpetrators of the crime. They cited that the prosecution did not present any witness
who can prove that the empty shells recovered from the crime scene belonged to Mandolado's
machine gun.

ISSUES/HELD:

1. W/N the trial court failed to acknowledge Ortillano and Mandolado's drunkenness -YES

The drunkenness of both appellants were put into consideration as a mitigating circumstance.
Consequently, the Court decided to reduce their sentences.
CRIMINAL LAW 1
(Sunday, Section 52) 207

2. W/N the trial court correctly qualified the crime as murder - YES

It is murder because of the presence of the qualifying circumstance of treachery. There is treachery
when the offender commits any of the crimes against the person, employing means, methods or forms
in the execution which tend directly and specially to insure its execution.

3. W/N the trial court correctly found that the aggravating circumstances of (a) advantage was taken
of being a draftee in the Philippine army and (2) abuse of confidence or obvious ungratefulness were
present in the commission of the crime -NO

While it is true that they were draftees, there is no evidence that when they stopped the jeep, the
accused already intended to shoot the occupants of the vehicle. Simply stated, there was no proof
that they intended to take advantage of being a draftee in order to commit the crime. They also
cannot be charged with the aggravating circumstance of abuse of confidence because there was no
proof that the victims had a "relation of trust and confidence" with the accused. As stated in People
vs. Comendador, in order for abuse of confidence to be considered an aggravating circumstance, it is
necessary that "there exists a relation of trust and confidence between the accused and the one
against whom the crime was committed and the accused made use of such relationship to commit
the crime."

PEOPLE OF THE PHILIPPINES vs. ANTONIO GARCIA Y CABARSE alias "TONY MANOK" and
REYNALDO ARVISO V REBELLEZA alias "RENE BISUGO
G.R. No. L-30449
October 31, 1979

FACTS:
The legal verdict hinges on the testimony of the lone eyewitness for the prosecution, Mrs. Corazon
DioquinoPaterno, sister of the deceased, ApolonioDioquino, Jr.

Before the incident which gave rise to this case, Corazon's husband informed her that he saw
Apolonio engaged in a drinking spree with his gang in front of an establishment known as Bill's Place
at M. de la Cruz Street. Pasay City. Corazon surmised that her husband must have been painting the
town red ("nag good time") in that same place. Upon learning this information from her husband,
Corazon obtained permission to leave the house at 3:00 a.m. so she could fetch her brother. At that
time, she had not been aware that Apolonio was in Pasay City; she had been of the belief that he was
with his family in Pampanga. She went to fetch him because she wanted him to escape the untoward
influence of his gang. In explaining the rationale for her noctural mission, she employed in her sworn
statement the following language: "DahilitongsiJunior ay meronnakamingnabalitaannanaaakay ng
barkadaniyasapaggawa ng hindimabuti."

On her way, as she rounded the corner of P.C. Santos Street, Corazon saw her brother fleeing a group
of about seven persons, including the two accused, Antonio Garcia and Reynaldo Arviso. She
recognized the two accused because they wereformergangmates of her brother; in fact, she knew
them before the incident by their aliases of "Tony Manok" and "Rene Bisugo, " respectively.
Corazon saw that the chase was led by the two accused, with Antonio carrying a long sharp
instrument. When she ventured to look from where she was hiding, about 20 meters away, she saw
the group catch up with her brother and maltreat him. Some beat him with pieces of wood, others
boxed him. Immediately afterwards, the group scampered away in different directions. Antonio was
left behind. He was sitting astride the prostrate figure of Apolonio, stabbing the latter in the back with
CRIMINAL LAW 1
(Sunday, Section 52) 208

his long knife. Corazon was not able to observe where Antonio later fled, for she could hardly bear to
witness the scene.

When Corazon mustered the courage to approach her brother, she saw that he was bathed in a pool
of his own blood.

ISSUE:
WON nocturnity (nighttime) is an aggravating circumstances?

HELD:
YES. The offense took place at 3:00 o'clock in the morning. It may therefore be said that it was
committed at night, which covers the period from sunset to sunrise, according to the New Civil Code,
Article 13. Is this basis for finding that nocturnity is aggravating? The Revised Penal Code, Article 14,
provides that it is an aggravating circumstance when the crime is committed in the nighttime,
whenever nocturnity may facilitate the commission of the offense. There are two tests for nocturnity
as an aggravating circumstance: the objective test, under which nocturnity is aggravating because it
facilitates the commission of the offense; and the subjective test, under which nocturnity is
aggravating because it was purposely sought by the offender. These two tests should be applied in
the alternative.

In this case, the subjective test is not passed because there is no showing that the accused purposely
sought the cover of night time. Next, we proceed and apply the objective test, to determine whether
nocturnity facilitated the killing of the victim. A group of men were engaged in a drinking spree, in
the course of which one of them fled, chased by seven others. The criminal assault on the victim at
3:00 a.m. was invited by nocturnal cover, which handicapped the view of eyewitnesses and
encouraged impunity by persuading the malefactors that it would be difficult to determine their
Identity because of the darkness and the relative scarcity of people in the streets. There
circumstances combine to pass the objective test, and the Court found that nocturnity is aggravating
because it facilitated the commission of the offense. Nocturnity enticed those with the lust to kill to
follow their impulses with the false courage born out of the belief that they could not be readily
Identified.

PEOPLE OF THE PHILIPPINES vs. ARMANDO RODAS, ET AL.


G.R. NO. 75881
August 28, 2007

Facts:
On August 9, 1996, the victim Titing Asenda was at Milaub, Denoyan, Zamboanga del Norte, to help
his brother, Danilo Asenda, in the harvesting of the latter's corn. On the same day, at around 8:00 in
the evening, a benefit dance at Milaub was being held. Among those roaming in the vicinity of the
dance hall were Alberto Asonda and Ernie Anggot. They stopped and hung out near the fence to watch
the affair. Titing Asenda was standing near them. They saw Charlito Rodas, Armando Rodas, Jose
Rodas, Jr., and Jose Rodas, Sr. surround Titing Asenda. Suddenly, without a word, Charlito Rodas,
armed with a hunting knife, stabbed Titing at the back. Armando Rodas then clubbed Titing with a
chako hitting him at the left side of the nape. Thereafter, Jose Rodas, Sr. handed to Jose Rodas, Jr. a
bolo which the latter used in hacking Titing. Alberto Asonda and Ernie Anggot tried to help Titing but
Armando Rodas prevented them by pointing a gun at them. Titing died instantly from his injuries.
CRIMINAL LAW 1
(Sunday, Section 52) 209

The lower court convicted the accused-appellants of the crime of murder, which the Court of Appeals
affirmed in toto.

Issue:
Whether evident premeditation, nocturnity and abuse of superior strength attended the killing.

Held:
For evident premeditation to be appreciated, the following elements must be established: (1) the
time when the accused decided to commit the crime; (2) an overt act manifestly indicating that he
has clung to his determination; and (3) sufficient lapse of time between decision and execution to
allow the accused to reflect upon the consequences of his act. The essence of premeditation is that
the execution of the criminal act was preceded by cool thought and reflection upon the resolution to
carry out the criminal intent during a space of time sufficient to arrive at a calm judgment. In the case
at bar, the prosecution failed to show the presence of any of these elements.

The aggravating circumstance of nocturnity cannot be considered against appellants. This


circumstance is considered aggravating only when it facilitated the commission of the crime, or was
especially sought or taken advantage of by the accused for the purpose of impunity. The essence of
this aggravating circumstance is the obscurity afforded by, and not merely the chronological onset
of, nighttime.

The aggravating circumstance of abuse of superior strength attended the killing. There was glaring
disparity of strength between the victim and the four accused. The victim was unarmed while the
accused were armed with a hunting knife, chako and bolo. It is evident that the accused took
advantage of their combined strength to consummate the offense. This aggravating circumstance,
though, cannot be separately appreciated because it is absorbed in treachery.

Judgment is affirmed with modification.

PEOPLE VS DAMASO
[G.R. No. L-30116]
November 20, 1978

Facts:
Fausto Damaso, Victoriano Eugenio Lorenzo Alviar, and Bonifacio Espejo are convicted of robbery
with double homicide.
On November 21, 1959, 9:00pm, Donata Rebolledo and her son-in-law Victoriano de la Cruz heard
their dogs barking outside their house, and two armed men entered, pointed their weapons at them,
tied up Victoriano and covered him; they asked Donata for the whereabouts of her daughter Catalina
Sabado. Donata kept silent and blocked the door leading to her daughter's room but was promptly
pushed aside. She was ordered to open an "aparador" from which they took jewelry, clothing,
documents, and cutting instruments. The two men brought Catalina Sabado down from the house
and then asked where they could find the other daughter Susana Sabado who was then in her store
nearby. Thereafter, Donata heard the men opening the door to Susana's store, and Donata untied the
hands of Victoriano and asked him to go to the store to see if her daughters were there; both women
could not be found. It was only the following morning, in a sugar plantation, the women were found
already dead with wounds in several parts of their bodies.
Contention of the Accused: The accused contends that the crime was not attended by the aggravating
circumstances of armed band, treachery and uninhabited place.
CRIMINAL LAW 1
(Sunday, Section 52) 210

Issue:
Whether or not there was aggravating circumstance to aggravate criminal liability

Decision:
AC of band exists whenever more than three armed malefactors act together in the commission of an
offense. It was found that Damaso, Eugenio, Alviar, and Gregorio were armed during the commission
of the crime. In this case, the presence of an armed band is to be considered as a generic aggravating
circumstance (Art 14(6), RPC) inasmuch as the crime committed was that provided for and penalized
in Art 294(1) and not under Art 295, RPC.

Treachery is present if the victim is killed while bound in such a manner as to be deprived of the
opportunity to repel the attack or escape with any possibility of success. The fact that the bodies of
Catalina and Susana were found dead with their arms tied behind their backs as well as the admission
of Gregorio in his confession that he killed the sisters while their arms were held by Eugenio and
Damaso lead SC to believe the offense was done under treacherous circumstances.

The uninhabitedness of a place is determined not by the distance of the nearest house to the scene of
the crime, but whether or not in the place of commission, there was reasonable possibility of the
victim receiving some help. Considering that the killing was done during nighttime and the sugarcane
in the field was tall enough to obstruct the view of neighbors and passersby, there was no reasonable
possibility for the victims to receive any assistance. That the accused deliberately sought the solitude
of the place is clearly shown by the fact that they brought the victims to the sugarcane field although
they could have disposed of them right in the house of Donata Rebolledo where they were found.

Decision is affirmed; penalty is to be imposed in its maximum period with three AC found by the trial
court, to wit: commission by a band, done with treachery, and in an uninhabited place. There is
likewise the additional aggravating circumstance that the robbery was committed in the dwelling of
the victim.

PEOPLE v BALDERA
April 24, 1950 | REYES
Plaintiff: People of PH | Defendants: Pedro Baldera, Miguel Blay, Jose Dela Cruz, et al
TOPIC: Aggravating Circumstances → Personal Circumstances of the Offender → RECIDIVISM

FACTS
Pedro Baldera was found guilty of robbery in band with homicide and serious and less serious
physical injuries. He was sentenced to death.

Dec 23, 1947, 4am: A Casa Manila bus left Batangas, bound for Manila. It was held-up by a group of 5-
6 men, one of them was Baldera.

Baldera, who was armed with a .45 caliber pistol, fired a shot, followed by a hail of bullets from
different directions. After firing had ceased, Baldera got on the bus and took the passengers’ money,
threatening them with his gun. He got off the bus and ordered the bus to proceed.

The driver headed for the municipal building of San Juan and reported the incident to the authorities.
Wounded passengers were taken to the hospital. One passenger died the following day.

Shortly after the commission of the crime, Baldera was arrested in Batangas for theft of a radio. His
features matched descriptions by the passengers.
CRIMINAL LAW 1
(Sunday, Section 52) 211

He made a written confession, admitting his participation in the crime as the one who, armed with a
pistol, boarded the bus and though intimidation, took the passengers’ money.

Later on, testifying in his own defense, he denied participation, claiming he spent the night in a house
of prostitution in Batangas, where he was employed by the prostitutes for drawing water

4 people were prosecuted and tried for the crime. Case was dismissed for 2 of the accused, but
Baldera and Miguel Blay were found guilty.
Baldera: death
Blay: life imprisonment

ISSUES/HELD/RATIO
W/N Baldera’s confession is admissible—YES
Baldera’s Confession was allegedly made for him to be given protection from his co-accused, and to
use him as a government witness. But he later on said the confession was taken through force and
intimidation. The latter was not proved.

But his confession still stands. Where one of several codefendants turns state's evidence on a promise
of immunity, but later retracts and fails to keep his part of the agreement, his confession made under
such promise may then be used against him.

W/N Baldera’s alibi stands—NO


His prostitute alibi cannot stand against the clear and positive testimony of one of the passengers,
Ponciano Villena, who has not shown to have any motive for falsely testifying against him.

W/N the following aggravating circumstances are present:


Crime committed is robbery in band and robbery was perpetrated by attacking a vehicle—YES

The fact that there were more than 3 armed men in the group that held up the bus appears in
Baldera’s own confession and is established by the uncontradicted testimony of one of the
government witnesses.

But this is immaterial because in the crime of robbery with homicide it is not essential that the
robbery be in band, but it may count as an aggravation in the imposition of the penalty.

Even if it is not taken into account, there remains the other aggravating circumstance that the robbery
was perpetrated by attacking a vehicle, which is not offset by any mitigating circumstance.

W/N the lower court erred in holding against the accused the circumstance of recidivism by reason
of his previous conviction for theft—YES

RECIDIVISM by reason of Baldera’s previous conviction for theft—NO

A recidivist is one who, at the time of his trial for one crime, shall have been previously convicted by
final judgment of another crime also punishable in the RPC. (Art 14 RPC)

No recidivism because Baldera’s crime of theft was committed on or about December 30, 1947 while
the offense now charged took place 7 days before that date.
CRIMINAL LAW 1
(Sunday, Section 52) 212

SENTENCE REDUCED TO LIFE IMPRISONMENT

PEOPLE V MELENDREZ
G.R. No. L-39913, December 19, 1933
AVANCEÑA, C.J

The text of the information filed against Ricardo Melendrez y Nieto and Elias Martinez in this case,
reads as follows:
That on or about the 15th day of June, 1933, in the municipality of Pasay, Province of Rizal, Philippine
Islands, within two and one-half (2 ½) miles from the limits of the City of Manila and within the
jurisdiction of this court, the said accused conspiring together and helping each other willfully,
unlawfully and feloniously forcibly broke open the door of the store located at No. 85 Cementina,
Pasay, an inhabited house belonging to and occupied by Tin Bun Boc, and once inside the said store,
with intent of gain and without the consent of the owner thereof, took, stole and carried away
therefrom the following personal properties of the said Tin Buc Boc:
Money amounting to P30.26
One(1) Elgin watch, gold plated and a gold filled chain,
25.00
valued at
One(1) Chinese ring, signet solid gold, valued at 13.50
One(1) buntal hat, valued at 4.50
Nine (9) small packages of "Camel" cigarettes 1.35
Nine (9) small packages of "Chesterfield" cigarettes 1.26
Three (3) cans of Milkmaid, valued at .81

Total 76.68

to the damage and prejudice of the said Tin Buc Boc in the total sum of seventy-six pesos and sixty-
eight centavos (P76.68), Philippine currency.

That the accused Ricardo Melendrez y Nieto is a habitual delinquent, he having been previously
convicted by final judgment of competent courts twice of the crime of theft and once of the crime
of estafa and having been last convicted of the crime of estafa on September 3, 1932.On the date of
the trial of this case, Elias Martinez had not been yet apprehended, for which reason only the other
defendant Ricardo Melendrez y Nieto, who pleaded guilty to the charge, was arraigned. Whereupon,
the court found him guilty of the crime charged in the information and sentenced him to eight years
and one day of prision mayor, and to serve an additional penalty of six years and one day of prision
mayor for being a habitual delinquent. From this judgment Ricardo Melendrez y Nieto appealed.In
this instance, counsel for the appellant contends that lack of instruction on the part of the appellant
should be considered as a mitigating circumstance in the commission of the crime. However, aside
from the fact that this court has repeatedly held in its various decisions that lack of instruction cannot
be considered as a mitigating circumstance in crimes of robbery, the records of the case do not afford
any basis on which to judge the degree of instruction of the appellant inasmuch as no evidence was
taken relative thereto, he having pleaded guilty.However, the fact that the appellant pleaded guilty
upon arraignment is a mitigating circumstance which should be considered in his favor.On the other
hand, the fiscal contends that the aggravating circumstance of recidivism should be taken into
account against the appellant. This claim of the fiscal is in accordance with the judgment rendered by
this court in banc in the case of People vs. Aguinaldo (47 Phil., 728) while the old Penal Code was in
force. But the enforcement of the Revised Penal Code has resulted in a difference of opinion regarding
CRIMINAL LAW 1
(Sunday, Section 52) 213

this point on the part of the members of this court. For this reason, after reviewing all the decisions
affecting the matter, rendered by this court both in banc and in division, it is now held that the
aggravating circumstance of recidivism should be taken into account in imposing the principal
penalty in its corresponding degree, notwithstanding the fact that the defendant is also sentenced to
suffer an additional penalty as a habitual delinquent.The facts alleged in the information constitute
the crime of robbery committed without the use of arms in an inhabited house, the value of the
articles taken being less than P250. In accordance with article 299 of the Revised Penal Code, the
penalty prescribed for said crime is prision correccional in its medium degree. Inasmuch as there is a
concurrence therein of one mitigating and one aggravating circumstance, this penalty should be
imposed in its medium degree.

Wherefore, it being understood that the principal penalty imposed upon the appellant is two years,
eleven months and eleven days, the judgment appealed from is hereby affirmed, in all other respects
with costs. So ordered.

UNITED STATES V MANALINDE


14 Phil. 77 (1909)
Evident Premeditation

Facts:
Between 2 and 3 o’clock on the afternoon of the 19th of January, 1909, Juan Igual a Spaniard, seated
on a chair by the door of Sousa’s store in Cotabato, Moro Province suddenly received a wound on the
head delivered from behind and inflicted with a kris. Moro Manalinde was the aggressor. He also
attacked Choa, a Chinaman, who was passing along the street. He attacked Choa with the same
weapon inflicting a severe wound in the left shoulder, on account of which he fell to the ground.

The Moro then escaped by running away from the town. The accused pleaded guilty and confessed
that he had perpetrated the crime. He said he was ordered by Datto Mupuck to go juramentado in
Cotabato in order to kill somebody. He was to be rewarded with a pretty woman if he was able to
escape after his arrest. He further stated that he had no quarrel with the assaulted persons.

Issue:
Whether or not premeditation is present in the crime committed? – YES

Held:
It is unquestionable that by the means and form employed in the attack the violent death of the said
Chinaman was consummated with deceit and treachery (alevosia), one of the five qualifying
circumstances enumerated in the aforesaid article as calling for the greatest punishment.

In the commission of the crime of murder, the presence of aggravating circumstances should be taken
into consideration in that the promise of reward and premeditation are present. There was also an
evident premeditation in the crime committed because upon accepting the order and undertaking
the journey, he deliberately considered and carefully and thoughtfully meditated over the nature and
the consequences of the acts, which under orders received from the said datto. He was about to carry
our, and to that end provided himself with weapon, concealing it by wrapping it up, and started on a
journey of a day and a night for the sole purpose of taking the life of two unfortunate persons whom
he did not know, and with whom he had never had any trouble. The person having been deprived of
his life by deeds executed with deliberate intent, the crime is considered a premeditated one as the
rm and persistent intention of the accused from the moment, before said death, when he received the
order until the crime was committed is manifestly evident. Even though in a crime committed upon
CRIMINAL LAW 1
(Sunday, Section 52) 214

offer of money, reward or promise, premeditation is sometimes present, the latter not being inherent
in the former, and there existing no incompatibility between the two, premeditation can not
necessarily be considered as included merely because an offer of money, reward or promise was
made, for the latter might have existed without the former, the one being independent of the other.
In the present case there can be no doubt that after the crime was agreed upon by means of a promise
of reward, the criminal by his subsequent conduct showed a persistency and firm intent in his plan
to carry out the crime which he intentionally agreed to execute, it being immaterial whether Datto
Mupuck did or did not conceive the crime, once Manalinde obeyed the inducement and voluntarily
executed it.

PEOPLE VS ILAOA
CRIMINAL LAW 1
(Sunday, Section 52) 215
CRIMINAL LAW 1
(Sunday, Section 52) 216

PEOPLE v. BIBAT
GR No. 124319, 13 May 1998, 290 SCRA 27
QUISUMBING, J.:

FACTS: Bibat stabbed Del Rosario to death. He was found guilty of murder in the RTC. Bibat claims
that the aggravating circumstance of evident premeditation should not have been appreciated. Court
ruled that based on the testimonies of the witnesses, evident premeditation has been established.
Bibat and his companions planned the killing in the house of Rogelio Robles and in his presence. On
the day of the killing, another witness Nona Cinco overheard Bibat planning the killing with his
companion while waiting for her bettor. A few hours later, Cinco personally saw Bibat kill the victim
Lloyd Del Rosario. From the time Nona Cinco heard the plan to kill someone at 11:30 up to the killing
incident at 1:30 in the afternoon of the same day, there was a sufficient lapse of time for Bibat to
reflect on the consequences. Such is evidence of evident premeditation

ISSUE: Whether or not there was evident premeditation

RULING: The presence of the requisites of evident premeditation appears to have been thoroughly
and sufficiently established. The essence of premeditation is that the execution of the criminal act is
preceded by cool thought and reflection upon the resolution to carry out the criminal intent during
the space of time sufficient to arrive at a calm judgment. The determination or conception of the plan
to kill the victim could be deduced from the outward circumstances that happened on the fateful day
of October 14, 1992. Records show that from the time Nona Cinco heard the plan to kill someone at
11:30 up to the killing incident at 1:30 in the afternoon of the same day, there was a sufficient lapse
of time for appellant to reflect on the consequences of his dastardly act. As held in the case of People
CRIMINAL LAW 1
(Sunday, Section 52) 217

v. Dumdum [20] "the killing of the deceased was aggravated by evident premeditation, because the
accused conceived of the assault at least one hour before its perpetration." In the case under
examination, two hours had elapsed from the time appellant clung to his determination to kill the
victim up to the actual perpetration of the crime

PEOPLE v. BACABAC
GR No. 149372, 11 September 2007
CARPIO MORALES, J.:

FACTS: Following a heated argument in a dance hall which resulted in a brawl, Jose Talanquines, Jr.,
and Edzel Talanquines, herein referred to as Talanquines brothers, proceeded to confront their
enemies armed with guns. They were accompanied by Jonathan Bacabac, Pat. Ricardo Bacabac, and
Jesus Delfin. In the dance hall, they encountered Hernani Quidato and Eduardo Selibio. After a
physical confrontation, The Talanquines brothers shot Quidato and Selibio. Quidato and Selibio later
died from their wounds. The Talanquines brothers, together with Jonathan Bacabac, Pat. Ricardo
Bacabac, and Jesus Delfin was charged and found guilty of the crime of Murder. Ricardo Bacabac
appealed his conviction, contending that he cannot be deemed to be in conspiracy with the other
accused because he was not the one who pulled the trigger. He also alleged that even if he was
convicted of Murder, in gratis argumenti, the correctness of the pronouncement of guilt should have
been attended by the mitigating circumstance of immediate vindication of a grave offense, in the
same manner as the other accused.

ISSUE: Whether or not there is conspiracy among Jonathan Bacabac, Pat. Ricardo Bacabac, and Jesus
Delfin in the murder of the victims

RULING: Bacabac‘s failure to assist the victims after the shooting reinforces this Court’s appreciation
of community of design between him and his co-accused to harm the victims. What is decisive in
treachery is that “the attack was executed in such a manner as to make it impossible for the victim to
retaliate.” In the case at bar, petitioner, a policeman, and his co-accused were armed with two M-16
armalites and a revolver. The victim and his companions were not armed. The attack was sudden and
unexpected, and the victim was already kneeling in surrender when he was shot the second time.
Clearly, the victim and his companion Eduardo had no chance to defend themselves or retaliate.
Conspiracy presupposes the existence of evident premeditation does not necessarily imply that the
converse ─ that evident premeditation presupposes the existence of a conspiracy ─ is true. In any
event, a link between conspiracy and evident premeditation is presumed only where the conspiracy
is directly established and not where conspiracy is only implied, as in the present case.

PEOPLE v. DUCABAO
GR No. 175594, 28 September 2007
CHICO-NAZARIO, J.:

FACTS: On October 24, 2000, at around 5:45 in the morning, Rolando, the brother of the victim, saw
Junjun Ducabao, the appellant walking back and forth, twice, in front of their house in Las Piñas City.
At around 6:18 in the morning, while he was sweeping inside their house, his brother Rogelio, the
victim, went out in front of their house also to sweep. While the victim was sweeping in a stooping
position at almost 90 degrees, the appellant suddenly appeared behind the victim, poked a gun and
shot the victim once at the back of his head, a little higher on his nape, causing the latter to fall on the
ground. Thereafter, Rolando immediately called for help from their neighbors to bring the victim to
the hospital. Unfortunately, the victim was pronounced dead on arrival at the Perpetual Help Hospital
in Las Piñas City. The victim died of hemorrhagic shock as a result of a gunshot wound on his head.
CRIMINAL LAW 1
(Sunday, Section 52) 218

ISSUE: Whether or not the lack of motive in killing the victim can prevail over the positive
identification made by the prosecution witness

RULING: In the crime of murder, motive is not an element of the offense. Motive becomes material
only when the evidence is circumstantial or inconclusive, and there is some doubt on whether a crime
has been committed or whether the accused has committed it. Hence, whether or not appellant had
any motive in killing the victim, his conviction may still follow from the positive and categorical
identification made by the witness. In the case at bar, Rolando’s positive identification of the
appellant as the perpetrator of the crime charged was categorical and consistent. Also, there was no
indication that he was improperly motivated when he testified against the appellant. As a rule, absent
any evidence showing any reason or motive for prosecution witnesses to perjure, the logical
conclusion is that no such improper motive exists, and their testimonies are thus worthy of full faith
and credit. The Court had previously said that aside from its intrinsic weakness, the defense of denial
cannot prevail over the positive identification made by the prosecution witness who had no improper
motive whatsoever to falsely testify against the accused. Between the self-serving testimony of the
accused and the positive identification by the prosecution witness, the latter deserves greater
credence. Hence, Junjun Ducabao is guilty of murder, qualified by treachery, for the wrongful death
of the victim with a penalty imposed of reclusion perpetua.

PEOPLE v. EMPACIS
GR No. 95756, 14 May 1993, 222 SCRA 59
NARVASA, C.J.:

FACTS: At about 9PM of Sept 16, 1986, as victim Fidel Saromines and his wife Camila were about to
close their small store in Cebu, Romualdo Langomez and Crisologo Empacis, came and asked to buy
some sardines and rice. After they finished eating, Langomez told Fidel to sell him some cigarettes.
He then announced a hold-up and ordered Fidel to give up his money. The latter started to hand him
PhP12,000.00 but suddenly decided to fight to keep it. A struggle followed in the course of which
Langomez stabbed Fidel about three times. Empacis joined in and with his own knife also stabbed
Fidel. At this time, gunshots were heard outside the house. It was only when Peter, Fidel’s 3-yr old
son, saw his father fighting for his life and rushed to his father’s defense with a pinuti (a long bolo)
striking Empacis and inflicting wounds on him did the two men flee. Fidel died from the fatal injuries,
which penetrated his lungs and heart. Empacis went to the clinic of Dr Eustaquio for the treatment
of his wounds inflicted by Peter. He told the doctor that he was assaulted without warning by a young
man near the Papan Market. The next day, the police officers arrested Empacis while Langomez
disappeared and could not be found.

ISSUE: Whether or not the aggravating circumstance of craft, fraud or disguise be appreciated in the
case

RULING: The aggravating circumstance of craft or fraud was properly appreciated against Empacis.
He and Romualdo pretended to be bona fide customers of the victim’s store and on his pretext gained
entry into the latter’s store and later, into another part of his dwelling. The Court finds the accused
Crisologo Empacis guilty of robbery with homicide as defined and penalized under Article 294 (1) of
the Revised Penal Code, and considering the attendance of the four generic aggravating
circumstances of dwelling, nighttime, craft or fraud and superior strength, not offset by any
mitigating or extenuating circumstance, hereby sentences the said accused Crisologo Empacis to the
supreme penalty of death.
CRIMINAL LAW 1
(Sunday, Section 52) 219

PEOPLE v. BIGCAS
GR No. 94534, 2 July 1992, 211 SCRA 631
REGALADO, J.:

FACTS: On July 28, 1988, Rodrigo Bigcas was at the store of a certain Efren Butron at Buyong, Pilar
which later accompanied by QuilianoButron. A few minutes later, AmbrocioPalapar, who was
apparently already intoxicated, arrived and drank 'tuba' with the group of Bigcas and Butron. Palapar
requested for more drinks but QuilianoButron refused as he had no more money. Palapar got angry
and called Butron stingy. He challenged Butron to fight but the latter remonstrated with him. Palapar
then placed his hand on Butron's shoulders and told him not to worry. He thereafter held the waist
of Butron, grabbed the knife that was hanging from the latter's waist and challenged everybody to
fight. Around fifteen minutes after Palapar had left, Bigcas and Butron left together with some other
persons. Butron reported to the police that he was waylaid by Palapar on his way home. In his
testimony which states that “on his way home, he saw Palapar standing in the middle of the road. He
greeted Palapar by his nickname "Boyax" but received no answer. As he was passing by Palapar, the
latter suddenly stabbed him with a bolo, hitting his stomach. He backtracked but the victim followed
him and gave him three stab thrusts which he parried. He was able to take hold of the victim's hand
holding the bolo and wrestled the same from him. Butron then repeatedly stabbed Palapar until the
latter fell. But according to two eye-witnesses RositoDoydoy and Jesus Calape, on July 25, 1988, which
was not so dark then as the moon was shining brightly while on their way, saw the victim Palapar
being attacked by the two appellants. Butron hit Palapar twice with a piece of wood at his back. Bigcas
told the victim to fight but the latter refused. Palapar pleaded for his life but appellant Bigcas instead
stabbed him twice, also at the back.

ISSUE: Whether or not self defense can be imposed to disclaim criminal liability

RULING: Yes, pursuant to paragraph 1 of Article 11 of the Revised Penal Code which states that
“Anyone who acts in defense of his person or rights, provided that the following circumstances
concur; First. Unlawful aggression. Second, Reasonable necessity of the means employed to prevent
or repel it. Third, lack of sufficient provocation on the part of the person defending himself. The
theory of self-defense is based on the necessity on the part of the person attacked to prevent or repel
the unlawful aggression. When said danger or risk ceased to exist, appellants had no justification in
law or in fact to attack the erstwhile aggressor. But in the case, self-defense is not credible where the
accused had succeeded in disarming the victim of the piece of wood which the latter was carrying,
hence the act of the accused in thereafter stabbing the victim with frequency, frenzy and force can no
longer be considered as reasonably necessary. Therefore, Bigcas disclaiming of criminal liability for
purposes of self-defense is unavailing and cannot be justified to disclaim criminal liability.

PEOPLE v. SANGALANG
L-32914, 30 August 1974, 58 SCRA 737
AQUINO, J.:

FACTS: At around 6am of June 9, 1968, Ricardo Cortez left his nipa hut located at Sitio Adlas, Barrio
Biluso, Silang, Cavite to gather tuba from a coconut tree nearby. Flora Sarno, his wife, was left inside
the hut. While he was on top of the tree gathering tuba, he was struck by a volley of shots. He fell to
the ground at the base of the coconut tree. Meanwhile, while inside their hut, Flora, wife of Ricardo,
heard three successive shots coming from the south of their hut. She went out, and from a distance
of about 25 meters saw five men armed with long firearms firing at her husband who was already
CRIMINAL LAW 1
(Sunday, Section 52) 220

lying on the ground at the foot of the coconut tree. She saw that the five men were about 5 meters
away from him. Looking at the five men, Flora recognized one of them as Laureano Sangalang, a
childhood acquaintance, and the others as Conrado Gonzales, Irineo Canuel, Perino Canuel and
Eleuterio Cuyom. Flora ran towards them and shouted why they were shooting her husband. As she
continued to approach them, the five men fired shots at Flora. Because of this she retreated to their
hut for cover. She heard more shots were fired, and after about 5minutes, the five men left the place.
Another witness to the event was Ricardo Sarno, brother of Flora Cortez, who lived in a hut about
20meters away from the hut of the Cortez’s. When the incident happened he and his family were
having breakfast, when he suddenly heard shots. He came out of his hut and saw his brother-in-law
being shot by the five men he also knew as Laureano Sangalang, Conrado Gonzales, Irineo Canuel,
Perino Canuel, and Eleuterio Cuyom. He saw Sangalang using a grand carbine to shoot Ricardo Cortez,
as he was on top of the coconut tree, and he also saw him fall to the ground and being fired at again
while lying helpless on the ground. He saw his sister attempting to approach the five men, and he saw
them fire shots at her. He wanted to join her, but he was also fired upon by the five men, so he
retreated to his own hut. When the five men left, Ricardo went to Flora’s hut and saw her sitting in
the hut. He followed her as she returned to the spot where her husband was shot; they both saw that
he was already dead. Noticing this, Ricardo he gathered his family and brought them to Sitio Biga,
and reported the killing to the chief of police who went to the scene of the crime with some policemen
and Constabulary men. The necropsy report of Ricardo Cortez showed that he sustained 23 gunshot
wounds on different parts of his body. Fourteen of these wounds were bullet entry wounds, while
the other nine were bullet exit wounds. Cause of death-multiple gunshot wounds. Sangalang’s alibi is
that in the afternoon of June 8, 1968 he and Crispulo Mendoza went to the house of Julian Gatdula at
Dapitan Street, Sampaloc, Manila. He arrived at Gatdula's place at six o’clock. He wanted to borrow
money from Gatdula to defray the matriculation fees of his children. As Gatdula had no money at that
time, he advised Sangalang to wait until morning. He would try to raise the sum of two hundred pesos
which Sangalang desired to borrow. Sangalang and Mendoza agreed. They allegedly slept in Gatdula's
house on the night of June 8th. The next morning, they breakfasted in that house. At about ten o'clock
on June 9, Gatdula delivered the two hundred pesos to Sangalang. He and Mendoza then went to the
Central Market in Manila and then to Quiapo. They returned to Cavite and arrived at seven o'clock in
the evening of June 9 in Barrio Capdula. Gatdula and Mendoza corroborated Sangalang's alibi.

ISSUES:
1. In criminal proceedings, which was given more weight, positive identification of the
siblings or the alibi of Sangalang?

2. What aggravating circumstance did the court identify and use as basis for the penalty of
reclusion perpetua?

RULING:
1. The trial court rejected appellant's alibi. It noted that although his witnesses, Mendoza
and Gatdula, learned of his arrest, and Mendoza even visited him in the municipal jail,
Sangalang and his witnesses did not interpose the defense of alibi when he was
investigated by the police and when he was summoned at the preliminary investigation.
Sangalang points to certain discrepancies in the declarations of Mrs. Cortez and her
brother Ricardo Sarno. The discrepancies may be attributed to deficiencies in observation
and recollection, or misapprehension of the misleading and confusing questions during
cross-examination or to the defective translation of the questions and answers but they
do not necessarily indicate a willful attempt to commit falsehood. The controlling fact is
that Mrs. Cortez and Sarno clearly and consistently testified that they saw Sangalang, a
CRIMINAL LAW 1
(Sunday, Section 52) 221

person already well-known to them, among the five armed persons who shot Ricardo
Cortez. That unwavering identification negates appellant's alibi.

2. The Court identified the killing of Ricardo Cortez to have the qualifying aggravating
circumstance of treachery or alevosia. This was manifested when the victim was shot
while gathering tuba on top of a coconut tree. He was unarmed, defenseless, and unaware
of any impending assault, and there was no provocation from him. The deliberate
surprise attack of Sangalang and his companions is a mode of execution which insured
the killing of Ricardo Cortez without any risk towards them, which could have arisen, has
Ricardo Cortez been able to defend himself.

PEOPLE v. SAN PEDRO


L-44274, 22 January 1980, 95 SCRA 306

FACTS: On June 2, 1970 between the barrios of Masaya and Paciano Rizal Municipality of Bay, Laguna,
Felimon Rivera, a jeepney driver, was killed. He died of profuse hemorrhage due to 23 lacerated and
stab wounds and multiple abrasions found on the different parts of the body. Four days prior to the
date of crime, Luisito, Artemio, and others planned to steal the jeep of the victim. They then rented
Rivera's jeep to haul coconuts (with him as driver), where they proceeded to Brgy. Puypuy in Bay,
Laguna. They were joined by Salvador Litan and Rodrigo Esguerra. At Esguerra's signal, Litan hit
Rivera at the nape with a water pipe. Rivera jumped out of the jeep but was chased by San Pedro and
Litan who stabbed him at the back several times with a dagger. Esguerra then drove the jeep and the
group proceeded to Makati, Rizal, where he was joined by Nelson Piso and Antonio Borja. The jeep
was sold for Php 2,000.00 at Cavite. Piso then went to Los Baños after four days and gave San Pedro,
Litan and Banasihan Php 50.00 each. On June 11, 1971, police caught Rodrigo Esguerra. He then
admitted his participation and named his companions. Artemio Banasihan was apprehended in 1972.

ISSUE:

1. Whether or not the aggravating circumstance of “Craft” can be joined as one with
“Treachery”

2. Whether or not the aggravating circumstances could be offset by the mitigating


circumstance of “Lack of Instruction”

RULING:
1. With the presence of two aggravating circumstances, craft and treachery, it would make
no difference even if the mitigating circumstance of lack of instruction were appreciated
in appellant's favor - which is even doubtful from the fact alone, as was allegedly proven
by the testimony of appellant that he cannot read and write but can only sign his name.
The "criteria in determining lack of instruction is not illiteracy alone, but rather lack of
sufficient intelligence." The trial court did not consider the mitigating circumstance
invoked because from appellant's testimony, and even more so from his given occupation
as a merchant, his alleged lack of intelligence never suggested itself to the trial court or to
his lawyer, as entitling him to the mitigating circumstance of lack of instruction. This,
apart from the fact that as held categorically in the case of People vs. Enot, 6 SCRA 325
(1962) lack of instruction is not applicable to crimes of theft and robbery, much less to
the crime of homicide. Treason is that robbery and killing are, by their nature, wrongful
CRIMINAL LAW 1
(Sunday, Section 52) 222

acts, and are manifestly so to the enlightened, equally as to the ignorant (People vs. Salip
Manla, et al., 30 SCRA 389 [1969]).

2. No. Treachery is when the victim did not have a chance to defend himself at the time of
the crime. Craft on the other had is intellectual trickery to lure the victim. There was
treachery because the criminals attacked the victim in surprise and he was unarmed.
There was craft because the criminals tricked the victim to bring them (and the jeep) to a
secluded area. Therefore, these two aggravating circumstances are separate for this case
and cannot be combined into one. With the presence of two aggravating circumstances,
the single mitigating

PEOPLE v. CASTILLO
GR No. 120282, 20 April 1998, 289 SCRA 213
PANGANIBAN, J.:

FACTS: Appellant was charged with murder in connection with the fatal stabbing of Antonio
Dometita in Cola Pubhouse in Quezon City. He pleaded not guilty and interposed the defense of denial
and alibi claiming that he was then asleep in his house at the time of the incident. Prosecution witness
Velasco testified that he was sitting outside the pubhouse when appellant suddenly arrived and
stabbed the victim on the left side of the chest causing his death. Another prosecution witness,
Mercado, testified that although she did not see the actual stabbing, she saw appellant wrapping a
bladed weapon in his shirt. However, defense witness Marcelino, a tricycle driver, testified that he
was about 25 meters away from the crime scene when he saw 2 persons ganging up on a person who
was later identified as the victim, and that appellant was not one of them. The trial court gave full
credence to the testimonies of the two prosecution witnesses and rendered judgment of conviction
with penalty of reclusion perpetua. Hence, this recourse. Appellant is questioning the credibility of
the prosecution witnesses and the partiality of the trial judge in favor of the prosecution as shown by
his participation in the examination of witnesses. One of the errors raised by appellant was that the
trial court in many instances showed its prejudice against the accused and that the trial judge took
over from the prosecution and asked questions in a leading manner, interrupted the cross-
examination to help the witness give answers favorable to the prosecution, and asked questions
which pertained to matters of opinion and allusions of bad moral character, which could not be
objected to by defense, counsel, because they have been ventilated by the judge himself.

ISSUE: Whether or not the judge showed partiality by asking questions during the trial

RULING: No. The trial court judge is not an idle arbiter during a trial. The impartiality of a judge
cannot be assailed on the mere ground that he asked such questions during the trial. The allegation
of bias and prejudice is not well-taken. It is a judge's prerogative and duty to ask clarificatory
questions to ferret out the truth. On the whole, the Court finds that the questions propounded by the
judge were merely clarificatory in nature. Questions which merely clear up dubious points and bring
out additional relevant evidence are within judicial prerogative. The propriety of a judge's queries is
determined not necessarily by their quantity but by their quality and, in any event, by the test of
whether the defendant was prejudiced by such questioning. In this case, appellant failed to
demonstrate that he was prejudiced by the questions propounded by the trial judge. In fact, even if
all such questions and the answers thereto were eliminated, appellant would still be convicted. There
is no showing of any interest, personal or otherwise, of the judge over the prosecution of the case. He
is therefore, presumed to have acted regularly and in the manner that preserves the ideal of the 'cold
neutrality of an impartial judge' implicit in the guarantee of due process.
CRIMINAL LAW 1
(Sunday, Section 52) 223

PEOPLE VS ARIZOBAL

PEOPLE OF THE PHILIPPINES, plaintiff-appellee, vs. JUAN GONZALES ESCOTE, JR. @ Jun
Mantika of Sta. Lucia, Angat, Bulacan and VICTOR ACUYAN y OCHOVILLOS @ Vic Arroyo of
Sto. Nio, Poblacion, Bustos, Bulacan, accused-appellants.

DECISION
CALLEJO, SR., J.:
Robbery with homicide is classified as a crime against property. Nevertheless, treachery is a generic
aggravating circumstance in said crime if the victim of homicide is killed treacherously. The Supreme
Court of Spain so ruled. So does the Court rule in this case, as it had done for decades.
Before the Court on automatic review is the Decision[1] of Branch 11 of the Regional Trial Court of
Bulacan in Criminal Case No. 443-M-97 convicting accused-appellants Juan Gonzales Escote, Jr. and
Victor Acuyan of the complex crime of robbery with homicide, meting on each of them the supreme
penalty of death, and ordering them to pay the heirs of the victim, SPO1 Jose C. Manio, Jr., the total
amount of P300,000.00 by way of actual and moral damages and to pay to Five Star Bus, Inc., the
amount of P6,000.00 by way of actual damages.

The Facts
The antecedent facts as established by the prosecution are as follows:
On September 28, 1996 at past midnight, Rodolfo Cacatian, the regular driver of Five Star Passenger
Bus bearing Plate No. ABS-793, drove the bus from its terminal at Pasay City to its destination in
Bolinao, Pangasinan. Also on board was Romulo Digap, the regular conductor of the bus, as well as
some passengers. At Camachile, Balintawak, six passengers boarded the bus, including Victor Acuyan
and Juan Gonzales Escote, Jr. who were wearing maong pants, rubber shoes, hats and jackets. [2] Juan
seated himself on the third seat near the aisle, in the middle row of the passengers seats, while Victor
CRIMINAL LAW 1
(Sunday, Section 52) 224

stood by the door in the mid-portion of the bus beside Romulo. Another passenger, SPO1 Jose C.
Manio, Jr., a resident of Angeles City, was seated at the rear portion of the bus on his way home to
Angeles City.Tucked on his waist was his service gun bearing Serial Number 769806. Every now and
then, Rodolfo looked at the side view mirror as well as the rear view and center mirrors installed
atop the drivers seat to monitor any incoming and overtaking vehicles and to observe the passengers
of the bus.

The lights of the bus were on even as some of the passengers slept. When the bus was travelling along
the highway in Plaridel, Bulacan, Juan and Victor suddenly stood up, whipped out their handguns and
announced a holdup. Petrified, Rodolfo glanced at the center mirror towards the passengers seat and
saw Juan and Victor armed with handguns. Juan fired his gun upward to awaken and scare off the
passengers. Victor followed suit and fired his gun upward. Juan and Victor then accosted the
passengers and divested them of their money and valuables. Juan divested Romulo of the fares he
had collected from the passengers. The felons then went to the place Manio, Jr. was seated and
demanded that he show them his identification card and wallet. Manio, Jr. brought out his
identification card bearing No. 00898.[3] Juan and Victor took the identification card of the police
officer as well as his service gun and told him: Pasensya ka na Pare, papatayin ka namin, baril mo rin
and papatay sa iyo. The police officer pleaded for mercy: Pare maawa ka sa akin. May pamilya
ako. However, Victor and Juan ignored the plea of the police officer and shot him on the mouth, right
ear, chest and right side of his body. Manio, Jr. sustained six entrance wounds. He fell to the floor of
the bus. Victor and Juan then moved towards the driver Rodolfo, seated themselves beside him and
ordered the latter to maintain the speed of the bus.Rodolfo heard one of the felons saying: Ganyan
lang ang pumatay ng tao. Parang pumapatay ng manok. The other said: Ayos na naman tayo
pare. Malaki-laki ito. Victor and Juan further told Rodolfo that after they (Victor and Juan) shall have
alighted from the bus, he (Rodolfo) should continue driving the bus and not report the incident along
the way. The robbers assured Rodolfo that if the latter will follow their instructions, he will not be
harmed. Victor and Juan ordered Rodolfo to stop the bus along the overpass in Mexico, Pampanga
where they alighted from the bus. The robbery was over in 25 minutes.

When the bus reached Dau, Mabalacat, Pampanga, Rodolfo and Romulo forthwith reported the
incident to the police authorities. The cadaver of SPO1 Manio, Jr. was brought to the funeral parlor
where Dr. Alejandro D. Tolentino, the Municipal Health Officer of Mabalacat, Pampanga, performed
an autopsy on the cadaver of the police officer. The doctor prepared and signed an autopsy report
detailing the wounds sustained by the police officer and the cause of his death:
Body still flaccid (not in rigor mortis) bathed with his own blood. There were 6 entrance wounds and
6 exit wounds. All the entrance were located on his right side. An entrance (0.5 cm x 0.5 cm.) located
infront of the right ear exited at the left side just below the ear lobe. Another entrance through the
mouth exited at the back of the head fracturing the occiput with an opening of (1.5 cm x 2 cm). Blood
CSF and brain tissues came out. Another fatal bullet entered at the upper right cornea of the sternum,
entered the chest cavity pierced the heart and left lung and exited at the left axillary line. Severe
hemorrhage in the chest cavity came from the heart and left lung. The other 3 bullets entered the
right side and exited on the same side. One entrance at the top of the right shoulder exited at the
medial side of the right arm. The other entered above the right breast and exited at the right lateral
abdominal wall travelling below muscles and subcutaneous tissues without entering the
cavities. Lastly another bullet entered above the right iliac crest travelled superficially and exited
above the right inguinal line.

Cause of Death:
Shock, massive internal and external hemorrhage, complete brain destruction and injury to the heart
and left lung caused by multiple gunshot wounds.[4]
CRIMINAL LAW 1
(Sunday, Section 52) 225

Rodolfo and Romulo proceeded to the police station of Plaridel, Bulacan where they reported the
robbery and gave their respective sworn statements.[5] SPO1 Manio, Jr. was survived by his wife
Rosario Manio and their four young children. Rosario spent P20,000.00 for the coffin and P10,000.00
for the burial lot of the slain police officer.[6] Manio, Jr. was 38 years old when he died and had a gross
salary of P8,085.00 a month.[7]
Barely a month thereafter, or on October 25, 1996, at about midnight, SPO3 Romeo Meneses, the team
leader of Alert Team No. 1 of Tarlac Police Station, and PO3 Florante S. Ferrer were at the police
checkpoint along the national highway in Tarlac, Tarlac. At the time, the Bambang-Concepcion bridge
was closed to traffic and the police officers were tasked to divert traffic to the Sta. Rosa
road. Momentarily, a white colored taxi cab without any plate number on its front fender came to
view. Meneses stopped the cab and asked the driver, who turned out to be the accused Juan Gonzales
Escote, Jr., for his identification card. Juan told Meneses that he was a policeman and handed over to
Meneses the identification card of SPO1 Manio, Jr. and the money which Juan and Victor took from
Manio, Jr. during the heist on September 28, 1996.[8] Meneses became suspicious when he noted that
the identification card had already expired on March 16, 1995. He asked Juan if the latter had a new
pay slip. Juan could not produce any. He finally confessed to Meneses that he was not a
policeman. Meneses brought Juan to the police station. When police officers frisked Juan for any
deadly weapon, they found five live bullets of a 9 millimeter firearm in his pocket. The police officers
confiscated the ammunition. In the course of the investigation, Juan admitted to the police
investigators that he and Victor, alias Victor Arroyo, staged the robbery on board Five Star Bus and
are responsible for the death of SPO1 Manio, Jr. in Plaridel, Bulacan. Meneses and Ferrer executed
their joint affiavit of arrest of Juan.[9] Juan was subsequently turned over to the Plaridel Police Station
where Romulo identified him through the latters picture as one of those who robbed the passengers
of the Five Star Bus with Plate No. ABS-793 and killed SPO1 Manio, Jr. on September 28, 1996. In the
course of their investigation, the Plaridel Police Station Investigators learned that Victor was a native
of Laoang, Northern Samar.[10] On April 4, 1997, an Information charging Juan Gonzales Escote, Jr.
and Victor Acuyan with robbery with homicide was filed with the Regional Trial Court of Bulacan. The
Information reads:
That on or about the 28th day of September 1996, in the municipality of Plaridel, province of Bulacan,
Philippines, and within the jurisdiction of this Honorable Court, the above-named accused,
conspiring, confederating together and mutually helping each other, armed with firearms, did then
and there wilfully, unlawfully and feloniously, with intent of (sic) gain and by means of force, violence
and intimidation, take, rob and carry away with one (1) necklace and cash in [the] undetermine[d]
amount of one SPO1 Jose C. Manio, Jr., to the damage and prejudice of the said owner in the said
undetermine[d] amount; that simultaneously or on the occassion (sic) of said robbery, said accused
by means of violence and intimidation and in furtherance of their conspiracy attack, assault and shoot
with the service firearm of the said SPO1 Jose C. Manio, Jr., thereby inflicting serious physical injuries
which resulted (sic) the death of the said SPO1 Jose C. Manio, Jr.
Contrary to law.

On the strength of a warrant of arrest, the police officers arrested Victor in Laoang, Northern Samar
and had him incarcerated in the Bulacan Provincial Jail. Assisted by Atty. Ramiro Osorio, their
counsel de parte, Juan and Victor were duly arraigned and entered their plea of not guilty to the
charge. Trial thereafter ensued. After the prosecution had rested its case on August 26, 1998, Juan
escaped from the provincial jail.[12] The trial court issued a bench warrant on September 22, 1998 for
the arrest of said accused-appellant.[13] In the meantime, Victor adduced his evidence.

Victor denied the charge and interposed the defense of alibi. He testified that in 1996, he worked as
a tire man in the vulcanizing shop located in Banga I, Plaridel, Bulacan owned by Tony Boy Negro. On
one occasion, Ilarde Victorino, a customer of Tony Boy Negro, ordered Victor to sell a tire. Victor sold
CRIMINAL LAW 1
(Sunday, Section 52) 226

the tire but did not turn over the proceeds of the sale to Ilarde. The latter hated Victor for his misdeed.
The shop was later demolished and after two months of employment, Victor returned to Barangay
Muwal-Buwal, Laoang, Northern Samar. On September 26, 1996, at 9:30 p.m., Victor was at the town
fiesta in Laoang. Victor and his friends, Joseph Iringco and Rickey Lorcio were having a drinking spree
in the house of Barangay Captain Ike Baluya. At 11:30 p.m., the three left the house of the barangay
captain and attended the public dance at the town auditorium. Victor and his friends left the
auditorium at 5:30 a.m. of September 27, 1996. Victor likewise testified that he never met Juan until
his arrest and detention at the Bulacan Provincial Jail. One of the inmates in said provincial jail was
Ilarde Victorino. Victor learned that Ilarde implicated him for the robbery of the Five Star Bus and
the killing of SPO1 Manio, Jr. to hit back at him for his failure to turn over to Ilarde the proceeds of
the sale of the latters tire.

On January 14, 1999, Juan was rearrested in Daet, Camarines Norte.[14] However, he no longer
adduced any evidence in his behalf.

The Verdict of the Trial Court


On March 11, 1999, the trial court rendered its Decision judgment finding Juan and Victor guilty
beyond reasonable doubt of the crime charged, meted on each of them the penalty of death and
ordered them to pay P300,000.00 as actual and moral damages to the heirs of the victim and to pay
the Five Star Bus Company the amount of P6,000.00 as actual damages. The decretal portion of the
decision reads:

WHEREFORE, this Court finds both accused, Juan Gonzales Escote, Jr. and Victor Acuyan GUILTY
beyond reasonable doubt of Robbery with Homicide as penalized under Art. 294 of the Revised Penal
Code as amended and hereby sentences both to suffer the supreme penalty of Death and to indemnify
the heirs of the late SPO1 Jose C. Manio, Jr., the amount of P300,000.00 as actual and moral damages
and to pay the Five Star Bus P6,000.00 as actual damage.

PEOPLE OF THE PHILIPPINES, plaintiff-appellee, vs. REGANDO VILLONEZ y PASCASIO,


RUEL SANTOS y LAPADA, JOHN DOE, PETER DOE, ELMER DOE, and ROY DOE, accused, vs. EDUARDO
N. SANTOS @ "EDDIE," REYNALDO N. SANTOS @ "REY," FERNANDO N. SANTOS @ "DEDE,"
EMERLITO N. SANTOS @ "ELMER," and RUDY N. SANTOS @ "BUDDA," accused, REGANDO VILLONEZ
y PASCASIO, EMERLITO N. SANTOS, and RUEL SANTOS, accused-appellants.

DECISION
DAVIDE, JR., J.:
Accused-appellants REGANDO VILLONEZ,[1] RUEL SANTOS,[2] and EMERLITO SANTOS pray for a
reversal of their conviction for MURDER decreed in a Joint Decision[3] rendered on 23 November
1995 by the Regional Trial Court (RTC) of Malabon, Metro Manila, Branch 170, in Criminal Cases Nos.
14943-MN and 15506-MN.

The information[4] in Criminal Case No. 14943-MN charged REGANDO and RUEL with the crime of
murder allegedly committed in the following manner:

That on or about the 3rd day of May 1994 in Malabon, Metro Manila, and within the jurisdiction of this
Honorable Court, the above-named accused [Regando Villonez y Pascasio, Ruel Santos y Lapada, John
Doe, Peter Doe, Elmer Doe and Roy Doe], conspiring together and mutually helping one another,
without any justifiable cause, with deliberate intent to kill, with treachery, taking advantage of
superior strength, and being armed with bladed weapons, did then and there wilfully, unlawfully and
CRIMINAL LAW 1
(Sunday, Section 52) 227

feloniously attack, assault and stab one GERARDO LONGASA on the different parts of the body,
thereby inflicting upon the latter serious physical injuries, which caused his death.

CONTRARY TO LAW.
The case was assigned to Branch 170 of the RTC of Malabon, Metro Manila.
Upon arraignment, REGANDO and RUEL entered a plea of not guilty.[5]
Meanwhile, accused-appellant EMERLITO, together with Eduardo, Reynaldo, Fernando, and Rudy, all
surnamed Santos, was likewise charged with murder in an information,[6] which was later docketed
as Criminal Case No. 15506-MN. The crime was allegedly committed as follows:
That on or about the 3rd day of May, 1994, in the Municipality of Malabon, Metro Manila, Philippines
and within the jurisdiction of this Honorable Court, the above-named accused [Eduardo N. Santos @
Eddie, Reynaldo N. Santos @ Rey, Fernando N. Santos @ Dede, Emerlito N. Santos @ Elmer, and Rudy
N. Santos @ Budda], conspiring, confederating with Regando P. Villonez and Ruel Santos who were
already charged for the same crime under Criminal Case No. 14943 and without any justifiable cause,
with deliberate intent to kill, with treachery taking advantage of superior strength and being armed
with a [sic] bladed weapons, did then and there willfully, unlawfully and feloniously attack, assault
and stab one GERARDO LONGASA on the different parts of the body, thereby inflicting upon the latter
serious physical injuries, which caused his death.

CONTRARY TO LAW.
Only EMERLITO was arrested; his co-accused have remained at large.
Evidently, the killing involved in Criminal Case No. 14943-MN was the same as that in Criminal Case
No. 15506-MN. Hence, the second case was transferred from Branch 72 to Branch 170 of the RTC of
Malabon, Metro Manila, and was consolidated and jointly tried with the first case pursuant to the
Order[7] of 22 March 1995.

At his arraignment, EMERLITO entered a plea of not guilty.[8]


The witnesses for the prosecution were Edgar Jimenez and Dr. Ronaldo Mendez, a Medico-Legal
Officer of the National Bureau of Investigation; and the witnesses for the defense were the accused-
appellants, as well as Arthur Aquino and Conrado Gungon. Edgar Jimenez testified that on 3 May
1994, at around 9:00 p.m., while he was resting inside his store at Hulo, Malabon, Metro Manila, a
certain Tonton informed him that his close friend GERARDO LONGASA had a fistfight with one
Rudy, alias Dede,[9] at Liwayway Street, Baritan, Malabon. Edgar proceeded to the area to mediate,
since LONGASA and Rudy were both his friends. Edgar passed through Javier II Street in going to
Liwayway Street. At Javier II Street, a group of seven armed men, including accused-appellants,
attacked Edgar. RUEL hit Edgar on his forehead and back with a bottle. Edgar was able to escape from
his attackers. While fleeing, he ran past LONGASA, who seemed drunk. When Edgar called LONGASA,
the attackers were already upon LONGASA. While he was about eight arms length away from
LONGASA, Edgar saw EMERLITO hit LONGASA with a 2 x 2 inches piece of wood. Simultaneously,
REGANDO and RUEL struck LONGASA with bottles. Rudy Santos and Eddie Santos then stabbed
LONGASA seven and eight times, respectively, even as two other persons named Rey and Budda held
LONGASAs arms. LONGASA fell to the ground. Edgar saw all these because the scene of the incident
was illuminated by a big fluorescent lamp located about three arms length away. Edgar rushed to
LONGASAs house and reported the incident to the latters parents. Dr. Ronaldo Mendez conducted an
autopsy on LONGASAs corpse. His findings are as follows: Abrasions: 1.0 x 0.6 cms., forehead, left
side; 6.0 x 4.0 cms., zygomatic area, left; 3.6 x 1.1 cms., nasal area, right side; 4.0 x 2.0 cms., maxillary
area, left; 2.0 x 0.6 cms., infranasal area, left side; 0.5 x 0.4 cms., mandibular area, left side; 6.6 x 4.2
cms., left upper quadrant, abdomen, 5.0 x 3.0 cms., anterior aspect, upper third, arm, left; 2.5 x 1.3
cms., left lumbar area; 5.0 x 2.5 cms., elbow, left. Contusions, purplish: 7.4 x 6.2 cms., anterior chest
CRIMINAL LAW 1
(Sunday, Section 52) 228

wall, left side; 4.4 x 2.3 cms., anterior aspect, upper third, arm, right. Lacerated wounds: 1.1 cms.,
supraorbital ridge, right; 1.2 cms., posterior aspect, upper third, forearm, left.

Stab wounds:
1) 3 in number, sizes ranging from 0.5 to 1.6 cms., elliptical clean-cut edges, with one extremity sharp
and the other blunt, located at the anterior chest wall, left side, over an area of 15.0 cms., x 7.0 cms.,
the farthest is located 10.0 cms., from the anterior median line, while the nearest is located 3.0 cms.,
from the anterior median line, directed backwards, upwards, downwards and medially, involving the
soft tissues, perforating the right ventricle and penetrating the lower lobe of the left lung with an
average depth of approximately 7.5 cms.
2) 1.4 cms., elliptical, clean cut edges, with sharp medial extremity and blunt lateral extremity,
located at the posterior chest wall, left side, 7.0 cms., from the posterior median line, directed
forwards, upwards and laterally, involving the soft tissues only with an approximate depth of 3.0 cms.
3) 1.5 cms., elliptical, clean-cut with sharp medial extremity and blunt lateral extremity, located at
the posterior chest wall, left side, 9.0 cms., from the posterior median line, directed forwards,
upwards and medially, involving the soft tissues, from the 6th intercostal space, into the left thoracic
cavity penetrating the upper lobe of the left lung with an approximate depth 5.0 cms.
4) 1.6 cms., elliptical, clean cut edges, with sharp lateral extremity and blunt medial extremity,
located at the anterior chest wall, right side, 3.0 cms., from the posterior median line, directed
forwards, upwards and medially, involving the soft tissue only with an approximate depth of 2.0 cms.
Hemopericardum, 230 cc.
Hemothorax, left, 1095 cc.
Other visceral organs, pale.
Stomach is almost empty.
CAUSE OF DEATH:
STAB WOUNDS.
Dr. Mendez explained that the abrasions were caused by hard, rough surface, possibly cement or a
piece of wood. The contusions and lacerations were caused by a blunt object, which could have been
a piece of wood, a bottle, a pipe, or any other hard object. The incise wounds or stab wounds were
caused by a sharp-bladed or sharp-edged instrument. Of the six stab wounds suffered by LONGASA,
stab wounds numbered 1 and 3 on LONGASAs chest caused the latters death.

The testimony of LONGASAs mother was dispensed with after the State and the defense agreed that
Longasas family incurred P8,500 in funeral expenses.

REGANDO interposed alibi and denial. He claimed that on 3 May 1994, between 7:30 and 8:00 p.m.,
he was having a conversation with Arthur Aquino at the premises of RUELs house. Someone passed
by the house and reported a slaying incident at Javier II Street. Curious, REGANDO and Aquino went
to the scene of the incident and there found LONGASA lying in a pool of his own blood. REGANDO
recognized LONGASA because the latter was a barber at REGANDOs neighborhood. He believed that
the victim was already dead, since the latter did not seem to be breathing. When policemen arrived,
REGANDO moved away from the scene; he did not want to be asked about the incident, as he knew
nothing about it. On 7 May 1994, he was arrested by Malabon policemen after Edgar Jimenez
identified him as one of the assailants. He opined that Edgar implicated him in the crime because they
had an altercation during a basketball game, which altercation could have erupted into a fistfight had
they not been pacified. Arthur Aquino, REGANDOs gangmate, corroborated the latters testimony and
declared that it was impossible for REGANDO to have taken part in the killing, since he was with
REGANDO before and after the incident. When they arrived at Javier II, they saw many people, none
of whom were known to him. He asked the people milling around LONGASAs body who the killer
was, but no one could tell him. RUEL, who was 16 years old at the time the crime in question was
CRIMINAL LAW 1
(Sunday, Section 52) 229

committed, also put up the defense of alibi. According to him, when the incident was taking place he
was at his grandmothers house in Javier II changing clothes, for he had just taken a bath. He heard
screams from outside of the house reporting that a killing had occurred at the corner of Javier II
Street. Out of curiosity, he immediately went to the reported scene of the incident. There he saw a
bloodied body lying on the ground, which he later found to be LONGASAs cadaver. RUEL was not
questioned by the authorities during the investigation. However, on 7 May 1994, while he and co-
accused REGANDO were watching television at his grandmothers house, they were arrested by the
police on the basis of Edgars information that they were among LONGASAs assailants.[17]
EMERLITO also relied on alibi for his defense. He declared that at the time of the incident he was at
Javier II to borrow P500 from his mother. On his way to his mothers place, someone informed him
that his brother Fernando Santos, alias Dede, was involved in a fight at Liwayway Street. He rushed
to the scene of the reported fight. There, he found his brother being ganged up on by Edgar Jimenez
and another person. EMERLITO grabbed Edgar and boxed him, but the latter retaliated. They
exchanged punches until Edgar ran towards a nearby alley. EMERLITO gave chase but failed to catch
Edgar, as the latter jumped into a river. EMERLITO waited for Edgar to come up for air. After ten to
fifteen minutes, EMERLITO got impatient and went back to Liwayway Street. After seeing no one in
the area, he went to Javier II Street. Along the way he saw people running, and then someone
shouted: Mang Emer, iyong kapatid ninyo nakasaksak namatay (Mang Emer, your brother stabbed
and killed a man). Another person advised him not to proceed to the scene of the incident and to go
home instead. EMERLITO followed the advice.He did not take his brother to the authorities because
not one of his brothers was at home when he got there. Neither did he go to the police to explain the
incident, as he did not know much about it. Conrado Gungon attempted to bolster EMERLITOs
account by claiming that at the time of the incident, he saw Fernando Santos and a certain Rey chasing
LONGASA at Javier II. He followed the three to a corner near General Luna Street. There he saw
Fernando and Rey stab LONGASA; after which the assailants ran towards General Luna Street. The
two attackers had no other companion. Conrado went home after the incident. In its Joint Decision,
the trial court found Jimenezs testimony to be credible and supportive of the theory of conspiracy
among the accused. It found the following circumstances to be more than sufficient to prove that the
accused-appellants and their co-accused had common design to kill LONGASA and were united in its
execution: (1) their simultaneous acts and concerted effort in surrounding the victim; (2) all of them
carried weapons, which they used against the victim; (3) they took turns in disabling the victim with
blows administered with a piece of wood and bottles; (4) the victims arms were restrained when the
death blows were inflicted; (6) none of the accused-appellants tried to dissuade their companions
from delivering fatal wounds on the victim; as a matter of fact, they continued attacking the victim
until the latter was already down and gasping for breath; and (7) the number of wounds inflicted on
the victim was a mute testimony of the vengeful fury and brutality of the deadly attack upon
him. Conspiracy having been established, the act of one was the act of all.

The trial court ruled against the presence of treachery, since LONGASA was engaged in a fight with
the accused before the fatal attack and was, therefore, sufficiently warned of the assault against
him. However, it appreciated against the accused the qualifying aggravating circumstance of taking
advantage of superior strength because of the superior number of the accused, most of whom were
armed with weapons; while the victim was alone, with his arms held behind him by two of the
assailants.

The trial court rejected the defense of alibi for failure of accused-appellants to prove that they were
so far away from the scene of the crime as to be physically impossible for them to be there when the
crime was committed.

The trial court thus ruled that the crime committed was murder and decreed; thus:
CRIMINAL LAW 1
(Sunday, Section 52) 230

WHEREFORE, all considered, the Court finds all the three (3) accused GUILTY beyond reasonable
doubt of the crime of MURDER and sentences each of them as follows:
a) Accused REGANDO VILLONES y PASCASIO and EMERLITO N. SANTOS, there being no aggravating
and mitigating circumstance, to suffer the penalty of RECLUSION PERPETUA;
b) Accused RUEL N. SANTOS, appreciating the privileged mitigating circumstance of minority in his
favor, being 16 years old at the time of the commission of the offense, to suffer an indeterminate
penalty of TEN (10) YEARS of prision mayor as minimum, to SEVENTEEN (17) YEARS of reclusion
temporal as maximum.

Likewise, all the accused are hereby ordered to indemnify, jointly and severally, the heirs of
GERARDO LONGASA in the amount of P8,500.00 as actual damages, and the additional sum
of P50,000.00, as civil indemnity for the death of the said victim, and the costs of suit.
Accused-appellants REGANDO, RUEL, and EMERLITO seasonably appealed to us.[21]
In their Brief, accused-appellants REGANDO and EMERLITO, represented by the Public Attorney's
Office, contend that the trial court committed the following errors:

I
... IN GIVING FULL WEIGHT AND CREDENCE TO THE OTHERWISE UNCORROBORATED,
INCREDIBLE AND FABRICATED TESTIMONY OF PROSECUTION WITNESS EDGARDO JIMENEZ.
II
IN FINDING ACCUSED-APPELLANT REGANDO VILLONES GUILTY BEYOND REASONABLE
DOUBT OF THE CRIME CHARGED.
III
IN FINDING THAT THERE EXISTS CONSPIRACY IN THE CASE AT BAR.
In his separate Brief, accused-appellant RUEL imputes upon the trial court the following errors:
I.
IN NOT HOLDING THAT EDGAR JIMENEZ IS NOT COMPETENT TO TESTIFY ON THE PARTICIPATION
OF RUEL SANTOS CONSIDERING HE WAS NOT AN EYEWITNESS AND HIS TESTIMONY IS,
THEREFORE, HEARSAY.
II
IN HOLDING THAT ACCUSED RUEL SANTOS WAS IN CONSPIRACY AMONG THE OTHER ACCUSED IN
THE KILLING OF GERARDO LONGASA.
III.
... IN DISREGARDING THE TESTIMONY OF EMERLITO SANTOS.
IV.
IN DISREGARDING THE TESTIMONY OF CONRADO GUNGON.
All accused-appellants attack the credibility of lone eyewitness Edgar Jimenez. REGANDO and
EMERLITO wonder why Edgar took a longer route through Javier II Street to get to Liwayway Street
instead of just crossing the bridge that separated Duhat from Liwayway Street, which was shorter
route. Another thing which they find illogical was Edgars failure to shout for help and to do anything
to save his friend LONGASA. As to Edgars testimony that he was mauled by the accused, accused-
appellants theorize that Edgar could have been a participant in the rumble, which made him a biased
and unreliable witness. Moreover, Edgar could not have witnessed the crime, as he was then running
away from the scene to escape further injuries. Additionally, no one corroborated Edgars testimony;
hence, it is self-serving.

RUEL claims that Edgar Jimenez committed inconsistencies on material points, especially on who
actually stabbed LONGASA and how many times he was so stabbed. Edgar was, likewise, unable to
make up his mind whether he was running away from the crime scene or staying at a safe distance
from the incident. RUEL finds illogical and incredible the story of Edgar that accused attacked him
CRIMINAL LAW 1
(Sunday, Section 52) 231

when he was merely looking for LONGASA, and that while Edgar was originally the target of the
accuseds aggression, they inexplicably vented their ire on LONGASA and allowed Edgar to witness
everything and walk away untouched. RUEL also stresses that the prosecution did not disprove
EMERLITOs testimony that Edgar was in a river near Liwayway Street while the incident was taking
place; hence, he could not have witnessed the crime. Besides, Edgars testimony was disproved by
Gungon, who testified that Reynaldo and Fernando Santos were the ones who stabbed LONGASA, and
that the said assailants had no other companions.

Accused-appellants insist on the credibility of the testimony of the defense witnesses. They further
claim that they were able to explain their whereabouts during the perpetration of the offense, and
that they had other witnesses to corroborate their respective versions.

Finally, accused-appellants assert that there is no sufficient proof of conspiracy. The short interval
between the attack on Edgar and the attack on LONGASA precluded the existence of a preconceived
plan among the accused to so assault LONGASA. Additionally, if there was indeed conspiracy among
the accused, all should have stabbed LONGASA, not just that some of them hit the victim with bottles
or a piece of wood.
In the Consolidated Brief for the Appellee, the Office of the Solicitor General (OSG) maintains that the
alleged lapses in Edgar Jimenezs testimony were duly explained and the alleged inconsistencies were
too trivial to impair his straightforward account of the crime. His failure to help his friend while the
latter was under attack was understandable considering that the aggressors had the strength of
number. At any rate, the trial court found Edgar credible.It is well-settled that a trial courts
assessment of a witnesss testimony is entitled to great respect on appeal.

As to RUELs claim of lack of logic in the version of Edgar, the OSG argues that it is of judicial
knowledge that persons have been assaulted for no apparent reason whatsoever.[22]
The OSG considers Gungons testimony undeserving of consideration. Gungon failed to explain why
he gave his account of the crime only on 1 August 1995 or fifteen months after the incident in
question. His long unexplained silence makes one suspicious of his motives; hence his testimony is
unworthy of belief.

Finally, the OSG agrees with the trial courts rejection of accused-appellants defense of alibi and with
the finding of conspiracy. It argues that there was no showing of physical impossibility for the
accused to be at the crime scene when the crime was committed; besides, they were positively
identified by Edgar Jimenez as among LONGASAs attackers. As to conspiracy, the same can easily be
deduced from the manner of the commission of the offense and from the concerted acts of the accused
to obtain a criminal objective.

As often happens in criminal cases on appeal, we are asked to disregard the testimony of a
prosecution witness for being incredible, and to give full credence to those of the defense and decree
accused-appellants acquittal.Among the discrepant accounts of the same incident, we choose to
believe the one certified by the trial judge to be credible, in this case, the testimony of Edgar
Jimenez. The judge had the distinct advantage of having personally heard the testimonies of Edgar
and the witnesses for the defense, and observed their deportment and manner of testifying during
the trial. It is settled that the trial judges findings on the credibility of witnesses will not generally be
disturbed unless said findings are arbitrary, or facts and circumstances of weight and influence have
been overlooked, misunderstood, or misapplied by the trial judge which, if considered, would have
affected the result of the case.[25] None of the exceptions have been shown to exist in the instant case.
Indeed, our perusal of the transcript of the testimony of Edgar Jimenez confirms
his trustworthiness. He told a consistent story throughout his two turns at the witness stand. He
CRIMINAL LAW 1
(Sunday, Section 52) 232

corrected misimpressions by the trial judge and examining counsel, and he satisfactorily explained
the apparent lapses in his testimony. He was frank about his dark history as a drug user who was
once the subject of a criminal case in court. It must be noted that a criminal record does not
necessarily make one an incredible witness.[26] Edgars honesty in revealing his past without
hesitation bolsters his credibility.

The inconsistencies between Edgars testimony and sworn statement given to the police were
likewise adequately explained. In any case, a sworn statement or affidavit, being taken ex parte by a
person other than the witness, is almost always incomplete and often inaccurate, sometimes from
partial suggestion or for want of suggestions and inquiries. Omissions and misunderstandings by the
writer are not infrequent, particularly under circumstances of hurry and impatience. The infirmity of
affidavits as a species of evidence is a matter of judicial experience. As such, an affidavit taken ex-
parte is generally considered to be inferior to testimonies made in open court.

Furthermore, as the OSG correctly opined, the findings of medico-legal officer Dr. Ronaldo Mendez
served to corroborate Edgars testimony. LONGASAs injuries, recorded in Dr. Mendezs report,
reflected the severe beatings LONGASA suffered at the hands of the accused as narrated by Edgar.
The bromidic defense of alibi cannot benefit accused-appellants. In the face of the positive
identification of the accused by Edgar, such defense is worth nothing. Besides, accused-appellants
were unable to prove that it was physically impossible for them to be at the crime scene at the time
the crime was committed. On the contrary, REGANDO and RUEL admitted that in just a short time
they were able to get to the crime scene by walking. For his part, EMERLITO acknowledged his
involvement in a fight which preceded LONGASAs killing, and he conceded that he was able to return
to the crime scene, or near the crime scene, at or about the time of the commission of the offense. The
alibi which is sufficient to acquit an accused of a criminal charge must be that which shows it was
physically impossible for him to be at the crime scene at the time of the commission of the crime.
As to the trial courts finding of conspiracy among the accused, we find the same to be supported by
evidence. For conspiracy to exist, it is not required that there be an agreement for an appreciable
period prior to the occurrence. It is sufficient that at the time of the commission of the offense, the
accused had the same purpose and were united in its execution. The agreement to commit a crime
may be gleaned from the mode and manner of the commission of the offense or inferred from the
acts of the accused which point to a joint purpose or design, concerted action, and community of
intent. In this case, the accused simultaneously attacked LONGASA, with two of them holding the
victims hands or arms. Some struck LONGASA with a piece of wood or bottles and two others stabbed
him. The attack continued until LONGASA fell dead. These acts clearly point to a joint purpose to
accomplish the desired end.

However, we do not share the assessment of the trial court that there was no treachery in this case
because the victim had engaged in a fight previous to the killing and was thus forewarned of an attack
against him. Treachery may still be appreciated even when the victim was forewarned of danger to
his person. What is decisive is that the execution of the attack made it impossible for the victim to
defend himself or to retaliate. The overwhelming number of the accused, their use of weapons against
the unarmed victim, and the fact that the victims hands were held behind him preclude the possibility
of any defense by the victim.

The other qualifying circumstance of abuse of superior strength, which the trial court appreciated,
will no longer be taken against accused-appellants, for it is absorbed in treachery.[32]
The penalty for the murder is reclusion perpetua to death pursuant to Article 248 of the Revised Penal
Code as amended by R.A. No. 7659. There being no mitigating or aggravating circumstance proved in
CRIMINAL LAW 1
(Sunday, Section 52) 233

favor of or against EMERLITO and REGANDO, the trial court correctly imposed the penalty
of reclusion perpetua.

As to RUEL, who was only 16 years old when the offense in question was committed, the trial court
correctly appreciated in his favor the privileged mitigating circumstance of minority. Pursuant to
paragraph 2 of Article 68 of the Revised Penal Code, the penalty next lower to that prescribed by law
shall be imposed; in this case the penalty shall be reclusion temporal.[34] Again, there being no proof
of any modifying circumstance, said penalty shall be imposed in its medium period.[35] Since RUEL is
entitled to the benefits of the Indeterminate Sentence Law, he shall be sentenced to suffer an
indeterminate penalty whose minimum shall be within the range of prision mayor and whose
maximum shall be within the range of reclusion temporal. The penalty imposed upon him by the trial
court, i.e., ten years of prision mayor as minimum to seventeen years of reclusion temporal as
maximum, is therefore correct.

The awards of P50,000 as indemnity for the death of LONGASA and of actual damages of P8,500 are
in conformity with current case law and with the agreement of the parties, respectively.

WHEREFORE, we DISMISS the appeal and AFFIRM the challenged Joint Decision of 23 November
1995 of Branch 170 of the Regional Trial Court of Malabon, Metro Manila, in Criminal Cases Nos.
14943-MN and 15506-MN convicting accused-appellants REGANDO P. VILLONES, EMERLITO N.
SANTOS, and RUEL L. SANTOS of the crime of murder and sentencing the first two accused to suffer
the penalty of reclusion perpetua and the third accused, to an indeterminate penalty of ten (10) years
of prision mayor as minimum to seventeen (17) years of reclusion temporal as maximum; and
ordering all accused-appellants to pay the heirs of the victim GERARDO LONGASA P50,000 as death
indemnity and P8,500 as actual damages. No pronouncement as to costs. SO ORDERED.

PEOPLE OF THE PHILIPPINES vs. NICOLAS GUZMAN


G.R. No. 169246 January 26, 2007

FACTS:
After attending a worship service at the Iglesia ni Kristo church in his barangay, Michael proceeded
home. While Michael was casually walking along the corner of Sto. Nino Street and Mactan Street,
appellant and his two companions, who were drinking nearby, suddenly approached and surrounded
Michael. Appellant positioned himself at the back of Michael while his two companions stood in front
of Michael. In an instant, they grabbed the shoulders of Michael and overpowered the latter. One of
the appellant's companions, whom the prosecution witnesses described as a male with long hair,
drew out a knife and repeatedly stabbed Michael on the stomach. Unsatisfied, the appellant's other
companion, whom the prosecution witnesses described as a male with flat top hair, took the knife
and stabbed Michael on the stomach. As the finale, appellant went in front of Michael, took the knife
and also stabbed Michael on the stomach. When Michael fell on the ground, appellant kicked him at
the body. Upon noticing that the bloodied Michael was no longer moving, appellant and his two
companions fled the scene. The appellant was convicted by the trial court with the crime of murder.
On appeal, appellant contends that even if he were held liable for the death of Michael, there was no
treachery which will qualify the killing as murder. According to him, there is no evidence to show
that appellant and his two companions had deliberately and consciously adopted their mode of attack
to ensure its execution without risk to themselves. The stabbing incident occurred in a place that was
properly lighted. There were many people in the area then walking in different directions. He claims
that if he and his two companions wanted to ensure that no risk would come to them, then they could
have chosen another time and place to attack Michael.
CRIMINAL LAW 1
(Sunday, Section 52) 234

ISSUE:
Can treachery be properly appreciated in the instant case?

HELD:
Yes. Treachery is a sudden and unexpected attack under the circumstances that renders the victim
unable and unprepared to defend himself by reason of the suddenness and severity of the attack. It
is an aggravating circumstance that qualifies the killing of a person to murder. Article 14, paragraph
(16) of the Revised Penal Code states the concept and essential elements of treachery as an
aggravating circumstance. There is treachery when the offender commits any of the crimes against
the person, employing means, methods, or forms in the execution thereof which tend directly and
specially to insure its execution, without risk to himself arising from the defense which the offended
party might make.

As can be gleaned from the foregoing, two essential elements/conditions are required in order that
treachery may be appreciated: (1) The employment of means, methods or manner of execution that
would ensure the offender's safety from any retaliatory act on the part of the offended party, who
has, thus no opportunity for self-defense or retaliation; (2) deliberate or conscious choice of means,
methods or manner of execution. Further, it must always be alleged in the information and proved in
trial in order that it may be validly considered.

In the instant case, treachery was alleged in the Information against appellant. Moreover, all the
essential elements/conditions of treachery were established and proven during the trial. The
suddenness and unexpectedness of the attack of appellant and his two companions rendered Michael
defenseless, vulnerable and without means of escape. It appears that Michael was unarmed and alone
at the time of the attack. Further, he was merely seventeen years of age then. In such a helpless
situation, it was absolutely impossible for Michael to escape or to defend himself against the assault
of appellant and his two companions. Being young and weak, Michael is certainly no match against
adult persons like appellant and his two companions. Michael was also outnumbered since he had
three assailants and was unarmed when he was stabbed to death. Appellant and his two companions
took advantage of their size, number, and weapon in killing Michael. They also deliberately adopted
means and methods in exacting the cruel death of Michael by first surrounding him, then grabbing
his shoulders and overpowering him. Afterwards, each of them repeatedly stabbed Michael with a
knife at the stomach until the latter fell lifeless to the ground. The stab wounds sustained by Michael
proved to be fatal as they severely damaged the latter's large intestine.

The fact that the place where the incident occurred was lighted and many people were walking then
in different directions does not negate treachery. It should be made clear that the essence of
treachery is the sudden and unexpected attack on an unsuspecting victim without the slightest
provocation on his part. This is even more true if the assailant is an adult and the victim is a minor.
Minor children, who by reason of their tender years, cannot be expected to put up a defense. Thus,
when an adult person illegally attacks a minor, treachery exists.

PEOPLE VS TORRIEFEL
CRIMINAL LAW 1
(Sunday, Section 52) 235

PEOPLE V. JAIME JOSE, G.R. NO. L-28232


Title: People v. Jaime Jose, G.R. No. L-28232
Subject Matter: Conspiracy

Facts:
On June 26, 1967, four principal-accused Jaime Jose, Basilio Pineda Jr., Eduardo Aquino and Rogelio
Cañal conspired together, confederated with and mutually helped one another, then and there, to
willfully, unlawfully and feloniously, with lewd design to forcibly abduct Magdalena “Maggie” dela
Riva, 25 years old and single, a movie actress by profession at the time of the incident, where the four
principal accused, by means of force and intimidation using a deadly weapon, have carnal knowledge
of the complainant against her will, and brought her to the Swanky Hotel in Pasay City, and hence
committed the crime of Forcible Abduction with Rape.

Having established the element of conspiracy, the trial court finds the accused guilty beyond
reasonable doubt of the crime of forcible abduction with rape and sentences each of them to the death
penalty.

Issue:
Whether or not the trial court made a proper ruling of the case considering the element of conspiracy.

Held:
No, the trial court’s ruling was not proper. The SC ruled that since the element of conspiracy was
present, where the act of one is the act of all, each of the accused is also liable for the crime committed
by each of the other persons who conspired to commit the crime. The SC modified the judgment as
follows: appellants Jaime Jose, Basilio Pineda Jr., and Eduardo Aquino are guilty of the complex crime
of forcible abduction with rape and each and every one of them is likewise convicted of three (3)
other crimes of rape. As a consequence thereof, each of them is likewise convicted with four death
penalties and to indemnify the victim of the sum of P10,000 in each of the four crimes. The case
against Rogelio Cañal was dismissed only in so far as the criminal liability is concerned due to his
death in prison prior to promulgation of judgment.

THE PEOPLE OF THE PHILIPPINES, plaintiff-appellee


vs. MICHAEL J. BUTLER, accused-appellant.

This is an automatic review of the Judgment of the Court of First Instance of Zambales, Third Judicial
District, Branch I, finding the accused Michael J. Butler in Criminal Case No. 2465 guilty beyond
reasonable doubt of the crime of murder qualified by abuse of superior strength, with the attendance
of aggravating circumstances of treachery and scoffing at the corpse of the deceased, without any
mitigating circumstance and sentencing the accused with the penalty of death, and ordering him to
indemnify the heirs of the victim with the sum of P24,000.00.

In an Information dated October 16, 1975, accused-appellant Michael J. Butler was charged with the
crime of murder committed as follows:

That on or about the 8th day of August, 1975, in the City of Olongapo, Philippines. and
within the jurisdiction of this Honorable Court the above-named accused, with intent
to kill and taking advantage o his superior strength, did then and there wilfully,
unlawfully and feloniosly assault, attack and hit with a statue of Jesus Christ oue
CRIMINAL LAW 1
(Sunday, Section 52) 236

Enriquita Alipo alias 'Gina Barrios' and after said Enriquita Alipo fell flat on her fare
the above-named, accused again taking advantage of superior strength then and there
apply force and pressure on the back of the head of said Enriquita Alipo thereby
forcing and sinking the latter's mouth and nose against the mattress of the bed, and
as a result thereof, the said Enriquita Alipo was not able to breathe and was choked,
thus directly causing the death of said Enriquita Alipo alias 'Gina Barrios'.

Upon arraignment, accused-appellant pleaded not guilty, hence the trial was conducted and at the
termination of which, judgment of conviction was rendered.

It appears from the records of the case that on August 7, 1975, at about 10:30 p.m., accused-appellant
Michael Butler and the victim, Enriquita Alipo alias Gina Barrios were together at Colonial Restaurant
in Olongapo City. They were seen together by Lilia Paz, and entertainer and friend of the victim, who
claimed to have had a small conversation with the accused, and by one Rosemarie Juarez, also a friend
of the victim. At about 1:00 of the same evening, the accused and the victim left the said
restaurant, 1 after the latter invited Rosemarie Juarez to come to her house that night.

Emelita Pasco, the housemaid of the victim, testified that, at about 11:30 p.m. or so of August 7, 1975,
her mistress (Gina Barrios) came home with the accused-appellant. As soon as she opened the door
for them, the victim and accused-appellant immediately entered the victim's bedroom. Shortly
thereafter, the victim left her bedroom holding an Id card and a piece of paper, and on the piece of
paper, the victim purportedly wrote the following words: MICHAEL J. BUTLER, 44252-8519 USS
HANCOCK. Said words were copied from the ID Card.

Pasco testified that the victim said she was copying the name of the accused because she knew he
would not be going back to her. Then she rushed back to her bedroom after instructing Pasco to wake
her up the following morning. 2 Before retiring, however, the victim's friend, Rosemarie Juarez, came
to the former's house and after having a small conversation, also left.

The following day, August 8, 1975, at about 4:00 a.m., Pasco rose to wake her mistress as instructed.
She knocked at the door. She found that the victim was lying on her bed, facing downward, naked up
to the waist, with legs spread apart, with a broken figurine beside her head. Immediately, Pasco called
the landlord and they called the authorities. 3

Patrolman Rudyard de los Reyes of the Olongapo Police Department arrived together with Fiscal
Llamado and Corporal Sobrepeña at about 6.00 a.m. of August 8, 1975. Pasco informed Patrolman de
los Reyes that the accused Butler slept with the victim the previous night, and the former gave the
latter the piece of paper where the name of the accused was written.

Sergeant Galindo of the Olongapo Police Department handed over to Jesus Bensales, a fingerprint
technician of the Police Department, a piece of cellophane together with the broken figurine for latent
print examination. The latent print examination report (Exh. E4) showed that there were three (3)
fragmentary latent prints that were lifted from the cellophane wrapping of the figurine. But only one
print was clear and distinguishable. This particular print was found Identical with the accused's left
middle fingerprint on thirteen (13) points. Bensales later testified that the latent print developed
from the piece of cellophane belonged to the accused Butler. 4

On the same day, officers of the Olongapo Police Department informed the Naval Investigation
Services Resident Agency (NISRA) in Subic Bay that an American Negro by the name of Michael J.
CRIMINAL LAW 1
(Sunday, Section 52) 237

Butler on board the USS Hancock- was a suspect in a murder case. Jerry Witt and Timothy Watrous
both special agents of NISRA went on board USS Hancock. They informed the legal officer that one of
the crew members was a suspect in a murder case. After being located, the accused was brought to
the legal office of the ship. Witt Identified himself, showed his credentials and informed the accused
that he was a suspect in a murder case. Then Witt informed the accused of his constitutional rights
to remain silent and right to counsel. Then the accused was searched, handcuffed, and was brought
to NISRA office.

Arriving at NISRA office at about 11:00 a.m. of the. same day, the investigation and interrogation were
started . by James Cox NISRA investigator, at about 2:55 p.m. According to Cox's testimony, before he
started the interrogation, he identified himself, informed the accused of his constitutional rights. At
the cross-examination, he stated it took him about 1-1/2 hours to finish the investigation. The first
45 minutes was accordingly devoted to interrogation, and for the next 45 minutes, he called James
Beaver who reduced the oral investigation into writing.

James Cox also testified that after apprising the accused of his constitutional rights to remain silent
and right to counsel he asked the accused if he needed a lawyer and if he understood his rights
(constitutional rights and rights under the military code of justice). The accused accordingly said he
understood his rights and that he did not need a lawyer.

The result of that investigation was thus a document taken from the accused consisting of three (3)
pages, signed and initialed on all pages by him and containing a statement that he was aware of his
constitutional rights, and a narration of the facts that happened on August 7, 1975

We are fully satisfied that the accused. appellant has behaved properly and has shown his capability
to be a useful member of the community. It is of no moment that the accused had not been specifically
committed by the court to the custody or care of the Department of Social Welfare then, now the
Ministry of Social Services and Development, or to any training institution operated by the
government or duly-licensed agencies as directed under Article 192 of P.D. 603. At any rate, the
Commander of the U.S. Naval Base in Subic Bay to whom the accused was committed in the Order of
December 3, 1976 pending the finality of judgment rendered in the case pursuant to the provisions
of paragraph 5, Article 13 of the Revised Base Military Agreement, may be considered a responsible
person to whom the accused may be committed for custody or care under the said Article 192 of P.D.
603. What is important is the result of such custody and care showing his conduct as well as the
intellectual, physical, moral, social and emotional progress made by the accused as shown in the
favorable recommendation of the Supervising Social Worker of the Ministry of Social Services and
Development who had visited him regularly and given counselling. We hereby approve the
recommendation of the Ministry that "Michael Butler be given a chance to enjoy his life fully outside
the jail, thus promoting Ms best interest and welfare" (Progress Report dated October 27, 1980);
"that Mr. Michael Butler is now fully rehabilitated, it is our recommendation that he be given an
opportunity to live happily and prove himself outside the Brig" (Progress Reported dated February
18, 1981); "with the above findings and Mr. Butler's desire to start life anew, this Final Report is
submitted." (Final Report dated September 14, 1981).

The dismissal of the case against the accussed Michael Butler is, therefore, meritorious and justifiable.
We hereby order his final discharge therefrom. His final release, however, shall not obliterate his civil
liability for damages in the amount of P24.000.00 to the heirs of the victim which We hereby affirm.
Such release shall be without prejudice to the right for a writ of execution for the recovery of civil
damages. (Article 198, P.D. 603).
CRIMINAL LAW 1
(Sunday, Section 52) 238

WHEREFORE, IN VIEW OF ALL THE FOREGOING, the case against the accused-appellant Michael J.
Butler is hereby DISMISSED and We hereby order his final discharge from commitment and custody.
The civil liability imposed upon him by the lower court shall remain.

SO ORDERED

PEOPLE v. RAFAEL SAYLAN +


215 Phil. 134

This is an automatic review of the decision of the defunct Court of First Instance of Misamis Oriental
in Criminal Case No. 52-M which imposed the death penalty.

RAFAEL SAYLAN was accused of the crime of rape in the sworn complaint of Eutropia Agno said to
have been committed as follows:
"That on or about the 23rd day of January, 1972, at more or less 7:00 o'clock in the evening, at Sitio
Craser, Malinao, Gingoog City, Philippines and within the jurisdiction of this Honorable Court, the
above-named accused, with deliberate intent to have sexual intercourse, did then and there wilfully,
unlawfully and criminally with the use of a dagger, force and intimidate Eutropia Agno y Arcay, to
remove her pantie and to lay down on the ground and with the use of a dagger, force and intimidation
succeeded in having sexual intercourse with Eutropia Agno y Arcay, a woman of good reputation and
against her will. That the commission of the foregoing offense was attended by the aggravating
circumstances of: abuse of superior strength, nighttime, uninhabited place, ignominy
and reiteracion." (Expediente, p. 27.)

The accused entered a plea of "not guilty" and after trial the court rendered the following judgment:
"WHEREFORE, the Court finds the accused guilty beyond reasonable doubt of the crime of rape,
penalized under Article 335 of the Revised Penal Code as amended by Republic Act No. 4111, and the
commission of the offense having been attended by three aggravating without any mitigating
circumstance, hereby sentences him to suffer the supreme penalty of death, to indemnify the
offended party in the amount of Six Thousand Pesos (P6,000.00), and to pay the costs. In view of the
fact that the offended party is a married woman, aside from the fact that she has not become pregnant
as a result of the commission of the rape, the Court makes no pronouncement as to acknowledgment
and support of offspring." (Id., p. 64.)
The factual version of the prosecution is summarized in the People's brief as follows:
"The complaining witness, Eutropia A. Agno, a married woman and a resident of Barrio Malinao,
Gingoog City, was a classroom teacher of the Malinao Elementary School (pp. 2, 3, tsn., Feb. 22, 1973).

"In the afternoon of January 23, 1971, Eutropia went to the public market in Gingoog City to buy
foodstuffs for her family and thereafter, she proceeded to the store of her mother to fetch her five-
year old daughter Nilsonita (p. 4, tsn., Id.). On their way home, Eutropia and Nilsonita boarded a
passenger jeepney and while inside the vehicle she (Eutropia) noticed that the other passengers were
Rudy Gonzales, a grade I pupil of the Malinao Elementary School, the appellant, Rafael Saylan, and a
couple whom she did not know (pp. 5, 6, tsn., Id. ). The jeepney went only as far as Malinas citrus farm
because the road to Barrio Malinao was not passable by vehicles (p. 5, tsn., Id.). It was almost 6:30
o'clock in the evening when the jeepney arrived at the Malinas citrus farm and so all the passengers
alighted and had to walk all the way to Barrio Malinao which was about three and a half kilometers
away (p. 5, tsn., Id.). After walking some distance and upon reaching a junction, the couple separated
from the group and took the road leading to their house while Eutropia's group took the opposite
road (p. 9, tsn., Id.). The appellant, however, joined the group of Eutropia and when they reached the
place where the road was plain, appellant who was then walking side by side with Eutropia suddenly
CRIMINAL LAW 1
(Sunday, Section 52) 239

pulled out a dagger about eight inches long and pointing it at the latter said, 'Do not shout, Nang, I
will kill you!' (pp. 11, 12, tsn., Id. ). At this juncture, appellant placed his right arm around the neck of
Eutropia with the dagger pointed at her left breast (p. 12, tsn., Id.), after which he dragged Eutropia
at some distance. When they reached the junction of the trail for men and a trail for carabaos, he
ordered everybody to stop and told the children (Nilsonita and Rudy Gonzales) to stay behind and
threatened to kill them if they persisted in following them (pp. 17, 18, tsn., Id.). Thereafter, appellant
again dragged Eutropia by her hand and brought her towards a creek near a coconut tree which was
about five meters away from where Nilsonita and Rudy Gonzales were (pp. 14, 15, 16, tsn., Id.). The
appellant then ordered Eutropia to remove her panty which she refused at first, but appellant
threatened to kill her, so she removed her panty after which appellant ordered her to lie down (pp.
18, 19, tsn., Id. ). Subsequently, appellant placed himself on top of the victim and inserted his penis
into her vagina and succeeded in having sexual intercourse with her by moving his buttocks up and
down (pp. 20, 21, tsn., Id.).

"After the first sexual act, appellant ordered Eutropia to stand up which the latter helplessly and
grudgingly followed (p. 23, tsn., Id.). Appellant again inserted his penis into her vagina and then
performed a push and pull movement (pp. 23, 24, 25, tsn., Id.). Not satisfied with the second
intercourse, appellant ordered Eutropia to lie down again preparatory to a third intercourse (p. 26,
tsn., Id.). Appellant again performed the sexual act with her (pp. 26, 27, tsn., Id.).

"After the third intercourse, appellant ordered Eutropia to stand up and then he bent her body
downwards with her hands and knees resting on the ground (p. 28, tsn., Id.). When the latter was
already in this position, appellant then placed himself behind her, inserted his penis into her vagina
and executed a push and pull movement in the dog's way of sexual intercourse (pp. 27, 28, tsn., Id. ).
"After performing this uncommon way of sexual intercourse, appellant ordered Eutropia to lie down
again which the latter reluctantly obeyed because appellant's dagger was always pointed at her and
thereafter he had carnal knowledge of her for the fifth time (pp. 29, 30, tsn., Id. ).

"After the fifth intercourse, and after satisfying his sexual lust, appellant asked Eutropia if she will
tell her husband what he did to her and the latter answered, 'I will not tell' (p., 31, tsn., Id.). But she
only said this so that appellant would let her go home (p. 33, tsn., Id.).
"Afterwards, Eutropia and appellant returned to the place where the children were left and upon
arriving thereat, they found Nilsonita (Eutropia's daughter) asleep with Rudy seated dozing beside
her (pp. 32, 33, tsn., Id.). Nilsonita who was sleeping was carried by the appellant and then they all
proceeded to Malinao (pp. 33, 34, tsn., Id.).

"After walking some distance, Eutropia saw the house of her friend "Ben" and upon approaching the
said house, she shouted, 'Ben, Ben, please give me hot water' (p. 34, tsn., Id.). Upon hearing her voice,
Ben, who was still awake at the time, opened the door of his house and allowed Eutropia to come up
(p. 34, tsn., Id.). Eutropia immediately went upstairs and went straight to the room of Ben as she was
feeling very bad (p. 34, tsn., Id.). Appellant, who was then carrying Nilsonita, and Rudy Gonzales, were
also allowed to go upstairs (p. 35, tsn., Id.). Meanwhile, Eutropia requested Ben to fetch her husband
(p. 35, tsn., Id.).
"When Eutropia woke up between 9:00 and 10:00 o'clock that evening, her husband was already
there (p. 36, tsn., Id.). She then asked him whether the appellant was still around, and in reply, he told
her that appellant had already left (p. 37, tsn., Id.). Eutropia then told her husband that she was raped
by the appellant (p. 37, tsn., Id.). Upon learning of the dastardly act committed by the appellant, he
advised his wife to submit herself to a medical examination (p. 37, tsn., Id.).
CRIMINAL LAW 1
(Sunday, Section 52) 240

"The following morning, the offended party was brought to the office of the City Health Department
of Gingoog City where she was examined by Dr. Ireneo O. Pascual, who after conducting a thorough
physical examination, issued a medical certificate with the following findings, to wit:
'(1) Multiparous.
'(2) Presence of viscid whitish secretions at vaginal fornix.
'(3) Microscopic examination of secretions reveals epithelial cells, but no spermatozoa identified.'
(pp. 10, 11, 12, t.s.n., Feb. 24, 1973; Exh. "A").

"Rudy Gonzales, a grade I pupil of the Malinao Elementary School and one of the witnesses for the
prosecution, testified that he met Mrs. Eutropia Agno in the afternoon of January 23, 1972 at the
public market of Gingoog City buying foodstuffs for her family (pp. 2, 3, tsn., Feb. 26, 1973). On their
way back to Barrio Malinao, they boarded a passenger jeepney and while he was inside the vehicle,
he noticed that the other passengers aside from Mrs. Agno, her daughter, and himself were the
appellant and a couple whose names he did not know (p. 4, tsn., Id. ). The jeepney, however, could
only travel up to the Malinas Citrus farm and so they had to walk all the way to Barrio Malinao (p. 4,
tsn., Id.) After walking some distance and upon reaching a trail for carabaos, the appellant suddenly
pulled a dagger and placed his arms around the neck of Mrs. Agno and then dragged her towards the
carabao trail (pp. 4, 5, tsn., Id.). Meanwhile, he and Nilsonita were left behind and they fell asleep
because it took a long time for the appellant and Mrs. Agno to come back for them (p. 5, tsn., Id.).
When Mrs. Agno and the appellant returned, he was already awake while Nilsonita was still asleep
and so appellant had to carry her in going home to Malinao (p. 6, tsn., Id.). After walking some
distance, Mrs. Agno saw the house of Mang Ben and because she was feeling bad, they all went to the
house of Mang Ben where Mrs. Agno spent the night (p. 7, tsn., Id. ). Afterwards, he and the appellant
left the house of Mang Ben and then they proceeded to his house at Malinao where both of them slept
(pp. 7, 21, tsn., Id. )." (At pp. 2-8.)

The accused did not deny having had sexual Intercourse with Mrs. Agno; in fact he admitted that he
copulated with her for three successive times in the early evening of January 23, 1972, but he claimed
that it was with her consent. Accordingly, he now claims that:
"I. THE COURT A QUO ERRED IN FINDING THA THE SEXUAL INTERCOURSE HAD BEEN COMMITTED
AGAINST THE WILL AND CONSENT OF THE COMPLAINANT.
"II. THE COURT A QUO ERRED IN FINDING THAT AGGRAVATING CIRCUMSTANCES HAD
ACCOMPANIED THE COMMISSION OF THE OFFENSE." (Brief, p. 5.)

The appeal must fail for the reasons stated hereunder.


This is a typical rape case. Only the participants could directly testify on the alleged sexual abuse and
the accused alleges consent on the part of the complainant. The question of credibility arises and
under the circumstances We have to rely heavily on the determination made by the trial judge who
observed the demeanor of the witnesses while before Us is only the cold transcript of what they said.
We accept the conclusions and findings of fact of the trial court that the complainant was in fact raped
by the appellant. There is no fact or circumstance in the record which will justify a different action.
The claim of the appellant that the sexual intercourse was mutually agreed is utterly incredible. If it
were true that Mrs. Agno consented to have coitus with the appellant, her conduct thereafter defies
understanding because it is contrary to reason and it has not been shown that Mrs. Agno, a school
teacher, was bereft of common sense. For if it was true that the sexual act was indeed mutually
desired and performed why did she complain not only to her husband but also to the authorities? An
affair such as that claimed by the appellant is carried out in a discreet manner. On the other hand, the
version of the complainant has indicia of credibility. For her version bared her shame to a small
community and her exposure was necessary only because she had to reveal the truth. No, We simply
cannot believe the appellant's version.
CRIMINAL LAW 1
(Sunday, Section 52) 241

We have said above that the findings and conclusions of the trial court are entitled to great respect.
In finding the appellant guilty, this is what the court a quo said in part:

"The testimony of the accused is incredible. When he told his love to the offended party for the first
time, they were only two in the latter's house. He had more time with her then. She refused him
because she is married. He tried for the second time. He was again refused because she is married. It
is unthinkable and highly improbable that on the evening of January 23, 1972, after only three
minutes, the offended party would rush to accept his love and go to the extent of thanking him for his
considering her daughter as his own, unless she was coerced, threatened, forced and intimated.
"It is highly improbable for a school teacher with several children to exchange her husband only 40
years old and with a good means of livelihood for one whom she does not know and whom she has
observed as doing nothing except to play basketball. It is subversive of the traits, character and nature
of Filipino women to say that the offended party, a school teacher and a girl scout accepted the love
of a man who is good for nothing and surrendered her whole body and virtue to him after an
accidental courtship of only three minutes. The offended party is an unsophisticated and
conservative woman, fixing her hair the old fashion way. She does not apply make-up on her face,
and her dress is up to her knees. This makes the pretensions of the accused all the more incredible."
(Expediente, p. 59.)

The complaint alleges the following aggravating circumstances: abuse of superior strength,
nocturnity, despoblado, ignominy, and reiteracion.

The trial court disregarded superiority because it "is inherent in the crime of rape or is absorbed in
the element of force." It also did not consider nocturnity "there being no evidence that the accused
purposely sought it to facilitate the commission of this rape." (Id., p. 63.)

Despoblado was present according to the trial court because: "The accused dragged the offended
party, at the point of a dagger, to the carabao trail, about 10 meters from the junction, but 40 to 50
meters below to better attain his purpose without interference, and to better secure himself from
detection and punishment (U.S. vs. Vitug, 17 Phil. 1). Even the junction where the two children were
left is already 400 meters from the nearest house. While there maybe occasional passersby, this does
not destroy its being an uninhabited place. (People vs. Bangug, 52 Phil. 87)." (Id., p. 62.) We hold that
the trial court for the reasons stated correctly held that the crime was committed in an uninhabited
place.

The trial court held that there was ignominy because the appellant used not only the missionary
position, i.e. male superior, female inferior, but also "The same position as dogs do" i.e., entry from
behind. The appellant claims there was no ignominy because "The studies of many experts in the
matter have shown that this 'position' is not novel and has repeatedly and often been resorted to by
couples in the act of copulation." (Brief, p. 24.) This may well be if the sexual act is performed by
consenting partners but not otherwise.
The trial court also held that "there is no reiteracion because one of the offenses, namely Robbery in
Band, for which the accused has been penalized, was committed after the commission of this rape
case, and the penalty imposed on the other offense of Frustrated Homicide, is lighter than the penalty
for rape." (Id., p. 63.)

Although not alleged in the complaint, the trial court stated that the offense was aggravated by
disregard of rank because it was a fact known to the appellant that Mrs. Agno was a school teacher.
The appellant claims that this circumstance cannot be assigned to him because there was no
CRIMINAL LAW 1
(Sunday, Section 52) 242

deliberate intent to offend or insult the rank of Mrs. Agno. The Solicitor General agrees with the
appellant for the same reason.

The judgment of the trial court is in accordance with the facts and the law but it cannot be affirmed
completely because of the lack of the necessary number of votes.
WHEREFORE, the judgment under review is modified in the sense that the appellant shall suffer the
penalty of reclusion perpetua instead of death and the indemnity to be paid to the offended party is
increased to P20,000.00. Costs against the appellant. SO ORDERED.

PEOPLE OF THE PHILIPPINES, plaintiff-appellee, vs. FERNANDO SULTAN y LATO, accused-


appellant.

FERNANDO SULTAN y LATO appeals from the Decision of the trial court finding him guilty of the
special complex crime of robbery with rape, sentencing him to reclusion perpetua and ordering him
to return to his victim one (1) wrist watch, one (1) ring, one (1) pair of earrings, and one (1) necklace
valued at P1,600.00, P850.00, P500.00, and P2,100.00, respectively, and cash of P130.00; otherwise,
to pay P5,180.00 if restitution be no longer feasible. He was further ordered to pay P50,000.00 for
moral damages.

The evidence for the prosecution was based principally on the testimony of complaining witness
Juditha M. Bautista. According to her, on 2 June 1997 at 9:00 oclock in the evening she was on her
way home from a visit to her cousin Cristina Mansilongan in Novaliches, Quezon City; when she
passed the dark alley in her cousin's compound she was accosted by someone, later identified as
accused-appellant Fernando L. Sultan, who pointed a sharp instrument at her neck and announcing
it was a "hold-up." He grabbed her and brought her to a house along the alley which turned out to be
his. Once inside the house, he made her sit down. He offered her a drink; she refused it. Then he
started divesting her of her watch, ring, earrings, and necklace the values of which are now reflected
in the Decision of the court a quo, and her cash of P130.00. After taking her valuables, he started
kissing her on the lips and cheeks. As if to discourage him from making further sexual advances, she
told him that she was married with two (2) children but accused-appellant was not dissuaded from
pursuing his intentions. While pointing an ice pick at her he ordered her to undress. She acceded for
fear that he would kill her as she was under constant threat. After she had completely undressed,
accused-appellant ordered her to lie down on the floor. He then kissed her again from head down.
Still she could not resist him because of fear. He went on top of her, held her two (2) hands on the
level of her head, spread her thighs and inserted his penis into her vagina. The coital encounter lasted
for ten (10) to fifteen (15) minutes.

After satisfying his lust, he ordered her to put on her bra and panty, tied her hands and went out of
the room to smoke. After ten (10) to fifteen (15) minutes, he came back, untied her, and once again
with threat and intimidation sexually abused her. Thereafter, he tied her hands to a protruding piece
of wood in the room and held her in his arms. She cried. He told her that he loved her and that he
would answer for what he had done to her. They talked until noon the following day without sleeping.

In her effort to release herself from his clutches she "agreed" to elope with him. Perhaps convinced
that she was going to run away with him, he allowed her to go home at noon to get her things. She
was then staying with her cousin Nita del Rosario, at No. 9 Sta. Eleuteria Street, Gulod, Novaliches,
Quezon City. He even accompanied her to the highway to get a ride home.

When Juditha arrived home she saw her sister Antonette in the house. She was not actually residing
there but went there only that day. Juditha lost no time in narrating her harrowing experience to her
CRIMINAL LAW 1
(Sunday, Section 52) 243

sister. Immediately Antonette called her brother SPO1 Fernando M. Bautista who resides in
Bulacan.[5] SPO1 Bautista arrived at around 3:00 or 4:00 oclock in the afternoon and was told about
what happened.[6] He then advised Juditha to go back to the house of accused-appellant for the
"planned elopement" so that he and his two (2) companions[7] could stage an arrest.[8]

On their way to the house of accused-appellant, Juditha rode in a passenger jeep with her sister
Antonette and cousin Nita while her brother and his two (2) companions followed them on board an
XLT Van. Juditha alighted near the house of accused-appellant while her companions waited for her
and accused-appellant along the highway. When she arrived at accused-appellants place, he was
already waiting for her outside the store nearby. They went inside his house and came out twenty
(20) minutes later. They boarded a passenger bus while SPO1 Bautista and his companions trailed
them. When the bus reached the corner of Forest Hill Subdivision, Gulod, Novaliches, it slowed down
because of the traffic thus making it easier for SPO1 Bautista and his companions to board the bus.
Upon seeing her brother and his companions, Juditha motioned to them. They immediately
approached accused-appellant and boxed him before they could arrest him. The other passengers of
the bus joined in hitting accused-appellant. This caused a commotion in the bus. Some policemen who
were in the barangay hall across the street saw the disturbance. They boarded the bus to find out
what happened. Then they assisted in facilitating the arrest of accused-appellant and brought him to
the barangay hall. He was later on transferred to the police headquarters for further interrogation.

At the police station the authorities investigated Juditha who readily identified accused-appellant as
her robber and rapist. The police then requested for physical examination to find signs of sexual
abuse. Medico-Legal Inspector Dr. Dennis G. Bellin found no external signs of violence although there
was a deep fresh laceration at 5 oclock position in Judithas hymen. He also discovered other
lacerations, deep healed, at 3, 7 and 9 oclock positions. Dr. Bellin also observed that Judithas external
vaginal orifice offered moderate resistance to his examining index finger and virgin-sized vaginal
speculum. She was no longer a virgin when the alleged rape transpired.[9]
On 5 June 1997 an Information[10] for the special complex crime of robbery with rape was filed
against accused-appellant Fernando Sultan y Lato, docketed as Crim. Case No. Q-97-71353. But
accused-appellant brushed aside the charge and claimed that it was simply a sexual congress of
consenting adults.

Finding the complaining witness version more credible, the trial court, on 5 June 1998, found
accused-appellant guilty as charged and sentenced him to reclusion perpetua. He was ordered to
return to Juditha Bautista one (1) wrist watch valued at P1,600.00, one (1) ring worth P850.00, one
(1) pair of earrings worth P500.00, one (1) necklace worth P2,100.00 and cash in the amount
of P130.00, or the payment of P5,180.00 if return was not possible. Accused-appellant was further
directed to pay his victim P50,000.00 for moral damages.[11] Misact
In this appeal, accused-appellant submits that there is no convincing proof that he is guilty of the
crime charged.

As to the robbery, he contends that the testimony of complainant that she was robbed of her personal
valuables should not be given weight and credence as (a) no evidence was presented in court to prove
her claim and that (b) if he had really robbed her, why did she not ask him for restitution of her
valuables after the alleged threat had ceased, i.e., when there was already an agreement between
them to elope?

These arguments fail to persuade us. The testimony of complainant as to the taking of her cash and
valuables is evidence enough to sustain a conviction for robbery considering that we find no fault in
the pronouncement of the trial court that her testimony is credible. The persuasive value of the
CRIMINAL LAW 1
(Sunday, Section 52) 244

declaration of credibility is bolstered by our own scrutiny of the testimony of complainant showing
her answers to the incisive questions propounded to her to be firm and straightforward.

While there may have been no effort on the part of complainant to retrieve her personal belongings
from accused-appellant even after all threats had ceased, her failure to do so does not under the
circumstances necessarily dispute the commission of robbery. Article 293 of the Revised Penal Code
provides that "[a]ny person who, with intent to gain, shall take any personal property belonging to
another, by means of violence against or intimidation of person, or using force upon anything, shall
be guilty of robbery." When accused-appellant divested complaining witness of her personal
belongings he committed the crime of robbery. All the elements necessary for its execution and
accomplishment were present, i.e., (a) personal property belonging to another, (b) unlawful taking,
(c) intent to gain, and (d) violence or intimidation. It is therefore immaterial that she failed to ask for
the return of her personal things. Moreover, her actuation could only be fairly interpreted to mean
that she did not want accused-appellant to be suspicious of her moves.
As for the charge of rape, accused-appellant maintains that the requisite force or intimidation was
not proved by the prosecution beyond reasonable doubt; that there was some form of consent to the
sexual intercourse as complainant did not put up tenacious resistance despite lack of threat on her
life during the alleged rape; and, that complainant on cross-examination was not certain whether
accused-appellant was armed at the commencement of the rape.

We likewise find these contentions of accused-appellant unconvincing. The prosecution for rape in
the instant case is based solely on the testimony of complaining witness. Thus, the basic issue that
must be addressed is her credibility. Doctrinally, the trial courts assessment of the credibility of
witnesses is accorded the highest respect and weight by the appellate courts. It is normally sustained
unless material facts and circumstances have been overlooked, misunderstood or
misapplied.[12] There is no such showing in this case.

Accused-appellant might not have employed force in committing the rape but he definitely used
intimidation which was sufficient to make complainant submit herself to him against her will for fear
of life and personal safety. Accused-appellant grabbed her and dragged her to his house. He was
armed with an ice pick and threatened to kill her with it if she did not follow his wishes. She was
naturally intimidated and her intimidation started from that moment on, and subsisted in her mind
when the rape was started until its consummation. Intimidation is subjective so it must be viewed in
the light of the victims perception and judgment at the time of the commission of the crime, and not
by any hard and fast rule. It is enough that it produces fear, as in the present case, fear that if the
complainant does not yield to the bestial demands of accused-appellant something would happen to
her at that moment or even thereafter. Thus, it is irrelevant that she was not certain when cross-
examined that accused-appellant was armed with an ice pick when the rape commenced; it was
enough that he was holding something that looked like an ice pick which engendered fear in her. With
fear instilled in her mind, it is understandable that she did not offer any resistance since any attempt
to do so would only be futile. Such failure on her part should not be taken to mean consent so as to
make her a willing participant in the sexual confrontation.

The Information charges accused-appellant with the special complex crime of robbery with rape. The
record shows that the prosecution has established that he committed both robbery and rape with the
intent to take personal property of another preceding the rape. Under Art. 294, par. (1), of the Revised
Penal Code, "x x x [a]ny person guilty of robbery with the use of violence against or intimidation of
persons shall suffer: 1. The penalty of reclusion perpetua to death, x x x when the robbery shall have
been accompanied by rape x x x x" Complaining witness Juditha Bautista was raped twice on the
occasion of the robbery. In this regard, this Court had declared in some cases that the additional rapes
CRIMINAL LAW 1
(Sunday, Section 52) 245

committed on the same occasion of robbery would not increase the penalty.[13] There were also cases,
however, where this Court ruled that the multiplicity of rapes committed could be appreciated as an
aggravating circumstance.[14] Finally, in the recent case of People v. Regala,[15] the Court held that the
additional rapes committed should not be appreciated as an aggravating circumstance despite a
resultant "anomalous situation" wherein robbery with rape would be on the same level as robbery
with multiple rapes in terms of gravity.[16] The Court realized that there was no law providing for the
additional rape/s or homicide/s for that matter to be considered as aggravating circumstance. It
further observed that the enumeration of aggravating circumstances under Art. 14 of the Revised
Penal Code is exclusive, unlike in Art. 13 of the same Code which enumerates the mitigating
circumstances where analogous circumstances may be considered, hence, the remedy lies with the
legislature. Consequently, unless and until a law is passed providing that the additional rape/s or
homicide/s may be considered aggravating, the Court must construe the penal law in favor of the
offender as no person may be brought within its terms if he is not clearly made so by the statute.
Under this view, the additional rape committed by accused-appellant is not considered an
aggravating circumstance. Applying Art. 63, par. (2), of the Revised Penal Code which provides
that "(i)n all cases in which the law prescribes a penalty composed of two indivisible penalties, the
following rules shall be observed in the application thereof x x x x 2. (w)hen there are neither mitigating
nor aggravating circumstances in the commission of the deed, the lesser penalty shall be applied," the
lower penalty of reclusion perpetua should be imposed on accused-appellant. S As to the award of
damages to the complaining witness, an additional amount of P50,000.00 may be given as
damages ex delicto in line with recent jurisprudence.

WHEREFORE, the Decision of the court a quo finding accused-appellant FERNANDO SULTAN Y LATO
GUILTY of the special complex crime of robbery with rape and sentencing him to reclusion perpetua,
to pay Juditha M. Bautista P50,000.00 for moral damages, P5,180.00 for actual damages representing
the value of the personal properties plus the cash amount of P130.00 taken from her is AFFIRMED
with the MODIFICATION that the amount of P50,000.00 be added as civil indemnity in conformity
with prevailing jurisprudence. Costs against accused-appellant. SO ORDERED.

PEOPLE v FERNANDEZ
(183 SCRA 511)
March 22, 1990

The accused entered the bathroom together with accused Fernandez. In the bathroom, the
latter tied a piece of cloth aroundthe victim’sneck while accused Conrado held her hands placing
them behind her body. Thereafter, they rapedthe victim one after the other. Hence ,the Court
was correct in imposing on each of the accused of the penalty corresponding to two crimes of
rape because each of them (accused) cooperated in the commission of the rape perpetrated by
the others, by acts without which it would not have been accomplished.
CRIMINAL LAW 1
(Sunday, Section 52) 246

PEOPLE VS. LUCAS

G.R. No. 80102. January 22, 1990


Plaintiff-appellee: People of the Philippines
Accused-appellant: Jovencio Lucas
Ponente: J. Cortes

FACTS:
Mauricia Lucas was then thirteen years old and working as a housemaid in Sampaloc, Manila.
Sometime in September 1985, she was fetched by her father, herein accused Jovencio Lucas, from her
place of work. They boarded a jeepney and alighted in a place which Mauricia found unfamiliar. She
was thereafter brought to a dark room where the accused tied both her hands and feet to a bed,
undressed her, burnt her face with a lighted cigarette, kissed her, fondled her private parts, pointed
a knife at her neck, and laughed while consummating the sexual act. The physical and genital
examination supported the fact of defloration and further testified that the findings were consonant
to that of a woman who had several experience with sexual intercourse. Nonetheless, as the
examinations were conducted about six months after the alleged rape took place, evidence of
violence can no longer be established.

ISSUE:
1) Whether or not the trial court correctly appreciated the aggravating circumstance of cruelty in the
case.
2) Whether or not the trial court correctly appreciated the aggravating circumstance of relationship
in the case.

HELD:
1) Yes. The Court held that there is cruelty when the offender enjoys and delights in making his victim
suffer slowly and gradually, causing unnecessary physical pain in the consummation of the offense.
Moreover, the absence of any evidence of force does not negate a finding that forcible sexual
intercourse actually took place. The trial court, having had the opportunity of hearing the witnesses
of both prosecution and defense, gave weight to the sincerity and conviction of the victim. The
appellant tied the victim to a bed, burnt her face with a lighted cigarette, and laughed while
consummating the crime. Undeniably, cruelty is present in this case.
2) Yes. Article 15 of the Revised Penal code provides that, in the case of rape, the alternative
circumstance of relationship shall be taken into consideration when the victim is the descendant of
the offender. The Court found that in order for the appellant to carry out the crime to his advantage,
the filial trust reposed in him by his daughter was undeniable abused. He personally fetched his
daughter, at her place of work, took her to the scene of the crime, and forced himself sexually. The
aggravating circumstance of relationship in the case was correctly applied in the case.
CRIMINAL LAW 1
(Sunday, Section 52) 247

PEOPLE V. FONCARDAS

People of the Philippines v. Ranil Duetes (at large), Basilio Quijada @ “Kokoy” (at large), Reyman
Foncardas & Ritchie Dequina
G.R. No. 144598 February 6, 2004

FACTS:
Appellant, together with Ranil Duetes, Basilio Quijada alias “Kokoy” and Ritchie Dequiña, was
indicted for murder (qualified by treachery and evident premeditation). After having a drinking
spree at a videoke Duetes, Quijada, Dequiña, Marco Mariaca (Mariaca) and appellant (the group)
walked some 50 meters to the corner of Trading Boulevard, where it stayed for five minutes.
Realizing that it had run out of cigarettes, the group proceeded to Carol’s Store, but returned to the
corner of Trading Boulevard, to sit, smoke and while the time away. Soon after, the victim who had
just purchased a bottle of Coke from Carol’s Store repaired to the corner of Trading Boulevard, where
he bought balut from a vendor. About 5 meters away from the group, the victim ate balut and drank
the coke. Quijada then approached the victim, and the two started talking while Duetes, Dequiña,
Mariaca, and appellant just watched and smoked.

Minutes later, Duetes approached the victim and Quijada and sat down behind the two. Not long after,
Quijada was heard shouting something in the Visayan dialect, allegedly angered by the victim’s not
acceding to his demands for money. Without any warning, Duetes pulled the victim from behind,
causing the latter to fall down on his back. Appellant and Dequiña rushed to join their companions
Duetes and Quijada. Apparently, the victim was able to rise. Appellant, Quijada, Duetes, and Dequiña,
however, pummeled him with their fists while Mariaca looked on in shock and disbelief. The mauling
of the victim continued even as Quijada left the scene momentarily. When Quijada returned bearing
a piece of wood about two and a half feet long, appellant and Duetes who were standing behind the
victim, held the latter, rendering him helpless, as Quijada struck the victim’s nape with the piece of
wood. The victim fell down after being struck. Duetes then told Mariaca, who was merely looking at
his companions, to run. Mariaca did as he was told and immediately ran away from the scene.

ISSUE:
Should the circumstance of drinking be appreciated as an aggravating or mitigating circumstance in
this case?

HELD:
No. Although Mariaca testified that appellant and his co-accused had been drinking before the
mauling of the victim, that does not suffice to aggravate or mitigate appellant’s criminal liability. The
trial court could not have appreciated intoxication – appellant’s drinking beer – as an aggravating
circumstance, therefore, as the same was not alleged in the information. Even assuming that the
aggravating circumstance of intoxication was alleged, appellant’s degree of intoxication was not
proven with certainty. Moreover, in the absence of clear and positive proof that appellant’s
intoxication was habitual or subsequent to the plan to commit the crime, it is improper to consider
the same as an aggravating circumstance.
Neither could appellant’s alleged intoxication be appreciated as a mitigating circumstance. To be
mitigating, it is necessary that appellant present proof of having taken a quantity of alcoholic
beverage prior to the commission of the crime sufficient to produce the effect of obfuscating reason.
At the same time, he must prove that he is not a habitual drinker and that he did not take the alcoholic
drink purposely to reinforce his resolve to commit the crime. In the absence then of clear and positive
CRIMINAL LAW 1
(Sunday, Section 52) 248

proof as to appellant’s state of intoxication, this Court cannot consider appellant’s drinking beer as a
mitigating circumstance.
In fine, appellant is indeed guilty of murder, penalized under Article 248 of the Revised Penal Code,
as amended by Republic Act No. 7569. There being neither mitigating nor aggravating circumstance,
the lesser penalty of reclusion perpetua was correctly imposed by the trial court, pursuant to Article
63(2) of the Revised Penal Code.

PEOPLE V AMADORE

Facts:
The instant case for automatic review brings to fore anew a decision promulgated by the Regional
Trial Court, Branch 275 (Las Pias), of the National Capital Judicial Region, convicting accused-
appellant RODRIGO AMADORE (Amadore) of, and imposing upon him the capital punishment for, five
(5) counts of rape. Accused-appellant denied all the accusations against him, stating that their house
is only 12 x 10 feet in area and that Maria Fe had her own room. He claimed that, throughout, he and
Maria Fe had a smooth relationship, and that the only reason he could think of why the cases were
filed against him was the misunderstanding that once arose when he scolded her after she had
refused to be sent on an errand. After having heard both parties, the court a quo gave the case for the
prosecution. In not giving credence to the denial proffered by accused-appellant, the court said that
neither the size of the house nor the misunderstanding between him and the young girl could hardly
overwhelm the strong evidence against him.

Issue:
Whether or not the alibi can prevail positive identification

Held: The denial of accused-appellant cannot prevail over the clear and convincing testimony of
Maria Fe. Neither can his alibi prosper. Accused-appellant himself has testified that he would
regularly go home from work. His work as kargador elsewhere, not really that distant away, is not
one that could have prevented him from going back to the house and committing his nefarious
deeds. Alibi cannot prevail over positive identification

LICYAYO VS. PEOPLE


Sufficient Provocation and Intoxication
G.R. No. 169425, March 4, 2008

FACTS:
Licyayo was charged of Homicide with the RTC when he stabbed Rufino in different parts of the body.
The RTC convicted Licyayo guilty of the crime Homicide there being no attending aggravating or
mitigating circumstances.

The petitioner appealed contending that sufficient provocation and intoxication should be taken as
mitigating circumstances attendant in the case. He insisted that there is sufficient provocation
because it was the deceased who punched him first and when the incident ensued he was intoxicated.

ISSUE:
Whether or not sufficient provocation and intoxication should be considered as mitigating
circumstances attendant in the case.
CRIMINAL LAW 1
(Sunday, Section 52) 249

HELD:
The records do not sufficiently establish who between Rufino and Aron started the brawl which
resulted in the stabbing of Rufino by petitioner. Granting arguendo that there was unlawful
aggression on the part of the victim, it is obvious that immediately he became the underdog, literally
even. He was easily overpowered by the bigger and sober Aron Licyayo. Sufficient provocation
therefore cannot be appreciated in favor of the petitioner.

As testified by the police officers, they said that petitioner indeed was drunk when the scuffle ensued.
However, these testimonies alone do not suffice as proof to appreciate intoxication as a mitigating
circumstance. In the case at bar, there is no plausible evidence showing that the quantity of liquor
taken by petitioner was of such quantity as to affect his mental faculties. On the contrary, the fact that
petitioner could recall the details that transpired during and after his drinking session with friends
is the best proof that he knew what he was doing during the incident.

PEOPLE VS. LADJAALAM


G.R. Nos. 136149-51. September 19, 2000

Appellee: People of the Philippines


Appellant: Walpan Ladjaalam alias “Warpan”
Ponente: J. Panganiban

FACTS:
Four Informations were filed against appellant Walpan Ladjaalam in the Regional Trial Court (RTC)
of Zamboanga City (Branch 16), three of which he was found guilty, to wit: 1) maintaining a drug den
in violation of Section 15-A, Article III, of Republic Act No. 6425 (Dangerous Drugs Act of 1972); 2)
illegal possession of firearm and ammunition in violation of Presidential Decree No. 1866 as amended
by Republic Act. No. 8294; and 3) direct assault with multiple attempted homicide. The following
information was provided by the prosecution:
1) In the afternoon of September 24, 1997, more than thirty (30) policemen proceeded to the house
of appellant and his wife to serve the search warrant when they were met by a volley of gunfire
coming from the second floor of the said house. They saw that it was the appellant who fired the M14
rifle towards them.
2) After gaining entrance, two of the police officers proceeded to the second floor where they earlier
saw appellant firing the rifle. As he noticed their presence, the appellant jumped from the window to
the roof of a neighboring house. He was subsequently arrested at the back of his house after a brief
chase.
3) Several firearms and ammunitions were recovered from appellant’s house. Also found was a pencil
case with fifty (50) folded aluminum foils inside, each containing methamphetamine hydrochloride.
4) A paraffin test was conducted and the casts taken both hands of the appellant yielded positive for
gunpowder nitrates.
5) Records show that appellant had not filed any application for license to possess firearm and
ammunition, nor has he been given authority to carry firearms.

ISSUE:
Whether or not such use of an unlicensed firearm shall be considered as an aggravating circumstance.

HELD:
No. Section 1 of RA 8294 substantially provides that any person who shall unlawfully possess any
firearm or ammunition shall be penalized, “unless no other crime was committed”. Furthermore, if
homicide or murder is committed with the use of an unlicensed firearm, such use of an unlicensed
CRIMINAL LAW 1
(Sunday, Section 52) 250

firearm shall be considered as an aggravating circumstance. Since the crime committed was direct
assault and not homicide or murder, illegal possession of firearms cannot be deemed an aggravating
circumstance.

CELINO, SR. VS. CA


ANGEL CELINO, SR. vs. COURT OF APPEALS, ET AL.
G.R. No. 170562 June 29, 2007

FACTS:
Two separate informations were filed before the RTC charging petitioner with violation of the gunban
and illegal possession of firearms. Petitioner filed a Motion to Quash contending that he "cannot be
prosecuted for illegal possession of firearms (R.A. 8294) . . . if he was also charged of having
committed another crime of [sic] violating the Comelec gun ban under the same set of facts.” The trial
court denied the motion to quash on the ground that "the other offense charged . . . is not one of those
enumerated under R.A. 8294" The denial was affirmed on appeal. Hence this petition, where
petitioner contends that the mere filing of an information for gun ban violation against him
necessarily bars his prosecution for illegal possession of firearms.

ISSUE:
Did the court err in denying the Motion to Quash?

HELD:
No. The law is clear: the accused can be convicted of simple illegal possession of firearms, provided
that "no other crime was committed by the person arrested." If the intention of the law in the second
paragraph were to refer only to homicide and murder, it should have expressly said so. As accusation
is not synonymous with guilt, there is yet no showing that petitioner did in fact commit the other
crime charged. Consequently, the proviso does not yet apply.

In sum, when the other offense involved is one of those enumerated under R.A. 8294, any information
for illegal possession of firearm should be quashed because the illegal possession of firearm would
have to be tried together with such other offense, either considered as an aggravating circumstance
in murder or homicide, or absorbed as an element of rebellion, insurrection, sedition or attempted
coup d'etat. Conversely, when the other offense involved is not one of those enumerated under R.A.
8294, then the separate case for illegal possession of firearm should continue to be prosecuted.

PEOPLE V. ATOP

FACTS:
Sec. 11 of RA 7659, which amended Art. 335 of the RPC, provides that the death penalty for rape may
be imposed if the “offender is a parent, ascendant, step-parent, guardian, relative by consanguinity
or affinity within the 3rd civil degree, or the common-law spouse of the parent of the victim”

ISSUE: Whether the common-law husband of the girl’s grandmother included

RULING: No! Courts must not bring cases within the provisions of the law which are not clearly
embraced by it. No act can be pronounced criminal which is not clearly within the terms of a statute
can be brought within them. Any reasonable doubt must be resolved in favor of the accused
CRIMINAL LAW 1
(Sunday, Section 52) 251

PEOPLE V. LUCAS

FACTS:
Mauricia Lucas was then thirteen years old and working as a housemaid in Sampaloc, Manila.
Sometime in September 1985, she was fetched by her father, herein accused Jovencio Lucas, from her
place of work. They boarded a jeepney and alighted in a place which Mauricia found unfamiliar. She
was thereafter brought to a dark room where the accused tied both her hands and feet to a bed,
undressed her, burnt her face with a lighted cigarette, kissed her, fondled her private parts, pointed
a knife at her neck, and laughed while consummating the sexual act. The physical and genital
examination supported the fact of defloration and further testified that the findings were consonant
to that of a woman who had several experience with sexual intercourse. Nonetheless, as the
examinations were conducted about six months after the alleged rape took place, evidence of
violence can no longer be established.

ISSUE:
1) Whether or not the trial court correctly appreciated the aggravating circumstance of cruelty in the
case.
2) Whether or not the trial court correctly appreciated the aggravating circumstance of relationship
in the case.

HELD:
1) Yes. The Court held that there is cruelty when the offender enjoys and delights in making his victim
suffer slowly and gradually, causing unnecessary physical pain in the consummation of the offense.
Moreover, the absence of any evidence of force does not negate a finding that forcible sexual
intercourse actually took place. The trial court, having had the opportunity of hearing the witnesses
of both prosecution and defense, gave weight to the sincerity and conviction of the victim. The
appellant tied the victim to a bed, burnt her face with a lighted cigarette, and laughed while
consummating the crime. Undeniably, cruelty is present in this case.
2) Yes. Article 15 of the Revised Penal code provides that, in the case of rape, the alternative
circumstance of relationship shall be taken into consideration when the victim is the descendant of
the offender. The Court found that in order for the appellant to carry out the crime to his advantage,
the filial trust reposed in him by his daughter was undeniable abused. He personally fetched his
daughter, at her place of work, took her to the scene of the crime, and forced himself sexually. The
aggravating circumstance of relationship in the case was correctly applied in the case.

US V MCMANN

Facts:
The defendant, McMann, and one McKay were packers at Camp Vicars in Mindanao, employed by the
Quartermaster's Department of the Army. On the day in question the defendant had charge of some
mules about one and one half miles from the camp. With his revolver in his hand he attempted to
enter the house, but the owner would not allow him to do so. A few moments later the defendant
arrived at the same house. He attempted to enter, but was unable to do so on account of the
opposition of the owner. He also carried his revolver in his hand with the hammer raised ready to be
discharged. That the defendant fired the shot which killed McKay is practically admitted by him in
his testimony and the fact is also proved by three or four eyewitnesses. It is accidental and that he
had no intention of killing McKay.
CRIMINAL LAW 1
(Sunday, Section 52) 252

Issue:
Whether or not his conditions could be taken into consideration for lessening the sentence?

Held:
The court below held that the defendant was drunk at the time the act was committed, but held also
that drunkenness was habitual with him and therefore his condition could not be taken into
consideration for the purpose of lessening the sentence. The defendant in this court claims that the
court erred in holding that drunkenness was habitual with the defendant.

People of the Philippines v. Ranil Duetes (at large), Basilio Quijada @ “Kokoy” (at large),
Reyman Foncardas & Ritchie Dequina
G.R. No. 144598 6 Feb, 1998

Carpio-Morales, J.
Topic: Alternative Circumstances (Art. 15, RPC) – Intoxication

FACTS:
Appellant, together with Ranil Duetes, Basilio Quijada alias “Kokoy” and Ritchie Dequiña, was
indicted for murder (qualified by treachery and evident premeditation). After having a drinking
spree at a videoke Duetes, Quijada, Dequiña, Marco Mariaca (Mariaca) and appellant (the group)
walked some 50 meters to the corner of Trading Boulevard, where it stayed for five minutes.
Realizing that it had run out of cigarettes, the group proceeded to Carol’s Store, but returned to the
corner of Trading Boulevard, to sit, smoke and while the time away. Soon after, the victim who had
just purchased a bottle of Coke from Carol’s Store repaired to the corner of Trading Boulevard, where
he bought balut from a vendor. About 5 meters away from the group, the victim ate balut and drank
the coke. Quijada then approached the victim, and the two started talking while Duetes, Dequiña,
Mariaca, and appellant just watched and smoked.

Minutes later, Duetes approached the victim and Quijada and sat down behind the two. Not long after,
Quijada was heard shouting something in the Visayan dialect, allegedly angered by the victim’s not
acceding to his demands for money. Without any warning, Duetes pulled the victim from behind,
causing the latter to fall down on his back. Appellant and Dequiña rushed to join their companions
Duetes and Quijada. Apparently, the victim was able to rise. Appellant, Quijada, Duetes, and Dequiña,
however, pummelled him with their fists while Mariaca looked on in shock and disbelief. The mauling
of the victim continued even as Quijada left the scene momentarily. When Quijada returned bearing
a piece of wood about two and a half feet long, appellant and Duetes who were standing behind the
victim, held the latter, rendering him helpless, as Quijada struck the victim’s nape with the piece of
wood. The victim fell down after being struck. Duetes then told Mariaca, who was merely looking at
his companions, to run. Mariaca did as he was told and immediately ran away from the scene.

ISSUE:
Whether or not circumstance of drinking be appreciated as
an aggravating or mitigating circumstance in this case

HELD:
No. Although Mariaca testified that appellant and his co-accused had been drinking before the
mauling of the victim, that does not suffice to aggravate or mitigate appellant’s criminal liability. The
trial court could not have appreciated intoxication – appellant’s drinking beer – as an aggravating
circumstance, therefore, as the same was not alleged in the information. Even assuming that the
aggravating circumstance of intoxication was alleged, appellant’s degree of intoxication was not
CRIMINAL LAW 1
(Sunday, Section 52) 253

proven with certainty. Moreover, in the absence of clear and positive proof that appellant’s
intoxication was habitual or subsequent to the plan to commit the crime, it is improper to consider
the same as an aggravating circumstance.

Neither could appellant’s alleged intoxication be appreciated as a mitigating circumstance. To be


mitigating, it is necessary that appellant present proof of having taken a quantity of alcoholic
beverage prior to the commission of the crime sufficient to produce the effect of obfuscating reason.
At the same time, he must prove that he is not a habitual drinker and that he did not take the alcoholic
drink purposely to reinforce his resolve to commit the crime. In the absence then of clear and positive
proof as to appellant’s state of intoxication, this Court cannot consider appellant’s drinking beer as a
mitigating circumstance.

LICYAYO VS. PEOPLE

Roberto Licyayo vs. People of the Philippines


G.R. No. 169425, 4 Mar. 1990
Chico-Nazario, J.
Topic: Alternative Circumstances (Art. 15, RPC) - Intoxication

FACTS:
Victim Rufino Guay (Rufino), along with his friends, Jeffrey and a certain Joel Dumangeng (Joel)
attended a wedding at Mabbalat, Kiangan, Ifugao. Petitioner, together with his friends, Paul and
Oliver, were also present at the same wedding. After the wedding reception, Rufino, Jeffrey and Joel
went to Natama’s Store at the Kiangan Public Market and ordered two bottles of gin. While the three
were drinking gin at the said store, petitioner, Paul and Oliver arrived and likewise ordered bottles
of gin. Later, petitioner, Paul and Oliver left the store. Subsequently, Rufino, Jeffrey and Joel likewise
adjourned their drinking session and left the store. Rufino, Jeffrey and Joel dropped by at Famorca’s
Store. Petitioner and his brother, Aron, as well as Paul and Oliver, were also present therein. While
Jeffrey was talking to the store’s owner, Larry Famorca (Larry), a brawl suddenly occurred between
Rufino and Aron. As a consequence thereof, Rufino fell to the ground. Aron thereafter placed himself
on top of Rufino and punched the latter several times. Jeffrey approached the two and tried to pacify
them. Paul entered the scene and punched Jeffrey on the head. Thereupon, a scuffle followed.
Afterwards, petitioner approached Rufino, who was then wrestling with Paul, and stabbed Rufino in
different parts of the body. Rufino was taken to a nearby hospital where he later died due to stab
wounds. Licyayo was charged of Homicide with the RTC when he stabbed Rufino in different parts of
the body. The RTC convicted Licyayo guilty of the crime Homicide there being no attending
aggravating or mitigating circumstances. The petitioner appealed contending that sufficient
provocation and intoxication should be taken as mitigating circumstances attendant in the case. He
insisted that there is sufficient provocation because it was the deceased who punched him first and
when the incident ensued he was intoxicated.

ISSUE:
Whether or not sufficient provocation and intoxication should be considered as mitigating
circumstances attendant in the case

HELD:
No. The records do not sufficiently establish who between Rufino and Aron started the brawl which
resulted in the stabbing of Rufino by petitioner. Granting arguendo that there was unlawful
aggression on the part of the victim, it is obvious that immediately he became the underdog, literally
even. He was easily overpowered by the bigger and sober Aron Licyayo. Sufficient provocation
CRIMINAL LAW 1
(Sunday, Section 52) 254

therefore cannot be appreciated in favour of the petitioner. As testified by the police officers, they
said that petitioner indeed was drunk when the scuffle ensued. However, these testimonies alone do
not suffice as proof to appreciate intoxication as a mitigating circumstance. In the case at bar, there
is no plausible evidence showing that the quantity of liquor taken by petitioner was of such quantity
as to affect his mental faculties. On the contrary, the fact that petitioner could recall the details that
transpired during and after his drinking session with friends is the best proof that he knew what he
was doing during the incident. For intoxication to be considered as a mitigating circumstance, it must
be shown that the intoxication impaired the willpower of the accused and that he did not know what
he was doing or could not comprehend the wrongfulness of his acts. The person pleading intoxication
must prove that he took such quantity of alcoholic beverage, prior to the commission of the crime, as
would blur his reason.

PEOPLE V. NABONG

Celino Nabong y Osenar (a.k.a. Salvador Abaquita), Alvin Laguit y Brendo and Nolfe Ladiao (a.k.a. Roel
Salutario),
G.R. No. 172324, 4 April 2007
Chico-Nazario, J.
Topic: Alternative Circumstances (Art. 15, RPC) - Intoxication

FACTS:
The four accused, Celino Nabong (Nabong), Alvin Laguit (Laguit), Nolfe Ladiao (Ladiao) and Arnel
Miraflor (Miraflor), were all construction workers employed as steel men by EEI, a construction firm,
and assigned at its OCW-RCBC Plaza Project located at Ayala Avenue, Makati City. The victim, was a
22-year old accountant employed as junior auditor at the Alba and Romeo Auditing Firm. At about
9pm, all accused went to a videoke bar in Amorsolo Street for a drinking session. Each of them
consumed six bottles of Colt 45 beer. After they stopped drinking at around 11pm, the four accused
proceeded to Ayala Avenue and stopped at a vacant lot in front of the Makati Medical Centre.
At about the same time, the victim was walking towards the centre island near the corner of Buendia
Avenue and Ayala Avenue with her officemate. When the victim’s officemate boarded a bus, Nabong
appeared from the dark portion of the vacant lot and stabbed the victim with a 7-inch metal bar
(kabilya). Simultaneously, Ladiao jumped at the victim and covered her mouth. The victim slumped
on the pavement while Ladiao, Laguit, Nabong and Miraflor crowded around the woman. At around
11:45 that night, the victim’s body was was found along Buendia Avenue near Tindalo Street, her
undergarment pulled down almost exposing her private parts. Her brassiere was torn off leaving her
left breast exposed. Her dress was torn and raised showing her belly. The police officer found the left
side of the victims body heavily bloodied. The centre part of the street was splattered with blood.
There were also drops of blood on the vacant lot where the concrete pipes were located as well as on
the extension of Tindalo Street.

ISSUE:
Whether or not sufficient intoxication should be considered as mitigating circumstances attendant in
the case

HELD:
No. For intoxication to be considered as mitigating circumstance, it must be shown that the
intoxication impaired the will power of the accused and that he did not know what he was doing or
could not comprehend the wrongfulness of his acts. The person pleading intoxication must prove that
he took such quantity of alcoholic beverage, prior to the commission of the crime, as would blur his
reason. Appellants failed to do this. The records are bereft of any evidence that the quantity of
CRIMINAL LAW 1
(Sunday, Section 52) 255

liquor they had taken was of such quantity as to affect their mental faculties. On the contrary, the fact
that appellants could recall details of what had transpired after their drinking session is the best
proof that they knew what they were doing during that occasion. The deception, the device, the place
and manner of perpetrating the crime all point to the fact that appellants had complete control of
their minds.

PEOPLE V. SAN PEDRO

The People of the Philippines vs.


Luisito San Pedro, et al., accused, Artemio Banasihan
G.R. No. L 44274.

Topic: Alternative Circumstances (Art. 15, RPC) - Degree of Instruction

FACTS:
On June 2, 1970 between the barrios of Masaya and Paciano Rizal Municipality of Bay, Laguna,
Felimon Rivera (a jeepney driver) was killed. He died of profuse hemorrhage due to 23 lacerated and
stab wounds and multiple abrasions found on the different parts of the body. Four days prior to the
date of crime, Luisito, Artemio, and others planned to steal the jeep of the victim. They then rented
Rivera's jeep to haul coconuts (with him as driver), where they proceeded to Brgy. Puypuy in Bay,
Laguna. They were joined by Salvador Litan and Rodrigo Esguerra. At Esguerra's signal, Litan hit
Rivera at the nape with a water pipe. Rivera jumped out of the jeep but was chased by San Pedro and
Litan who stabbed him at the back several times with a dagger. Esguerra then drove the jeep and the
group proceeded to Makati, Rizal, where he was joined by Nelson Piso and Antonio Borja. The jeep
was sold for Php 2,000.00 at Cavite. Piso then went to Los Baños after four days and gave San Pedro,
Litan and Banasihan Php 50.00 each. On June 11, 1971, police caught Rodrigo Esguerra. He then
admitted his participation and named his companions. Artemio Banasihan was apprehended in 1972

ISSUE:
Whether or not the aggravating circumstances could be offset by the mitigating circumstance of “Lack
of Instruction”.

HELD:
No. Lack of Instruction is not applicable to the crimes of theft and robbery, much less to the crime of
homicide. The reason is that the robbery and killing are, by their nature, wrongful acts and are
manifestly so to be enlightened, equally as to the ignorant.
The "criteria in determining lack of instruction is not illiteracy alone, but rather lack of sufficient
intelligence." It is significant that neither to the trial court nor to the appellant's counsel has the
mitigating circumstance of lack of instruction entered the mind. No attempt was made to prove it, as
direct proof, not mere inference, is required, and must be invoked in the court below (People vs.
Mongado, et al., 28 SCRA 642, [1969]), the reason being that the trial court can best gauge a person's
level of intelligence from his manner of answering questions in court (People v. Manuel, 29 SCRA 337
[1969]). If the trial court did not consider the mitigating circumstance invoked for the first time here
on appeal, it must be because from appellant's testimony, and even more so from his given occupation
as a merchant (T.S.N., p. 3, Sept. 1, 1975), his alleged lack of intelligence never suggested itself to the
trial court or to his lawyer, as entitling him to the mitigating circumstance of lack of instruction.
CRIMINAL LAW 1
(Sunday, Section 52) 256

PEOPLE VS.YANSON-DUMANCAS

Jeanette (Ginette) Yanson-Dumancas, Pol. Col. Nicolas Torres, Pol. Insp. Adonis Abeto, Pol. Officer
Mario Lamis Y Fernandez, Dominador Geroche Y Mahusay, Jaime Gargallano, Rolando R. Fernandez,
Edwin Divinagracia, Teody Delgado, Cesar Pecha, Charles Dumancas (Acquitted), Pol. Officer Jose
Pahayupan (Acquitted), Vicente Canuday, Jr. (Acquitted), Accused, Jeanette (Ginette) Yanson-
Dumancas, Pol. Col. Nicolas Torres, Pol. Insp. Adonis Abeto, Pol. Officer Mario Lamis Y Fernandez,
Dominador Geroche Y Mahusay, Jaime Gargallano, Rolando R. Fernandez, Edwin Divinagracia, Teody
Delgado, Cesar Pecha, G.R. No. 133527-28 December 13, 1999
Melo, J.

Topic: Principals (Art. 17, RPC)

FACTS:
One Rufino Gargar, Jr. was abducted and detained. He was thereafter shot and killed while
blindfolded, by a group of persons; most of whom were members of the police force, alleged to be
under the direction, and undue influence of one P/Col. Nicolas Torres, and acting upon the
inducement of accused Yanson-Dumancas.

ISSUE:
Whether accused Yanson-Dumancas could be held liable of the crime as a principal by inducement.

HELD:
There are two ways of directly forcing another to commit a crime, namely: (a) by using irresistible
force or (b) by causing uncontrollable fear. There is nothing to conclude that the accused used either
of the methods on the accused-appellants. Likewise, there are two ways of inducing another to
commit a crime, specifically: (a) by giving a prize, or offering a reward or promise, and (b) by using
words of command. There was no evidence to show that the accused committed any of the foregoing
acts prior to the commission of the crime. In order that a person may be convicted as principal by
inducement, the following must be present: (1) the inducement be made with the intention of
procuring the commission of the crime, and (2) such inducement be the determining cause of the
commission by the material executor. There must exist, on the part of the inducer, the most positive
resolution and the most persistent effort to secure the commission of the crime, together with the
presentation to the person induced of the strongest kind of temptation to commit the crime.

PEOPLE VS. MALUENDA


The People of the Philippines vs. Daniel Maluenda alias Dongkoy; Gil Bueno; Raul Mondaga alias
Bobong; and Rodrigo Legarto, Daniel Maluenda and Rodrigo Legarto
G.R. No. 115351, 27 Mar. 1998, 288 SCRA 225
Panganiban, J.
Topic: Principals (Art. 17, RPC)

FACTS:
That on the 19th day of August 1992, at about 9:00 o’clock in the evening, more or less, at Barangay
Diatagon, Municipality of Lianga, Province of Surigao del Sur, Philippines, and within the jurisdiction
of this Honourable Court, the above-named accused, conspiring, confederating and mutually helping
one another, did, then and there, wilfully, unlawfully and feloniously kidnap Engr. Miguel Resus for
the purpose of extorting money from Engr. & Mrs. Resus, and detaining said Engr. Miguel Resus for a
CRIMINAL LAW 1
(Sunday, Section 52) 257

period of four (4) days, to the damage and prejudice of the victim in the amount of P200,000.00,
Philippine Currency.

Appellant Legarto, the Resus couples former part-time driver, denies any criminal involvement in the
kidnapping. He avows that he participated only in the delivery of the ransom money at the insistence
of Dr. Resus herself. The trial court convicted Legarto, Maluenda and Mondaga, holding that they
successfully perpetrated a clear case of kidnapping. The lower court held that Mondaga was the
mastermind of the kidnapping.

ISSUE:
Whether or not Legarto was a co-principal by indispensable cooperation

HELD:
No. Principal by indispensable cooperation is defined by Article 17 of the Revised Penal Code thus:
ART. 17. Principals. The following are considered principals:
xxxxxxxxx
3. Those who cooperate in the commission of the offense by another act without which it would not have
been accomplished.
The appeal is partially meritorious as regards Legarto who, in the light of the evidence presented,
should be held liable only as an accessory.
Legarto may not have had a direct hand in the kidnapping, but he received part of the ransom and
used it to pay off his arrears in his motorcycle loan. Thus, having knowledge of the kidnapping for
ransom and without having directly participated therein, he took part in the crime subsequent to its
commission by profiting from its effects. He may not be the devil with the face of an angel that the
trial court described, but he is definitely not a saint. He is criminally liable as an accessory to the
crime of kidnapping for ransom.

PEOPLE V. MONTEALEGRE
People of the Philippines v Napoleon Montealegre
L-67948, 31 May 1988
Cruz, J.
Topic: Principals (Art. 17, RPC)

FACTS:
Edmundo Abadilla was eating in a restaurant when he detected the smell of marijuana coming from
a nearby table. Intending to call a policeman, he quietly went outside and saw Pfc. Renato
Camantigue. Camantigue joined Abadilla in the restaurant and they both smelled marijuana from the
table of Vicente Capalad and Napoleon Montealegre. Camantigue collared the 2 and said,
“Nagmamarijuana kayo, ano?” He forced them up, holding 1 in each hand but
Capalad pulled out a knife and started stabbing Camantigue at the back. Camantigue let go of
Montealegre to get his gun but Montealegre restrained Camantigue’s hand to prevent the latter from
defending himself. They grappled and fell on the floor. Capalac fled and Camantigue pursued him
firing some shots. Then he stopped and asked to be brought to the hospital. Capalac was found
slumped in the street with a bullet to his chest. Both he and Camantigue died the next day.
Montealegre on the other hand, escaped through the confusion. He was later apprehended.

ISSUE:
Whether or not Montealegre was rightly considered a co-principal for having corroborated with
Capalad in killing the police officer
CRIMINAL LAW 1
(Sunday, Section 52) 258

HELD:
Yes. The two acted in concert, with Capalad actually stabbing Camantigue 7 times and Montealegre
holding on to the victim’s hands to prevent him from drawing the pistol and defending himself, as
Abadilla testified. While it is true that Montealegre did not himself commit the act of stabbing, he was
nonetheless equally guilty thereof for having prevented Camantigue for resisting the attack against
him. Montealegre was a principal by indispensable cooperation under ART. 17 (3) of the RPC.

PEOPLE V. MADALI
People of the Philippines v Pat. Ricarte Madali and Annie Mortel Madali
G.R. Nos. L-67803-04
July 30, 1990
Fernan, C.J.
Topic: Principals (Art. 17, RPC)

FACTS:
Husband and wife Patrolman Ricarte Madali and Annie Mortel Madali appeal from a decision of the
Regional Trial Court finding them guilty beyond reasonable doubt of killing father and son Cipriano
and Felix Gasang, and seriously wounding Agustin Reloj and Cipriano's daughter, Merlinda. The
crimes stemmed from an altercation between the son of the Madali spouses, and the group of Felix
Gasang, who was twenty years old when he was killed.
Madali spouses pray for their acquittal arguing that the lower court erred in: [a] finding Annie Mortel
Madali guilty as principal by direct participation; [b] not finding that the Gasangs and their kins were
motivated by revenge.
Annie’s participation in the crime consisted of beaming her flashlight at the victims and warning her
husband of the presence of other persons in the vicinity

ISSUE:
Whether or not Annie Mortel Madali is guilty as principal by direct participation.

HELD:
No. The Court finds that proof beyond reasonable doubt has not been established as to the existence
of conspiracy between the Madali spouses. While direct proof is not essential to prove conspiracy as
it may be shown by acts and circumstances from which may logically be inferred the existence of a
common design among the accused to commit the offense(s) charged, the evidence to prove the same
must be positive and convincing considering that conspiracy is a facile devise by which an accused
may be ensnared and kept within the penal fold.
With this and the principle that in criminal prosecution, doubts must be resolved in favour of the
accused, as guides, the Court rules that the liability of Annie Mortel Madali with respect to the crimes
committed herein, is only that of an accomplice.
CRIMINAL LAW 1
(Sunday, Section 52) 259

ABEJUELA V. PEOPLE
People of the Philippines and Court of Appeals
G.R. No. 80130, 19 Aug. 1991, 200 SCRA 806

Teehankee J.
Topic: Accomplices (Art. 18, RPC)

FACTS:
This is a case of estafa. Abejuela became close friends with Balo, a Banco Fililipino employee. Abejuela
has an account with Banco Filipino. Balo borrowed Abejuela’s passbook with Banco Filipino and used
it to deposit and withdraw money even amounting to almost 200K. He told Abejuela that what he
was depositing were from the insurance proceeds of his grandfather but that’s as a Banco Filipino
employee he could not open his own account and so had to use Abejuela’s. It was found that Balo
used his being an employee of the bank to post false deposits. Abejuela did not know of this and when
he started getting suspicious, he actually even closed his account with Banco Filipino to prevent Balo
from continuing. Both Balo and Abejuela were charged with estafa. During pendency of the case, Balo
was purportedly killed by the NPA. The trials continued for Abejuela. RTC ruled that Abejuela is an
accomplice to the crime. CA affirmed the RTC decision.
• This is a case of estafa.
• Abejuela became close friends with Balo, a Banco Filipino employee. Abejuela has an account with
Banco Filipino.
• Balo borrowed Abejuela’s passbook with Banco Filipino and used it to deposit and withdraw money,
even amounting to almost 200k. He told Abejuela that what he was depositing were from the
insurance proceeds of his grandfather but that as a Banco Filipino employee he could not open his
own account and so had to use Abejuela’s.
• It was found that Balo used his being an employee of the bank to post false deposits. Abejuela did
not know of this and when he started getting suspicious, he actually even closed his account with
Banco Filipino to prevent Balo from continuing. Both Balo and Abejuela were charged with Estafa.
• During pendency of the case, Balo was purportedly killed by the NPA. The trials continued for
Abejuela.
• RTC ruled that Abejuela is an accomplice to the crime. CA affirmed the rtc decision.
• This is a case of estafa.
• Abejuela became close friends with Balo, a Banco Filipino employee. Abejuela has an account with
Banco Filipino.
• Balo borrowed Abejuela’s passbook with Banco Filipino and used it to deposit and withdraw money,
even amounting to almost 200k. He told Abejuela that what he was depositing were from the
insurance proceeds of his grandfather but that as a Banco Filipino employee he could not open his
own account and so had to use Abejuela’s.
• It was found that Balo used his being an employee of the bank to post false deposits. Abejuela did
not know of this and when he started getting suspicious, he actually even closed his account with
Banco Filipino to prevent Balo from continuing. Both Balo and Abejuela were charged with Estafa.
• During pendency of the case, Balo was purportedly killed by the NPA. The trials continued for
Abejuela.
• RTC ruled that Abejuela is an accomplice to the crime. CA affirmed the rtc decision.
• This is a case of estafa.
• Abejuela became close friends with Balo, a Banco Filipino employee. Abejuela has an account with
Banco Filipino.
• Balo borrowed Abejuela’s passbook with Banco Filipino and used it to deposit and withdraw money,
even amounting to almost 200k. He told Abejuela that what he was depositing were from the
CRIMINAL LAW 1
(Sunday, Section 52) 260

insurance proceeds of his grandfather but that as a Banco Filipino employee he could not open his
own account and so had to use Abejuela’s.
• It was found that Balo used his being an employee of the bank to post false deposits. Abejuela did
not know of this and when he started getting suspicious, he actually even closed his account with
Banco Filipino to prevent Balo from continuing. Both Balo and Abejuela were charged with Estafa.
• During pendency of the case, Balo was purportedly killed by the NPA. The trials continued for
Abejuela.
• RTC ruled that Abejuela is an accomplice to the crime. CA affirmed the rtc decision.

ISSUE:
Whether or not Abejuela is an accomplice to the crime.

HELD:
No. Knowledge of the criminal intent of the principal in this case is essential in order that petitioner
Abejuela can be convicted as an accomplice in the crime of estafa thru falsification of commercial
document. To be convicted as an accomplice, there must be cooperation in the execution of the
offense by previous or simultaneous acts. However, the cooperation which the law punishes is the
assistance rendered knowingly or intentionally, which assistance cannot be said to exist without the
prior cognizance of the offense intended to be committed.

In a number of cases decided by the court, it has been held that knowledge of the criminal intention
of the principal is indispensable in order to hold a person liable as an accomplice. It has been
satisfactorily
established that Banco Filipino suffered damage. Although Abejuela was unaware of the criminal
workings of Balo, he nevertheless contributed to their eventual consummation by recklessly
entrusting his passbook
to Balo and by signing the withdrawal slips. He failed to exercise prudence and care. Therefore he
must be held civilly accountable.

This is a case of estafa.


• Abejuela became close friends with Balo, a Banco Filipino employee. Abejuela has an account with
Banco Filipino.
• Balo borrowed Abejuela’s passbook with Banco Filipino and used it to deposit and withdraw money,
even amounting to almost 200k. He told Abejuela that what he was depositing were from the
insurance proceeds of his grandfather but that as a Banco Filipino employee he could not open his
own account and so had to use Abejuela’s.
• It was found that Balo used his being an employee of the bank to post false deposits. Abejuela did
not know of this and when he started getting suspicious, he actually even closed his account with
Banco Filipino to prevent Balo from continuing. Both Balo and Abejuela were charged with Estafa.
• During pendency of the case, Balo was purportedly killed by the NPA. The trials continued for
Abejuela.
• RTC ruled that Abejuela is an accomplice to the crime. CA affirmed the rtc decision
This is a case of estafa.
• Abejuela became close friends with Balo, a Banco Filipino employee. Abejuela has an account with
Banco Filipino.
• Balo borrowed Abejuela’s passbook with Banco Filipino and used it to deposit and withdraw money,
even amounting to almost 200k. He told Abejuela that what he was depositing were from the
insurance proceeds of his grandfather but that as a Banco Filipino employee he could not open his
own account and so had to use Abejuela’s.
CRIMINAL LAW 1
(Sunday, Section 52) 261

• It was found that Balo used his being an employee of the bank to post false deposits. Abejuela did
not know of this and when he started getting suspicious, he actually even closed his account with
Banco Filipino to prevent Balo from continuing. Both Balo and Abejuela were charged with Estafa.
• During pendency of the case, Balo was purportedly killed by the NPA. The trials continued for
Abejuela.
• RTC ruled that Abejuela is an accomplice to the crime. CA affirmed the rtc decision

PEOPLE VS DOBLE
PEOPLE VS DOCTOLERO
PEOPLE VS DE VERA
GARCES VS PEOPLE
PEOPLE VS TALINGDAN
DIZON-PAMINTUAN VS PEOPLE
PEOPLE VS TABACO
PEOPLE VS VALDEZ
PEOPLE VS SANCHEZ

PEOPLE VS. HERNANDEZ


G.R. No. 6025-26 July 18, 1956

Facts:
Amado Hernandez and several others were accused of the crime of rebellion with multiple murder,
arsons and robberies. They were convicted of the crime and sentenced to suffer the penalty of life
imprisonment.

Issue:
Whether or not there is a complex crime of rebellion with murder.

Ruling:
NO. Under the allegations of the amended information, the murders, arsons and robberies described
therein are mere ingredients to the crime of rebellion allegedly committed by said defendants as
means “necessary” for the perpetration of said offense of rebellion. The crime, therefore, is simple
rebellion.
Motion for bail is GRANTED.

ENRILE VS SALAZAR
G.R. No. 92163 June 5, 1990

Facts:
In the afternoon of February 27, 1990, Senate Minority Floor Leader Juan Ponce Enrile was arrested
by law enforcement officers led by Director Alfredo Lim of the National Bureau of Investigation on
the strength of a warrant issued by Hon. Jaime Salazar of the Regional Trial Court of Quezon City
Branch 103, in Criminal Case No. 9010941.

The warrant had issued on an information signed and earlier that day filed by a panel of prosecutors
composed of Senior State Prosecutor Aurelio C. Trampe, State Prosecutor Ferdinand R. Abesamis and
Assistant City Prosecutor Eulogio Mananquil, Jr., charging Senator Enrile, the spouses Rebecco and
Erlinda Panlilio, and Gregorio Honasan with the crime of rebellion with murder and multiple
frustrated murder allegedly committed during the period of the failed coup attempt from November
CRIMINAL LAW 1
(Sunday, Section 52) 262

29 to December 10, 1990.

Senator Enrile was taken to and held overnight at the NBI headquarters on Taft Avenue, Manila,
without bail, none having been recommended in the information and none fixed in the arrest warrant.
The following morning, February 28, 1990, he was brought to Camp Tomas Karingal in Quezon City
where he was given over to the custody of the Superintendent of the Northern Police District, Brig.
Gen. Edgardo Dula Torres.

On the same date of February 28, 1990, Senator Enrile, through counsel, filed the petition for habeas
corpus herein (which was followed by a supplemental petition filed on March 2, 1990), alleging that
he was deprived of his constitutional rights.

Issue:
Whether the petitioner has committed complex crimes (delito compleio) arising from an offense
being a necessary means for committing another, which is referred to in the second clause of Article
48 of the Revised Penal Code?

Held:
There is one other reason and a fundamental one at that why Article 48 of the Penal Code cannot be
applied in the case at bar. If murder were not complexed with rebellion, and the two crimes were
punished separately (assuming that this could be done), the following penalties would be imposable
upon the movant, namely: (1) for the crime of rebellion, a fine not exceeding P20,000 and prision
mayor, in the corresponding period, depending upon the modifying circumstances present, but never
exceeding 12 years of prision mayor, and (2) for the crime of murder, reclusion temporal in its
maximum period to death, depending upon the modifying circumstances present. In other words, in
the absence of aggravating circumstances, the extreme penalty could not be imposed upon him.
However, under Article 48 said penalty would have to be meted out to him, even in the absence of a
single aggravating circumstance. Thus, said provision, if construed in conformity with the theory of
the prosecution, would be unfavorable to the movant.

The plaint of petitioner's counsel that he is charged with a crime that does not exist in the statute
books, while technically correct so far as the Court has ruled that rebellion may not be complexed
with other offenses committed on the occasion thereof, must therefore be dismissed as a mere flight
of rhetoric. Read in the context of Hernandez, the information does indeed charge the petitioner with
a crime defined and punished by the Revised Penal Code: simple rebellion.

Petitioner finally claims that he was denied the right to bail. In the light of the Court's reaffirmation
of Hernandez as applicable to petitioner's case, and of the logical and necessary corollary that the
information against him should be considered as charging only the crime of simple rebellion, which
is bailable before conviction, that must now be accepted as a correct proposition. But the question
remains: Given the facts from which this case arose, was a petition for habeas corpus in this Court
the appropriate vehicle for asserting a right to bail or vindicating its denial? The criminal case before
the respondent Judge was the normal venue for invoking the petitioner's right to have provisional
liberty pending trial and judgment. The original jurisdiction to grant or deny bail rested with said
respondent. The correct course was for petitioner to invoke that jurisdiction by filing a petition to be
admitted to bail, claiming a right to bail per se by reason of the weakness of the evidence against him.
Only after that remedy was denied by the trial court should the review jurisdiction of this Court have
been invoked, and even then, not without first applying to the Court of Appeals if appropriate relief
was also available there.
CRIMINAL LAW 1
(Sunday, Section 52) 263

The Court reiterates that based on the doctrine enunciated in People vs. Hernandez, the questioned
information filed against petitioners Juan Ponce Enrile and the spouses Rebecco and Erlinda Panlilio
must be read as charging simple rebellion only, hence said petitioners are entitled to bail, before final
conviction, as a matter of right. The Court's earlier grant of bail to petitioners being merely
provisional in character, the proceedings in both cases are ordered remanded to the respondent
Judge to fix the amount of bail to be posted by the petitioners. Once bail is fixed by said respondent
for any of the petitioners, the corresponding bail bond flied with this Court shall become functus
oficio. No pronouncement as to costs.

PEOPLE VS. GARCIA


GR. No. 141125

Facts:
The victim Cleopatra Changlapon who was 19 years old, was on her way home from schoolat around
6:30 pm. On her way home she was abducted, when she awoke she was lying flat naked on a bed and
there were four men in the room. The men raped her. She struggled to escape, but the men burned
her using a cigarette and punched her abdomen whenever she tried to resist. She was sprayed with
something which made her vision blurry and when she woke up she was on the streets. She rode a
cab going home, she told her family what had happened and the next day she was brought to the
police station.

After sometime accused appellant Jeffrey Garcia was captured for another crime of rape filed by Gilda
Mangyo. A police officer found appellant familiar and Cleopatra was asked to come to the police
station. Cleopatra was able to positively identify appellant as one of her assailants. After the trials the
court convicted Garcia for the complex crime of abduction with rape on 3 counts.
Garcia then appeals his case arguing that the court rendered an erroneous judgement. He argues that
the court was wrong in convicting him when the other assailants are unknown, that the cour barely
considered that he was only a look alike of the assailant and that the court was wrong for finding that
victim was able to positive identify him as he has an alibi.

ISSUE:
Whether or not the court has made an erroneous decision.

HELD:
No, the court did not make an erroneous decision. In the current case, Cleopatra was able to positively
identify the assailant. Furthermore, the testimony of Cleopatra and the medical findings were
cohesive. In addition an alibi in a criminal case must not only prove that the accused was not present
at the time of the crime but also that is physically impossible for the accused to be present during the
crime. In the case of Garcia the location where in he was at was near the place where Cleopatra was
taken. In criminal law it is a well-established rule that the positive identification backed by evidence
and testimony free from ill-motive has weight. Thus, appeal was denied.
CRIMINAL LAW 1
(Sunday, Section 52) 264

LEONILA BATULANON VS. PEOPLE OF THE PHILIPPINES


G.R. NO. 139857 September 15, 2006

Facts:
Polomok Credit Cooperative Incorporated (PCCI) employed Leonila Batulanon as its
Cashier/Manager from May 1980 up to December 22, 1982. She was in charge of receiving deposits
from and releasing loans to the member of the cooperative.
During an audit conducted in December 1982, certain irregularities concerning the release of loans
were discovered. It was found that Batulanon falsified four commercial documents, all checks/cash
vouchers representing granted loans to different persons namely: Omadlao, Oracion, Arroyo and
Dennis Batulanon, making it appear that said names were granted a loan and received the amount of
the checks/cash vouchers when in truth and in fact the said persons never received a grant, never
received the checks, and never signed the check vouchers issued in their names. In furtherance,
Batulanon released to herself the checks and received the loans and thereafter misappropriated and
converted it to her own use and benefit.

Thereafter, four Informations for Estafa through Falsification of Commercial Documents were filed
against Batulanon. The prosecution presented Medallo, Gopio, Jr. and Jayoma as witnesses. Medallo,
the posting clerk whose job was to assist Batulanon in the preparation of cash vouchers testified that
Batulanon forged the signatures of Omadlao, Oracion and Arroyo. Gopio, Jr. stated that Oracion is
Batulanon’ sister-in-law and Dennis Batulanon is her son who was only 3 years old in 1982. He
averred that membership in the cooperative is not open to minors.

On April 15, 1993, the trial court rendered a Decision convicting Batulanon of Estafa through
Falsification of Commercial Documents. The Court of Appeals affirmed the decision of the trial court,
hence this petition.

Issue:
Whether the crime committed by Batulanon was Falsification of Private Documents.

Held:
Yes. Although the offense charged in the Information is Estafa through Falsification of Commercial
Documents, Batulanon could be convicted of Falsification of Private Documents under the well-
settled rule that it is the allegation in the information that determines the nature of the offense and
not the technical name given in the preamble of the information.
As there is no complex crime of Estafa through Falsification of Private Documents, it is important to
ascertain whether the offender is to be charged with Falsification of a Private Document or with
Estafa. If the falsification of a private document is committed as a means to commit estafa, the proper
crime to be charged is falsification. If the Estafa can be committed without the necessity of falsifying
a document, the proper crime is Estafa. We find that the Court of Appeals correctly held Batulanon
guilty beyond reasonable doubt of Falsification of Private Documents in the cases of Omadlao,
Oracion and Arroyo.

In the case of Dennis Batulanon, records show that Batulanon did not falsify the signature of Dennis.
What she did was to sign: “by: Ibatulanon” to indicate that she received the proceeds of the loan in
behalf of Dennis. Said act does not fall under any of the modes of Falsification under Article 171
because there is nothing untruthful about the fact that she used the name of Dennis and that as
representative of the latter, obtained the proceeds of the loan from PCCI. The essence of falsification
is the act of making untruthful or false statements, which is not attendant in this case. As to whether,
CRIMINAL LAW 1
(Sunday, Section 52) 265

such representation involves fraud which caused damage to PCCI is a different matter which will
make her liable for estafa, but not for falsification. Hence, it was an error for the courts below to hold
that Batulanon is also guilty of Falsification of Private Document with respect to the case involving
the cash voucher of Dennis Batulanon.

PEOPLE VS MADRIGAL-GONZALES

Facts:
Pacita Madrigal-Gonzalez was charged with malversation of public funds amounting to Php 104,000.
On the same day Pacita and her co-accused where charged with the crime of falsification of public
documents on 27 separate information. The court then ordered that the 27 separate cases for
falsification of public documents be condensed to into one case. Later on the decision was revered
and all 27 cases were distributed to multiple trial courts. Accused then claimed that the other cases
should be quashed based on double jeopardy. Accused claims that the 27 cases filed stemmed from
only one criminal impulse. Furthermore other cases were already decided upon thus the other cases
which are still ongoing is considered as double jeopardy.

Issue:
Whether or not the judgement on the 3 cases would in effect dismiss the other pending cases?
Held:
"In respect of the defense of former jeopardy, made to a prosecution for forgery, it must appear by
the plea that the offense charged in both cases was the same in law and in fact. The plea will be bad
if the offenses charged in the two indictments are perfectly distinct in point of law, however nearly
they may he connected in fact. As to several acts of forgery, each generally constitutes a separate
crime, even though they are committed in the course of a continuous transaction, on the same date,
or even on the same piece of paper, unless each act constitutes merely component part of an
indivisible instrument. * * *." (23 Am, Jur., p. 700-701).

Moreover, under the facts and circumstances appearing In the record, the grounds upon which the
appellees their defense of double jeopardy in the motion to quash, are not clear and indubitable. One
cannot build up the defense of double jeopardy on mere hypothesis.

Wherefore, the Order of the lower court (Branch XVIII) dismissing Criminal Cases Nos. 36894, 36899
and 36904 on the ground of double jeopardy is set aside and another entered remanding the said
case for further proceedings. The Motion for Leave to Withdraw Appeal, presented by the Solicitor
General should be, as it is hereby denied. No special pronouncement as to costs.”

JOSE GAMBOA VS. CA


G.R. No. L-41054 November 28, 1975

Facts:
This is a petition to review on the judgment of CA involving the theory of continuous crime.
Haycu was a former employee of Units optical with 124 complaints of estafa files against him by the
company with the office of city fiscal of manila. after the procedural PI, fiscal filed 75 cases of estafa
against Haycu, except as to the dates and amounts of conversions, the 75 informations commonly
charge that Haycu having collected and received customers of the company the purchase of goods
has converted the sum of money he collected to his own personal account with a bank.
A civil action for accounting was filed also by the owner of the company against Haycu complaining
that Haycu initiated discharging the business functions and prerogatives of the company thru deceit
CRIMINAL LAW 1
(Sunday, Section 52) 266

and machinations that the owner affixing his signatures to the power of attorney to open account in
the bank.

Haycu appealed to the CA to reversed the order of the lower court asserting that the 75 criminal cases
is not only oppressing but also out of the jurisdiction of the city fiscal of manila. Asserting also that
the 75 were mere components of only one crime.

Issue:
Whether or not the accused committed the crime of estafa.

Held:
CA ruled that, with intent Haycu defraud his employer to sign the power of attorney, and the series
of deposits made by Haycu constitutes one crime of estafa, there being one criminal resolution and
the different acts were aimed at accomplishing the purpose of defrauding his employer.

But the characterization of the estafa as continuing offense cannot be validly seize by haycu because
the elements of estafa may separately take place in different jurisdiction until the crime is
consummated and the series of deposits of the company's funds to his personal account is treated
separately from that of the case of the power of attorney. CA is hereby set aside and reversed.

Consuelo Mallari vs People of the Philippines and CA, respondents


G.R. No. L-58886 December 13, 1998

Facts:
Mallari with 3 others was accused of crime of estafa thru falsification of public document before the
court of first instance of manila. Upon arraignment, Mallari pleaded not guilty. Trial was conducted
then court rendered that Mallari is guilty of the crime charged.
Mallari then appeal to CA but resulted to CA affirmation of the CFI's decision with modification as to
the penalty.

Mallari then motion for reconsideration and contended that the decision placed her in jeopardy of
being punished for the same offense. Unconvinced the appellate court denied the motion.

Issue:
whether the accused was placed in double jeopardy?

Ruling:
requisites of double jeopardy are the following: (1) first jeopardy must have attached prior to the
second (2) first jeopardy must have been validly terminated and (3) second jeopardy must be for the
same offense as that in the first.

The question lies on the 3rd requisite because obviously 1st and 2nd is present in the case.The
information filed in the two cases were compared and showed that they refer to the same series of
acts, which is considered as continuing crimes.

Continued crime is a single crime consisting of series of acts but all arising from one criminal
resolution.
CRIMINAL LAW 1
(Sunday, Section 52) 267

The singularity of the offense committed by the petitioner was demonstrated by the fact that the
falsification of the two documents were performed on the same date, place and at the time on the
same occasion.The petitioner having already been convicted of the complex crime of estafa stands to
reason that she can no longer be held liable for the same crime in this case.
Petition granted.

MIRIAM DEFENSOR SANTIAGO, vs. HON. JUSTICE FRANCIS GARCHITORENA, SANDIGANBAYAN


(First Division) and PEOPLE OF THE PHILIPPINES

Facts:
Petitioner was charged in the Sandiganbayan with violation of Sec. 3(e) of RA 3019, Anti-Graft and
Corrupt Practices Act, allegedly committed by her favoring “unqualified” aliens with the benefits of
the Alien Legalization Program. Petitioner filed this case to enjoin Sandiganbayan from proceeding
with the case, on the ground that it was intended solely to harass her as she was then a presidential
candidate. After her petition was dismissed, she then filed a motion for inhibition of Presiding Justice
Garchitorena.

[A lot of procedural issues and controversies were discussed, but for the purpose of limiting this
digest to Criminal Law 1, the author did not include it.]

Petitioner next claims that the Amended Informations did not charge any offense punishable under
Section 3 (e) of R.A. No. 3019 because the official acts complained of therein were authorized under
Executive Order No. 324 and that the Board of Commissioners of the Bureau of Investigation adopted
the policy of approving applications for legalization of spouses and unmarried, minor children of
“qualified aliens” even though they had arrived in the Philippines after December 31, 1983. She
concludes that the Sandiganbayan erred in not granting her motion to quash the informations.
In a motion to quash, the accused admits hypothetically the allegations of fact in the Information.
Therefore, petitioner admitted hypothetically in her motion that: 1) she was a public officer; 2) she
approved the application for legalization of the stay of aliens, who arrived in the Philippines after
January 1, 1984; 3) those aliens were disqualified; 4) she was cognizant of such fact; and 5) she acted
in evident bad faith and manifest partiality in the execution of her official functions; thereby
constituting the elements of the offense defined in Sec. 3(e) of RA 3019.
It bears noting that the public prosecutors filed a total of 32 Informations against the petitioner for
the violation of such law.

Issue:
1. How is the violation of Sec. 3(e) of RA 3019 committed?
2. Whether or not the filing of 32 Amended Informations against petitioner was proper.

Held:
1. There are two ways of violating Section 3 (e) of R.A. No. 3019. These are: (a) by causing undue
injury to any party, including the Government; and (b) by giving any private party any
unwarranted benefit, advantage or preference.
2. NO. Only one crime was committed in petitioner’s case, and hence, there should only be one
Information to be filed against her.

The 32 Amended Informations charge what is known as delito continuado or “continued


CRIMINAL LAW 1
(Sunday, Section 52) 268

crime” and sometimes referred to as “continuous crime.” A delito continuado consists of


several crimes but in reality there is only one crime in the mind of the perpetrator. See full
text for the discussion and examples of delito continuado as discussed by SC.

In the case at bench, the original information charged petitioner with performing a single
criminal act — that of her approving the application for legalization of aliens not qualified
under the law to enjoy such privilege. The original information also averred that the criminal
act : (i) committed by petitioner was in violation of a law — Executive Order No. 324 dated
April 13, 1988, (ii) caused an undue injury to one offended party, the Government, and (iii)
was done on a single day, i.e., on or about October 17, 1988. The 32 Amended Informations
reproduced in verbatim the allegation of the original information, except that instead of the
word “aliens” in the original information each amended information states the name of the
individual whose stay was legalized.

The 32 Amended Informations aver that the offenses were committed on the same period of
time, i.e., on or about October 17, 1988. The strong probability even exists that the approval
of the application or the legalization of the stay of the 32 aliens was done by a single stroke
of the pen, as when the approval was embodied in the same document. Likewise, the public
prosecutors manifested at the hearing the motion for a bill of particulars that the Government
suffered a single harm or injury.

SC ordered the Ombudsman to consolidated the 32 Amended Informations into one


Information charging only one offense.

IVLER vs. HON. MODESTO


G.R. No. 172716, November 17, 2010

FACTS:
Following a vehicular collision in August 2004, petitioner Jason Ivler was charged before the
Metropolitan Trial Court of Pasig City (MeTC), with two separate offenses: (1) reckless imprudence
resulting in slight physical injuries for injuries sustained by respondent Evangeline L. Ponce; and (2)
reckless imprudence resulting in homicide and damage to property for the death of respondent
Ponce’s husband Nestor C. Ponce and damage to the spouses Ponce’s vehicle.
Crimes charged: 1) reckless imprudence resulting in slight physical injuries; and 2) reckless
imprudence resulting in homicide and damage to property
On September 7, 2004, Ivler pleaded guilty to the charge in reckless imprudence resulting in slight
physical injuries and was meted out the penalty of public censure. Invoking this conviction, Ivler
moved to quash the Information of reckless imprudence resulting in homicide and damage to
property for placing him in jeopardy of second punishment for the same offense of reckless
imprudence.
MeTC: denied the motion to quash
RTC: denied Ivler’s Petition for Certiorari in dismissing his Motion to Quash
CRIMINAL LAW 1
(Sunday, Section 52) 269

ISSUE:
Whether or not Ivler’s constitutional right under the Double Jeopardy Clause bars further
proceedings in the information charging him with reckless imprudence resulting in homicide and
damage to property (YES)
Defense: Ivler argues that his constitutional right not to be placed twice in jeopardy of punishment
for the same offense bars his prosecution in reckless imprudence resulting in homicide and damage
to property having been previously convicted in reckless imprudence resulting in slight physical
injuries for injuries for the same offense. Ivler submits that the multiple consequences of such crime
are material only to determine his penalty

HELD:
The Supreme Court reversed the ruling of the RTC. Petitioner’s conviction in the case of reckless
imprudence resulting in slight physical injuries bars his prosecution in criminal reckless imprudence
resulting in homicide and damage to property
1) Reckless Imprudence is a Single Crime; its Consequences on Persons and Property are Material
Only to Determine the Penalty
Quasi-offenses penalize “the mental attitude or condition behind the act, the dangerous recklessness,
lack of care or foresight, the imprudencia punible,” unlike willful offenses which punish the
intentional criminal act. These structural and conceptual features of quasi-offenses set them apart
from the mass of intentional crimes.
2) Prior Conviction or Acquittal of Reckless Imprudence Bars Subsequent Prosecution for the Same
Quasi-offense
Once convicted or acquitted of a specific act of reckless imprudence, the accused may not be
prosecuted again for that same act. For the essence of the quasi-offense of criminal negligence under
Article 365 of the Revised Penal Code lies in the execution of an imprudent or negligent act that, if
intentionally done, would be punishable as a felony. The law penalizes thus the negligent or careless
act, not the result thereof.
The gravity of the consequence is only taken into account to determine the penalty, it does not qualify
the substance of the offense. And, as the careless act is single, whether the injurious result should
affect one person or several persons, the offense (criminal negligence) remains one and the same,
and cannot be split into different crimes and prosecutions.
3) Article 48 Does Not Apply to Acts Penalized Under Article 365 of the Revised Penal Code
Article 48 is a procedural device allowing single prosecution of multiple felonies falling under either
of two categories: (1) when a single act constitutes two or more grave or less grave felonies (thus
excluding from its operation light felonies); and (2) when an offense is a necessary means for
committing the other. The legislature crafted this procedural tool to benefit the accused who, in lieu
of serving multiple penalties, will only serve the maximum of the penalty for the most serious crime.
In contrast, Article 365 is a substantive rule penalizing not an act defined as a felony but the mental
attitude behind the act, the dangerous recklessness, lack of care or foresight, a single mental attitude
regardless of the resulting consequences. Thus, Article 365 was crafted as one quasi-crime resulting
in one or more consequences. Article 48 is incongruent to the notion of quasi-crime resulting in one
or more consequences.
Article 48 is incongruent to the notion of quasi-crimes under Article 365. It is conceptually impossible
for a quasi-offense to stand for (1) a single act constituting two or more grave or less grave felonies;
or (2) an offense which is a necessary means for committing another.
Prosecutions under Article 365 should proceed from a single charge regardless of the number or
severity of the consequences. In imposing penalties, the judge will do no more than apply the
penalties under Article 365 for each consequence alleged and proven. In short, there shall be no
splitting of charges under Article 365, and only one information shall be filed in the same first level
court.
CRIMINAL LAW 1
(Sunday, Section 52) 270

PEOPLE VS BON

PEOPLE OF THE PHILIPPINES, plaintiff-appellee,


vs. CONRADO LUCAS Y BRIONES, accused-appellant.

DAVIDE, JR., J.:


In a sworn statement 1 taken on 16 February 1991, Chanda Lucas y Austria, then seventeen years old,
charged her natural father, accused Jose Conrado Lucas, of attempted rape committed against her on
12 February 1991. She revealed therein that she was first raped by him when she was only nine years
old, or, as disclosed in a handwritten note at the left-hand margin of her sworn statement, "noong
Nov. 26, 1982 . . . at naulit ng maraming beses."
On 19 February 1991, Chanda, assisted by her mother, Ofelia Austria-Lucas, filed two separate sworn
criminal complaints for rape 2 and for attempted rape 3 against her father with the Regional Trial
Court of Quezon City. The complaints, docketed as Criminal Cases Nos. Q-91-18465 and Q-91-18466,
were subsequently assigned to Branch 104 of the said court.
The accusatory portion of the complaint for rape in Criminal Case
No. Q-91-18465 reads:
That on or about the 26th day of November 1982 and sometime thereafter in Quezon City, Philippines
and within the jurisdiction of this Honorable Court, the above-named accused, with lewd designs and
by means of violence and intimidation did then and there, wilfully, unlawfully and feloniously have
sexual intercourse with the undersigned CHANDA LUCAS Y AUSTRIA, who was then nine (9) years
old, now 17 yrs. of age, against her will, to her damage and prejudice in such amount as may be
awarded to her under the provisions of the New Civil Code.
while that for attempted rape in Criminal Case No. Q-91-18466 reads:
That on or about the 12th day of February 1991, in Quezon City, Philippines and within the
jurisdictionof this Honorable Court, the above named accused, did then and there wilfully, unlawfully
and feloniously with lewd design and by means of force and intimidation, commence the commission
of the crime of rape directly by overt acts by then and there taking advantage of complainant's tender
age and innocence, by then and there putting his hand inside the panty of the undersigned and
mashing her vagina while his other hand was pressing her nipples and at the same time kissing her
on the lips, face and neck, thereafter accused placed himself on top of her but said accused did
not perform all the acts of execution which should produce the said offense of rape by reason of the
fact that the brother and sister of the undersigned was awakened and shouted upon the accused, a
cause other than the spontaneous desistance of the said accused, that the aforesaid act of the said
accused was done against the will of the undersigned, to her damage and prejudice in such amount
as may be awarded to her under the provisions of the New Civil Code.
The cases were jointly tried after the accused had pleaded not guilty upon his arraignment. 4 The
prosecution presented as witnesses the complainant herself; her sister, Cynthia; and Dr. Emmanuel
Aranas. The defense presented only the accused.
As found by the trial court and fully supported by the evidence, the accused had carnal knowledge of
his daughter Chanda — then below twelve years old — on 26 November 1983. We are not persuaded
by the arguments of the accused that if indeed she were raped on that date and several times
thereafter, she should not have kept her silence until she was seventeen years old since she had all
the available remedies for redress as well as relatives who could help her. The equanimity or the
wisdom of more mature persons cannot be expected from a young and immature girl like Chanda.
We have said before that the workings of a human mind when placed under emotional stress are
unpredictable and that people react differently to various situations. 30
In addition to her tender age and immaturity, Chanda was, to say the least, a victim of unfavorable
circumstances not of her own making. These prevented her from exposing earlier the evil deeds of
CRIMINAL LAW 1
(Sunday, Section 52) 271

her father. All that she could proudly claim was a beautiful name — Chanda. She had no decent home.
Her father and her mother were not married and were untrammeled by the bonds of lawful wedlock.
When she was born, her father was under detention for gunrunning and it was only when she was
four years old (1977) when he rejoined his "family." Since then, all the members of the family slept
in one room. Chanda had no choice of another home, for it does not appear that another was available
to the family or that she was prepared to leave it because she had the means to face life alone or that
a kind soul had offered her shelter. She was a victim of poverty and a virtual captive in the only
"home" her natural parents could provide, for she was entirely dependent upon them.
Verily, she was completely under the moral ascendancy and control of her father and the fear alone
of a harsher life outside such a "home" and of what her father would do if she would expose his evil
deeds, made her suffer in silence for a long time the excruciating pains his assaults inflicted upon her.
Then too, although she told her mother about the abuse committed by her father on 26 November
1983, 31 her mother only got angry but did not do anything. Chanda must have felt despair at such
indifference.
Her delay in reporting the sexual assaults to the authorities is thus understandable and does not
affect her credibility. We do not believe that she would fabricate a story of defloration against her
own father, make public her painful and humiliating experiences which are better kept in secret or
forgotten, allow her private parts to be examined, and eventually bring to shame her own family and
jeopardize her chances of marriage unless she was not telling the truth and was motivated by nothing
but the desire to obtain justice for the grievous wrongs committed against her. 32
There was a consummated rape on 12 February 1991. According to Chanda's testimony, at 3:00 a.m.
that day, the accused, who had a balisong with him, laid down beside her, threatened her that she had
only one life which he can take away any time; removed her shorts and panty and then moved on top
of her and inserted "his organ to her organ." Thereafter, he stood up holding his balisong and
reiterated his earlier threat. 33
Considering, however, that the complaint for this incident subject of Criminal Case No. Q-91-18466
charges the accused with the crime of attempted rape, then, as correctly pointed out by the accused
in his second assigned error and concurred in by the Office of the Solicitor General, he cannot be
convicted of consummated rape.
Section 4, Rule 120 of the Rules of Court provides that "[w]hen there is variance between the offense
charged in the complaint or information, and that proved or established by the evidence, and the
offense as charged is included in or necessarily includes the offense proved, the accused shall be
convicted of
the offense proved included in that which is charged, or of the offense charged included in that which
is proved." The offense charged in Criminal Case
No. Q-91-18466 (attempted rape) is necessarily included in the offense that was proved
(consummated rape). Accordingly, the accused should be convicted of attempted rape only. The
penalty for attempted rape is prision mayor, which is two degrees lower than that provided by law
for rape. 34 The accused is entitled to the benefits of the Indeterminate Sentence Law, and for
attempted rape he may be sentenced to a penalty whose minimum should be within the range
of prision correccional and whose maximum should be within the range range of prision mayor, taking
into account the modifying circumstances. The alternative circumstance of relationship provided for
in Article 15 of the Revised Penal Code should be appreciated against the accused considering that
the offended party, Chanda, is his descendant. In crimes against chastity, such as rape, relationship is
aggravating. 35
Prior to R.A. No. 7659, 36 the presence of modifying circumstances would not affect the penalty
of reclusion perpetua prescribed for the crime of rape because such a penalty was then indivisible
and under Article 63 of the Revised Penal Code, when the law prescribes a single indivisible penalty,
it shall be applied by the courts regardless of any mitigating or aggravating circumstances that may
have attended the commission of the deed. However, pursuant to Section 21 of R.A. No. 7659, which
CRIMINAL LAW 1
(Sunday, Section 52) 272

amended Article 27 of the Revised Penal Code, reclusion perpetuahas now a defined duration, i.e.,
from twenty (20) years and one (1) day to forty (40) years. There is, however, no corresponding
amendment to Article 76 of the same Code for the purpose of converting reclusion perpetua into a
divisible penalty with three specific
period — minimum, medium, and maximum — and including it in the table provided therein showing
the duration and the time included in each of the periods.
It may thus be said that although the law has now fixed the duration of reclusion perpetua, it did not
make explicit its intention to convert it into a divisible penalty. In any event, Article 65 of the Code
which provides:
Art. 65. Rules in cases in which the penalty is not composed of three periods. — In cases in which the
penalty prescribed by law is not composed of three periods, the courts shall apply the rules contained
in the foregoing articles, dividing into three equal portions of time included in the penalty prescribed,
and forming one period of each of the three portions.
may be applied. Accordingly, the time included in the penalty of reclusion perpetua (twenty [20] years
and one [1] days to forty [40] years) can be divided into three equal portions, with each composing
a period. The periods of reclusion perpetua would then be as follows:
minimum — 20 years and 1 day to 26 years and 8 months
medium — 26 years, 8 months and 1 day to 33 years and 4 months
maximum — 34 years, 4 months and 1 day to 40 years
Taking into account the presence of the aggravating circumstance of relationship in Criminal Case
No. Q-91-18465, the accused may finally be sentenced to thirty-four (34) years, four (4) months and
one (1) day of reclusion perpetua.
Considering again such aggravating circumstance, the accused may be sentenced in Criminal Case No.
Q-91-18466 to an indeterminate penalty ranging from four (4) years, two (2) months and one (1)
day of prision correccional maximum as minimum to ten (10) years and one (1) day of prision
mayor maximum as maximum.
There should also be awards for damages in each of the two cases.
WHEREFORE, the challenged Decision of 28 October 1992 of Branch 104 of the Regional Trial Court
of Quezon City in Criminal Case
No. Q-91-18465 and Criminal Case No. Q-91-18466 is hereby AFFIRMED, subject to the modifications
indicated above. As modified:
(1) In Criminal Case No. Q-91-18465, accused JOSE CONRADO LUCAS y BRIONES is hereby sentenced
to suffer the penalty of Thirty-four (34) years, Four (4) months and One (1) day of reclusion
perpetua and to pay the offended party the sum of P50,000.00 as civil indemnity; and
(2) In Criminal Case No. Q-91-18466, said accused is hereby found GUILTY beyondreasonable doubt
of the crime of ATTEMPTED RAPE only and is hereby sentenced to suffer an indeterminate penalty
ranging from Four (4) years, TWO (2) months and One (1) day of prision correccional maximum
as minimum to Ten (10) years and one (1) day ofprision mayor maximum as maximum and to pay
the offended party the sum of P30,000.00 as civil indemnity.
CRIMINAL LAW 1
(Sunday, Section 52) 273

PEOPLE OF THE PHILIPPINES, plaintiff-appellee,


vs. CONRADO LUCAS Y BRIONES, accused-appellant.

In the decision in this case, promulgated on 25 May 1994, the First Division touched on the nature of
the penalty of reclusion perpetua in the light of Section 21 of R.A. No. 76591 which amended Article
27 of the Revised Penal Code by specifically fixing the duration of reclusion perpetua at twenty (20)
years and one (1) day to forty (40) years. It opined that since no corresponding amendment to Article
76 of the Revised Penal Code was made, the said laws has not made explicit an intention to
convert reclusion perpetua into a divisible penalty. Nevertheless, it applied Article 65 of the Revised
Penal Code2 and stated:
Accordingly, the time included in the penalty of reclusion perpetua (twenty [20] years and one [1]
day to forty [40] years) can be divided into three equal portions with each composing a period. The
periods of reclusion perpetua would then be as follows:
minimum — 20 years and 1 day to 26 years and 8 months
medium — 26 years, 8 months and 1 day to 33 years and 4 months
maximum — 34 years, 4 months and 1 day to 40 years

Taking into account the presence of the aggravating circumstance of relationship in Criminal Case
No. Q-91-18465, the accused may finally be sentenced to thirty-four (34) years, four (4) months and
one (1) day of reclusion perpetua.
It then modified the challenged decision of the trial court by changing the penalty in Criminal Case
No. Q-91-18465 from reclusion perpetua, as imposed by the trial court, to "imprisonment of 34 years,
4 months and 1 day of reclusion perpetua."
In a motion for clarification seasonably filed by the appellee on 28 June 1994 which was not opposed
by the accused-appellant in his comment, the appellee asks the Court to correct the duration of the
maximum period of reclusion perpetua from thirty-four (34) years, four (4) months and one (1) day
to forty (40) years, as stated in the decision, to thirty-three (33) years, four (4) months and one (1)
day to forty (40) years.
Since the issue of whether the amendment of Article 27 of the Revised Penal Code by Section 21 of
R.A. No. 7659 has made reclusion perpetua a divisible penalty is one of first impression and of
sufficient importance, the First Division referred the motion for clarification to the Court en banc. The
latter accepted the referral.
After deliberating on the motion and re-examining the legislative history of R.A. No. 7659, the Court
concludes that although Section 17 of R.A. No. 7659 has fixed the duration of reclusion perpetua from
twenty (20) years and one (1) day to forty (40) years, there was no clear legislative intent to alter its
original classification as an indivisible penalty. It shall then remain as an indivisible penalty.
R.A. No. 7659 is a consolidation of Senate Bill (SB) No. 8913 and House Bill (HB) No. 62.4 SB No. 891
seeks to amend Article 27 of the Revised Penal Code by inserting therein what are to be considered
heinous crimes and to penalize these not with the death penalty, but which reclusion perpetua only,
with the qualification that "any person sentenced to reclusion perpetua for . . . [such heinous] crimes
under this Code shall be required to serve thirty (30) years, without entitlement to good conduct time
allowance and shall be considered for executive clemency only after service of said thirty (30) years."
HB No. 62 defines and enumerates the heinous crimes and seeks to penalize them with the death
penalty.
An amendment by substitution to SB No. 891 was introduced by the Senate Special Committee on
Death Penalty. The amendment was entitled "An Act to Impose the Death Penalty on Certain Heinous
Crime, Amending for that Purpose some Articles of Act No. 3815, as Amended, and for other Purposes."
CRIMINAL LAW 1
(Sunday, Section 52) 274

The substitute amendment sought to amend (a) Article 25 of the Revised Penal Code by providing in
the scale of penalties the following:

CAPITAL PUNISHMENT: DEATH


Afflictive Penalties: LIFE IMPRISONMENT
Reclusion Perpetua
Reclusion Temporal
and (b) Article 27 of the same Code by inserting therein the penalty of life imprisonment and
providing a specific duration therefor as well as for reclusion perpetua. The proposed amended
Article 27 pertinently reads as follows:
Art. 27. LIFE IMPRISONMENT. — THE PENALTY OF LIFE IMPRISONMENT SHALL BE FROM THIRTY
YEARS AND ONE DAY TO FORTY YEARS.
RECLUSION PERPETUA — THE PENALTY OF RECLUSION PERPETUA SHALL BE FROM TWENTY
YEARS AND ONE DAY TO THIRTY YEARS.
Thus, life imprisonment, therefore a penalty imposed by special penal statutes, was sought to be
incorporated as penalty in the revised Penal Code with a specific duration.
In his sponsorship of this substitute bill, Senator Arturo M. Tolentino explained the incorporation of
life imprisonment as follows:
But a very basic amendment was made, and that is, an amendment that will create a new penalty,
known in this bill as life imprisonment. The new penalty was created in order to enable the
committee to provide, in some crimes, a three-grade penalty that would be composed of reclusion
perpetua, as now provided by the Revised Penal Code, as the lowest grade; on top of that, would be
life imprisonment; and the third highest grade would be death penalty. With this new grade of
penalty, it became possible for this bill now under consideration to impose a penalty ranging
from reclusion perpetua to death, composed of actually three periods or
grades.5

However the Bicameral Conference Committee eliminated from the proposed amendment of Article
27 the penalty of life imprisonment but extended the duration of reclusion perpetua from twenty (2)
years and one (1) day to forty (40) years. Thus, in his sponsorship of the Conference Committee
report on both the substitute SB No. 891 and HB No. 62, Senator Tolentino stated:
By this, Mr. President, we have this new consolidated session that is before the Members of this
Chamber. There is one part or one portion of the Senate version that we have agreed to be eliminated
and that is the creation of the new penalty known as "life imprisonment." Even in this Chamber, there
were some doubts as to the creation of this new penalty of life imprisonment because reclusion
perpetua, which is in the Revised Penal Code and retained in this bill, also means the same thing. It is
a perpetual imprisonment.
So in order to still accommodate the increase of imprisonment by means of life imprisonment —
while we eliminated the new penalty of life imprisonment which would last from 30 years and one
day to forty years — what we did was simply to extend the period of reclusion perpetua by adding 30
to 40 years imprisonment to the original 20 to 30 years, making the reclusion perpetua in this new
bill range from 20 years to one day to 40 years. This would be what we had called one day before a
"flexible or divisible penalty."6
CRIMINAL LAW 1
(Sunday, Section 52) 275

PEOPLE OF THE PHILIPPINES, plaintiff-appellee, vs. GERARDO LATUPAN y SIBAL, alias


JERRY, accused-appellant.

The case is an appeal from the decision[1] of the Regional Trial Court, Tuao, Cagayan, Branch 11
convicting Gerardo Latupan y Sibal, alias Jerry of the complex crime of double murder and sentencing
him to life imprisonment and to indemnify the heirs of the two victims in the amount of fifty thousand
(P50,000.00) pesos each. The court also convicted accused Gerardo Latupan of inflicting physical
injuries to Jaime Asuncion, and sentenced him to ten days imprisonment and to pay two hundred
(P200.00) pesos as indemnity. On April 13, 1992, Provincial Prosecutor Alejandro A. Pulido of
Cagayan filed with the Regional Trial Court, Tuao, Cagayan four separate informations charging
Gerardo Latupan y Sibal alias Jerry with two counts of frustrated murder and two counts of murder,
committed as follows:

Criminal Case No. 379-T


That on or about April 29, 1991, in the Municipality of Tuao, Province of Cagayan, and within the
jurisdiction of this Honorable Court, the said accused, Gerardo Latupan alias Jerry, armed with a
pointed knife, with intent to kill, with evident premeditation and with treachery did then and there
willfully, unlawfully and feloniously attack, assault, box, maul, kick and hit with his aforesaid arm one
Leo Asuncion, inflicting upon him injuries on the different parts of his body.
That the accused had performed all the acts of execution which would have produced the crime of
Murder as a consequence but which, nevertheless, did not produce it by reason of causes independent
of his own will.
Contrary to law.[2]

Criminal Case No. 380-T


That on or about April 29, 1991, in the Municipality of Tuao, Province of Cagayan, and within the
jurisdiction of this Honorable Court, the said accused, Gerardo Latupan alias Jerry, armed with a
pointed knife, with intent to kill, with evident premeditation and with treachery did then and there
willfully, unlawfully and feloniously attack, assault, box, maul and kick one, Jaime Asuncion inflicting
upon him injuries on the different parts of his body.
That the accused had performed all the acts of execution which would have produced the crime of
Murder as a consequence but which, nevertheless, did not produce it by reason of causes
independent of his own will.

Criminal Case No. 381-T


That on or about April 29, 1991, in the Municipality of Tuao, Province of Cagayan, and within the
jurisdiction of this Honorable Court, the said accused, Gerardo Latupan alias Jerry, armed with a
pointed knife, with intent to kill, with evident premeditation and with treachery did then and there
willfully, unlawfully and feloniously attack, assault and stab one, Jose Asuncion inflicting upon him
stab wound on his body which caused his death.
Contrary to law.

Criminal Case No. 382-T


That on or about April 29, 1991, in the Municipality of Tuao, Province of Cagayan, and within the
jurisdiction of this Honorable Court, the said accused, Gerardo Latupan alias Jerry, armed with a
pointed knife, with intent to kill, with evident premeditation and with treachery did then and there
willfully, unlawfully and feloniously attack, assault and stab one, Lilia Asuncion inflicting upon her
stab wounds on her body which caused her death.
CRIMINAL LAW 1
(Sunday, Section 52) 276

Contrary to law.[5]
At the arraignment on May 25, 1993, accused pleaded not guilty to the charge of frustrated
murder.[6] During the pre-trial conference of the four cases, accused offered to change his plea of not
guilty to guilty of the complex crime of double murder and frustrated murder. The prosecution did
not interpose any objection. Thus, on July 20, 1993, the trial court re-arraigned the accused. He
withdrew his plea of not guilty and instead pleaded guilty to the single offense of multiple murder
with multiple frustrated murder.[7]

Thereafter, the trial court ordered the prosecution to present evidence to establish the culpability of
the accused.

The facts are as follows:


On April 29, 1991, at around 4:00 in the afternoon, Ceferino Dagulo (hereafter Ceferino) was
chopping firewood outside his house in Angang, Tuao, Cagayan. Suddenly, he heard the shouts of a
woman and a child coming from the north.
Moments later, Ceferino saw accused Gerardo Latupan y Sibal walking in his direction, carrying a
thin, bloodied knife. Accused Latupan entered the house of Ceferino and started chasing Ceferinos
wife, who was able to run to another house nearby. Unable to catch Ceferinos wife, accused Latupan
turned to Ceferino and said, I will kill you all. At that time, accused Latupans clothes, chest, hands and
legs were full of blood. Accused Latupan attempted to thrust the knife into Ceferino, who was able to
parry it. Later on, accused Latupan told Ceferino to bring him to the authorities and tried to give the
knife to Ceferino. Ceferino refused to touch the knife and told accused to go to the authorities by
himself. Hearing this advice, accused ran away.
The house of Emilio Asuncion (hereafter Emy) was 100 meters from Ceferinos house. At around 4:00
in the afternoon of the same day, Emy Asuncion was returning to his house from a store. He reached
his house and found his wife, Lilia, dead on the ground with several stab wounds on her body. His
one-year old son, Leo, was lying on top of Lilia Asuncion. Emy picked up Leo and saw that the left side
of Leos face was lacerated. He saw Jaime, his three-year old son and asked where Jose, his eldest son,
was. At that moment, Emy heard the voice of Jose from upstairs of the house, asking for medicine. He
ran upstairs and saw that Jose was wounded. He asked Jose who stabbed him. Jose replied, Uncle
Jerry, Tatang. Seeing that Jose needed immediate medical treatment, Emy brought him to the house
of Ceferino and then returned to his house to get his two other children, Leo and Jaime. They left the
corpse of Lilia Asuncion inside Emys house.
Lilia Asuncion was the sister of Ceferinos wife.

Meanwhile, Ceferino tried to ask a barangay councilman for assistance. Failing to obtain assistance,
Ceferino went back to his house and found Emy Asuncion and his children there. Then, Ceferino went
to a military camp to borrow a vehicle to bring the children to the hospital. The military men provided
them with a jeep. Thus, the three children were taken to the Nuestra Seora de Piat Hospital in
Cabalansan. Riding in the jeep were five soldiers, the accused Latupan, Emy Asuncion, Ceferino
Dagulo, Ceferinos wife, and the three children, Leo, Jaime and Jose Asuncion.
During the trip to the hospital, Emys son, Jose, saw accused Latupan inside the jeep. Jose pointed to
accused Latupan as the one who stabbed him.
At the hospital, the doctors treated the injuries of Leo and Jaime. However, the doctors advised Emy
and Ceferino to bring Jose to another hospital due to the seriousness of his wounds. So, they
proceeded to Cagayan Valley Regional Hospital. Sadly, Jose was dead on arrival.[8] He was only nine
years old.
CRIMINAL LAW 1
(Sunday, Section 52) 277

Jaime, 5 year-old son of Emy Asuncion, testified that he was three years old when the incident
occurred. He stated that accused Latupan stabbed his mother, stepped on him, threw his brother, Leo,
outside the window and stabbed his other brother, Jose.[9]
After presenting testimonial and documentary evidence, the prosecution rested its case. The defense
did not present any testimonial or documentary evidence, merely relying on accuseds plea of
guilty. Thus, the case was considered submitted for decision.

WHEREFORE, the Court AFFIRMS the decision of the Regional Trial Court, Tuao, Cagayan, Branch 11
in Criminal Case Nos. 112453-56 with MODIFICATION. The accused-appellant Gerardo Latupan y
Sibal is convicted of two counts of murder, for the death of Lilia Asuncion and Jose Asuncion, and is
sentenced to reclusion perpetua in each case, and to indemnify the heirs of Lilia and Jose Asuncion in
the amount of P50,000.00 pesos, each case, and in addition thereto, the amount of P50,000.00 pesos,
each case, as moral damages. Accused-appellant is further convicted of two counts of slight physical
injuries and is sentenced to twenty (20) days of arresto menor, in each case, plus P2,000.00 pesos as
indemnity to each of the two victims, Jaime and Leo Asuncion.
Costs against accused-appellant. SO ORDERED.

PEOPLE OF THE PHILIPPINES, Plaintiff-Appellee,


vs. RODEL LANUZA y BAGAOISAN, Accused-Appellant.

On appeal is the Decision1 dated April 27, 2009 of the Court of Appeals in CA-G.R. CR. No. 31406,
which affirmed the Judgment2 dated January 30, 2008 of Branch 14 of the Regional Trial Court (RTC)
of Laoag City in Criminal Case No. 13388-14, finding accused-appellant Rodel Bagaoisan Lanuza
guilty beyond reasonable doubt of the crime of frustrated homicide. The RTC, taking into
consideration the mitigating circumstance of voluntary surrender and applying the indeterminate
sentence law, sentenced accused-appellant to imprisonment from four (4) years of prision
correccional, as minimum, to seven (7) years of prision mayor, as maximum.

The criminal information, charging accused-appellant with the crime of frustrated homicide, as
defined and penalized under Article 249 in relation to Article 6 of the Revised Penal Code, reads:
That on or about the 1st day of April 2007 in the City of Laoag, Philippines, and within the jurisdiction
of this Honorable Court, the said accused, with intent to kill, did then and there willfully, unlawfully
and feloniously attack, assault and shoot by the use of a 12 gauge shotgun, Joel G. Butay, performing
all the acts of execution which would produce the crime of homicide as a consequence, but which
nevertheless did not produce it by reason of causes independent of the will of the accused and that is
the timely medical attention extended to said Joel G. Butay.3
On July 11, 2007, accused-appellant was arraigned and he pleaded not guilty to the criminal charge.4
During the preliminary conference, the parties admitted, among other facts, that accused-appellant
shot private complainant Joel G. Butay; that as a result of the shooting, private complainant sustained
a gunshot wound which caused his confinement at the provincial hospital for 12 days; that accused-
appellant voluntarily surrendered to the Philippine National Police (PNP), Laoag City, surrendering
a shotgun, five live bullets, and one empty shell; and that private complainant suffered actual
damages amounting to ₱70,000.00. Accused-appellant, however, asserted that the shooting was
accidental, as contemplated under Article 12(4) of the Revised Penal Code, which exempts from
criminal liability "any person who, while performing a lawful act with due care, causes injury by mere
accident without fault or intention of causing it." Private complainant, however, insisted that
accused-appellant pulled the trigger of the gun with the intention of killing him.
CRIMINAL LAW 1
(Sunday, Section 52) 278

In view of accused-appellant’s assertion of an exempting circumstance in his favor, the RTC, in its
Pre-Trial Order5dated July 26, 2007, ordered a reverse trial of the case.
Thereafter, trial ensued.
Accused-appellant testified on November 21, 2007,6 while private complainant testified on
November 29, 2007.7The RTC summarized the evidence presented by the parties as follows:
From the defense evidence, it appears that the incident subject of this case took place at the basement
of the BIR office in Laoag City in the morning of April 1, 2007, while the private complainant as
outgoing security guard was handing his shotgun to the accused, the incoming security guard.
Because the accused did not report for duty on the scheduled time, the private complainant
reprimanded him. After the accused had affixed his signature on the pertinent portion of the logbook
enumerating the items turned-over to him by the outgoing security guard, the private complainant
handed to him their service firearm, a shotgun. Allegedly, the private complainant held it with both
hands, with the muzzle pointed at him and the butt towards the accused. At that moment, the accused
gripped the firearm with one hand, with his pointer finger inside the trigger guard and on top of the
trigger itself. In his affidavit which was adopted as part of his direct testimony, the accused stated
that "I immediately held opposite the muzzle of the gun where the trigger is, I almost slip with it while
in the act of gripping and then immediately the gun went off; the incident happened so fast that I was
stunned then realized that I accidentally shot my fellow guard." The private complainant was hit on
the left side of his waist. With the private complainant bleeding and unconscious, the accused went
to the telephone upstairs to call for an ambulance. There, however, the accused heard the sound of a
motorcycle leaving the BIR premises. He went down and discovered that the private complainant
was no longer at the place where he had left him. The accused, thereafter, proceeded to the Laoag
City police station and surrendered.
The prosecution presented a different scenario. According to the private complainant, he did not
actually hand the shotgun to the accused. Instead, he merely placed it, together with one bullet, on
top of the security guard’s table. Although he was turning over six bullets to the accused, the private
complainant asserted that the five others were inside a drawer on the security guard’s table at their
office upstairs. While the private complainant who was about to go home was asking why the accused
did not report on his scheduled shift, the latter got the shotgun, placed the ammunition inside it, and
shot him. The private complainant fell down on his buttocks. The accused went near the private
complainant and pulled the trigger a second time, but the shotgun did not fire and the private
complainant heard only a click. The accused ran upstairs, and the private complainant crawled to his
motorcycle and drove it himself to the provincial hospital. The medical certificate issued by his
attending physician, Dr. Frankie Pete Albano, shows that the private complainant sustained the
following:
"- Gunshot wound 3cm. in diameter left lumbar area thru and thru left paravertebral area
- Fractured spleen / Hemoperitoneum 100 cc thru and thru left kidney (2 points)."
The medical certificate also indicated that exploratory laparotomy was conducted on the private
complainant, his spleen was repaired, and a drain was placed on his left perirenal area.8
At the end of the trial, the RTC promulgated its Judgment dated January 30, 2008, finding accused-
appellant guilty beyond reasonable doubt of the crime charged. The dispositive portion of the said
Judgment reads:
WHEREFORE, the accused RODEL LANUZA y BAGAOISAN is hereby found GUILTY beyond reasonable
doubt of frustrated homicide under Article 249 in relation to Article 6 of the Revised Penal Code and,
with the mitigating circumstance of voluntary surrender, is hereby sentenced to an indeterminate
penalty ranging from four years of prision correccional as minimum to seven years of prision mayor
as maximum. He is further ordered to pay the private complainant ₱70,000.00 as actual damages and
₱25,000.00 as moral damages. Costs against the accused.9
Accused-appellant filed his Appellant’s Brief10 before the Court of Appeals on July 23, 2008 to assail
the foregoing judgment of conviction rendered against him by the RTC.
CRIMINAL LAW 1
(Sunday, Section 52) 279

In his Brief, accused-appellant maintained that he shot private complainant by mere accident. In the
event the Court of Appeals is not convinced that accused-appellant acted with due care, one of the
elements for the exempting circumstance of accident under Article 12(4) of the Revised Penal Code,
accused-appellant urged the appellate court to impose upon him a sentence in accord with Article 67
of the same Code, which specifically provided for the "[p]enalty to be imposed when not all the
requisites of exemption of the fourth circumstance of Article 12 are present."
In the alternative, accused-appellant contended in his Brief that, at the most, he could only be held
accountable for the crime of physical injuries in the absence of proof of his intent to kill private
complainant.
Accused-appellant argued that if he really had the intent to kill, he could have shot private
complainant with precision. Accused-appellant claimed that private complainant’s version of events
immediately after the latter was shot was incredible. By private complainant’s own admission,
accused-appellant did not say anything to him, did not hit him with the gun, and did not kick him
while he sat on the floor after being shot. Private complainant even pleaded for help from accused-
appellant after sustaining the gunshot wound.
Accused-appellant further raised doubts as to the credibility of private complainant given the
inconsistencies in the latter’s testimony. The private complainant allegedly testified that he placed
the shotgun and one bullet on top of the security guard’s table for turn-over to accused-appellant.
The five other bullets for the shotgun were in a drawer in another security guard’s table on the upper
floor. Private complaint claimed to have seen accused-appellant load one bullet in the shotgun.
However, during cross-examination, private complainant said that all six bullets for the shotgun
could not be seen during the turn-over. Thus, private complainant could not have seen accused-
appellant load any bullet into the shotgun. Private complainant also initially narrated that he was
about to board his motorcycle when he was shot by accused-appellant; yet, when cross-examined,
private complainant stated that he had already boarded his motorcycle at the time he was shot.
In its Brief11 filed on November 27, 2008, plaintiff-appellee People of the Philippines countered with
the following arguments:
I.
ACCUSED-APPELLANT FAILED TO PROVE THAT HE IS ENTITLED TO THE EXEMPTING
CIRCUMSTANCE OF ACCIDENT.
II.
THE PROSECUTION PROVED BEYOND REASONABLE DOUBT THAT THE OFFENSE COMMITTED WAS
A RESULT OF A DELIBERATE AND INEXCUSABLE ACT.
III.
ACCUSED-APPELLANT WAS CORRECTLY FOUND BY THE TRIAL COURT GUILTY OF THE CRIME OF
FRUSTRATED HOMICIDE.12

Questions on whether or not private complainant had actually seen accused-appellant load the
shotgun with a bullet, or whether or not private complainant was already on board his motorcycle
when he was shot by accused-appellant, would have no bearing on the fact that private complainant
was shot by accused-appellant with the service shotgun turned-over by the former to the latter. The
Court stresses that accused-appellant himself admitted the fact of shooting, and only disputed any
intent to kill private complainant. The conclusion of the RTC, as affirmed by the Court of Appeals and
this Court, that accused-appellant intended to kill private complainant was not based entirely on
accused-appellant deliberately loading the shotgun, but also on the existence of motive on accused-
appellant’s part, the location and severity of private complainant’s injury, and accused-appellant’s
behavior immediately after the shooting.

Finally, the Court likewise sustains the penalty and damages imposed against accused-appellant.
CRIMINAL LAW 1
(Sunday, Section 52) 280

The penalty prescribed by law for the crime of frustrated homicide is one degree lower than that
prescribed by law for the crime of homicide.25 Under the indeterminate sentence law, the maximum
of the sentence shall be that which could be properly imposed in view of the attending circumstances,
and the minimum shall be within the range of the penalty next lower to that prescribed by the Revised
Penal Code.

Considering that the penalty prescribed by law for the crime of homicide is reclusion temporal, the
penalty for the crime of frustrated homicide would be prision mayor. Applying the indeterminate
sentence law, there being the mitigating circumstance of voluntary surrender and no aggravating
circumstance, the maximum of the sentence should be within the range of prision mayor in its
minimum term which has a duration of six (6) years and one (1) day to eight (8) years, and that, on
the other hand, the minimum should be within the range of prision correccional which has a duration
of six (6) months and one (1) day to six (6) years. Thus, the imposition of imprisonment from four
(4) years of prision correccional, as minimum, to seven (7) years of prision mayor, as maximum, is in
order. There is similarly no reason for the Court to disturb the award of damages made by the court
a quo. Accused-appellant shall compensate private complainant for actual damages in the amount of
₱70,000.00 as the parties voluntarily stipulated during the pre-trial conference that private
complainant incurred actual expenses in said amount because of his injuries. Accused-appellant shall
also be liable for moral damages suffered by private complainant in the amount of ₱25,000.00, in
accordance with jurisprudence. WHEREFORE, the instant appeal of accused-appellant is DENIED for
lack of merit and the Decision dated April 27, 2009 of the Court of Appeals in CA-G.R. CR. No. 31406
is AFFIRMED.
SO ORDERED.

TALAMPAS VS PEOPLE

By petition for review on certiorari, Virgilio Talampas y Matic (Talampas) seeks the review of the
affirmance of his conviction for homicide (for the killing of the late Ernesto Matic y Masinloc) by the
Court of Appeals (CA) through its decision promulgated on August 16, 2007.[1]

The Regional Trial Court, Branch 25, in Bian, Laguna (RTC) had rejected his pleas of self-defense and
accident and had declared him guilty of the felony under the judgment rendered on June 22, 2004.[2]

The information filed on November 17, 1995, to which Talampas pleaded not guilty, averred as
follows: That on or about July 5, 1995, in the Municipality of Bian, Province of Laguna, Philippines
and within the jurisdiction of this Honorable Court, accused VIRGILIO TALAMPAS, with intent to kill,
while conveniently armed with a short firearm and without any justifiable cause, did then and there
willfully, unlawfully and feloniously attack, assault and shoot one Ernesto Matic y Masinloc with the
said firearm, thereby inflicting upon him gunshot wound at the back of his body which directly caused
his instantaneous death, to the damage and prejudice of his surviving heirs.

CONTRARY TO LAW.

The State presented as witnesses Jose Sevillo, Francisco Matic, Jerico Matic, Dr. Valentin Bernales,
and Josephine Matic. The CA summarized their testimonies thuswise:[4]

Prosecution witness Jose Sevillo (Jose) who allegedly witnessed the incident in question, testified
that on July 5, 1995 at about 7:00 oclock in the evening, he together with Eduardo Matic (Eduardo)
and Ernesto Matic (Ernesto) were infront of his house, along the road in Zona Siete (7), Wawa,
Malaban, Bian, Laguna, repairing his tricycle when he noticed the appellant who was riding on a
CRIMINAL LAW 1
(Sunday, Section 52) 281

bicycle passed by and stopped. The latter alighted at about three (3) meters away from him, walked
a few steps and brought out a short gun, a revolver, and poked the same to Eduardo and fired it hitting
Eduardo who took refuge behind Ernesto. The appellant again fired his gun three (3) times, one shot
hitting Ernesto at the right portion of his back causing him (Ernesto) to fall on the ground with his
face down. Another shot hit Eduardo on his nape and fell down on his back (patihaya). Thereafter,
the appellant ran away, while he (Jose) and his neighbors brought the victims to the hospital. On June
6, 1995, Jose executed a Sworn Statement at the Bian Police Station.

Another witness, Francisco Matic, testified that prior to the death of his brother Ernesto who was
then 44 years old, he (Ernesto) was driving a tricycle on a boundary system and earned P100.00
daily, although not on a regular basis because sometimes Ernesto played in a band for P100.00 per
night.

Jerico Matic, eldest son of Ernesto, alleged that he loves his father and his death was so painful to him
that he could not quantify his feelings in terms of money. The death of his father was a great loss to
them as they would not be able to pursue their studies and that nobody would support them
financially considering that the money being sent by their mother in the amount of P2,000.00
to P2,500.00 every three (3) months, would not be enough.

Dr. Valentin Bernales likewise, testified that he was the one who conducted the autopsy on the body
of Ernesto and found one gunshot in the body located at the back of the costal area, right side, sixteen
(16) centimeters from the spinal column. This shot was fatal as it involved the major organs such as
the lungs, liver and the spinal column which caused Ernestos death.

The last witness, Josephine Matic, wife of Ernesto, testified that her husband was laid to rest on July
18, 1995 and that his untimely death was so painful and that she could not provide her children with
sustenance. She asked for the amount of P200,000.00 for her to be able to send her children to school.

On his part, Talampas interposed self-defense and accident. He insisted that his enemy had been
Eduardo Matic (Eduardo), not victim Ernesto Matic (Ernesto); that Eduardo, who was then with
Ernesto at the time of the incident, had had hit him with a monkey wrench, but he had parried the
blow; that he and Eduardo had then grappled for the monkey wrench; that while they had grappled,
he had notice that Eduardo had held a revolver; that he had thus struggled with Eduardo for control
of the revolver, which had accidentally fired and hit Ernesto during their struggling with each other;
that the revolver had again fired, hitting Eduardo in the thigh; that he had then seized the revolver
and shot Eduardo in the head; and that he had then fled the scene when people had started swarming
around.

Ruling of the RTC

On June 22, 2004, the RTC, giving credence to the testimony of eyewitness Jose Sevilla, found
Talampas guilty beyond reasonable doubt of homicide,[5] and disposed:

WHEREFORE, premises considered, the court finds the accused guilty beyond reasonable doubt of
the crime of Homicide, with one mitigating circumstance of voluntary surrender, and hereby
sentences him to suffer an indeterminate penalty of IMPRISONMENT ranging from TEN (10) years
and One (1) day of prision mayor, as minimum, to FOURTEEN (14) years and EIGHT (8) months of
reclusion temporal, as maximum. He is likewise ordered to pay the heirs of Ernesto Matic y Masinloc
the following sums, to wit:
CRIMINAL LAW 1
(Sunday, Section 52) 282

1. P50,000.00 as and for death indemnity;


2. P50,000.00 as and for moral damages;
3. P25,000.00 as and for actual damages; and
4. P30,000.00 as and for temperate damages.

Furnish Public Prosecutor Nofuente, Atty. Navarroza, the private complainant and accused with a
copy of this decision.

SO ORDERED.

Ruling of the CA

Talampas appealed to the CA, contending that:

I
THE COURT A QUO GRAVELY ERRED IN FINDING THAT THE GUILT OF THE ACCUSED-APPELLANT
FOR THE CRIME CHARGED HAS BEEN PROVEN BEYOND REASONABLE DOUBT.

II
THE COURT A QUO GRAVELY ERRED IN NOT FINDING THAT THE DEATH OF ERNESTO MATIC WAS
MERELY ACCIDENTAL.

III
THE COURT A QUO GRAVELY ERRED IN NOT FINDING THAT THE ACCUSED-APPELLANT ACTED IN
DEFENSE OF HIMSELF WHEN HE GRAPPLED WITH EDUARDO MATIC.

Still, the CA affirmed the conviction based on the RTCs factual and legal conclusions, and ruled that
Talampas, having invoked self-defense, had in effect admitted killing Ernesto and had thereby
assumed the burden of proving the elements of self-defense by credible, clear and convincing
evidence, but had miserably failed to discharge his burden.[7]

The CA deleted the award of temperate damages in view of the awarding of actual damages, pointing
out that the two kinds of damages were mutually exclusive.[8]
Issue

Hence, Talampas is now before the Court, continuing to insist that his guilt was not proven beyond
reasonable doubt, and that the lower courts both erred in rejecting his claim of self-defense and
accidental death.

Ruling

The Court finds to be unnecessary the increment of one day as part of the minimum term of the
indeterminate sentence. It may be true that the increment did not constitute an error, because the
minimum term thus fixed was entirely within the parameters of the Indeterminate Sentence Law. Yet,
the addition of one day to the 10 years as the minimum term of the indeterminate sentence of
Talampas may occasion a degree of inconvenience when it will be time for the penal administrators
concerned to consider and determine whether Talampas is already qualified to enjoy the benefits of
the Indeterminate Sentence Law. Hence, in order to simplify the computation of the minimum penalty
CRIMINAL LAW 1
(Sunday, Section 52) 283

of the indeterminate sentence, the Court deletes the one-day increment from the minimum term of
the indeterminate sentence.
WHEREFORE, the Court AFFIRMS the decision promulgated on August 16, 2007 finding VIRGILIO
TALAMPAS y MATICguilty beyond reasonable doubt of the crime of homicide, and IMPOSES the
indeterminate sentence of 10 years of prision mayor, as minimum, to 14 years, eight months, and one
day of reclusion temporal, as maximum.

The petitioner shall pay the costs of suit.


SO ORDERED.

PEOPLE OF THE PHILIPPINES, appellee,


vs. BETH TEMPORADA, appellant.

Before us for review is the February 24, 2006 Decision1 of the Court of Appeals (CA), affirming with
modification the May 14, 2004 Decision2 of the Regional Trial Court (RTC) of Manila, Branch 33,
convicting accused-appellant Beth Temporada of the crime of large scale illegal recruitment, or
violation of Article 38 of the Labor Code, as amended, and five (5) counts of estafa under Article 315,
par. (2)(a) of the Revised Penal Code (RPC).

The antecedents, as found by the appellate court, are as follows:


From September 2001 to January 2002, accused Rosemarie "Baby" Robles, Bernadette Miranda,
Nenita Catacotan and Jojo Resco and appellant Beth Temporada, all employees of the Alternative
Travel and Tours Corporation (ATTC), recruited and promised overseas employment, for a fee, to
complainants Rogelio Legaspi, Jr. as technician in Singapore, and Soledad Atle, Luz Minkay, Evelyn
Estacio and Dennis Dimaano as factory workers in Hongkong. The accused and appellant were then
holding office at Dela Rosa Street, Makati City but eventually transferred business to Discovery Plaza,
Ermita, Manila. After complainants had submitted all the requirements consisting of their respective
application forms, passports, NBI clearances and medical certificates, the accused and appellant, on
different dates, collected and received from them placement fees in various amounts, viz: a) from
Rogelio Legaspi, Jr. – 57,600.00; b) from Dennis Dimaano – P66,520.00; c) from Evelyn Estacio –
P88,520.00; d) from Soledad Atle – P69,520.00 and e) from Luz Minkay – P69,520.00. As none of them
was able to leave nor recover the amounts they had paid, complainant lodged separate criminal
complaints against accused and appellant before the City Prosecutor of Manila. On November 29,
2002, Assistant City Prosecutor Restituto Mangalindan, Jr. filed six (6) Informations against the
accused and appellant, one for Illegal Recruitment in Large Scale under Article 38 (a) of the Labor
Code as amended, and the rest for five (5) counts of estafa under Article 315 paragraph 2 (a) of the
Revised Penal Code.

The Information for large scale illegal recruitment reads:


Criminal Case No. 02-208371:
"The undersigned accuses ROSEMARIE "BABY" ROBLES, BERNADETTE M. MIRANDA, BETH
TEMPORADA, NENITA CATACOTAN and JOJO RESCO x x x.
That in or about and during the period comprised between the months of September 2001 and
January 2002, inclusive, in the City of Manila, Philippines, the said accused, representing themselves
to have the power and capacity to contract, enlist and transport Filipino workers for employment
abroad, did then and there willfully, unlawfully for a fee, recruit and promise employment to
REGELIO A. LEGASPI, JR., DENNIS T. DIMAANO, EVELEYN V. ESTACIO, SOLEDAD B. ATTE and LUZ
CRIMINAL LAW 1
(Sunday, Section 52) 284

MINKAY without first having secured the required license from the Department of Labor and
Employment as required by law, and charge or accept directly or indirectly from said complainant[s]
the amount of PH57,600.00, PH66,520.00, PH88,520.00, PH69,520.00, PH69,520.00, respectively, as
placement fees in consideration for their overseas employment, which amounts are in excess of or
greater than that specified in the scheduled of allowable fees prescribed of the POEA and without
reasons and without fault of the said complainants, failed to actually deploy them and failed to
reimburse them the expenses they incurred in connection with the documentation and processing of
their papers for purposes of their deployment.
Contrary to law."
Except for the name of private complainant and the amount involved, the five (5) Informations
for estafacontain substantially identical averments as follows:
Criminal Case No. 02-208372:
"The undersigned accuses ROSEMARIE "BABY" ROBLES, BERNADETTE M. MIRANDA, BETH
TEMPORADA, NENITA CATACOTAN and JOJO RESCO x x x.
That in or about and during the period comprised between November 23, 2001 and January 12, 2002,
inclusive, in the City of Manila, Philippines, the said accused, conspiring and confederating together
and helping one another, did then and there willfully, unlawfully and feloniously defraud ROGELIO
A. LEGASPI, JR., in the following manner, to wit: the said accused, by means of false manifestations
and fraudulent representations which they made to said ROGELIO A. LEGASPI, JR., prior to and even
simultaneous with the commission of the fraud, to the effect that they have the power and capacity
to recruit and employ ROGELIO A. LEGASPI, JR., as technician in Singapore and could facilitate the
processing of the pertinent papers if given the necessary amount to meet the requirements thereof,
induced and succeeded in inducing said ROGELIO A. LEGASPI, JR., to give and deliver, as in fact he
gave and delivered to said accused the amount of P57,600.00 on the strength of said manifestations
and representations said accused well knowing that the same were false and fraudulent and were
made solely for the purpose of obtaining, as in fact they did obtain the amount of P57,600.00, which
amount, once in their possession, with intend to defraud, they willfully, unlawfully and feloniously
misappropriated, misapplied and converted the same to their own personal use and benefit, to the
damage and prejudice of said ROGELIO A. LEGASPI, JR. in the aforesaid amount of P57,000.00
Philippine Currency.
Contrary to law."
The other four (4) Informations for estafa involve the following complainants and amounts:
1. DENNIS T. DIMAANO P66,520.00

2. EVELYN V. ESTACIO P88,520.00

3. SOLEDAD B. ATLE P69,520.00

4. LUZ T. MINKAY P69,520.003


Only appellant was apprehended and brought to trial, the other accused remained at large. Upon
arraignment, appellant pleaded not guilty and trial on the merits ensued. After joint trial, on May 14,
2004, the RTC rendered judgment convicting appellant of all the charges:
WHEREFORE, the prosecution having established the GUILT of accused Beth Temporada BEYOND
REASONABLE DOUBT, judgment is hereby rendered CONVICTING the said accused, as principal of
the offenses charged and she is sentenced to suffer the penalty of LIFE IMPRISONMENT and a fine of
Five Hundred Thousand Pesos (P500,000.00) for illegal recruitment; and the indeterminate penalty
of four (4) years and two (2) months of prision correctional as minimum, to nine (9) years and one
(1) day of prision mayor, as maximum for the estafa committed against complainant Rogelio A.
Legaspi, Jr.; the indeterminate penalty of four (4) years and two (2) months of prision correctional as
CRIMINAL LAW 1
(Sunday, Section 52) 285

minimum to ten (10) years and one day of prision mayor as maximum each for the estafas committed
against complainants, Dennis Dimaano, Soledad B. Atte and Luz T. Minkay; and the indeterminate
penalty of four (4) years and two (2) months of prision correctional as minimum, to eleven (11) years
and one (1) day of prision mayor as maximum for the estafa committed against Evelyn Estacio.
The accused is also ordered to pay jointly and severally the complainants actual damages as follows:
1. Rogelio A. Legaspi Jr. P57,600.00

2. Dennis T. Dimaano 66,520.00

3. Evelyn V. Estacio 88,520.00

4. Soledad B. Atte 66,520.00

5. Luz T. Minkay 69,520.00

WHEREFORE, the Decision of the Court of Appeals is MODIFIED with respect to the indeterminate
penalties imposed on appellant for the five (5) counts of estafa, to wit:
(1) In Criminal Case No. 02-208372, the accused is sentenced to an indeterminate penalty of 4 years
and 2 months of prisión correccional as minimum, to 9 years, 8 months and 21 days of prisión
mayor as maximum.
(2) In Criminal Case Nos. 02-208373, 02-208375, and 02-208376, the accused is sentenced to an
indeterminate penalty of 4 years and 2 months of prisión correccional as minimum, to 10 years, 8
months and 21 days of prisión mayor as maximum for each of the aforesaid three estafa cases.
(3) In Criminal Case No. 02-208374, the accused is sentenced to an indeterminate penalty of 4 years
and 2 months of prisión correccional as minimum, to 12 years, 8 months and 21 days of reclusión
temporal as maximum.
In all other respects, the Decision of the Court of Appeals is AFFIRMED.
SO ORDERED.

IN THE MATTER OF THE PETITION FOR HABEAS CORPUS OF PETE C. LAGRAN,


PETE C. LAGRAN, petitioner.

On April 18, 1994, petitioner Pete C. Lagran was convicted by the Regional Trial Court of Quezon City
of three (3) counts of violation of Batas Pambansa (BP) Blg. 22. He was sentenced to suffer
imprisonment of one (1) year for each count and to pay a fine of P125,000.00, with subsidiary
imprisonment in case of insolvency.[1] He appealed the decision of the trial court to the Court of
Appeals but the appeal was dismissed on July 11, 1997 for failure to file appellant's brief. The decision
became final and executory on August 6, 1997 and entry of judgment was made on March 5, 1998.[2]

By virtue of a Commitment Order issued by Hon. Elsa I. De Guzman, Presiding Judge, Regional Trial
Court of Quezon City, Branch 93, petitioner was committed to the Quezon City Jail on February 24,
1999.[3] On April 3, 1999, he was transferred to the New Bilibid Prison[4] where he has been serving
his sentence until the present.

Petitioner filed the instant petition for habeas corpus on March 19, 2001. He prayed for his immediate
release as he had allegedly completed the service of his sentence. Citing Article 70 of the Revised
Penal Code, he argued that if the penalties or sentences imposed on the accused are identical, and
such penalties or sentences emanated from one court and one complaint, the accused shall serve
CRIMINAL LAW 1
(Sunday, Section 52) 286

them simultaneously. He stated that he has been incarcerated for two (2) years and four (4) days,
counted from February 28, 2001, thus, his detention in the New Bilibid Prison is now without legal
basis.

Petitioner's argument deserves scant consideration.


Section 70 of the Revised Penal Code provides:
ART. 70. Successive service of sentences.--When the culprit has to serve two or more penalties, he shall
serve them simultaneously if the nature of the penalties will so permit; otherwise, the following rules
shall be observed:
In the imposition of the penalties, the order of their respective severity shall be followed so that they
may be executed successively or as nearly as may be possible, should a pardon have been granted as
to the penalty or penalties first imposed, or should they have been served out.

For the purpose of applying the provisions of the next preceding paragraph the respective severity
of the penalties shall be determined in accordance with the following scale:
1. Death,
2. Reclusion perpetua,
3. Reclusion temporal,
4. Prision mayor,
5. Prision correccional,
6. Arresto mayor,
7. Arresto menor,
8. Destierro,
9. Perpetual absolute disqualification,
10. Temporary absolute disqualification,
11. Suspension from public office, the right to vote and be voted for, the right to follow profession or
calling, and
12. Public censure.

Notwithstanding the provisions of the rule next preceeding, the maximum duration of the convict's
sentence shall not be more than threefold the length of time corresponding to the most severe of the
penalties imposed upon him. No other penalty to which he may be liable shall be inflicted after the
sum total of those imposed equals the same maximum period.
Such maximum period shall in no case exceed forty years.

In applying the provisions of this rule the duration of perpetual penalties (penal perpetua) shall be
computed at thirty years.

Article 70 of the Revised Penal Code allows simultaneous service of two or more penalties only if the
nature of the penalties so permit.[5] The penalties that can be simultaneously served are: (1)
perpetual absolute disqualification, (2) perpetual special disqualification, (3) temporary absolute
disqualification, (4) temporary special disqualification, (5) suspension, (6) destierro, (7) public
censure, (8) fine and bond to keep the peace, (9) civil interdiction, and (10) confiscation and payment
of costs. These penalties, except destierro, can be served simultaneously with imprisonment. The
penalties consisting in deprivation of liberty cannot be served simultaneously by reason of the nature
of such penalties.[6] Where the accused is sentenced to two or more terms of imprisonment, the terms
should be served successively.[7]

In the case at bar, petitioner was sentenced to suffer one year imprisonment for every count of the
offense committed. The nature of the sentence does not allow petitioner to serve all the prison terms
CRIMINAL LAW 1
(Sunday, Section 52) 287

simultaneously. Applying the rule on successive service of sentence, we find that petitioner has not
yet completed the service of his sentence as he commenced serving his sentence only on February
24, 1999. His prayer, therefore, for the issuance of a writ of habeas corpus has no basis.
IN VIEW WHEREOF, the petition is DISMISSED.

COLINARES VS. PEOPLE OF THE PHILIPPINES (2011)


G.R. No. 182748December 13, 2011

Facts:
Arnel Colinares was charged and found guilty beyond reasonable doubt of frustrated homicide by the
RTC of Camarines Sur. He was sentenced to suffer imprisonment from two years and four months of
prison correccional, as minimum, to six years and one day of prison mayor, as maximum. Since the
maximum probationable imprisonment under the law was only up to six years, Arnel did not qualify
for probation. On appeal by Colinares, the Court of Appeals sustained the RTC’s decision. Unsatisfied
with the Court of Appeal’s decision, petitioner then appealed to the Supreme Court and took the
position that he should be entitled to apply for probation in case the Court metes out a new penalty
on him that makes his offense probationable, which was strongly opposed by the Solicitor General
reiterating that under the Probation Law, no application for probation can be entertained once the
accused has perfected his appeal from the judgment of conviction. The Supreme Court, however,
found that Colinares is guilty of attempted homicide and not of frustrated homicide.

Issue:
Whether or not Arnel Colinares may still apply for probation on remand of the case to the trial court

Ruling:
Yes, The Supreme Court ruled that Colinares may apply for probation upon remand of his case to the
RTC. Ordinarily, an accused would no longer be entitled to apply for probation, he having appealed
from the judgment of the RTC convicting him for frustrated homicide. But in this case the Supreme
Court ruled to set aside the judgment of the RTC and found him only liable for attempted homicide,
if the Supreme Court follows the established rule that no accused can apply for probation on appeal,
the accused would suffer from the erroneous judgment of the RTC with no fault of his own, therefore
defying fairness and equity.

FRANCISCO V. CA
G.R. No. 108747. April 6, 1995

FACTS:
Pablo Francisco was accused of multiple grave oral defamation by his employees. The Metropolitan
Trial Court of Makati sentenced him of prision correccional in its minimum period in each crime
committed on each date of each case. Francisco then elevated the case to the RTC in which they
sentenced him only of eight straight months for appreciating mitigating circumstances.

Francisco failed to make an appeal on the RTC’s decision making it final. The MTC issued a warrant
of arrest, but before Francisco was to be arrested, he filed an application for probation which the MTC
denied. He went to the Court of Appeals on certiorari which was also denied.

ISSUE: Whether Pablo Francisco is still qualified to avail of probation.


CRIMINAL LAW 1
(Sunday, Section 52) 288

RULING:
No. Petitioner is no longer eligible for probation. First, Francisco violated Sec.4 of the Probation Law
in which no application for probation shall be entertained after the judgement is final.

Second, Francisco misunderstood when he thought that his prison sentence held by the MTC was not
qualified for probation. Multiple prison terms should not be added up. Consequently, Francisco lost
his right to probation when he appealed the MTC decision to the RTC. The law considers appeal and
probation mutually exclusive remedies.

Third, Francisco’s appeal to the RTC was not for reducing his penalties but for his assertion of his
innocence. The Probation Law prevent opportunism when petitioners apply for probation when their
appeal was dismissed.

Lastly, the application for probation was filed way beyond the period allowed by law.

LAGROSA V. PEOPLE
G.R. No. 152044. July 3, 2003

Decision on : PETITION FOR REVIEW IN THE DECISION OF THE COURT OF APPEALS RE CA-GR
NOS 67308 and THE DENIED MOTION FOR RECONSIDERATION

FACTS:
Petitioners were convicted for violation of the Revised Forestry Code and sentenced to suffer
imprisonment with minimum periods of prision correccional to maximum of prision mayor. Raised
in the Court of Appeals, decision was affirmed although modified by reducing the sentence to
indeterminate penalty ranging from prision correccional, minimum to its maximum period. Said
decision became final and executory.
Petitioners after conviction, applied for probation but was denied both by the lower court and Court
of Appeals so as the Motion for Reconsideration filed by the same.

ISSUES:
WHETHER OR NOT THE APPLICATION FOR PROBATION WAS MERITORIOUS?
WHETHER OR NOT BOTH COURTs ERRED IN THEIR DECISION BY DENYING THE APPLICATION FOR
PROBATION APPLIED BY BOTH PETITIONERS?

SC RULINGS:

1. PD 968 as amended by PD 1990 allows application for probation in a conviction on appeal only if
the accused has not served his sentence yet. When petitioners perfected their appeal, ipso facto they
relinquished alternative remedy for probation. The contention of the petitioners that their right to
apply for probation only accrued by the time Court of Appeals released its’ decision is deemed
without merit. They could have applied for probation solely, but having to raise appeal to prove their
innocence precluded them to avail such privilege.

2. Petitioners failed to establish that the appeal they made in their conviction was due to erroneous
penalty imposed by the court. An accused, who although already eligible does not at once apply for
probation but did so after his appeal on his conviction failed clearly denotes intent to make the
probation as an escape. Both courts issued sound decision for denying their application .
CRIMINAL LAW 1
(Sunday, Section 52) 289

SERMONIA V CA
GR No. 109454 (1994)

Facts:
26 May 1992, Jose C. Sermonia was charged with bigamy before the RTC of Pasig for contracting
marriage with Ma. Lourdes Unson on 15 February 1975 while his prior marriage to Virginia C.
Nievera remained valid and subsisting. Petitioner moved to quash the information on the ground that
his criminal liability has been extinguished by prescription. RTC denied motion to quash. Petitioner
filed a petition for certiorari and prohibition to the CA assailing that since the second marriage
contract was duly registered with the Office of the Civil Registrar in 1975, such fact of registration
makes it a matter of public record and thus constitutes notice to the offended party as of 1975, and
that prescription commenced to run on the day the contract was registered. For this reason, the
information should have been filed on or before 1990. He also holds that the second marriage
ceremony was held at Our Lady of Nativity Church in Marikina and was open to inspection by any
interested party. The prosecution maintains that the prescriptive period does not begin from the
commission of the crime but from the time of discovery by the complainant which was in July 1991.

Issue:
WON prescription applies in cases of bigamy;
WON prescription commences at the time of registration

Ratio:
NO; the rule on constructive notice in civil cases may be applied in criminal actions if the factual and
legal circumstances warrant, BUT, it cannot apply in the crime of bigamy because a bigamous
marriage is generally entered into by the offender in secrecy in order to conceal his legal impediment,
that even though his second marriage may be contracted in an open place, it may be done so in a place
and among people who do not know of his original subsisting marriage.

NO; it is reasonable that the prescriptive period for the crime of bigamy should be counted only from
the day on which the said crime was discovered by the offended party, the authorities or their agency,
otherwise, the prosecution of such offense would be impossible and would encourage a fearless
violation of a social institution cherished and protected by law.

REPUBLIC V. DESIERTO
[G.R. No. 136506. August 23, 2001]

Facts:
Before us is a petition for certiorari[1] which seeks to annul the Review and Recommendation[2] dated
August 6, 1998 of Graft Investigation Officer I Emora C. Pagunuran, approved by Ombudsman Aniano
A. Desierto, dismissing the petitioners complaint in OMB-0-90-2808 against private respondents for
violation of Republic Act No. 3019 otherwise known as the Anti-Graft and Corrupt Practices Act as
well as the Order[3] dated September 25, 1998 denying petitioners subsequent motion for
reconsideration of the said Review and Recommendation.
The complaint alleged, inter alia, that respondent Cojuangco, Jr., taking advantage of his close
relationship with then President Marcos, had caused the latter to issue favorable decrees to advance
his personal and business interests, had caused the government through the National Investment
Development Corporation (NIDC) to enter into a contract with him under terms and conditions
grossly disadvantageous to the government, and, in conspiracy with the aforenamed members of the
CRIMINAL LAW 1
(Sunday, Section 52) 290

UCPB Board of Directors, in flagrant breach of the fiduciary duty as administrator-trustee of the
Coconut Industry Development Fund (CIDF), manipulated the said Fund resulting in the successful
siphoning of Eight Hundred Forty Million Seven Hundred Eighty-Nine Thousand Eight Hundred Fifty-
Five Pesos and Fifty-Three Centavos (P840,789,855.53) of CIDF to his own corporation, the
Agricultural Investors, Inc. (AII); and that respondents were directly or indirectly interested for
personal gain or had material interest in the transactions requiring the approval of a board, panel or
group of which they were members, in violation of the Anti-Graft and Corrupt Practices Act to the
grave damage and prejudice of public interest, the Filipino people, the Republic of the Philippines,
and the coconut farmers.

Issue:
WHETHER THE OMBUDSMAN ACTED WITH GRAVE ABUSE OF DISCRETION IN DECLARING THAT
THE OFFENSE CHARGED IN THE COMPLAINT FOR VIOLATION OF R.A. NO. 3019 HAD ALREADY
PRECRIBED WHEN THE COMPLAINT WAS FILED.
WHETHER THE OMBUDSMAN ACTED WITH GRAVE ABUSE OF DISCRETION IN DECLARING THAT
THERE IS NO BASIS TO INDICT PRIVATE RESPONDENTS FOR VIOLATION OF THE ANTI-GRAFT LAW
BASED ON THE CONTRACT IN QUESTION.

Held:
Statutes regulating procedure of the courts will be construed as applicable to actions pending and
undetermined at the time of their passage. In that context and in view of the retroactive application
of procedural laws, the instant petition should thus be considered timely filed.

On the matter of prescription, before B.P. Blg. 195, which was approved on March 16, 1982, the
prescription period for violation of the Anti-Graft Practices Act was ten (10) years. The complaint for
violation of R.A. No. 3019 was filed before the PCGG on February 12, 1990 or more than fifteen (15)
years after the birth of the allegedly illegal contract.

As a rule, if the commission of the crime is known, the prescriptive period shall commence to run on
the day it was committed.[32] However, in cases where the time of commission is unknown,
prescription shall only run from its discovery and institution of judicial proceedings for its
investigation and punishment. Ordinarily, there is no problem in determining the date when the
crime consists of a series of acts, especially when some or all of these acts are innocent in themselves.

Yes. We do not subscribe to the Ombudsmans view that P.D. Nos. 961 and 1468 ipso facto served to
insulate the private respondents from prosecution. The legislative imprimatur allegedly granted by
the then President Marcos to the MOA is not necessarily inconsistent with the existence of a violation
of R.A. No. 3019.

The task to determine and find whether probable cause to charge the private respondents exists
properly belongs to the Ombudsman. We only rule that the Office of the Ombudsman should not have
dismissed the complaint on the basis of prescription which is erroneous as hereinabove
discussed. The Ombudsman should have given the Solicitor General the opportunity to present his
evidence and then resolve the case for purposes of preliminary investigation. Failing to do so, the
Ombudsman acted with grave abuse of discretion.

This Courts pronouncement in the case of Domingo v. Sandiganbayan is quite relevant and instructive
as to the date when the discovery of the offense should be reckoned, thus:
CRIMINAL LAW 1
(Sunday, Section 52) 291

In the present case, it was well-nigh impossible for the government, the aggrieved party, to have
known the violations committed at the time the questioned transactions were made because both
parties to the transactions were allegedly in conspiracy to perpetrate fraud against the
government. The alleged anomalous transactions could only have been discovered after the February
1986 Revolution when one of the original respondents, then President Ferdinand Marcos, was ousted
from office. Prior to said date, no person would have dared to question the legality or propriety of
those transactions. Hence, the counting of the prescriptive period would commence from the date of
discovery of the offense, which could have been between February 1986 after the EDSA Revolution and
26 May 1987 when the initiatory complaint was filed.

ROMUALDEZ V. MARCELO
July 28, 2006

FACTS:
Petitioner is being charged with violations of Section 7 of RA No. 3019 for failure to file his Statements
of Assets and Liabilities for the period 1967-1985 during his tenure as Ambassador Extraordinary
and Plenipotentiary and for the period 1963-1966 during his tenure as Technical Assistant in the
Department of Foreign Affairs.
The said offenses were said to be discovered by SolGen Francisco Chavez on May 8, 1987, the date
when he filed a complaint against the petitioner with the PCGG.
The preliminary investigations conducted by the PCGG over the 24 offenses ascribed to Romualdez
(of failure to file annual statements of assets and liabilities) were held invalid by the court for lack of
jurisdiction of said offenses.
The nullity of the proceedings initiated by then Solicitor General Chavez in 1987 with the PCGG and
by the PCGG with the Sandiganbayan in 1989 was judicially settled. The only proceeding that could
interrupt the running of prescription of the 1987 case is that which is filed or initiated by the offended
party before the appropriate body or office.
The complaint was filed with the wrong body, the PCGG. Thus, the same could not have interrupted
the running of the prescriptive periods.
On March 3, 2004, Office of the Special Prosecutor initiated the preliminary investigation of Crim
Case Nos. 13406-13429 by requiring the petitioner to submit his counter-affidavit.

ISSUE:
Whether the filing of the complaint with the PCGG as well as the filing of the information with the
Sandiganbayan to initiate Criminal Case interrupted the running of the prescriptive period such that
when the Ombudsman directed petitioner to file his counter-affidavit, the offenses have already
prescribed.

HELD: YES.
Section 11 of RA No. 3019 provides that all offenses punishable therein shall prescribe in 15 years.
However, prior to the amendment of Section 11 of R.A. No. 3019 by B.P. Blg. 195 which was approved
on March 16, 1982, the prescriptive period for offenses punishable under the said statute was only
ten (10) years.
For offenses allegedly committed by the petitioner from 1962 up to March 15, 1982, the same shall
prescribe in 10 years. On the other hand, for offenses allegedly committed by the petitioner during
the period from March 16, 1982 until 1985, the same shall prescribe in 15 years.

BUT Since the petitioners were absent in the Philippines from 1986 to April 27, 2000, the
respondents were saying that the charge could not have prescribed citing Article 91 of the Revised
CRIMINAL LAW 1
(Sunday, Section 52) 292

Penal Code which provides that "[t]he term of prescription should not run when the offender is absent
from the Philippine Archipelago."
This was supported by the dissent of Justice Carpio. He stated that Article 10 of the same Code makes
Article 91 "x x x supplementary to [special laws], unless the latter should x x x provide the contrary."
Nothing in RA 3019 prohibits the supplementary application of Article 91 to that law. Hence, applying
Article 91, the prescriptive period in Section 11 of RA 3019, before and after its amendment, should
run only after petitioner returned to this jurisdiction on 27 April 2000. There is no gap in the law.
Where the special law is silent, Article 10 of the RPC applies suppletorily.
HOWEVER, Court ruled that the law on prescription of offenses is found in Articles 90 and 91 of the
Revised Penal Code for offenses punishable thereunder. For those penalized under special laws, Act
No. 3326 applies. Section 2 of Act No. 3326 provides that the prescription shall begin to run from the
day of the commission of the violation of the law, and if the same be not known at the time, from the
discovery thereof and the institution of judicial proceedings for its investigation and punishment.
The running of the prescriptive period shall be interrupted when proceedings are instituted against
the guilty person, and shall begin to run again if the proceedings are dismissed for reasons not
constituting jeopardy. Clearly, Section 2 of Act No. 3326 did not provide that the absence of the
accused from the Philippines prevents the running of the prescriptive period. Thus, the only
inference that can be gathered from the foregoing is that the legislature, in enacting Act No. 3326, did
not consider the absence of the accused from the Philippines as a hindrance to the running of the
prescriptive period. Expressio unius est exclusio alterius.

In view of the foregoing, the applicable 10-and-15-year prescriptive periods in the instant case, were
not interrupted by any event from the time they began to run on May 8, 1987. The offenses committed
by the petitioner for the years 1963-1982 prescribed 10 years from May 8, 1987 or on May 8, 1997.
On the other hand, the alleged offenses committed by the petitioner for the years 1983-1985
prescribed 15 years from May 8, 1987 or on May 8, 2002.

Also, when the Office of the Special Prosecutor initiated the preliminary investigation of Criminal
Case Nos. 13406-13429 on March 3, 2004 by requiring the petitioner to submit his counter-affidavit,
the alleged offenses subject therein have already prescribed.

CRUZ III VS GO

PEOPLE OF THE PHILIPPINES, plaintiff-appellee, vs. ROGELIO BAYOTAS y CORDOVA,


accused-appellant
G.R. No. 102207. September 2, 1994

FACTS:
Rogelio Bayotas, accused and charged with Rape, died on February 4, 1992 due to cardio respiratory
arrest. The Solicitor General then submitted a comment stating that the death of the accused does not
excuse him from his civil liability (supported by the Supreme Court’s decision in People vs
Sendaydiego). On the other hand, the counsel of the accused claimed that in the Supreme Court’s
decision in People vs Castillo, civil liability is extinguished if accused should die before the final
judgement is rendered.

ISSUE:
Whether or not the death of the accused pending appeal of his conviction extinguish his civil liability.
CRIMINAL LAW 1
(Sunday, Section 52) 293

RULING:
The Court decided on this case through stating the cases of Castillo and Sendaydiego. In the Castillo
case, the Court said that civil liability is extinguished only when death of the accused occurred before
the final judgement. Judge Kapunan further stated that civil liability is extinguished because there
will be “no party defendant” in the case. There will be no civil liability if criminal liability does not
exist. Further, the Court stated “it is, thus, evident that… the rule established was that the survival of
the civil liability depends on whether the same can be predicated on the sources of obligations other
than delict.

In the Sendaydiego case, the Court issued Resolution of July 8, 1977 where it states that civil liability
will only survive if death came after the final judgement of the CFI of Pangasinan. However, Article
30 of the Civil Code could not possibly lend support to the ruling in Sendaydiego. Civil liability ex
delicto is extinguished by the death of the accused while his conviction is on appeal. The Court also
gave a summary on which cases should civil liability be extinguished, to wit:
Death of the accused pending appeal of his conviction extinguishes his criminal liability as well as the
civil liability based solely thereon. Therefore, Bayotas’s death extinguished his criminal and civil
liability based solely on the act complained of.
CRIMINAL LAW 1 CASE DIGESTS
(Sunday, Section 52)

Das könnte Ihnen auch gefallen